Вы находитесь на странице: 1из 424

ONTARIO HIGH SCHOOL

ALGEBRA

AUTHORIZED BY THE MINISTER OF EDUCATION

Storage

Item

EDUCATION
LP6
-

%H COMPANY OF CANADA LIMITED

J35A
Library

RONTO

UBC

C5am

f^ iz

Digitized by the Internet Archive


in

2010 with funding from

University of British

Columbia Library

http://www.archive.org/details/highschoolalgebOOcraw

HIGH SCHOOL ALGEBBA

HIGH SCHOOL ALGEBRA^''

BY
T. CRAWFORD, B.A. chief instructor ix mathematics, university schools, professor of mathematics, ontario college of education t:niversity of Toronto
J.

REVISED EDITION

TORONTO

The macmillan company of Canada, limited

Copyright. Canada, ini

By

THE iL\CMILLAX COMPANY OF CANADA LIMITED

PRINTED IX CANADA

PREFACE
This text covers the work prescribed
Universities
for entrance to the

and Normal Schools. The book is written from the standpoint of the pupil, and in such a form that he will be able to understand it with a

minimum
method
is

of

assistance

from

the

teacher.

frequently used in developing the theory.


is

The question The


is

purpose of this

to lead the pupil to think for himself.

The

close

connection between algebra and arithmetic

constantly kept in view, and in

manv

cases the arithmetical

and algebraic processes are shown


trating the theory,
of the

in parallel columns.
of
illus-

There are numerous diagrams for the purpose

and algebraic methods are applied to many


is

theorems which the pupil meets in elementary geometry.


placed upon the verification of results.

Special emphasis

In the past, sufficient attention has not been given to this

important part of mathematical work.


Provision
is

made

for oral work,

many

of

the exercises

being introduced by a
class.

number

of oral

examples for use in


simple problems are

The equation and the


'jitroduced
in

solution

of

is hoped that the become interested much earlier in the work. Long multiplications and divisions are not included in the work of the first year. They are difficult for the beginner and of little interest, as there is not much to offer in the

the second Chapter.

It

pupil will thus

way

of practical illustrations.

VI

PREFACE
Chapter X., with which the pupil would begin the second

year's work, contains a thorough review of the simple rules.

Here the more complicated processes are dealt with. The graphical work is introduced naturally in illustrating
the negative quantity and
in

the

solution

of

equations.
ruler

Only

graphs

which

can

be

drawn

with

the

and

compasses are included

in the book.

More attention
extraction
of

is

given to methods of inspection in the

roots.

eUminated, as cube root

The long process for cube root is not now required in arithmetic.
is

is

The work on

ratio

and proportion
is

presented in as simple

a form as possible, and

intended

onl}' as

an introduction to
illustrations

the senior work in this subject.

which are given should make

it

The geometrical more interesting.

The

division

method
it

of finding highest

common

factor has

been discarded, as

is

usually performed mechanically and

not understood by pupils.


is

The elimination method which


Finding
the
highest

used will be found easy to apply with expressions which


not
too

are

comphcated.

common
is

factor of expressions of
little

the fourth or higher degrees


of

of

algebraic

value,

and few examples

such problems

will

be found in the book.


exercises at the end of each Chapter will be

The review
work

found useful, particularly for the purpose of reviewing the


of a previous term.

On

the recommendation of experienced teachers the answers

are not given to simple examples, or to such examples as the


pupil can verify without difficulty.

CONTENTS
CHAPTER
Algebraic Notation

I
PABV
1

Arithmetical and Algebraic Signs and Symbols (1). Fundamental Laws (3). Factor and Pro-

duct
like

Coefficient

Power and Index (7). Terms (8). Addition and Subtraction of (9). Terms (9). Use of Brackets (11). Review
(6).

Exercise (13).

Simple Equations

....:....
II

CHAPTER

16

Meaning of Equation, Solving an Equation, Root of an Equation (16). Axioms used (18). Verifying Results Problems solved by (19). Equations (21). Review Exercise (25).

CHAPTER
Positive

III

and Negative Numbers Graphical Representation of Positive and Negative Numbers (28). Concrete Examples of Negative Numbers (30). Signs of Operation and Signs
of

28

Quahty

(32).

Absolute Value

(32).

Review

Exercise

(34).

CHAPTER
Addition and Subtraction

IV
36

Addition of Quantities with like Signs (36). Compound Addition (37). Addition with unlike Signs (39). Indicated Additions (41). Subtraction the Inverse of Addition (42). Rule for Subtraction (43). Removal of Brackets (45).

Review Exercise

(47).

CONTENTS

CHAPTER V
Multiplication

and Division

49

Multiplication of Simple Positive Quantities (49). Index Law for Multiplication (49). Rule of

Signs

(50).

Compound
(55).

Multiplication

(53).

Verifications

Division

Quantity

(57).

Index
(60).

Law

a Simple for Division (58).

by

Review Exercise

CHAPTER
Simple Equations {continued)

VI
62

Equation and Identity' (62). Transposing Terms Simple Fractional Equations (67). Steps (63). in the Solution of an Equation (67). Problems
(69).

Algebraic
(74).

statements

of

Arithmetical
(75).

Theorems

Review Exercise

CHAPTER
Simultaneous Equations

VII
78
(78).

Equations with two Unknowns


Solution (79).

Method

of

Ehmination
(85).

(80).

Fractional
(83).

Simultaneous Equations

(82).

Problems

Review Exercise

CHAPTER Vin
Type Products and Simple Factoring
88
of

Monomial Factors
(89).

(88).

Product

two Binomials

Factors of Trinomials (90). Radical Square of a Binomial (94). Square Root of a Trinomial (95). Difference of two Squares (97). Numerical Applications (99). Review Exercise (101).
Sign
(92).

CONTENTa

ti

CHAPTER IX
Simple Applications of Factoring

Highest

Common

.....
Algebraic Fractions
Multiplication and

10?

Factor (103).
(105).

(104).

Lowest Terms

Division of Fractions (106). Multiple (107)

Lowest

Common

Fractions (108). Review Exercise (111).

Addition and Subtraction of Mixed Expressions (110).

CHAPTER X
Review of the Simple Rules Collecting Coefficients Brackets (114). Multiplication with Detached Coefficients Division by a Compound Quantity (121). fying Division (122) Inexact Division
114
(117). (118).

Veri(124).

Review Exercise

(125).

CHAPTER XI
Factoring (continued)

128

Factors by Grouping (128).

Complete Squares (130). Incomplete DifEerence of Squares (132). Sum. and Trinomials (137), Squares (135). Differenc of Cubes (140). The Factor Theorem Easy Quadratic Equations 1144). (141).

Review Exercise

(146).

CHAPTER Xn
Simultaneous Equations {continued)

Elimination by Substitution and by Comparison


(149).

...
(161).

149

Special

Forms

Equations with three Unknowns (152). Solution of of Equations (154).

Problerj> '156).

Review Exercise

CONTENTS

CHAPTER
Geometrical Representation of

Xm

Number Function of x (165). Variables and Constants (165). Arithmetical Graphs (166). The Axes (169). Graph of an Equation (170). Coordinates (170). Plotting Points (171). Linear Equation (173). Graphical Solution of Simultaneous Equations Special Forms of Equations (176). (175). Review Exercise (177).

....

PAOX

165

CHAPTER XIV
Highest

Common
of

Factor and Lowest


(180).

Common
(185).

Multiple

180

By Factoring
the

By Elimination

(183).

Product

H.C.F. and L.C.M.

Review

Exercise (187).

CHAPTER XV
Fractions

....
in the

188

Changes

Form

of a Fraction (188).

Lowest

Terms
Special

(190).

Types

Addition and Subtraction (191). Cyclic Order (194). (193).

Multiplication and Division (197).

Complex
(201).

Fractions (199).

Review Exercise

CHAPTER XVI
Fractional Equations

...
(204).

204

Cross

Multiplication

Special

Forms

of

Fractional Equations (206). Literal Equations with one Unknown (209), with two Unknowns
(212).

Review Exercise

(214).

COlfTENTS

Jd

CHAPTER XVn
PAGE

Extraction of Roots

216
(216),

Square Root by Inspection

by the Formal Cube of a Binomial (222). Cube Root (223). Higher Roots (224). Review Exercise (226).

Method

(217).

Verifying Results (218).

CHAPTER XVIII
Quadratic Surds

228

Surd and Rational Quantities (228). Mixed and Entire Surds (229). Like Surds (23D. Addition and Subtraction (231). Square Root Table (232). Conjugate Surds (233). Rationalizing a Denominator (235). Surd Equations (237). Review Exercise (238).

CHAPTER XIX
Quadratic Equations

240

Standard Form (241). Solution by Factoring (242), Irrational by Completing the Square (244). Roots (247). Inadmissible Solutions (248).

Review Exercise

(250).

CHAPTER XX
Ratio and Proportion

253
(253).

Methods

of

Comparing Magnitudes

Com-

parison of Ratios (254). Proportion (256). Finding a Ratio by Solving an Equation (257).

Mean

Proportional (261). Ratio Theorems (263).


(266).

Review Exercise

:m

CONTENTS

CHAPTER XXT
PAOI

The General Quadratic Equation Solution of Literal Quadratics (268).

268
Solution by
(271).

Formula
tions

(270).

Imaginary Roots
Quadratics

Solved like Exercise (276).

(273).

EquaReview

CHAPTER XXII
Simultaneous Quadratics

279

Three Types
f288).

Simultaneous Quadratics (279/. Graphical Solutions Special Methods (284).


of

Review Exercise

(290).

CHAPTER XXIII
Indices

293

The

Index Laws (294). Fractional, Zero and Operations with Negative Indices (297). Fractional and Negative Indices (301). Contracted Methods (303). Review Exercise (305).

CHAPTER XXIV
Surds and Surd Equations

308

Varying Forms of Surds of different Orders (308). Surds (309). Surd Equations (312). Extraneous Roots (313). Square Root of a Binomial Surd Impossible Imaginary Surds (320). (317).

Problems

(323).

Review Exercise

(324).

CHAPTER XXV
Theory
of

Quadratic Equations

327

Sum and

Reciprocal Product of the Roots (327). Functions of the Roots (331). Roots (328). Character of the Roots (335). The Discriminant Factors of the Quadratic Expression (337).
(338).

Review Exercise

(341).

CONTENTS

CHAPTER XXVI
Supplementary Chapter

344

Factors of the Product of two Trinomials (344). Sum and Difference of Cubes (346). Factors The Factor Theorem by Grouping (349).
(351),

Symmetry
(355).

(354).

metry
f362).

Identities (359).

Factoring by SymReview Exercise

Answers

365

HIGH SCHOOL ALGEBRA

HIGH SCHOOL ALGEBRA


CHAPTER
I

ALGEBRAIC NOTATION
Use of Arithmetical Signs. In arithmetic, signs are I. used to abbreviate the work. In algebra the same signs are used, with the same meanings and for the same purpose.

BZSRCISB
Write
tiie

following statements in the shortest

way you

can, using

the signs and symbols with which you are familiar in arithmetic.
1. 2.
3.

Two and two make

four.

The sum of five, ten and Six and four is the same
Seven times eight
is

twentj'
as four

is

thirty-five.
six.

and

4.
5.
6.

the same as eight times seven.


five is seven.

The

difference

between twelve and

7.

Ten exceeds six by four. The excess of twenty over

fifteen is five.
is

8.
9.

The

defect of thirty from a hundred

seventy.

Thirty-six divided

by four
is

is

nine.

10. II. 12.

Three score and ten

seventy.

One half The sum

of the

sum

of seven

and

five is six.

or the product of three, five

and seven

is

the

same

in

whatever order they are written.


13.

Three multipUed by four


is

is

twelve, therefore twelve divided

by three
14.

four.

The square
is

of

four

is

sixteen, therefore

the square root of

sixteen

four.

In the preceding exercise you have Algebraic Symbols. 2. used symbols to represent the numbers stated and signs tc show the operations performed on those numbers. In algebra, sj-mbols are used more extensively than ic
Arithmetic.

A
I

H
it

If

the length of this line be measured


inches.

will

be found tc

But without measuring it, we may say that be two the measure of its length is some definite number which might be represented by the letter a. The measure of the length of another line might be represented by h. The cost of an article might be c cents, or the cost of a farm might be x dollars, or the weight of a stone
might be m pounds. Here a, b, c, x, m are algebraic number-symbols, or briefly algebraic numbers. The symbols 1, 2, 3, etc., used to represent numbers in
arithmetic are called
arithmetical

number-symbols

or

arith-

metical numbers. In algebra the number symbols of arithmetic are also used. For the present, when letters are used to represent numbers,

will

be understood that each letter represents some integral

or fractional
3.

number.

In this square the measure of Signs of Multiplication. What is the is a. the length of the side C

AB

measure
;

of

the

length

of

BC

of

CD

of

In algebra, ' multipUcati a being understood. dot repreijnting multipHcation.


Thus, Thus,

AB-}-BC of AB-{-BC-\-CD 1 The measure of the perimeter (sum of all the a+a-j-a+a or 4 times a or 4xa. sides) is xa or a X 4 is usually written 4a, the sign oi
It
is

also written 4

a, the

4xa=4.a=4a, and
if

as in arithmetic,
of

is

a short way of writing


ie

a+a+a+a.
a =6, the measurf
the perimeter cf the squar?

+6-|-6+6=4xt = M,

ALOBBRAIC NOTATION
ft will

and 4
it

is

may
The

be observed that in algebra the multiplication of a only indicated in the form 4a, while in arithmetic be actually performed as in the result 24.
pupil

{twenty-four)

must recognize the difference between 24 and the product of 2 and 4 or 2 x 4 or 2 4. When
.

two numerical quantities are to be multiplied, the sign of multiplication must be used, so that as stated, 24 may be When both factors are not numerical distinguished from 2x4. as 4xa or axb, the sign is omitted and these are written in the form 4a, ah.
4.

Signs of Division.

tained

As in arithmetic, the quotient obby dividing one number by another may be TVTitten

in the fractional form.

In arithmetic the division may be actualty performed, as which may be written | or 2, but it is frequently only indicated as in 6-^7, which is written 5. So in algebra, the quotient obtained on dividing a by h,
in 6-T-3,

or a-i-6,

is

written

-r,

and

here, as in multipUcation, the division

can only be indicated unless the numerical values of a and


h are
5.

known. Some
Fundamental
Laws.

Since the letters

used in
laws
of

algebra

represent

arithmetical

numbers,

all

the

arithmetic must be true also in algebra.

In

arithmetic.

ALGEBRA
From
(3)

and

(4)

it

follows that a series of additions and

subtractions, or of multiplications

and

divisions,

may

be

made

in

any

order.

3a4-46 2c

In finding the numerical value of an expression hke for given values of a, b and c, the operations are performed in the same order as in arithmetic, the multiphcations being performed first and then the additions and
subtractions in any order
Thus, when a = 2, 6 = 3, c=l,

3a+46-2c=3x 2+4x3-2x1 = 6+12-2=16.


Similarly, for the

same values of a, ab+bc _ 2x3 + 3x

b, c,
1

a+b "

+3

_ 6+3 _ 9 5 ~5*

Note. Many of the examples in the following exercise may be taken orally. The pupil, however, is advised to write the algebraic forms so that he may thereby become familiar with themi.

EXSROISB
1.

When a = 6, what
_

are the numerical value? of:


1

o
-,

5
a,

6a, -a,

12

2
, ,

5a.,

-7
3

2
2.

When x=5 and When

j/=3,

what

are the values of

x+y, xy,
3.

xy, 3x+2i/,

2x3y, Ixy
of

m4i, n=6, r=2, find the values

m-\-n-\-r, m-{-r--n, mn-{-mr,


4. of a

mrn, 4n 3to 6r.

and

Express algebraically the sum, the difference and the product What are their values when a=8 and fc=3 ? h.

5.

The

quantities a, b

and

c are to
is

be added together.

Express
!

the

sum
6.

algebraically.
is

What

its

value

when a=6, 6=4, c=12


is

When a

divided by 6 the quotient

expressed in the form


is

-.

When c is added to the quotient of x by y, how What is its value when z=12, ?/=4, c=10 '
7.

the result expressed

A boy has p marbles


has he

he wins q marbles and then loses


if

r marbl***.

How many

now

How many

p=5, q\\, r=4

ALOBBRAIC NOTATION
8,

When o=4 and 6=5,


The

find the numerical value of

12a-56 + 6o-76-i-10,
9.

sides of a triangle are a, b


tlie

and

express algebraically the

perimeter and

semi-perimeter.

What do

they become

if

a=13,

6=14, c=15?
10.

Find the cost of 8


;

articles at 5 cents

cents each

of x yards of cloth at b cents a

each of 7 articles at k yard of m tons of coal at


; ;

n dollars a ton.
11.
dollars

How many
and y cents
;

cents are there in 4 dollars


in a quarters

in

x
?

dollars

in x

and

ten-cent pieces
;

12.
in

Find the number


;

of inches in 2 yards

in 3 feet
;

and

7 inches

a yards
inches.
13.

in 6 feet

in z feet

and y inches

in

yards n feet and

What What

operations are to be performed to find the numerical

ealue of ma-\-nb,
14.

when

a =2, 5=5,

m=3, n=6

What

is

the value

operations are to be performed to find the value of -^^^


a-\-b
?

when a =5, 6=6, x=15, t/=7


15.

What

is

the value
in

By
In

varying the order of the


?

letters,

how many ways can


f

you write a+b-^c


16. 17.

how many
figure,

different

ways can you write zyz

In the
I

BC

is

twice as long as
is

^5.

If
1

B
'

C
'

AB \& of AC
18.

units in length,

what

the length of

BC

In the

figure,

BC

is is

three times as long as

AB

and

CD
C
1

is

twice

as long as
length,

AB. what are

If

AB

x units in

/\
'

b
^

D
'

the lengths of

BC

CDt BDl AD1


19.
s

In the following statements


:

c represents

the cost of an article,

the selling price, and g the gain


(1)

s-c=g,

(2)

c-\-g=s, (3)

sg=c.

Read them and explain


20.

their meanings.

What
is

is

the next integer above 27


?

What

the next integer above n

The next below 27 ? The next below n 1


is

21. If n is an even integer, what and the next even inteser below it ?

the next even integer above

it


22.
If

ALGEBRA
I
is

any number,, what


?

is

the

number which

is

5 greater than

less

than x

23.

A
?

boy

is

10 years old.
old

How

old will he be in 6 years


?

In

m years
24.

How
is

was he

4 years ago

n years ago

A man

x years
? ?

old.

How

old will he be in

n years

How

old

was he

m
is

years ago

In

how many

years will he be three times

as old as he

now

25.

A boy wae p years


?

old 3 years ago.

How old will

he be 15 years

from now
26.

Explain the difference between


c

a+c

and a.-.
c

What

are their values


27.

when a=6, 69, c=3


is

The

side of one square

the difference in their perimeters.

a and of a smaller one is 6. What Is the difference if

Indicate

a=10 and

6=6?
28.

The
is

sides of

one rectangle are a and

b,

and

of another are c
first rect-

and

d.

angle
29.

Indicate the difference in their areas, (1) when the the larger, (2) when the second is the larger.

What
?

arithmetical

number does

10x4-?/ represent

when x=5

V=3
30.

When x^l, y=^ ? When a=3, 6=4, c=5, d=0,


(1)

find the values of


(2)

10a+46-5c+3d.

5ab+2cd-3ac.
2o-t-6 c+a

(3)

\ac+^c-lad.
Product.
is

(4)

-r-.

6.

Factor

and

together the result

called

WTien numbers are multiplied the product, and the numbers


factors of the product.

which were multipUed are called the


Thus,

3x5=15,

therefore the feictors of 15 are 3

and

5,

so a

xb = ab,

therefore the factors of ab are a

and

b.

The
factor

factors of 3z are 3

and

x.

The

is

called a numerical

and the

x,

literal factor.

X 2 X 3,

Just as 12 may have different sets of factors as 3x4, 2x6, so 3xy has the factors Sxxy,3xxy, xxZy or Sxxxy.

The prime

factors of 12 are
3.

2,

and

3.

and the

simplest

factors of 3ry are

x and

y.

ALGEBRAIC NOTATION
the same, but

In whatever order the factors are written the product is it is usual to write the numerical factor first and the literal factors in alphabetical order.
7.

Power and Index.


is

What
?

is

the area of a square whose

of the area of thesquare in art. 3 is axa, which is written a^, and is read " a square," or " a to the second power." The product when 2 a's are multiphed together is called the power, and the 2 is called the index or exponent of the
side

7 inches in length

The measure

power. If the edge of a cube is 6 inches, what is the sum of all the edges ? What is the area of each face of the cube ? What is the area of all the faces ? What is the volume of the cube ? If the edge of a cube is a, the sum of all the edges is 12a. The area of each face is a^, and of all the faces is Ga^. The volume is axaxa or a^, which is read "a cube," or " a to the third power."

The pupil must distinguish between 3a and means 3xa, and the latter axaxa. Thu8,ifa = 5, 3o=3x5=15,
but
0^

a^.

The former

= 5x5x5= 125,
3 (1-14, Oral)
? ?

BXERCISE
1. 2. 3. 4.
5. 6.
2,

What What

are the prime factors of 35, of 42, of 75 are the simplest factors of 5xy, of

6mn

Express 3a6c as the product of two factors in four different ways.

Give two

common

factors of 15a6

and
2*

256c.

Find the values

of 3^

2^,

10^

x 5\

x5,3^x2^x 5.
10, 16 as

27 as a power of
7.

Using an index, express 100 as a power of 3, 625 as a power of 5.

a power

oJ

What What

is

a short

way

of writing

a+al a+a+al axal axaxat


8.
is
7

aaaa
is

the area of a square whose side

6 inches

whose

side

is

x inches

B
9.

AW)SBRA
What
is

is

the volume of a cube whose edge


?

is

3 inches

whose

edge
10.

inches

When a=4, what


?

is

the value of a^

of 2a

What

is

theii

difference

11.
12.

When x=2, what


is

is

the difference b^iween x^ and 3x

What when x=ll ?


13.
If

the difference between

"x

square-' and "twice

x"

Tn=\0, what

is

the difference between the square of

3m and

three times the square of


14.

m?
is

The
is

side of a square

x inches and of a smaller one


?

is

y inches.

What
15.*

the

sum

of their areas

What

is ?

the difference

What do

these results
If

become when
and y=2,
3x2,

x 10 and y=6

x=6

find the numerical values of

x^+y, x+y-, x^-y\ 2x^-3y^.

16.

Find the values of x'+x^-f-x for the following values of x


2, 3, 0.

x=l,
17.
18.

If

j/=4x- 7,
unit of

find the value of


is

if

x=2,
'

if

x=3,
is,

if

x=2^.

The

work

" a day's work,

that

the work which

one
in 5

man can do in one daj. How many units of work can 3 men do days ? 6 men in x days ? m men in n days ? a men in a days T
If

19.

a=3, 6=2, c=l,


that

find the quotient

when

a^-\-b^-}-c^ is divided

by 2a+b c.
20.

Show
If If

x^+26x has

the

same value
greater
is

as 9x^-1-24

when x=2
?

or 3 or 4.
21.

x=10 and y=5, how much

x^+i/- than 2xy

22.

we know
23.
If

d represents the diameter of a circle and c the circumference, Find d when c = 22. that c=S}d. Find c when d=14.

represents the area of a circle and r the radius,


;

A = 3]r^.

Find

A when r 7

when r=14.

24. By arranging the factors in the most suitable order, find th( values of 2*. 53. 252.43, 125.25.

The parts of an algebraic Terms of an Expression. which are connectec' by the signs of addition or expression
8.

subtraction are called the terms of the exuression

ALGEBRAIC NOTATION
ix*

Thus, the expression 2a+36 has two terms, and the expression 3xyy* has three terms.

Quantities which are connected by the signs of multipUcation


or division are not different terms.
Thus,
9.
4oa; is

only one term, so

a*
is -r-

CoefiBcient.
x.

or co-factor, of

In the product 4a;, 4 is called the coefficient, In ah, a is the coefficient of b and b is
is

the coefficient of

a.

The 4

is

a numerical coefficient, and the a or 6


is

literal

coefficient.

In any product, any factor


rest of the product.

called the coefficient of the

and 5ax

Thus, in 5abx, 5 is the coefficient of abx, 5a is the coefficient of b.

is

the coefficient of bx,

the coefficient

In any term where the numerical coefficient 1 is understood.


is

is

not stated,

Thus, in xy the numerical coefficient

Addition and Subtraction of Like Terms. 10. When terms do not differ or differ only in their numerical coefficients, they

are called like terms.


Thus, 2ab, 5ab, ^ab are like terms, but 3a, 46, Gab are unlike terms.

may

In arithmetic, quantities which have the same denominations be added or subtracted.


3ft. -f 4ft.

Thus,

- 2ft. = 5ft.
denomina-

$12-$10+$8-$3 = $7.

We cannot add or subtract quantities of different


tions, unless

we can first reduce them to the same denomination.

Similarly, in algebra, like terms

may

be added or subtracted.

Thus,

5a+ 2a = 7a, 5a 2a = 3o, 6a6-f 5a6- 3a6= 1 lo6- 3o6 = 8a6,


Sx*-6x*+9x*-2x*=\lx'~Sx'=--9x*.
in

In the laat example we may, of course, perform the operations ^der in which they occur and obtain the same result

the

10

ALGEBRA
Unlike terms can not be added or subtracted.

Thus, the sum of 3a and 56 can be indicated in the form 3a4-56, but they can not be combined into a single term vmless the numerical values of a and 6 are given.

BXBRCISE
1.

(1-8,

Oral)

What What

is

the numerical coefficient of each term in the expression

5a3+a2+|o
2.
is

the

sum

of the numerical coefficients in

2x^
3.

+ 3xy+x+y

Which are like terms in the expression

5a24-26-3a+76-4a2
4. 5.

In 66cy, what

is

the coefficient of bey


of
:

oi cy

oi by

of 6

What

is

the

sum

(1) 2a, 3a, 4a.


(3) 4a2,

(2)

5m, ^m, fm.

la\

5a^.

(4) 3xy, 4xy, 2xy.

6.

If

x=2,

find the numerical value of the

sum

of Sa;^

and 4x^

in

two
7.

different

ways and compare the

results.

Simplify the expression

3a+86+2a+6+a+3fc by combining

like terms.

8.

Express in as simple a form as possible


(1)
(3)

5TO4-7m 3m 2m.

(2) (4)

%ah3ab + 2abab.
15a+106-7a-(-46.
:

3x+a+2x+a.

9.*

Combine the like terms in the expression 2x+7t/+5z-a:+2i/-3z+3a;-4t/-z

and

find its value


If

when

a;=3, i/=5, z=10.

10.

a =6, find the value of

15o2 lOa^ 3a2+8a-5a 20.


11.

What

arithmetical
?

number does lOOa+106+c


?

represent

when

a=2, 6=3, c=4


12. 13.

When a=9, 6=5, c=7

SimpUfy

2x^-\-3x-\-l-x'^-{-\\x-2-x^-4x+b.

walks 4x feet East, then x feet West, then 3a; feet East, then 5x feet West. How far is he now from the starting point and in what direction from it ?

A man

ALOEBRAIC NOTATION
14.

11

began to work for a firm on a salary of x dollars a year. was double the salary for the preceding year, how much did he earn in four years ?
If his

A man

salary for each year

15.

If x-\-Sz-\-5x is

equal to 72, what


is

is

the value of x

How

do

you know that your answer


16.

correct

Write in the shortest form you can


aaa+aaa-{-aa-\-aa-\-a-\-a-\-a.

17.

Find the average

of (1) 10, 8, 15, (2) 3x, Ix, 5x.

11.

Use

of Brackets.

In algebra brackets are used for the

same purpose and with the same meanings as in arithmetic. In finding the value of 10+8+5, we may perform the additions in any order, but if we wi'ite it 10+ (8+5), it is understood that the 8 and 5 are first to be added and the sum of 10 and the result is to be taken. Similarly, a-\-{b-\-c) means that the sum of the numbers represented by b and c is to be added to the number represented by a.
In the expression 7+5x2, the multipUcation is to be performed first, and then the addition. If, however, we wish the value of (7+5) x 2, we must add the 7 and 5 before
multiplying by
2.

Although 10+(8+5) is equal to 10+8+5, it is clear that (7+5) X 2 is not equal to 7+5x2, the former being equal to 24 and the latter 17. When a is to be multiplied by b, the sign of multiphcation
omitted the indicated product so when (7+5) is to be multiphed by 2 we may write 2(7+5) or (7+5)2, the sign of multiphcation being understood.
is
;

It

is

the order in

thus seen that one of the uses of brackets which operations are to be performed.

is to indicate

Thus, 10 (7 3) means that 3 is to be subtracted from 7 and the is to be subtracted from 10. If the values of the letters were given, what operations would you perform to find the values of
result
:

a+{b+c), a-{b+:), a-{b-c), {a-b)-{c-d)t

The

pupil should recognize that 3a^

is

not the same as

12
(3a) 2.

ALGEBRA
The
latter

means that
is

is

first

to be multiplied

by

3 and the product

to be squared.
3a* = 3x4=12, = 3ax3a = 6x6 = 36.

Thu8,ifa=2,

and

(3a)*

are to be considered as a single quantity, that

Brackets also indicate thai the numbers within the brackets is,_ they are used

for the purpose of grouping.

The dividing

line

between the numerator and denominator

of a fraction has the

same value

as a pair of brackets.
is c

Thus, in

-, a-\-b is

a single quantity and so

c-\-d

+ d.

The

fractional

form

is

another way of writing (o+6)-7-(c+d).

E3XB3RCISB 5
Perform the operations indicated
1.
:

(1-18, Oral)

ALGEBRAIC NOTATION
26.

13

(3o+26 c)(o 36).

27.

a^+b^+c^2{ab-\-bc-\'Ca).

2a 36 2a5b+2c~ a+b2c
a-\-Zhc
30.

a 6
'

'

be

'^

bc c+a

c+a
o+26

When a=6 and 6=3, show that R(o 6) +3(a+6) = 2(4a-6).

EXERCISE
1.
If
a;

6 (Review of Chapter

I) 4a;

represents a certain number,


?

what does
will

represent

^xl
2.

X*

3x*

If
?

number
3.

a represents a number, what y times the number ?

represent 5 times the

How

do you indicate that y


?

is

to be

added to x
a.

That x

is

to be subtracted from y
4. 5.

Indicate the
If

sum

of x

and y diminished by

one yard of cloth costs x cents, how


dollars
?

many

cents will 10 yards

cost

How many
If

6. 7.

a yard of ribbon

is

worth y cents, how much


?

is

a foot worth
it

A man
If I

bought an
did he

article for

x dollars and sold

at a loss of

dollars.

What

sell it for

8.

paid a dollars for


c articles

b articles,

how much
price
?

did I pay for each

What would

cost at the

same

A boy has a dollars. He 9. How many cents has he left


?

earns b cents and then spends

c cents.

10.

have x

dollars.

If I
left
?

pay two debts


is

of a dollars

and

b dollars,

how much
11.
If

shall I

have

one number

is

x and another

5 times as large,

what

is

the

sum

of the
If

numbers

12. 13.

one part of 10
rn

is x,

what

is

the other part

A man worked
how much

per hour,

hours a day for 6 days. did he earn ?

If

he was paid $2 In a

How far can a man walk in 5 hours at 4 miles per hour 14. hours at b miles per hour ?
15.

A man

bought x acres

of land at a dollars per acre

and sold
7

it

at a loss of 6 dollars per acre.


16.

What

did he
?

sell it for

What number By how much

is

15 greater than x

15 less than x
6

17.

does a* exceed 6*

when a = 7,

=3

i4

ALGEBRA
z

When x=l, j/=2, 18. 2x+5y-3z, 7x-3y+z T


If a = 3, 19. 6a-26*+4c.

= 3,

what are the values

of

x-\-y-"t,

= 4,

c=0,

find the values of 2ab, Aac, a'+b*+c>,

20.

If

x=^
?

and y=s, what are the values

of 3x

2y,

6xy, 2x*

3y*, c*),

8x- 272/3
If 21. c(a-6).

a= 10, 6 = 5, c = 3,
is

find the values of a(6+c),

a(6 c),

a{6*

22.
23.

What What
d
?

the

sum

of 2x, 5x, Ix

and 3x

Simplify
is

5a 3a+lla+o-10o.
the average of 20, 15,
0, 8,

24.
a, 6, c,

12

Of

2a, 3a,

la

Ot

25.

In 8 years a

man

will

be x years

old.

How

old

was he
If

8 years

ago

26.
$,

has $20 more than A,


7

has $20 more than B.

has

how much has C

27.

What

is

the
?

sum

of the numerical coefficients in the expression

3a+ ^a6+ac+ fad


28.

power
29.

of 3

Express 1000 aa a power of 10 64 as a power of 4.


;

32 as a power of 2

81 as a

Express

15, 105, 3a6, 35x*y, as


it

the products of simple factors.

30.*

How

long will

take

me

to

walk o miles at 3 miles per houi


?

and
at

ride 6 miles at 12 miles per

hour

A farmer buys 5 lb. of tea at x cents per lb. and 20 lb. of sugar y cents per lb. He gives in exchange 7 lb. of butter at z cents per lb. If he still owes something, how much is it ?
31.
32.
If I

per

lb.,

buy 100 lb. of nails at a cents per what is the average cost per lb. ?

lb.

and 200

lb.

at 6 centf

What is the total number of cents in x five-cent pieces, y ten33. cent pieces and z half-dollars ?
34.

What number
c

is

represented by

lOOOa+1006-f lOc + d, when


?

a=I, 6=2,
35.

= 3, d=4 ? When a=4, 6 = 0, c=l, d = 9 When a = -2 and 6 = -I. what are the values
7

of

a+6,

ab, ti

^, a+6, a-6 ab

ALGEBRAIC NOTATION
36.
if

16

fraction of the

can do a piece of work in 10 days and S in 15 days, what work can they together do in 1 day ? What fraction could do it in x days and B in y days ?
li

37. If a = 20, 6=15, c=10, d = 5, find the difference between (0+6) (c+d) and (o 6) (c d), also between 3(a+6) 5(c d) and 6(0 -(f)- 3(6 -c).
38.

When
If

x = 7 and y=\, the product of

much
39.

greater than the product of x y,

x-\-y, x+2y, x by x2y, x3y ?

is

how

= 3,

find the value of

-\

CHAPTER

II

SIMPLE EQUATIONS
12.

Idea

see that the scales

draw

If

weight (w) balance ?


4
lb.

article, when you what conclusion do you a 5 lb. bag of salt is placed in one scale pan, what must be placed in the other pan to restore the What must w be to balance a 3 lb. bag and a
of

Equality.

In weighing an

are balanced,

bag

the scales are balanced in each of the following figures, what must w be equal to ?
If

Fia.

1.

Pio.

2.

Fio. 3.

what is w equal to ? what is iv equal to ? If 1^+3 = 5+2, as in fig. 3, what is w equal to ? If the scales are balanced and I add 2 lb. to one side, what else must I do to preserve the balance ? What, if I take away 3 lb. from one side ? If I halve the weights on one side ? If I double the weights on one side ?
If

U)+4 = 9, as

in fig.

1,

If w-\-io= 12, as in fig. 2,

13.

The Equation.
is

When
What
:

a certain number
the
in

is

added to

10 the result
briefly

27.

is

number

The condition expressed shown in the form

this

problem might be more

or in the

10+ a certain number =27, form 10+? =27, where the question mark

stands for

the required number. Any other symbol would answer the same purpose as the

SIMPLE EQUATIONS

17

question mark. Thus, if x represents the required number, then the problem states that

10+a;=27.
an equation and is merely a short given in the arithmetical problem preceding. In order that the statement may be true, it is easily seen that the symbol x must stand for the number 17.
This statement
of stating
is

called
is

way

what

Ex.

When
?

number

is

multiphed by
is

3,

and 5

is is

sub-

tracted from the product, the result

19.

What

the

number
Here,

if

x stands for the number, the problem states that

3a:-5=19.
Before the 5 was subtracted the product was evidently 5 more than
19 or
If 3

19+5

or 24.

times the
solution

number

The
If

may

is 24, then the number must be be written more briefly thus


:

\ of

24 or

8.

3a;-5=19,
.-.

3a;=

19+5 = 24,

x=Jof24 = 8.
That 8
fact,
is

the correct value for the


it
is

that

when

multiplied
is

number is shown by the by 3 and 5 is subtracted

from the product, the result


14.

19.

of

Solving an Equation. The process of finding the value such that 8a:: 5=19, is called "solving the equation," and the value found for x is called the root of the equation.
X,

ESXSRCISS
1.

7 (Oral)
in

State the
:

number

for

which the question mark stands

each

of the following
(1) (4)

5+? = 12.
15=-8 + ?.

(2) (5)

+ 12=20.

(3) (6)

40=62-?.
8+a;=32.
I0-a:=8.

10-V=2. ?-8=42.
:

2.
(1)

What

is

the

number

for

which x stands in each of the following


(3) (6)

a:+6=20.
a:-16=7.

(2)

25=x+6.
12=17-x.

(4)

(5)

IB
3.

ALGEBRA
The
first

liquation in Ex. 2 states that


is

creased by 6 the result

20.

What

when a number is in does each of the other equa^Bone

say

of a

If 3 times a number is 45, what is the number ? If one-hali number is 16, what is the number ? If n stands for a given numberj what would represent \ of the number ? \ of the number ?

4.

5.

For what number does


(1)

n stand in each of the following equations*


(3)

4=24.

(2)

^n = 10.
is is

|m=36.
? ?

(4)

ln=U.

6.
7.

If

2x+5=ll, what
what

the value of 2x

oi

xl

If 3/rt 2=13, If

the value of

3m
?

of of

mT
?
?

8. 9.

^p+3=10, what
-^x 11=7, what
2(a;+4)

is is

the value of |p
the value of
;ix ?

If
If

of
?

a;

10.

= 14,
:

what

is

the value of a:+4

of

a;

Solve the equations


11.
14. 17.

a;+10=30.

12. 15, 18.

3x 2=16.
2n = ll.

13.

5)/

+ 2---17.

4<-5=27.
3m;+2 = 38.

16.
19.

7?i-4=24.

^x 1=4.
iM;+2=5.
5(x-2)=45.
Equations.

2/i+l=4.

20.

3w J=5J.
3(x+l) = 30.

21. 24.

22.
25.

|x-5=15.
i(x-l)=3.

23.
15.

Axioms used

in Solving

If

equal,
to

what
?

is

the result

when the same number


T

two numbers are is added

each
Thus,

if

x=6, what
is

is

x+2

equal to

What

the result
or

from two equal numbers

same number
ia

same number is subtracted when each is multiplied by the when each is divided by the same number ?
the
;

when

or

Thus, if x= \x equal to

10,
?

what

is

x 4

equal to

What

is

3x equal to

What

The preceding
(1)

conclusions

may

be stated thus
to

// the same

number

he

added

equal numbers, the

sums

are equal.
(2)

// the same number he subtracted from equal numbers^

(he remainders art equal.

SIMPLE EQUATIONS
(3)

19

// equal numbers be multiplied by the same number, the // equal numbers be divided by the same number, the

products are equal.


(4)

quotients are equal.

These statements are called axioms, or self-evident truths, and are used in solving equations. The method is illustrated by the following examples
:

Ex. 1.Solve Add 7 to each side,


Divide each side by

.-.

3a:-7=35. 3x 7+7 = 35 + 7, axiom


.-.

(1),

3x = 42.

3,

.".

x= V = l'i> axiom
ia;+2=34.

(4).

Ex. 2. Solve
Subtract 2 from each side,
Multiply each side
.-.

ix+ 2-2 = 34-2, axiom


^x=32. x=64, axiom
(3).

(2),

by

2,

.".

Ex. 3.Solve Add 3 to each side,


Subtract 2x from each side,

5x 3=2a;+12.
.'.
.".

5x=2a;+15.

5x 2x=15,
3x=15.
x=
5.

Divide each side by

3,

.'.

The object
equations
{x) to
is

of the

changes which have been

to get the quantities containing the

made in these unknown

one side and the remaining quantities to the other side. The unknown quantities are usually' transferred to the left side, but sometimes it is better to transfer them to the right.

Ex. 4. Solve

3m+20=5m 16.
.*.

Add

16 to each side,

Subtract

3m from

each

side,

.'.

3m+36=5m, 3m+36 3m = 5m 3m,


36 = 2m,
18

=m

or

m= 18.

16.
first

Verifying the Result.

side of the last

we substitute equation we get


If

18 for

in the

3m+20=3x 18 + 20=74.
If

we

substitute in the second side

we

get

5m-16=5x 18-16=74.

02

20

ALGEBRA
This process
is

called verifying or testing the correctness

of the result.

If

the root obtained

is

the correct one, the

same number when the value found for the unkno\^Ti is substituted. The equation is then said to be satisfled. The beginner is advised to verify the result in every case. Verify the results obtained in Ex.'s 1, 2 and 3.
two
sides of the equation should be equal to the

BXBRCISB
i.

8 (Oral)

3x=lo, what does x equal ? What axiom is used ? What axiom is used 2. If oar -1-2 = 17, what does 5^ equal ? What does x equal ? What axiom Ls used ? What two axioms are used If 2?/ 3 = 13, what does ?/ equal ? 3.
If

4.

If

Ax 4=6, what
?

does ^x equal

What

does x equal

What

two axioms are used


5.
If

?x=6, what does \x equal


?

What

does x equal

What

two axioms are used

What
6.

is

the value of x in the following equations


7.

2x=18.

6x=72.
2x4-1

8.

5x=:16.

9.

3x=6-9.

10.
14.

x+20=25.

11.

= 15.

12.
16.

3x-l=20.
Jx=2|.
9

13.
17.

6x+5=29.
^x=15.

ix=8

15.

x=12.

BXHIRCISB
Solve the following equations, giving
In each case verify the result
1. 4.
7.

full

statements of the methods

3x-rll=47. 3x- 10=65.

2.
5.

7x=60+3x.
10x-L3=3x-f
66.

8.

10.

11.

2x+5=27. 4x=x+21. ix+5=50. 6a 3a=a+5.


14.

3.

13.

8m=36 4m.
12x-652=7x+428.

20+6x-|-5=50-3x-fll.

15.
17.

16.

764x-9=680x-f 12.

Nine blocks of equal weights (w) together with a 20-gram weight Express this by are balanced by weights of 50 grams and 10 grams. an equation and find the weight of each block.
18.

19.

17x 11 is equal in value to 5x-t-121, what is the value What value of will make 11?/ +60 equal to 20t/ 30 7
If
j/

of x

SIMPLE EQUATIONS
17.

21

statemeni

As we have already shown, an equation is merely the in algebraic form of the condition given in an
is

arithmetical problem.

The

solution of the problem

thus obtained by solving the

equation.

EXBRCISS
an equation
1.
:

10
af

State the condition in each of the following problems in the form

2.
3.

4.
5.
is

What must be added to 33 to make 50 ? What must be taken from 90 to leave 40 ? What is the number which when doubled is 36 ? Five times a certain number is 45. What is
If

the
is

number
25.

a number
?

is

doubled and 3 added, the result doubled by adding 27

What

the
6.
7.

number

What number What number


If 8 is
is

is

? ?

is

halved by subtracting 20

8.

subtracted from f of a certain number, the result


?

is

What
9.

the

number

Solve the equation in each of the preceding examples.

18.

Problems Solved by Equations.

The

following examples

will illustrate the

method

of solving problems

by means

of

equations

Ex.
result

1.
is

When

double a certain number and add


is

16, the

40.

What

the

number

Let X represent the required number. Then 2xtS the double of the number. Then 2x+16 is the double with 16 added. But the problem states that this is 40,
.-.

'

2a;+ 16

= 40,

2x=24,
a;=12.

Therefore the required

number

is 12.

The

result should be verified


satisfies

by showing that the number


I

obtained
I got 40=

the given problem.


12
is

Verification;

When

doubled
is

get 24 eind

when

16

ia

added

Therefore the result

correct

22

ALGEBRA

Note that the substitution is made in the original, problem, There might be an error in writing down the equation and then the sohition obtained might satisfy the equation, but would not necessarily satisfy the given
not in the equation.

problem.

Ex. 2. The number of pupils in a class is 33, and the number of boys is 7 greater than the number of girls. Find the number of each.
Let
.'.

x = the number of

girls,

x+7 = the number


x-\-x-{-l

of boys,

= the

total

number,

.-.

x+a;+7 = 33.
2a;

= 33-7 = 26,
.-.

a;=I3,
.'.

x+l = 20,
of

the

number
:

of girls

is

13

and the number

boys

is

20.

Verification

20+13 = 33,20-13 = 7.
100 among A, as A, and
number

Ex.

3.

Divide
x=
3x= 3x+30=

B
C

receive 3 times as

much
the
.-.

and C, so that B may S30 more than B.

Let
:.
.'.

of dollars

receives,

B c

they

all

receive (x+3a;+3x+30) dollars,

a;+3x+3x+30=100, 7x+30=100,
7x = 70, a;=10,

.-. A receives $10, Verify this result.

$30 and

$60.

19.

Steps

in

the

Solution
will

of

Problem.

The examples
problem
:

which have been given

show that

in solving a

the steps in the work are usually in the following order


(1)

Read

the

problem carefully

to see

what quantity

is to be

found.
(2)

Represent this

unknmvn by a

letter.

If there be more than one quantity to be found, represent the others in terms of the same letter.
(3)

SIMPLE EQUATIONS
(4)

23
the

Express

the condition stated in the

problem in

form

of

an equation.
(5)

Solve the equation

and draw

the conclusion.

(6)

Verify the solution by substitution in the problem.

On referring to Ex. 1, we see that there was only one quantity to be found, and therefore step (3) did not appear in the solution. In Ex. 2 there were two quantities to be found, and when we represented the number of girls by z, we could represent the number of boys by
x+1.

The pupil
English, as to

is

advised to
let

make

full

statements, in plain

what the unknown

represents.

Thus, in Ex. 3 to say, only lead to difficulties.


Note.

x=A,

or let x=^A's money, will

The examples

in the following exercise are to

be solved by

means

of the equation

and the

results should be verified in every

case. Although the answers to many of them may be given mentally, the pupil is advised to give complete solutions, so that he may become familiar with algebraic methods.

E3XERCISE!
1.
If 37 is
?

11
is

added to a certain number, the sum

63.

What
What

is

the

number
2.

If 27 is subtracted
?

from a number, the

result

is

5.

is

the

number
3.

A
sum

If the

number was doubled and the result was is now 73, what was the number ?
a number
is is

increased by

27.

4.

When
If five

product, the result


5.

59.

multiplied by 7, and 25 subtracted from the Find the number.

times a

as

if

twice the

number be increased by 6, the sum is the same number were increased by 15. Find the number.
if

6.

What number
If

trebled
?

and the

result diminished

by 36

gives

twice the original


7.

number

you add 19

to a certain

number the sum

is

the

same

aa

if

you add
8.

7 to twice the

number.

Find the number.

Five times a number, plus 19, equals nine times the number,
41.

minus

What

is

the

numbw

24
9,

ALGEBRA
Two numbers
The sum
of
differ

by

11

and their sum

ia

51.

Find the

numbers.
10.
15.

What
S1500.

are the

two numbers numbers ?

is

47 and one exceeds the other by

11.
is

^'s salary is three times B's and the difference of their salaries Find the salary of each.

12.

twice as
13.
other.

much

The carriage horse and carriage are worth $360. Find the value of each. as the horse.
will

is

worth

Divide 93 into two parts so that one part

be 27

less

than the

14.

The length
is

of a rectangle

is

three times the width.

The

peri-

meter
15.

72
is

feet.

Find the

sides.

twice as old as B.

In 10 years the sum of their ages wUl


?

be 41 years.
16.

What

are their ages

Divide $500 between

and

so that

will receive

$20 more

than twice what

wUl

receive.

The sum of two consecutive numbers is 59. What are 17. numbers? (Let x be the smaller number, then x+1 will be
greater.

the the

18.

Find three consecutive numbers whose sum


^'s age
is

is

150.

19.

is

twice B's and

is

7 years older than A.


of each.

The sum

of

their ages

67 years.
difference

Find the age

20.

The

10 feet and the perimeter


21.

between the length and width Find the sides. is 68 feet.

of a rectangle

is

much
.

as A,

Divide S468 among A, B and C, so that and C three times as much as B.

B may

get twice as

22 A railwav train travels miles per hour. If it goes from Toronto to Montreal, a distance of 333 miles, in 9 hours 15 minutes, what is the value of
2.3.

m? A line

20 inches long
is

of the longer part

^ inch

The length is divided into two parts. more than double the shorter one. Find
5x-|-6 equal to 3x -f 40
is
?

the lengths of the parts.


24. 25. 26. oost
7

What
If

value of x will

make

5"o of a

sum

is

$48,

what

the

sum

An

article sold for

$2-61, the loss being

10%. What was the

SIMPLE EQUATIONS
27.

26

Divide $1496

among A,

J's share and


28.

wili get

B and C, so that B will get three times $100 more than A and B together.
as B.

A
If
is

has five times as

he has only twice as


29.

much

much money as B. After A How much has B ?


is

has spent $63

$20

less
?

than | of a sum of money

$10 more than ^ of

it

what

the

sum

30. Three boys sold 42 papers. The first sold J as many as the and the second sold \ as many as the third. Kow many did each

third
sell ?
is

The sum of ^ What is the number ?


31. 32.

of a

number and

\ of the

same number

55.

A man
much
?

paid a debt of $4500 in 4 months, paying each


as the

month

twice as

month

before.

How much

did he pay the hrst

month
33.

The
52.

make

half, third and fourth parts Find the number.

of a certain

number together
part
is

34. Divide 72 into three parts so that the second and J of the third.
35.

first

J of the

What number
its third,

is

that to which

if ?

you add

its half

and take

away
36.

the remainder will be 98

3a = 46c,
.

(1)

(2) (3)

Find Find Find

a,
b,
c,

when when when

6:= 10,
o

=12,
8.

o=

c= 15. c= 3. 6= |.

BXBROISE)
1.

12

(Review

of

Chapter

II)

State the four axioms which are used in solving equations.

Show that 3a; 7 = 2a;+ll.


2.
3.

x=18
if

is

the

correct

solution

of

the

equation

Determine
Solve
(a)

is

4.
(d)

5x4-3
(e)

7x+l = 9a;-9;
5.

a root of 3(a;+6) = 5(a; 1). = 2x+ 9; (6) l+2x=9-2x \\x\ = 5x+\.


;

(c)

3x--7 = 8-2x
five

My

much
6. fehsm

as the lot.

house and lot cost $16,800, the house costing Find the cost of each.
If
5^

times as

A horse and carriage cost $520. the horse, what did the horse cost

the carriage cost $60 more

26
7.

ALUEBKA
Three farmera together raised 2700 bushels of wheat.

raised

three times as

much
?

as B,

and C raised twice as much as A.

How

much
8.

did each raise

What
where

value of x will
r is
Tr

make

136 3a; equal to 172 9x


c
is

9.

Where

the radius of a circle and

the circumference,

c=2irr,

= 3y.
Find Find
c, r,

(a)
(6)

when r= 7 when c = 88

when r=42. when c=ll.


<=
10

10.

If

5=J/<, find 5

when

= 4 and/=32; when
women.

and /= 32-2,

11.

In a company of 98 persons, there are twice as

as men, and twice as are there 7

many

children as

many women How many children


If

Six boys and 15 men earn 5264 a week. 12. four times as much as each boy, how much does a

each

man
in

boy earn

earns a week T

Five times a certain number, increased by 47 is equal to eight 13. times the number, diminished by 43. What is the number ?
14.
If his

An agent charges 3 % commission for collecting an account. charge is $11-13, what was the amount of the account ?
Solve (a) 05x=4;
(6)

16.
\d)

x+-04x=208
is its

(c)

x-06x=235

a;+6%x=630.
height, the area (o)
is

If 6 is the base of a triangle and h 16. given by the formula a=\bh.


(i)
(ii)

(iii)

Find Find Find

a,
6,

h,

when b= 8, /t= 4. when a = 36,71=12. when a=176, 6=22.


is

17 .

The sum
is

of the

difference

15 feet.

unequal sides of a rectangle Find the area of the rectangle.


is

65 feet and their

18. 19.
is

If

6x i/ = 2x + 2/> what

the value of y

if

x=6

?
if

For what number does the question mark stand,

satisfied

when x = 3

5x+i = 3x+?
7

20.
21.

If 4

of X together with 3

of x

is

equal to 35, find


is

x.

State a problem the condition of which

expressed by the

equation
22.

3x 20=x.
Together

has $10 more than A, and C has $20 more than B. How much has each 7 they have $190
23.

turkey costs as

much

as three chickens.

If 2

turkeys and

3 chickena cost $7-20, find the cost of

a chicken.

SIMPLE EQUATIONS
24. 25.

27
?

What number

increased

by

of itself

is

equal to 60

Divide $6400 among A, B and G, so that more than A, and C $160 more than A.
26.

will

get

$120

years.

enterprise doubled each year for five the total net income for the five years was $7750, what was the income for the first year ?
If

The net income from an

27.

If2a6 = 3mn,
(1)
(2)

(3) (4)

Find a, when 6=15, 5. 6, n= Find b, when a= 12, 2. 2, n= Find m, when a= i^, 6 = 6, n = J. Find n, when o= -3, 6 =-6, m = -12.
6
is

m= w=

28.

Show that

a root of the equation

2(x-l)(a;+2) = 4(x+3)(a;-5) + (x-2)(a;+5).

The area of the United States is 4000 square miles more than 29. seventy times the area of England. If the area of the United States is 3,560,000 square miles, find the area of England.
30
Solve and verify
:

(1)

6850+a; = 27a;+350.

(2)

(3)

Jx+Jx+}x = 3380. 16O7x+20=1762z-ll.

CHAPTER in
POSITIVE

AND NEGATIVE NUMBERS

62
51

60
59 58 57

66 65 54
53 62

POSITIVE
3.
4.
6.

AND NEGATIVE NUMBERS


? ?

29
?

What was What was

the highest temperature the range of temperature


it

What

the lowest

Between what hours was

rising

? ?

6.
fall

How much
When was

did
7
it
?

it rise

between 10 and 11

How much

did

it

between 6 and
7.

60, 58, 55

8.
fall

Between what hours did most rapidly ?

it rise

most rapidly

When

did

it

The percentage of games won by a baseball team, up to the 9. beginning of each month of the playing season, was as follows
:

June, 66

July, 63

Aug., 60-5

Sept., 62

Oct., 61-5.

Draw a graph showing


10.

these changes.

boy's height in inches, for each year from the age of 7 to the

age of

14,

was

44, 47, 50, 51, 52-5, 54, 56-5, 58.

Draw a graph

to

illustrate the variations in his height.

21. Negative Numbers. temperature for a week.

This diagram shows the

average

Thus on Monday it was 25 above zero, while on Thursday it was 15 below zero.

We
ically

might express

this algebra-

by saying that on Monday the temperature was +25, and on


Thursday
it

was

15.

The
positive

number +25 is called a number and is read " posi25


or
is

tive "

number, and

" plus " 25, while 25 is called a negative read " negative " 25 or " minus " 25.

A negative number is therefore one which is measured on the opposite side of zero from a positive number.
sxsrcise:
1.

14

in the diagram.

Using algebraic signs, write down the temperature for each day Also read the temperature. days was the temperature negative
?

2.

On what

30
3.

ALOE BR A
How much higher was it on Monday than on Thursday lower on Tuesday than on Saturday ?
If the
? ?

How
will it
?

much
4.

temperature

is

be then
5.

If it

had

fallen 10",

30 and it rises how much Avould

40,
it

how much

have been then

that

The temperature at which mercury freezes is 39C. What does mean ? How much lower is it than the normal temperature
blood which
is

of the

4-37C.

sells above par is positive and the price below par is negative, make a diagram similar to the preceding, showing the prices of a certain stock for a week, when the record was as follows

6.

If the price at

which a certain stock

Mon., 4 above par Tues., 2 below Thurs., at par; Fri., 3 below; Sat.,
;

Wed.,
below.

above,

22.

Distances measured on a Horizontal Line.

9
I \

B
1 \
1

O
+
\
\

AC
\

-5

-4

-3

-2

-1

*l

+2

^3

+4

+5

On this diagram the distance between each successive marking represents one foot.
What
differ
is

the length of

OA

of

OB

In what respect does


this difference
?

OA

from
is

OB

How

might we use signs to show

It

usual to consider measurements

made

to the right as

positive

and

to the left as negative.


T

point is +5 feet from ? What one is 5 feet from a point moves from O, 4 feet to the right and then 7 feet to the how far is it then from O ? Is the distance positive or negative ?

What

If

left,

We

thus see that in addition to the numbers of arithmetic

which begin with zero and extend indefinitely in one direction, we now have another series of numbers which also begin with zero and extend indefinitely in the opposite direction. In each series all integral and fractional numbers are included.
23.
(1)

Further Example^ of Negative Numbers.

A man

has property worth $100, and debts amounting

POSITIVE
to $60.

AND NEGATIVE NUMBERS

31

When

he has paid his debts he wiU have property

worth $40.
Thus,
If,

$100- $60=

$40.

however, he has debts amounting to $100, are paid he will have nothing left.

when

these

Thus,
If

$100- $100=

$0.

he has debts amounting to $140, when he has paid a'.' Ae can he will still owe $4G. We express this algebraicall thus $100- $140= $40.
:

In the first case we say that his net assets are $40, in the second they are zero, and in the third they are miniis $40. When we say his assets are $40, we mean he is ji40 in debt. It will be seen that the difference in mea'iing between

+40 and 40 when


same
as

referring to dollars

is

practically the

the
in

difference

between

+40 and

40,
Line,

when
as in

referring to degrees of temperature, as in art. 21, or to distances

measured
art. 22.
(2)

opposite directions on a horizontal

If

man

on another, what

gains $20 on one transaction and loses $15 If he had lost $25 on is the net result ?

the second transaction what would have been the net result ? If we attach a plus sign to the result when it is a gain, how

may we
If

indicate a loss

represents a sum gained ^nd L a sum lost, state the result in each of the following, attaching tt proper sign

1.

$30G+$20(?.
$30
If

2.
5.

$30

4.

L+ $20
a

O.

04 $40 O^

$20 i.
$4f L.

3.
6.

$30Z,+ S20i.
$20

(3)

represent a
lost,

game won game lost.

is

represented

6+ $60 L. by +1, then 1 would


is
:

In a series of games I find that my record won, lost, won, lost, won, won. This might be represented thus
:

won,

lost,

+ 1-1-1 + l-l + l-l + l + = +5-4= + l.


l

What

does this result

mean

?
:

Write in a similar way the following record lost, lost, won, lost, won, lost, lost, won. Also the following won, lost, drawn, won, won.
:

32
(4)

ALGEBRA

In locating points on the earth's surface, the distance north of the equator (north latitude) is said to be positive, and south of the equator negative.
in degrees
Thiis, the latitude of

What

is

44 and of Rio de Janeiro is - 23. Toronto is the distance in degrees of latitude between these two cities ?

The preceding
differs

illustrations

show that

a positive

number

from a negative number

in direction or quality.

Thus, if +10 means 10 yards measured to the right; or 10 east longitude or 10 miles a boat goes up stream or 10 games won or 10 minutes a clock is fast or $10 in bank balance or 10 what would 10 mean in the correspounds lifted by a balloon
; ;

my

ponding cases

24.

Signs of Operation and Sighs of

Quality.

The numbers
but
differ

+25 and 25
When

are

the same in magnitude,

in

direction or quality.

the

a number is preceded by the sign +, it means that number is taken in the positive direction or sense, and when preceded by the sign that it is taken in the
,

negative direction.

be seen that we use the signs + and with When they are used to indicate the operations of addition or subtraction, they are called signs of operation. When they are used to indicate direction or quality, they are sometimes called signs of quality. The beginner might think that this ambiguity would lead to confusion, but he will find that such is not the case. When we read a quantitj'' hke 25, we should say " negative 25," but this is not followed in practice, as it is usually read " minus 25."
It will thus

two

different significations.

When

no sign precedes a number,

it

is

understood to be a

positive number. Absolute Value.

25.
its

The absolute value

of

a number

is

value without regard to sign.


4- S

Thus,

and

have the same absolute

valu''

POSITIVE

AND NEGATIVE NUMBERS


15 (1-15, Oral)

33

BXEBCISB
1.

What

$47,
2.

(6)

is the net property of a man who, (a) has $60 and owes has $40 and owes $50, (c) has $65 and owes $65 ?

What is the value of, (a) $40- $30, (6) $40 $60, (c) $30 $20, $20- $30, (e) $10- $0, (/) $0-$10? 3. The temperature was 10 at 6 p.m. and 4 at 10 p.m. How many degrees did it rise in the interval ?
(d)

4.
is

A Uquid

whose temperature
?

is

20

is

cooled through 30.

What

the final temperature

6. A vessel sailed on a meridian from latitude 15 How many degrees did it sail and in what direction ?

to latitude 5.

What is the distance between a place 90 miles due east of 6. Toronto and another 60 miles due west ?
7.

am overdrawn
balance $100

at the
?

bank

$20.

What must

deposit to

make my
If

20-|-x=100,
What would

what

is

a negative number mean in stating the height of a tree above the window of a house ? The height above sea level of the bottom of a well ?
8. 9.

A man
and

biiys a horse for

$100 and

sells

him

for $80.

What

is

his gain

his

gain %

10.

from A.
11.
If

A man travels 20 miles from A How far are they apart ?


What
What
is

and

his friend travels

10 mile*
?

the rise in temperature from

30

to

10

30-t-a;= 10,
b miles

what

is

12.

and

is the distance between two places which are a Miles west of Montreal, (1) if a is greater than b, (2) if b is
?

greater than a
13.

Denoting a date a.d. by + and of years between these pairs of dates


:

B.C.

by

state the

number

(1) (4)

+1815 to +1915. -120 to +60.


Augustus was

(2)
(5)

-20

to to

+75.
+200.

(3)
(6)

-65

to

-37.
to

-200

+1900

+1800

14.

years was he

Emperor

Roman Emperor from 31 to +14. How many What is the difference betweei 14 and 31 ? 1

34
16,

ALGEBRA
The
last
?

First Punic

War

lasted

did

it

What

is

the difference between

from -264 to 241. How long 241 and 264 ?

16. A boy adds 15 marbles to his supply, gives away 10, buys 5 and gives away 12. How many has he thus added to his supply ? 17.
I

have Sn

in the

bank.
is

If I issue
?

balance

when the cheque


?

paid

If

a cheque for S6, what is my a=40 and 6=50, how do you

mterpret the result


18.

How
19.

has S50 and B has $20. A owes B $10 and much will each have when his debts are paid ?

owes

A
If

$40.

The weights

of

two pieces

of iron are 65 lb.

and 147

lb.

they

are attached to a balloon with an

upward
?

pull of 239 lb.,

how would

you represent the combined weight


20.

Represent graphically the following changes

in

the price of

a stock

POSITIVE
6.

AND NEGATIVE NUMBERS


A
?

35

In a 100 yards handicap race yards start. What do these mean

has 3 yards start and B has How far has each to run ?

7.

many
8.

In solving a problem in which it is required to find in how years A will be twice as old as B, I get the answer 10. What

does this answer

mean

Find the average noon temperature for a week in which the noon temperatures were 20, 10, 15, 0, 4, -6, 15.
:

9.

train
it

three did
10.

was due at 10 minutes to 3. How many minutes arrive if it was half an hour late ?

before

11
11.

miles.

A man travels How far


and
?.

8 miles, then 6 miles, then 4 miles, then has he travelled ? How far is he from the

starting point

in

what

direction (positive or negative)

from

it ?

Egjrpt was a

years was this


12.

Roman province from 30 to 616. How many What is the difference between 616 and 30 ?
:

The
2,

daily average temperature for 14 days were


0,
5,
1,

6, 5,

0,

-4,

-6, -2,

-1,

-6, -3,

3.

Show

these

variations
13.
If

by means
?

of a graph.
will represent

a gain of a dollar be the positive unit, what


of a patient's

a loss of $3-50
14.
at 12

The record
noon was:

temperature for each hour beginning


102,

100,

100-5,

101,

104,

101-5,

99-5,

98,

97-5, 97.

Represent these changes graphically, taking two spaces on the vertical line to represent one degree.
15.
If the

in the preceding question using positive

normal temperature of the body is 98-5, write the record and negative signs.
for the first 15

16.

The minimum temperatures

were:

26, 22, 14, 25, 21, 18-5, 13, 7-5, 11, 6,

days of December -4. -6, -1,

10. 12-5.

Make

a chart to show these variations.

d2

CHAPTER IV
ADDITION AND SUBTRACTION
26.

Addition of Positive Quantities.


:

What

is

the result ol

combining
(1)
(2)

A A
A

gain of $20 with another gain of $10

measurement
?

of 5 feet to the right with another of


rise of 8

3 feet to the right


(3)
(4)

temperature of 10 with a 6 points won with 4 points won ?


rise in

As explained

in

to be positive quantities,

Chapter III., we will consider all of these and we might show this by attaching
four

the positive sign to each. We might write these


addition, thus
:

questions

as

problems

in

+ $20 + $10 + $30


Similarly, the

+5 +3 +8
of 6a

feet

+10

feet
feet
is

+8
+18
10a,

+ +

6 points

4 points

+10

points

sum

and 4a

and the sum

of 2x*, 5x*

and

6xi3 13x'.

Here we have not prefixed any


positive sign
is

sign,

and when that

is

the case the

understood.

We
is

see then that the always positive.

sum

of

any number of positive

quantities

27. Addition of Negative Quantities. We might change the data of the four questions in the preceding article so that all the quantities would be negative.

ADDITION AND SUBTRACTION


Thus, the
result of

ST
is

"What first might be changed to " combining a loss of $20 with a loss of $10 ?
.

the

Read the other


As problems

three questions

making

similar changes.
?

What would now be


-$20

the answer to each question

in addition

they would
-10

now appear

thus

$10 $30
Similarly, the

-5 -3

feet

feet

5x

6 points

4 points

-8

feet

18
is

- 10

points
of

2o^,
and
is

sum of and a^ and 6a' is 14a'.

7x

12x,

and the sum

5a*,

Thus,

the sum of any number of negative quantities is negative, found by adding their absolute values and prefixing the

negative sign to the result.

EXERCISE
1.

17 (Oral)
:

State the results of the following additions

38

ALGEBRA

We
in the

proceed in a similar same column.

way

in algebra, writing like

terms

In arithmetic.
2 yd.
3 yd.
1 ft. 1 ft.

In
in.

algebra.

2a-\-

h+

6c 4c

4
10

in.

3a+ 6+

5 yd. 2
If

ft.

in.

5a+26 + 10c.
same
order, the}'

the like terms are not in the

must be

properly arranged for addition.

Ex. Add 5x+3^ 22,

4;/ 52

+ 3x,
:

32+4x4-?/.

Here the problem might be written thus


5x-l-3i/3a;

2z

4x-l-

+ 4!/y-

5z Zz

Sum=12x+8i/-102.

EXERCISE
Add:

18 (1-6, Oral)

AUDITION AND SUBTRACTION


30.

39

Addition of Quantities with Unlike Signs.


is

What

the result of combining

(1) (2)

(3)
(4)

A A A A

gain of $20 with a loss of $10 gain of $5 with a loss of 815 ?


loss of loss of

$8 with a gain of S6 ? $7 with a gain of $12

These might be written as problems

in addition,

thus
7

+ $20
-$10

-r% b

$8
+$6
-$2

$
-8

-$15 -$10

+S12
5

+ $10
It is

thus seen, that when we add two quantities differing in sign, the sum is sometimes positive and sometimes negative.

When

is it
is

positive

and when
is it

is it

negative

How

the numerical part of the


?

sum found when

the

signs are alike

found when the signs are different ? The answers to these questions might be combined into
:

How

the following rule

When
addition,

the signs are alike, the

and

the

common
is

different, the

sum

found by arithmetical when the signs are ; found by arithmetical subtraction, and the
is

sum

sign

is

affixed

sign of the greater

is affixed.

Ex.

1.

Find the sum of 6 and


result
is

8.
between
8

Here the

2,

since the difference


is

and

is

and

the one with the greater absolute value

negative.

If there is doubt in any case, it is advisable to make the problem concrete by substituting for +6, a gain of $6 and for 8, a loss of $8, when the result will at once be evident.

Ex. 2. Find the sum of 5a, 8a, 7a, 6a, 2a. The sum of the positive quantities is llo. The sum of the negative quantities is 17a. The sima of 1 la and 1 7a is 6a,
.'.

the required

sum

is

6a.

which they come. Thus, the sum of 5a and 8a is 3a, of 3a and la of lOo and 6a ia 4a, of 4a and 2a ia - 6o.
also be eidded in the order in

They might

ia

10a

40

ALGEBRA
Ex. 3.Add

3a- 116+ 5c, 66-5a, 56-c+a.


in

Write the expressions

columns as already explained.

a==b=c= 1

5a+

3a-116 + 5c
66
c

a+ 56
The sum
zero.
is

-a +4c a + 4c or 4c o,

= -3 = +1 = +5 = +3
the

sum

of the second

column being

We may check the result by substituting


for the letters.

particular

numbers

the
of

first

Thus, if we substitute unity for each letter quantity becomes 311+5 or 3, the second is +1,
is

the third

3,
Add:
1.

+1

+5, and the sura ( a+4c) is +3. Since the sum and +5 is +3, we assume that the work is correct.
EXE3RCISB
19 (1-12. Oral)

+6 ft. -3 ft.
-3
7

-$10

+ $27

9.

3a-26
5a + 36

10.

ADDITION AND SUBTRACTION


24.
If

4t

the

sum

of

13x 7, 2x+5 and 6 4a;


3a; 6 and 5a; 6
find
is

is

48, find the valne of

X and verify.
25. If the sum of i 6, 12 z, 12 3a; and 12 6a;, 31.

the same as the

sum

of

x and verify.
If
is

Indicated

Additions.

we wish
to be

to

use the sign of


a,

addition to indicate that b

added to

we

write

it

thus
to

a-4-6.
if

Similarly,
11,

we wish
it

to indicate the

sum when

is

added

we

write

ll-!-( 7),

the negative quantity being

enclosed in brackets.

To
which

find the value of


is

done

ll+( 7), we must add 11 and 7, by subtracting their absolute values and
ll+(-7)=ll
(
-

prefixing the positive sign.


Thus,
Similarly,
-7 = 4. 3a) = 6a 3o=3a, 5w+( 3m) + m^ = 67w ?m w=m, o+{ fe)=o 6.

6a + (

and

We
If

to subtract

to add a negative ouaniity is the same <m a positive quantity of the same absolute value. we wish to simplify a quantity hke

thus see that,

(3o-26)+(2a-36),
or

we may write 2a 36 under 3a 26, and add in the usual way, we may remove the brackets and say that tht quantity

==3a 26+2a 36,

=5a 56, when

the like terms are collected.

EXERCISE
Simplify
1. 4. 7.
:

20

(1-12.

Oral)

-3+4.
(-2)+(-3).

2.
5. 8.

10+(-6).

3.
6. 9.

3+(-4).
76+(-4fc).

5a+(-4o).

-8a+( + 7a).
10.
12.

-5ab+(-2ab).
11.

9x^+{-Zx'

-p-\-(-3p).

(-3m)+(-8m).

-o+(-2a)+(-3c).
10xy-\-{-^xy)+(-Axy)-xy+(-5xy).

13.*

4i
14.

AIMEBRA
-6+(-26)+(-36)+(-46) + 106.
(2m+3) + (5m-ra)+(3m-5).
(6x+3y-42) + (x+2y-2)+(y+z-7x).

16.
17.
18. 19.

a+{-6) + 6 + (-c)+c+(-a).

z+(a-26 + c) + {6-2c + a)4-(c 2a-}r&),


is

20.

When 20
is

that this

true

by adding the
addition,

subtracted from 10, the difference is 30. Show difference to the quantity which was

subtracted.
21.

Show by

that
is

when 2ah-\-5c

is

subtracted from

3a 464-3c
22. 23. 32.

the remainder

a 36 2c.
(Verify.)

Solve (2x+3) Solve (8x-7)

+ (3z-5)f(5a;-l)=57. (Verify.) + (-4a;-3) = (-5x-7)+(7a;-2).


is

Subtraction
is

the Inverse of

Addition.

from 7
4 will

equivalent to finding the


7.

To subtract 4 number which added to

make

in subtraction may be changed into a corresponding problem in addition. If we wish to subtract 4 from 7, we enquire what number added to 4 will make 7. We might make the problem concrete by finding what must be added to a loss of $4 to result in a gain of $7, and the answer is evidently a gain of

Thus every problem

$11.
.*.

when
7

4 is subtracted

from 7 the remainder

is 11.

Thus,

less 4=11, because -10 less -3 = -7, because


66 less

4+ 11 = 7.
-3 + {-7)=
-10.

46=106, because 46+106 = 66.

BXBRCISB
What must
1.

21 (Oral)

be added to

2.

A A

gain of SIO to give a gain of $15


gain of $8 to give a gain of $3
?
7

3.

gain of %5 to give a loss of $4

ADDITION AND SUBTRACTION


4.
5. 6.

43

A A
A

loss of
loss of loss of

$6 to give a gain of $3

20 to give a

loss of

$15
?

$5 to give a loss of $8

7.

44

ALGEBRA
EXERCISE
Re-write the
following

22
in

problems in subtraction as problems

addition and find the result


1.

ADDITION AND SUBTRACTION


17.

46

Subtract the

sum

of

3x^ 5a;+6 and

5x*-|-4a; 3 from 6a;^ x+3-

Check when x\.


18.

What must

be added to

2m+3n 4p

to give

5m--n2p
3a* lla + 4

(Verify.)

19. 20.

By how much is 7a* 15a 11 By three subtractions simplify

greater than

(6a+106-c)-(3a+46-2c)-(a-36-[-4c)-(2a+76-3c).
Subtract the sum of 2p5q3r, p-'r3q2r and 4^ + 6^ 4r from the sum of 3p4:q-'r5r, 3q4:r-\-5p and 3r4ip-{-5q.
21. 22.
23.

Subtract

2a 36+ 5c from
a-2b
sum
?

zero.
?

What
?

is

the excess of 15 over 10

8 over

? ?

over

11
24.

a+b
Add

over

3a;2-5x+2 over 2^2 llx+7

a*-|-2a 5 to the excess of 2a^ 4a+3 over


of

a^ 3a+10.
from

25.
zero.

Subtract the

o 36+c, 6 3c+a,

and

c 3o+6

26. What must be added to the sum of x^5x, 6a; 3x* so that the result wiU be unity ?
27.

2x^ 3a:+4 and

From 2z-3x^+o-x^
Indicated

take 3

Ux^-5x^6x.
we wish we write it:
to

34.

Subtractions.

If

indicate

that

is

to be subtracted from 9
is

9 (3). From

the preceding this


Also,

at once seen to be equal to 9


(

+ (-)-3) = 12.

-8-(-5)= -8 + + 5)= -3. 3o 5a)= 3a + of = 2a. o-(-6)= a + + 6) = a + 6.


( (

We

thus see that

to subtract

as to add a positive quantify of the


Similarly, (3o

a negative quantity is same absolute value.

the

same

+ 4fe)-{2a-36) = (3a + 46) + (-2o + 36), = 3a + 46 2a + 36,


=a+76.
a(b+c)a-\-{ b c) abc,

Also

and

a (6 c) = a + 6 + c)=a 64-c.
(

Thus, brackets which are preceded by a minus sign may be removed if the signs of all the quantities within the brackets be
changed.

an

ALGEBRA

Ex. Simplify 5x 3?/+42-(3.r 2?/4-2z).


consider this as an ordinary problem in subtraction and proceed in the usual way.

We may

5x-3y + 4z 3x2y + 2z

2x-

y + 2z

We

may, however, remove the brackets using the


:

rule

and then

collect the like terms, thus

The expression

=5x Zy + 4z 3x + 2y 2z, = 2xy + 2z.

EXERCISE
Simplify
1.
:

24 (1-10. Oral)

10-(-3).

2. 5.

-5-(-6).
(-2m)-{-3m).
8.

3.
6.

-7a-(-4a).

4.
7.

8z-(-3x).

-{-b)+b.

8-(-4)-(-2)
??i-(-3m)-(-5m).

8ab-l0ab-{-lab).
-4:X^+(-3x^)-{-lx^).
.3a-116-(5a-86).

9.

10.
12.

11.* {5x-2y)-{2x4.y).
13. 14. 15.
16. 17.

2a-36+5c-(a-46+5c).
(a+6)-f(2a-36)-(4a-36).
a-rbc{b-\-ca) + {a^bc).

(6x^3x+5)+(2x^5x6){5x^8x+2).
Find the value of
5a-\-b,

when a = 2,

= 3.
l,

18.
19.

Find the value of 2a+3bc, when a =

b^2,

c= 3.

By two

different

methods

find

the value of a- (bc), whei

0=20, 6=10, c=7.


20.

Solve and verify


(1) (2) (3)

2a;-3-(x-4)=-8.

3x-l-(a:-3)-(z+7)=40. l-(4-x)-(5-x)-(6-x)=52.
make 5x 6 exceed 3x 11 by 70
?

21. 22.

What

value of x will

Find the value of 3x* 2*+5 (2x+x 1), when*=0,

1, 2, 3, 4.

ADDITION AND SUBTRACTION


23.

47

Remove
(1) (2)
(S^
(4)

the brackets and simplify

(a+36-llc)-(6 + 3c-8a)-(c+5a-26). (a-36)-(6-3c)+(c-3d)-(d-3a).

{Zxy+2z){2xZy+^) {'iy-%zbx).
-{a-h)-{b-c)-{c-d)-{d-a).

E3XERCISB
1,* Find the
2.
least.

25 (Review of Chapter TV)

sum

of 5a,

3a, la and 8a.


which x
is

Find the sum

of four consecutive integers of

the

3.

Find the sum

of five consecutive integers of

which n

is

the middle

one.
4.
5. 6. 7.

Add 3a-26 + 7c,


Subtract

56

3c 2o

and c-a-36.
5a.

106 from 66, 3a from

From

4a 36 -f 5c subtract 2a 56 c.

Subtract

5x 3t/ + 42 from 4:X2y z.


be added to 3a 56 + 6c to give 6a 76 + 4c
is
?

8.

What must
If
2t/ ?

9.

x-\-y\Q and x j/ = 4, what


the

the value of 2x

What
?

is

the value of
10. 11. 12.

What

is

sum

of the coefficients in

6a
?

116

+ c 3d
when

What must

be added to x j/ to give
i/

When x=3 and


7

= 4,

what

is

the remainder

x*

y*

is

subtracted from 2xy


13.

What
To
the

is

the difference between 2a 6 c and a 6 + c

Give

two answers.
14.

sum
4x*

of

3m 4n and 2m 3n add
subtract the

the

sum

of

m+7n
7x 5

and

3m -2n. 15. From


2x-8x+7.
16.
of

+ 3x 7
is

sum

of

2x* +

and

By how much

3x 7
7

greater than

2x+5

For what value

X would they be equal


17. 18.

Simplify a + (26 3c)-(c


If

+ a).

x = a+26 3c, i/ = 6 + 2c 3a and z = c + 26-3a, find the value 0f(l)x+(y + 2), (2)x-(j/-2). (3)x-(2/ + 2).

48

ALGEBRA

From the sum of bx-\--^y and Zx^y subtract the sum of 19. 2x\-\y and Ax-\-\-Zy. By how much is 3x* 5a;4-ll greater than 3a;* 8x+17 7 20. What is the meaning of the result when x = 2 7 when x = 1 7
21. 22.

From
If
c
7

%a^b-\-\c take

Ja J6 Jc.
6

a + 6 + c = 0when

a=3x 4;/ and

= 4)/ oz,

what

is

the

value of
23. 24.
25.

5x-3-(x-4)-(x-2) = 27. (Verify.) value of x will make 3x 2 exceed x 7 by Whena=l, 6 = 2, c = 3, the sum of x-f-a 36 + 4c,
Solve

What

63

2x + 6-3c+4o

and 3x + c a 6
26.

is

124.

Find the value


d.

of x.

Using

breickets,

indicate that the


If

diminished by the sum of c and d = I 6x, what is the result ?


27.

sum of a and b is to be a = 2x 3, 6 = 5 3x, c = 3x j,

Solve

2-(x-i)-(i-3x) = 16-25.

28. 29.

Subtract

2m 7n 4x

from

zero.

What must

be added to a

(1 6) (1 c)

to

produce unity

Subtract the sum 30. excess of 4a 6-j-c over a + 6-t-c.

of 2a -|- 36

4c

-j-

d and a 26 -c d from the

CHAPTER V
MULTIPLICATION AND DIVISION
35.

Multiplication of Simple Positive Quantities,

The

factafs

of

a product
Thus, Also

may

be taken in any order.

3x5x4 = 3x4x5 = 5x4x3 = etc. axbxc=axc X 6 = 6 xaxc =etc.


product

The

latter

may be written abc, acb, 3ax26=3xax2x6, = 3x2xax6,

etc

ai

= 6ab.
Make a diagram
Similarly,

to

show that ixx2y = 8xy. 3a6 x 5cd 3xaxbx5xcxd,

= 3x5xax6xcxa, = I5abcd.

flO

ALOBBRA
BXBRCISE
26 (Oral)

Find the product of


1.

MULTIPLICATION AND DIVISION


EXERCISE
State the product of
1.
6,
3,
:

61

27 (Oral)

-7. 5.
2?/.

2.
5.

-4,
3x,
2t7i,

3.

3.
6. 9.

-4, -5.

4.
7.

2.

X,

y.

X,

8.

3n.

x,

3x7/

10. 13.
16.

-o(-6).

11.
v).

(-6)2.

12.

-ayi-x).

2?ni;(
x2,

14. 17.

\x,\2x.

15.
18.

X.
2x2.

2a2,

3a.

2.r2x Sx" ofcx


3crf.

19. 22. 25.

oxy,
5x32/3,

20. 23.

a^ a.
Za^b^c^,

21.
24.

3a%Xah.
x'^yz^, 5x^yh.

X2/2.

abH.

Since

fou expect

4x3 = 12, 4x2= 8, 4x1 = 4, what would 4x0 to be equal to Also 4x 1 and 4x 2
? ?

39.

Multiplication of several Simple Factors.

Ex. 1. Multiply 2a, -36, 4ab, b. The product of 2a and 36 is 6ab. 6ab and 4ab is 24a*6*. 24a6 and -& is -24a26S
.*.

the required product

is

24o'6^.

Of course the factors may be multiplied in any order we choose. we multiply all the negative factors first, what sign will their product have ? What sign will the product have if we multiply four negative factors ?
If

Five negative factors

Twenty negative

factors

factors

The product will be negative vvhen the number of negative is odd and will be positive when the number of negative factors is even. Any number of positive factors will evidently
not affect the sign of the product.

Ex. 2. Multiply 3x, 5xy, -6x2, _y^ QyS


(1) The sign of the product is since there is an odd number of negative factors. (2) The numerical coetficient = 3 x 5 x 6 x 6 = 540. (3) The literal part of the product = x. xy x* .y y^ x*y^, the complete product 540xH/*'
,
.

,",

E 2

52

ALGEBRA
Ex. 3. Find the values of (-2)3, {-2)^ (-2)^
In
(

2)*

the

number

of factors
/.

is

odd,

(_2)3=-2=-8.
16

Similarly,

(-2)*=

and (-2)6= -32.

EXERCISE

28 (1-18 Oral)

MULTIPLICATION AND DIVISION

53

Compound Multiplication. We multiply a compound 40. quantity by a simple one in a manner similar to the method
in arithmetic.

In
3yd. 2
6yd.
If

arithmetic.

In
2

algebra.

1ft. 4in.

23 = 2.10+3
2 2

3a+46-

5c

2ft. Sin.

46 = 4. 10 + 6

60+86 10c
and
be
y-\-z,

we wish
x{y-\-z),

to indicate the product of x

we

write

it

in the

form

which we see is equal to The diagram shows how this product

xy-\-xz.

may

illus-

y
^

trated geometrically.

Make a

similar
a(6

diagram to show that

Similarly,

+ c + d)=o6+oc+od. x{y z)==xy~xz.


diagram
is

Can you

see that the


7

a geometrical

'^

illustration of this

Ex.^implify 3(a-6)-4(6 c)-2(a-6+c).


The expression

= (3a- 36) -(46 -4c) -{2a -26 + 2c), = 3o-36 46 + 4c 2a+26 2c, =o-66 + 2c.

54
16.
17.

ALOEBRA
4(o-26+c)-3(6-2c+a)-2(5c-4a-56)
i(2a-36) + i(2a+56) + i(5a-f&).
x(a:-l)4-2x(x 3)+3x(x4-5).

18. 19.

a(a2-a+l)+3(o2+o-2)-2(a2+2a-3).
3x(a;2-2x+2)-2a;(3x2+4a;-5)+x(4a;2+5x-6).

20.

21.

2a(6 c+d)-3a(c-d+6) a(d-6-c).


:

Solve and verify


22.
24.
26.

3(x-l) = 2(x+4).

23.

5(x-2)-2(x+2)=70.
2(5x-9)4-4(x-ll) = 36.

6(2x-3)-3(x-3)=0.

25.

3(x+2)+5(x-3)=2(x-4)+4(x-l) + 13.
Find the sum of x(x+l), 3x(x 2), 2x(x 5).
Subtract a(2a2 a+1) from 2a(a2+3a-2).
If

27. 28.

29.
of

a stands for x^^xy-^-y"^ and 6 for x^xy-\-y'^, find the values

ab,
41.

2a-\-b,

3a 26.
Compound
Quantity.

Multiplication by a

The measures of the sides of the large rectangle are a-\-h and x-\-y. The measure of the area is the X y product of a-\-h and x-\-y, which is seen to ax
be ax-\-ay-\-hx^hy,
:.

[a-[-h){x-\-y)=ax-{-ay-\'hx-^hy.

This diagram shows how to find the product of x + 3 and x + 2. What does it show the product to be ?

Make a
of a-f-6

similar figvire which will


a-\-b,

show the product

and

and thus
6.

find the value of (a

+ 6)*,

or the square of a +

The method
is

of obtaining the product without the

diagram

similar to that used in arithmetic.

MULTIPLICATION AND DIVISION

50

Thus, (he product of any two expressions is obtained by multiplying each term of the multiplicand by each term of the multiplier. The proper signs are attached to these partial
products,

and

the

sum

of the partial products is then taken.


in

In multiplying in arithmetic we begin at the right, but algebra it is usual, but not necessary, to begin at the left.

Ex.Multiply
Check
=

(1)
(2)

2a-3b by 3a-26. 3x-5y by 4x+y.


Check
36 26
(2)

(1)

6=1
1 1

2o 3o

x=y = \ -2
5

3x
4a;

5y
1/

6a*1

9ab 4a6 + 66

-10

I2x^-20xy + 3xy-5y12x*
l'Jxy

6o*-13a6 + 66
42.

5y*

Results. In Chapter II. we saw how U which we obtained in solving an equation. We might verify our work in subtraction by addition. As in addition, the work in multiplication is easily checked by substituting particular numbers for the letters involved.

Checking

verify the root

Thus to check the work in the first example we might substitute 1 for each letter involved.

in

the preceding article,

Whena = 6=l,
and

2a-36 = 2-3= -1, 3a-26 = 3-2 = l,


6a*-13a6 + 66* = 6-13 + 6= -1.

Since the product of 1 and 1 is the work is likely correct. 1 A convenient way of exhibiting the test is shown. Of course any numbers might be used in checking, but we naturally choose the
,

simplest ones.

EXERCISE
x+3 2+4
2x+7

30

Find the product of the following and check


1.
2. 3.

x+5
2x+2

4.

3a;+4

x+1

2x+3

MULTIPLICATION AND DIVISION


34.

67

Find the sum of the squares of x+1, ar+2, z-f3.

Check by

putting x=2.

Solve and verify


36.

{x-\-5)(x-l)={x-5)ix+2).
(2a;-l)(3x-l) = (x-2)(6x+4).

36.

{x-l)^={x-6){x-\-2).

37. 38.

(a:+ll)(a;-2)=(a;-7)(x-l)+107.
a;(a;+l)+(a:+l)(x+2)=2(a;+l)(x+3).

39. 40.

(x+l)2+(x+2)2+(ar+3)2=3(x-2)2+14.
Division by a Simple Positive Quantity.

43.

To

divide 24
division

by
is

is

the same as to find the


24.

number by

which 6 must be multiplied to produce

Thus
is

the inverse of multiplication as subtraction

the inverse of addition.

Since
.'.

axb=ab,

ab^a=h and

ai)^b=a.

If we wish to divide 6xy by 2x, we must find what 2x must be multiplied by to produce 6xy.
(1) 2 (2)

must be multiplied by 3

to produce 6,

y
:.

xy.

6xy-^2x=3y.

Similarly,

I5abc-^3bc = 5a.

problem

in division

may

be written in the fractional

form, the dividend being the numerator of the fraction and

the divisor the denominator.


Thus. 24
6
.

--='*.

o6

= a

6x1/

f>,

-^ = 32/. 2x

As in arithmetic, we may remove or cancel from the dividend and divisor any factor which is common to both
Thus, -^r- = 4a, on removing the factors 3 and
6.

gioul^y,

l-^.= 3ay and

^=1^.

68

ALGEBRA
44. Index Law tor Division. Since a^xa^=a'' by the index law for multiplication,
.'.

a'''^a-=a^ or a^^ a^^a-,


.
.

a^

\ .k .a .a .a

a^

h .k .k .a .a

Thus,
is

the index of the quotient of

powers of

the

found by

subtracting the index of the divisor

same quantity from the index

of the dividend. Thus, a;*-:-x' = x*~* = x*


Similarly,

a''-^a'

= a''~*=a^.

^^^ = 5a^-*b^-^ = 5ab*.


multiplication.
correct,

The work in division may be verified by Thus the preceding division is seen to be 5ab^x3a-b = 15a^b^.

since

EXERCISE
Copy and supply the
multiplication
^
:

31

quotients, verifN-ing

the results by mental

3x1/

2.

5abc

24mn
3n
12a*

25xyz
5z

ab
,

4a2
.

O,

b,

42x3
.

7.

o.

BSm^ra
.

2a
9.
??:!'''^

Ix
10.
'

2a2
11.
'

Vimn
12.
'

i!^'.
6p^
14. 17.

ahc
13.
6xz/-i-2x.

i"-. ha
15.

l^^V
.

\xy

lOa'-f-Sa.

^miJ^^^r.
ISx'j/V^Sx^t/z*.

16.
19.

10x5^-2x3.

16a36^4a6.

18.

22a%^^Ua-h-.
Rule of Signs for Division.

45.

(+a)x(^i) = +aA, ^aJb, {a)x{b)=-^ab,


Since

(-a)x(+6) = oA,
it
,

(-t-a)x(-6)

follows that
,

ab +aZ> - =+0, 06 =+0, -r^=Oy a -\-a +a


,

;r"=~
-{-ab

and when the sign of the quotient What then is the rule of signs for division ? Compare it with the rule of signs for multipUcation

When

is

is

it

(art. 38).

MULTIPLICATION AND DIVISION


Ex.

S&

Divide
(1)

XOx'^y^ by

2xy'^.
numerical coefficient hteral part ? complete quotient 7
?

(2) (3) (4)

What What What What

is is is is

the the the the

sign of the quotient ?

EXERCISB
Perform the indicated divisions
1.
:

32 (Oral)

12+-3.

2.
5.

-12+-4.
0^

4.
7.

7+7.
0^5.
ah-.

10.

13.

lOa*-^


a.

S.

11.
2a2.

axy-.

16.
19.

\2m^n-.

i4.

6mw.

17. 20.

x^y^z-.

-6a3+3a.
2a-.
5.

3.
6.

a.

-10+2. 12 + 2^
6a2^

9.

x.

12.
15. 18.

45^


2a.
5-i

3.

3.

27a;*^

^x\

xyz.

40^ + 2a'.
^'^^^

~y V
pq

?^*!^
4imn^

21

2x
(5)

22.

FUl in the blanks in the following


(1)

Dividend:
Divisor
:

6a^
2a-

lOa;^

(2)

(3)

(4)

lOaftc 5c
2ac

35m^n

5m

Quotient
46.

2x

6a

Division of a

Compound Quantity by a Simple One.


2

we divide 6 ft. 4 in. by by 6 we get 2 lb. 1 oz.


If
Similarly,

we

get 3

ft.

in.,

or 12

lb.

6 oz.

3)9 3

ft. ft.

in.

2 in.

to

ALGEBRA
BXBRCISE
88
(1-16,
:

Oral)

Divide the
1. 4.
7.

first

quantity by the second


2. 5.

9a 2+ 6a,

3.

Qx^-\-<ix^~2x,

x.

3.

ISz^-lOx,

Sr.

16m' 4m, 4m.


ax-\-ay, a.

x^y+xy^,

xy.

6. 9.

12a2 4a6,

2o.

8.

a*-\-a^a,a.
11.

-6x2 4xy,

2x.

10.

6aA 6o,

3a.

60'- Sa^-f 4a, 2a.

12.

a^h^a^h'^, ab^.

13.
15.

5a* 10a', 5a\


3y^2y^,
\y.

14.

4a: -(-10x2 ex', _2a;.


:

Simplify
.

ID.

3i-|-6 3

lOx 15
\

._ 17.

oA-f-oc

hc-\-ah

ac+bc

-|

a
^^ '

18* g'+3a
o
20.
a:-f-xy
^

Sa'+6a
3a

(x+2)(a;-2)+(a;-2)(z-4)
*

2
^^^ *

y'-xy
y
,

(a+2)(a-h3)-(a-3)(o-2)
'

X
22

2a

abac

be ah

a^a^_^a^a.
a

-b
Subtract 24. remainder by x.
26.

(x+3)(x 8)

from

(2x-4)(x4-6)

and

divide

the

Solve and verify

^^-10^
X

^ 3x2+15x _^ lOx-lS^^^^
3a;

EXERCISE
1.

84 (Review of Chapter V)

State the rule of signs for multiplication and for division.

2.* If a = 3 and

6= 4,

find the values of

a, 6, ab, a*

+ b*,

a*-b*,

a, 6,

a-6.

3.

What

are the values of (-1),

(-lj%

(-1)^", (-2)*,

(-3)t

4.
5.

Simplify 3ax

-46x 2o6^ Safes.

Simplify 2a(a + 3)-f-3a(2a-5).


is (a-\-b)

What is the area in square feet of a rectangle which 6. long and (a 6) yards wide 7
7.

feet

Make a diagram

to

show that 3xx4a;=12z'.

MULTIPLICATION AND DIVISION


8.

61

A merchant

bought a pieces of
If

pieces at 80 cents a yard.


total cost in dollars.

silk at 60 cents a yard and b each piece contained 50 yards, find the

9.

To

the product of 3x 2 and 2x 3 add the product of

3a;

+2

and

2a;

+ 3.
From
the product of ox Zy and

10.
of 3x

2x+y

subtract the product

2y

and 2x 3t/.
to

11.
a*

Make a diagram

show that the product

of a

+3

and

a+1

is

+ 4o+3.
12. 13. 14.

Divide 4o' 6o* 8a by

2a and

verify.

To
is

the square of 2?n 3/. idd the square of


is

3m 2n.
5x6y
the

Prove that when I5x* 8xyl2y*


3x + 2y.

divided by

quotient
15.

stituting 3 for a

Find the product of a b, a + b and and 2 for b.


Simplify ^ '
^.
,.,

a*-\-b*.

Check by sub-

^^ 16.
17.

4x3-8x+12x
4x

15x+10a;-15x
J 5x 3).

Solve (2x + 3)(3x+2) = (6x-l)(x +

(Verify.)

18.
19.

Simplify (2a-36)(a + 6) + (a-6)(3a + 6).

value of x will make (x+3)(x + 9) equal to Could (x+3)(x+9) be equal to (x+4)(x+8) ?

What

(x

+ 5)(x+6)

20.
21.

Find the sum

of (a-l),

(o-2)* and (a-3).

3x 47/
22.

Subtract the product of and 4x+3i/.


Simplify

2x~3y and

3x + 2y from the product

4a'+2a
^^

+(3-|-2a)(l-a).
1,

23. 24. 25.

Find the value of 2x*+3x


Find the product of
If

when

x= 3; when x=

4.

x=o* 3a+l

x 2, x+2 and x* + 4. and y = 2a* al, find the

values of 2x + 3y,

*x-2y,'-y.
26.
If

x=2a-f-6 and y = a-f 26, find

in

terms of a and

b the values of

ax by ~2
27.

'

4x 21/ x*y* 3"~' 3o~' If x=36-2c and y = 2b-3c,


If

find the value of

(2xy){3x-2y).

28.

x=2, 2/= 2, 2= -4,

find the value of

x*+y*+z*-3xyz.

CHAPTER VI
SIMPLE EQUATIONS
47.

{continued
is

from Chapter
the

11.)

Definition.

An
is

equation

statement

of

the

equality of two algebraic expressions.

Thus, 2.r+3=13
true.

consists in finding a value of x

an equation, and the solving of it which will make the statement


between the

The beginner should

clearly see the difference

value of X in an expression like

2x+3 and

the value of x in

an equation like 2a:;+3=13. In the expression 2a:+3, x may represent any number, and for different values of x the expression has different

But in the equation 2.r+3 = 13, x can not represent any number we please, but some .particular number, in this case 5, which when substituted for x will make 2x-\-3 have
values.

the value 13.


Identity. The statement 4(x 2)=4a; 8 is an equation 48. according to the definition we have given. If the first side of this equation be simplified by multi-

plication,

we obtain 4a; 8, which


It
is

is

identically the

same
is

as

the second side.

at once seen that this equation

true

for all values of x.

An

equation which

is

true for

all

values of the letters

involved is called an identical equation or briefly an identity, while an equation which is true only for certain values of the The usual letters involved is called a conditional equation. method, however, is to call all conditional equations .simply " equations," and all identical equc^^ons, " identities."

SIMPLE EQUATIONS
Thus,

93

and

2 = 3x+10, (x+3)(a; 3)=x* 9,


5a;

is is

an equation, an identity.

We cannot always see mentally whether a given statement is an equation or an identity. Thus, (x+2)(x + 3) = (a;-l)(x-3) + 3(3x+l) might appear to be an equation, but if we simplify each side, we find that each becomes x*-\-5z-\-6; and this statement is therefore an identity.
BXBRCISEI 85
Which
Identities
1.

of the following statements


?

are

equations and which are

8(x+3)=4x+4(a;+6),.

2. 3.

3x{x+l)=x{x+l) + 2x{x+5)+lQ.
(a:-3)2-5=x(x-6)+4.
(2x-4)(a;-5)+(x-2)(x-3) = (3x-2)(a:-7)+40.
(x-\-a){x^-\-a^)=x^-\-ax{x-^a)-\'a^,

4.
5.

6.

(a;+2)(a;-3)=a:(z+5)+3(a;-l).
of

49.

Transposition

Terms.

In Chapter

II.

the method

of solving easy equations of the four

was dealt with.

entirely on the proper use axioms of art. 15. The following examples will show how the methods of Chapter II. may be abbreviated.

The method depended almost

Ex. 1.Solve Add 6 to each side,


Subtract 4x from each, Collect terms on each side, Divide each side oy 3,

7x-6=4x-f 12. 7x = 4a; + 1 2 + 6. 7x 4x=12 + 6.


3x= 18.
x=6.

Here we added 6
the

to each side with the object of causing

to disappear

from the

first

side of the equation, so

that

we might

hs-ve only

unknown

quantities on that side.

+6 to appear on that side. We might say then, that the 6 was transposed from the first side and written on the other side with its sign
But the addition
of 6 to the second side caused

64

ALOBBRA
4a-

changed, and similarlj', that the

was transposed from the


:

second side to the

first,

with

its

sign changed.

We
Any

therefore have the following rule

quantity

may

be transposed

from one

side of

an equation

to the other if the sign

of the quantity he changed.


1

Using the

rule,

the solution of Ex.

might appear thus

7x-6 = 4x+12.
Transposing terms,
7a;

4x = 1 2 + 6,
3x=18, x=6.

.-. .-.

Verify this result.

Ex. 2. Solve
Removing
brackets,

2(3a:-5)
6z

+ 3(a;-5) = 7(a: 1).


1

+ 3a; 1 5 = 7a; 7.
10+ 15
.-. ..

Transposing terms,

6x + 3x 7x=

2x=18,
x = 9.

Verification,
first

when x=9

side

second side

=2x22+3x4 = 66, =56. =7x8


3(i/-2)-5(i/-3) = 17.
3?/

Ex. 3.Solve
Removing
brackets, Transposing terms,

6 5?/ +15=17. 5j/ = 6 15+17, 3^/ -23/ = 8,


.-.

y=2 =

-^'

Verification

first

side

= 3( 6) - 5( - 7 = -18 + 35=17.
performed.

Ex. 4.Solve (2a; l)2-(x-3)(x-2)=3(a:-2)2 4.


Here the indicated multiplications are
first

(2x-l)* = 4x*-4x+l,

(x-3)(x-2) = x*-5x+6,
(x-2)i' = x*-4x + 4, 4x-4x+l-(x*-5x+6) = 3(x*-4x+4)-4, 4x*-4x+l-x* + 5x-6 = 3x-12x+12-4, 4a;-x-3x-4x+5x+12x= 12-4- 1 + 6,
.',

/.

13x=13,

.-.

=1.

SIMPLE EQUATIONS

65

Here the product of x 3 and x 2 is first found and enclosed in brackets with the minus sign preceding. In the next line the brackets are removed and the signs changed. In 3(x 2)*, the x 2 must first be squared and the product multiplied
by
3.

The beginner should not attempt to perform these double operations together.
Note.

EXERCISE] 86
Solve and verify
1.

4x-4=2x+8.
3-3a;=9-5a;.
5(y-2)=3(2/+4).
ll(4x-5)=7(6a:-5).

2. 4. 6. 8.

3x-7=8-2x.
2(x-5)=:a;+20.
10(a:-3)=8(a;-2).

3. 5.
7.

7x-ll+4x-7=3x-8.
3(5a;-6)-9x=30.
5(a;-7)

9.

14+5x=9a:-ll+3.
7(x-3)=9(a;+I)-38.
72(x-5)=63(5-a:).

10. 12. 14.

11.

+ 63=9a;.

13.
15.

28(x+9)=27(46-a;).

7(4x-5)=8(3a;-5)+9.
(a;+7)(a;-3)=(x-l)(x+l).

16.
18.

4(x+2)=3-3(2x-5).
(x-8)(x+12)==(x+l)(x-6),

17. 19. 21.


23.

20(x-4)-12(a;-5)=x-6.

20. 22. 24.

5(2x-l)-3(4x-6)=7.
5(3;i+l)-7;i-3(;i-7)=6.

(2m-o)(4m-7)=8m2+52.
(x+5)2-(x+3)2=40.
4(2i/-7)-3(42/-8)-22/-7.

(x+5)2-(4-x)2=21x.

25.
26.

(x+4)(x-3)-(x+2)(x+l)=42.
(2x-7)(x+5)=(2x-9)(x-4)+229.

27.

28.
29.
30. 31.

(x+l)2+(x+2)2+(x+3)2=3(x+l)(x+4)-7.
2(x-l)2-3(x-2)(x+3)=32-(x-3)(x-4).

What
Prove
x.

value of x will
that

make lOx+11 equal

to

5x 9
is

3(x-2)+4(3x-5)=5(3x 6)+4

true

for

all

values of
*

32.
33.

What

value of a will
of x

make 5(a 3) exceed 3(a 7) by 28


wiU the sum
of

For what value be wern^

12+7x, 4a;+3 and 95*

66
3-4,

AWEBRa
If
a;

=2

is

a solution of the equation

(a;+l)(x+2)=(a;-4)(x-5) + 4,
find the value of
k.
is

35.

Prove that 10

a root of the equation

(x+3)(a:+4) + (x+5)(x+6)=422.
36.

When (3x+2)(4x 5)
is

is

subtracted

from (-2x+7)(6a;+3) the


exceed (y+4)(y 7)

remainder
37.

141.

Find

x.

What
?

value of y will

make make

(2/ 3)(?/+3)

by 40
38.

What

value of k will

(5

3A-)(7 2t)
?

equal to

(ll-6it)(3-it)
39.

What

is

peculiar about the equation

(x-5)2-(x-3)(x-7)=0?
40.
of

Under what condition


1

is

the square of

x+3

equal to the product

X
41.
is

and
If

x-|-6

3(2x 1)

is

greater than 12(x 3)


x.
is

b}'

the same

amount

that

5x

greater than 22, find


If

42. 43.

4ax 4a2=ax+2a^ what


lever in the diagram
is

the value of x

The
F

when PaQl.
6

balanced by the weights P and Q The point of support F is call d

the fulcrum.

If

P=10 lb., ^=15


?

lb.

and a = 12

in.,

1
p
of support
is

what
44.

is

the length of 6

Two

boys balance on a teeter 16 feet

in

length.

The heavier boy weighs 85 lb. and the point


Find the weight of the

6 feet from his end of the teeter.

other boy.
45.
if

How

far

weights of 8
?

lb.

from the larger weight must the fulcrum be placed, and 16 lb. balance at opposite ends of a lever 12 feet
to change Fahrenheit readings
If

long

46.
of a

The formula C=f,{F32)is used

thermometer to Centigrade readings.

F=n,

find the value of

0.

47

Change the following readings


0C., 40C., 100C.,

to Fahrenheit readings

-10C., -50C.
the two scales indicate equal

48.

What
?

is

the temperature

when

numbers

SIMPLE EQUATIONS
50.

67

Equations with Fractional Coefficients.

Ex. 1.Solve
Since
^ + ^ = 1,

lx-{-\x=2Q.
.-.

gx
a;

.-.

= 20, = 20H-| = 24.


we might
6.

Instead of adding the fractions,


ing each term of the equation

get rid of

them by multiply-

by

Then

^xx

B
.-.

+ ix X 6 = 20x 6,
3a;+2x=120, 5x=120,
a;

= 24.

Verify

by substituting

in the original equation.

Ex. 2.Solve

|(a;+l)+i(a;+2)-|(a;+14).
2, 3, 4),

Multiply each quantity by 12 (the L.C.M. of


.-.

^(x+l)xl2 + i(x + 2)xl2 = J(x+14)xl2,


6(x+l) + 4(x + 2) = 3(x+14).
verify.

Complete the solution and

Ex. 3.Solve
Multiply by 30,
'

x2
~5
.-.

x^_xl
6^ """To"
x 30

X 30

^
.-,

= %r^

30,

.-. .-.

6(a;-2)-5(a;-3) = 3(x-7),

6z-12-5x+15 = 3x-21, 6x-5x-3x= 12- 15-21, -2x=-24,


x=12.

Verification rfirstside

=1^ ^ = 2 1^ = ^.

second side

= ^^^

= J.

Note. omit the


line

not to attempt to with the brackets. He may, however, omit the preceding when he feels that he can safely do so.
line

In

this solution the beginner is advised

Steps in the Solution of an Equation. 51. equation the steps in the work are
:

In solving an

(1) Clear the equation of fractions by multiplying each term by the L.C.M. of the denominators of the frad,ions. (2) Remove any brackets which appear. F 2

68
(3)

ALOSBRA
Tran.spose
all the

unknown

quantities to one side

and

the

known
(4) (5) (6)

quantities to the other.

Simplify each side by collecting like terms. Divide each side by the coefficient of the unknown. Verify the result by substituting the root obtained in the

original equation.

BXBROISES
Solve and verify
1.
3.

87

p=x+5.
ix-Jx-lO.
ta;+|x=a;+5.

2.

^x=^x-\-2.

4.

^z + ix+ix=2Q.
I

5.

6.

^x

= 2x - ^ -^

7.

2/=i2/+i
X

8.

+ l + l^x-4.
^

2-5 = 4 +

3ot

1-

10.

7m ^-^ =

4.

11.

|-| = ^-15.
|

12.

ix+lx=li-x.
ix-|+7x=3a;+li.

13.

+ 2=H + ^-|-

14.

16.

|-j = 2.
7a;+2

16.

i(x-3)=20.

17.

-5- = -2-a;

4x-l

x+1
18-

^r-^ =
,

^-

a; 8

^=
5

^
^-

^^22.

a; 1

-4-

+ -5- = x+4 x+6

a;+3

21.

i(x-3)+i(x-5)=0.

J{x-6) = Kx+5) + i(a;-13).


X4-2

3x-l
23.

^r- + To = 7 + ^12 "4"^ 3

2z+l
5

3^5
,

'7
c\
1

26.

X 3 2x 4 3x + -^i 8" -i- =

-^

-,

26.

i(2/-3)-i(y-6)=l

o\

1/

27.

70

ALGEBRA
Ex.
2.

is

3 times as old as

2 years ago
their ages.

was 5

times as old as

was 4 years ago.

Find

Let X years represent B's age.

What will now represent A's What will represent ^'s age, What will represent jB's age,

age

2 yea>'s

ago

?
?

4 years ago

Now
Let

express that 3x 2

is

5 times x

4.
:

The complete

solution might appear thus

X years = .I's age,


3a;
.'.

,,

(3a; 2)

.'.

(x 4)
.

=4's =^'s =B's

age,
age, 2 years ago, age, 4 years ago,

.-.

.-.

3x-2 = 5(a;-4), 3x-2 = .5x-20,


18
a;

= 2x,
=9.
is

.*.

B's age

is

9 years

and A'a

27 years.

5%

Ex. 3. How do you represent 2i% of $(x+50) ? of a: ?


:

3%

of 130

4%

of $27

" Divide S620 into two parts so that Solve the problem 5% of the first part together with 6% of the other part will make $34."
Let
i, of
.'

$x = the

first

part,

$(620-x) = the other part,

iL

of S((>20-x)
.-.

Sx=5% = 6%

of the first part, of the other part,

T-T)a;+i^(620-x) = 34,

5x+6(620-x) = 3400.
Complete the solution and verify the
result.

Ex.

4.

What

is

defect of 30 from

.50

What is the the excess of 73 over 50 ? What is the excess of x over 50 ? ?


?

The

defect of x from 89
:

is

" The excess of a number over 50 Solve the problem Find the number." 11 greater than its defect from 89.

SIMPLE EQUATIONS
Let
then

71

x = the number,

X 50 = its
.-.

and

excess over 50, 89 x = its defect from 89,


verify.

x-50 = 89-x+ll.

Complete the solution and

Ex. 5. The value of 73 coins consisting of 10c. piecep and 5c. pieces is $5. How many are there of each ?
Let
.-.

x=the number

of 10c. pieces,

73-x=

5c.

The value of the 10c. pieoes= lOx cents. The value of the 5c. pieces = 5(7 3 x) cents, 10x+5(73-x) = 500.
.-.

Complete the solution and

verify.

The pupil should be careful to express each term of the equation in the same denomination. Why would it be incorrect to say that
10a;+5(73-a:) = 5
?

SXSRCISB
All results should be verified.
1.

88

A number
is

is

multiplied by 23 and 117

is

then added.

The
is

result
2.

232.

Find the number.


the double of a

From

number
is

is

taken.

The remainder

95.
3.
is

Find the number.


Three times a number Find the number.
subtracted from 235 and the result

217.
4.

Five times a number with 33 added


18 added.

is

equal to 7 times the

number with
5.

Find the number.


its

Find a number such that the sum of

third

and fourth parts


and they together

may

be 35.

6.

have $250.
7.

has $10 more than 3 times as How much has each ?


of two numbers Find the numbers.
is

much

as B,

The sum
4.

81.

The greater exceeds 6 times


its

the less by
8.

Find a number whose seventh part exceeds

eighth part

by

2.

72
9.
59.

/SL&EBEa
The
excess of a

number over 42

is

the

same

as its defect from

Find the number.

10.

Find 3 consecutive numbers whose sum

is

129.

Divide 114 into three parts so that the by 15 and the third exceeds the first by 21.
11.
12.

first

exceeds the second

than
13.

Divide S176 among A, and 8 more than C.

and C so that

naay have 816 less

A man

sold a lot for $2280


?

and gained 14%

of the cost.

What
first

did the lot cost


14.

Divide 420 into 3 parts so that the second


third
is

is

double the

and the
6.

the

sum

of the other two.


${x-\-5)
x.

A man

$(a;+25) each.
16.

buys 8 horses at $x each, 5 at The total cost is $2020. Find

each and 3 at

Find a number which exceeds 31 by the same amount that

J of the 17.

number exceeds

1.

Find a number which when multiplied by 6 exceeds 35 by as much as 35 exceeds the number.
18.
for

A farmer sells 7 cows and 17 pigs for $754. Each cow $70 more than each pig. What is the price of each cow ?
If less

sells

19.
is

30
20.

than the number.

10 be subtracted from a number, 40 more than J the remainder Find the number.
of J of the

less

Find two consecutive numbers such that the sum and ^ of the greater is 44.
Divide 46 into two parts so that
3,
if

21.

the greater part


is

is

divided

by

and the other by


J times.

the

sum

of the quotients

10.

22.

Divide 237 into two parts so that one part


1

may

be contained

in the other

23.

horse was sold for $116-25 at a loss of 7%.

What

did he

cost

24.
is 17.

The difference between the squares of two consecutive numbers Find the numbers.
in half-dollars

A box contains two equal sums of money, one 25. If the number of coins is and the other in quarters. money is in the box ?
26.

30,

how much
years will

is

35 years old

fi

is

7 years old.

In

how many

be twice as old as

SIMPLE EQUATIONS
27.

IZ
it

What
28.

is

My age in 20 j'ears will my age ? A is 35, 5 is 7 and C is 5


is

be double what

was 10 years ago.

years old.

How
1

long wUl

it

be before

4's age
29.

the

sum

of the ages of

and

a fourth of the
30.

Find three consecutive even numbers such that the sum of first, a half of the second and a fifth of the third is 17.
^'s share of $705
?

is *

of S's

and 5's

is

f of C"s.

What

is

the

share of each

The simple interest on a sum at 2% together with the interest 31. on a sum twice as large at 3J% is $135 per annum. What are the sums ?
32.
is

Three % of a certain sum together with 4% of a sum which $50 greater is $12'50. I'ind the sums.

33.
is

The value

of 52 coins

made up
of each
?

of quarters

and ten-cent pieces

$10.
34.

How many are there A square floor has a


The area
room.
of the

carpet.
of the

margin 2 feet wide all around a square margin is 160 sq. ft. Find the dimensions

35.

angle
angle.

is

In any triangle the sum of the angles is 180. The greatest 35 larger than the smallest angle and 10 larger than the other

Find the angles.

36. The length of a room exceeds the width by 4 feet. If each dimension be increased by 2 feet the area wUl be mcreased by 52 sq. ft. Find the length.
37.
If I

walk

miles at 4 miles per hour

and

m+2

mUes

at 3 miles

per hour, the whole journey wUl take 15 minutes longer than
at the uniform rate of 3| miles per hour.

if I

walked
have

Find the length

of the journey.

38.
$95.

A and B together have How much has each ?


5x-10=60.
?-l-?=x-10.

$65,

and

have $100,

C and A

39

State problems which wiU give rise to che following equations


(1) (2)

4a;-a;=24.

(3)

(4)

23-5a;=4a;-4.
of 5 for 3 cents

40.

fruit dealer

buys apples at the rate

and

sells

them

at the rate of 3 for 5 cents.

How many must

he seU to gain

$1-28?

74
41.

ALGEBRA
The sum
\ of

of

two numbers
as

is

147 and J of the

less is

9 greater

than
42.

the other.
\

Find the numbers.

John has

much n-oney

as his brother, but

spent 25 cents, John has only | as has each ?


53.

much

as his brother.

when each has How much

Algebraic

Statements of Arithmetical TJieorems.


13,

If

we

take any two numbers, say 23 and


their

and add together

sum and

their difference,

we

will find the result is twice

the larger number.

Thus,

and

23 + 13=36 and 23-13=10, 36 + 10=46, which is twice 23.


it is

We
would
sure

see that
find
it

true for the

numbers 23 and

13,

and we
are not

true for other pairs of numbers, but


all

we

it is

true for

pairs of numbers.

By

the use of algebraic s^'mbols and methods,


is

we may show

that the statement

true for every two numbers.

Let the larger number be a and the smaller 6. Their sum is a + 6 and their difference is a b. But (a + fe) + (a-6) = tv + 64-o-o = 2a, and 2a is twice the larger number.

Thus the statement (a+6) + (a &) = 2a represents


form the theorem stated at the beginning of this Besides stating it in a concise form it shows that it
brief
is

in

article.

true

generally.

EXERCISE
Show
1.

39
all

that the following statements are true for

numbers

two numbers by twice the smaller number.


of
2.

The sum

is

equal to their difference increased

The

difference

between the sum


is

of

two numbers and the

difference of the
3.

same two numbers

twice the smaller number.

difference
4.

Half of the sum of two numbers increased by half of their is equal to the larger number.
of

The sum

two numbers multiplied by one

of

them

is

equal

to the square of that one, plus their product.

SIMPLE EQUATIONS
5.

lb
to the square

The square The sum


If

of the

sum

of their difference

increased

of t.vo numbers is equal by four times their product.

6.

of three consecutive

numbers

is

equal to three times

the middle one.


7.

two

integers differ
is

between them
integers.

less

by 2, twice the square of the integer by 2 than the sum of the squares of the two

8. Read the statement {a-^b)^-\-(ab)^=2{a^+b^) without using symbols and prove that it is true.

BXBR0ISE3
1.

40 (Review of Chapter VI)


?

2. 3.

What What

is

an equation
is

An

identity

7 ?

rule

followed in transposing terms

Solve and verify: 6x(2x+3) = (3x+2)(4a;+


Is

3).

4.
5.
6. 7.

^x ^ 3

=
x-1 -5
H

8x - 6

an equation or an identity

What
Solve

value of x will
a;- 10
7

make 5(x 3) 4(a; 2)


a;-ll
h
2.
t>

equal to zero

= =

The sum
oi_

of

two numbers
a;
,

is 50.

If 5

times the less exceeds 3


?

times the greater by 10, what are the numbers


o
8.

Show XI ^ x that

+3 = -
6

5x + 6
o

1-

x2
-

is

true for

,,

all

values

of X.

What value of x will make the product of 9. equal to the product of 11 6x and 3 x ?
2x
10.
If
_

5 3x and

2x

3x

|-

-262, find

x correct to two decimal places.

A and B invested equal sums. A gained $200 and 11. gained $2600. If B then had 3 times as much as A, how much did each invest ?

12.

From a
full.

cask which

is

|ths

full,

then half
13.

How much will the cask Show that x=6 is a root of


A man

36 gallons are drawn and hold ?

it is

(x-l)(x-2)(x-3) = 2x(z-5)(2x-7).
has $115 in $2 bills and $5 Altogether, how many of each has he 7
14.
bills.

If

he has 35

bills

76
15.

ALGEBRA
If

x4

- =

31

and a

i,

find x.
If

In a ntairway there are 45 steps of equal heights. 16. been one inch higher, there would have been only 40 steps. ia each step ?
<-f

How

they had high

17.

o Solve
1

x5

x2 6-=^-

Divide 150 into two parte such that if the smaller be divided 18. by 23 and the other by 27 the sum of the quotients will be 6.
19.
is 51.

The

difference between the squares Find the numbers.

of

two consecutive numbers


five years

A father is 30 years older than his son 20. four times as old. Find the son's present age.
21.
If

ago he was

the

sum

of the fractions
7

2x4-3
;;

and

= 47
a;

is 9,

what

is

the

numerical value of each fraction

Show that the difference between the squares of any two 22. Show also consecutive numbers is equal to the sum of the numbers. that the sum of their squares is one more than twice their product.
23. 24.
of

Solve 2-(a;-4-|-3x-5)=10-x.
If

the product of

x+2

and 2x+5

is

greater than the product

2x+l and x + 3 by
25.

127, find x.

Solve |(2-3a;)-|(x-4) = i-(x-5).


three

Divide -75 into two parts so that 26. exceeds six times the less by -75.
27.

times

the

greater

Solve

X 3

_+-f -._=0.
24-x
1

2x

28

A man walked a certain distance at


?

by

train at 33 miles per hour.

3 miles per hour and returned His whole time was 4 hours. How far

did he walk
29.

" Take any Prove the accuracy of the following statement number, double it, add 12, halve the result, subtract the original number, and 6 will remain."
:

30. 31.

Solve|

^^-^^=x-8.
minutes
of
is it

How many
it

to 10 o'clock
8
'

if

three-quarters of an

hour ago
32.

was twice aa many minutes past


value

What
T

will

make 2(6x+o)-3(2x+a) = 4(lix-6)

n identity

SIMPLE EQUATIONS
33. 34. width.

77

Solve

(6x-2)(2x-l)-(4x 2)(3a;-2) = 4.
length 5 yards longer than its 5 yards longer and 3 yards Find the dimensions of the first.
is

A rectangular grass-plot has its A second plot, of equal area,


first.

narrower than the


35. 36.

Solve (a;+l)(x+2) + (x + 3)(x4-4) = 2x(x+12).

A man

among his wife, two sons and three daughters. A son as much as a daughter, and the wife $500 more than
together.

leaves his property amounting to S7500 to be divided is to have twice


all

the children

Find the share


,
,

of each.

37.
38.

x-2 4x+5 Solve +-g


by
37.
,

7x-8 = ^

g-

0.

Find an integer whose square

is less

than the square of the next

higher integer
_39.

If

2x+l -

3 exceeds 3x-2 by x-2 4 6


, -

find x.

-^

40.

How

far can I

at 10 miles per

walk at 3 miles per hour and return on a bicycle hour and be absent 6 hours 4 minutes ?
If

41.

A man

invested f of his

remainder at 6%. did he invest ?

money at 3%, J at 4%, ^ at 5% and the he receives an annual income of $516, how much

Prove that the product obtained by multiplying the sum of 42. any two numbers by their difference is equal to the difference of their
squares.

CHAPTER

\T1

SIMULTANEOUS EQUATIONS
54.

Equations with two Unknowns.


of

The sum
It
is

two numbers

is

10.

What
1

are the
2

numbers
8,

evident that there are

many

different answers to this

problem.
etc.,

The numbers might be

and

9,

and

3 and

7,

3 and 13, etc. or J and 9|, If we are also given that the difference of the
4,

numbers

is

condition.

new The numbers would evidently be 7 and 3. If we follow the method previously adopted and represent the required numbers by x and y, where x is the greater, the first condition would be expressed by the equation
then only one of these

answers

will

satisfy this

x+?/=10.

As
If

stated,

any number

of pairs of values of x

and y

will

satisfy this equation.

the second condition be expressed in terms of the same

unknowns, we have another equation

xy=4:.
It
will
is

now

required to find a pair of values of x and y which

satisfy

x-\-y=lO,

and
If

xy=
we add
2x=14,
.'.

4.

the corresponding sides of the equations


/,

we

get

x=l and

/.

y=3,

and 3 are Mxq required numbers.

SIMULTANEOUS EQUATIONS
55.
fied

79

Simultaneous Equations.

Any
of

equations which are satis-

by

the

same

values

the

unknowns

are

called

simultaneous equations.

Thus, x1,

y=3

satisfy both of the equations

x-\-y=\0 and xy=4:.

To find a definite pair of values of x and y it is seen that we must have two equations containing these letters. To solve any problem where two numbers are to be found we must have two conditions given, from which the required equations

may

be obtained,

Ex. 1. If 5 men and 4 boys earn $43 in a day, and 3 men and 4 boys earn 29 in a day, what sum does each earn in a day ?

Why
We
Let

do the

first set of

workers earn more than the second

How
earn
?

much more do they earn ? -How much then does one man How can we now find how much a boy earns ?
might solve
this

problem algebraically, thus


wages wages
of

$a;

and

= the $2/ = the


'

a man for a day, of a boy for a day.


algebraicall}-

The conditions
by the equations
:

of the

problem would now be expressed


$5a;+$42/=$43,
$3a;-|-$4y=$29.

Or, omitting the $ sign

and using only the numbers,


5x + 4?/ = 43,

Subtract the terms of the second equation from the corresponding terms of the first,
.-.

2x=14,

.-.

x=

7.

Substitute

x=7

in the first

equation and

35 +42/ = 43,
.-.
.-. .'.

41/

= y=

8,

2,

.'.

the roots of the equations are x=7, y=2, a man earns $7 and a boy S2 per day.

Verify by showing that these results satisfy the conditions of the given problem.

80

ALOEBEA
Ex.
2.

For

and

for 5 lb. of tea

the price of

of tea and 2 1b. of sugar I pay $1-30, and 4 lb. of sugar I pay $2 20. What is one pound of each ?

1b.

does this problem differ from the preceding ? in the first statement so that the number of pounds of sugar would be the same -as in the second statement ?

How

What change might we make

Let

and

X cents = the price of a y cents = the price of a

lb. of tea, lb. of

sugar.
(1) (2)

Then and
Multiply the
first

3a;+2j/=130, 5x + 42/ = 220.


equation by 2 and

we

get
(3)
(2)

6x + 42/ = 260,

5x+42/ = 220.

Now
esults

solve (2)

and

(3) as in the preceding example

and verify the

you

get.

Ex. 3.Solve

3x+4y=39,
4x+3i/=38.
(2)

(1)

(2)

Multiply

(1)

by 4 and

by

and we get

12a;+162/

= 156,

12x+ 9y=114.
Complete the solution and
verify.

Ex. 4.Solve

5x-2y^U,
3x+4?/=42.
2,

(1)
(2)
(3)

Multiply

(1)

by

10x-42/ = 88.

get rid of the term containing y, When we do so subtract.

To

we must now add

instead of

SIMULTANEOUS EQUATIONS

81

Thus in Ex. 4 we eliminated the y. We might have ehminated the X equally well. Solve Ex. 4 by first eliminating the x. After performing the necessary multiplications, when do we add ind when do we subtract to eliminate the unknown 1

EXE1ROISE3 41
Solve for x and y and verify 1-21
1.
:

x+22/=8, x+ y=5.
2x \-3y=25,

2.

3x+5y=13,

3.

Qx+5y=23,

3x+2y=
6.

7.

3x+2y=n.
6,

4.

5x2y=lS,

5x+2i/=24,

2x3y^

7.

2x- y=
8.

7.

2x+3y=U.

^.
10.

3x+5y=lS,
2x-\-3y=12.
x-{-y=i,

5x6y=3l, 6x-3y=33.

' 3x-2y=2A,
2x-3y=\\
12.

11.

3x+ iy=

5,

3x+2^^24,

x-y=3.
13.

6x+12y=13.
tk.

2a;+3i/=10.
15,

3x 4t/=16,
lx+3y=62.

14.

2x+5y=

0,

2i/ 3a;= 22,

yi^^

3x-'iy=23.

2x+3i/=32
18.

16.

3x=2y+

7,

O*^ 17.

2x='6y-l2.
19.

x^3y+20, y=2x-20.
20.
21.

3a;=2y,

2x-by=-33.
22;+3i/= 5x ^=17. 4x-52/=10?/-14a;=-ia

2x+l3y=

275,

14x-17t/=1385.
22.* If
23. 24.
25.
If

5x 1/=8 and

5?/ x=20, find the values of x-\-y and


find

j/,

2a:-5^-31=6(/-9a;+57=0,

the value of 19a;4-13y.

Solve

x+3=4 2i/,
equals 39
find

7(x l) + lli/=6.

If ax-\-by
is

when o
y.

is

3 and b

is 4,

and equals 13 when

is

5 and 6

2,

x and

26.

What
7

values of x and y will

make 16x y and

4:X-\-2y

each

equal to 6
27.

Solve 2(->/)+3(x+?/)--3l, 3(2x-y)-f 5(a;-22/)=53,

82

ALOEBRA

If the equations 57. Fractional Equations in two Unknowns. contain fractional coeffioients of x or y, the fractions may be

removed by

multiplication.

Ex.Solve
Multiply Multiply
(1)
(2)

^x^^y^
6,

8,

(1)

^a;+f?/=32.
by by

(2)

3z+ 2y=
verify.

48.

4,

z+10!/=128.

Complete the solution and

EXERCISE
Solve and verify 1-20
1.
:

42

SIMULTANEOUS EQUATIONS
21.*

83

^
x2y

+ :^= 2i

22.

z^-ky-l^y+Kx+y),

23.

ar+|y=y-2,

24. 5(a;+2/)-7(a;-2/)=26,

26. 8a;-7i/-12,

26.

x+l ^
10

- 3w 5 - a; ^. ^

2xy 4^3 _

27.

|?/-ix+24=f2/+^a;+ll=0.

EXBRCISB
Solve,
1.

48

by using two unknowns, and verify The sum of two numbers is 40 and their

difference

is 12.

Find

the numbers.

The sum of two numbers is 19. The sum 2. and 4 times the second is 64. Find the numbers.
3.
If

of 3 times the first

41b. of tea

and 7

lb. of

sugar cost $2-42, and 5

lb. of

tea

and

3 lb. of sugar' cost $2'68, find the cost of each per lb.

4.

Find two numbers such that 7 times the


first

first

is

greater than

twice the second by 23, and 5 times the

and 3 times the second

make
5.

136.
If 5 horses

and 6 cows cost $840, and 3 horses and 2 cows cost

$440, find the cost of a horse.


6. If either 9 tables
is

and 7

chairs, or 10 tables
?

and 2

chairs,

can be

bought for $156, what


7.
If

the cost of each

men and

women

earn $164 in 4 days and 5

women
woman.
8.
is

earn $135 in 3 days, find the daily wages of a

men and man and of

2
a

26,

Find two numbers such that \ of the first and \ of the second and \ of the first and \ of the second is 8.

Three bushels of wheat cost 20 cents more than 5 bushels ot of wheat and 1 bushel of com cost $2 '30. What is the price of each per bushel ?
9.

com, and 2 bushels

6 2

84
10.

ALGEBRA
In 10 years a

man

will

be twice as old as his son, but 8 years

ago the
11.

man was
If the
is

8 times as old as his son.


of

Find their present ages.

sum
;

the result
is

18

if

two numbers be added to 3 times their difference twice the sum be added to their difference the result

26.

Find them.

12.

at $25,
13.

merchant sells 33 suits, some at $35 each- and the others and receives $945. How many did he sell at each price ?

of the second

Find two numbers such that 5% of the first is greater than 6% by 3, and 7 % of the second is greater than 4 % of the first
3 algebras

by

7-5.
If

14.

and 4 arithmetics cost

$2-95,

3 arithmetics cost $2' 10, find the cost of 6 algebras

and 2 algebras and and 2 arithmetics.

15.

bull's

eye counts 5 and an inner

4.

In 10 shots a marks-

man
16. there

scores 46 points, each shot being either a bull's eye or an inner.


of each

How many

kind did he

make

A
is
?

classroom has 25 seats, some double and some single.

If

seating

accommodation

for 42 pupils,

how many double

seats

are there
17.

A man

at a gain of

$1030.
18.

bought 8 cows and 50 sheep for $900. He sold the cows 20% and the sheep at a gain of 10%, and received in all Find the cost of a cow ?
10

If

men and

8 boys receive $37, and 4


receive
?

men

receive $1

more

than 6

bo3's,

how much does each boy

19. A man bought 20 bushels of wheat and 15 bushels of corn for $36 and 15 bushels of wheat and 25 bushels of corn, at the same rate, for $32'50. How much did he pay per bushel for each ?

20.

will be twice the second,

Find two numbers such that, if the first be increased by 8 it and if the second be increased by 31 it will
first.

be three times the


21.

farmer bought 100 acres of land for $4220, part at $37 and How many acres were there of each kind ? the rest at $45 per acre.
22.

Find two numbers such that 7 times the greater and 5 times

the less together


23.

make

332,

and 51 times

their difference

is

408.
is

The quotient

is

20 when the sum of two numbers


7

divided
2.

by

3,

and the quotient

is

when

their difference

is

divided by

Find

the numbers.

SIMULTANEOUS EQUATIONS
24.

86
30c., the total

grocer bought tea at 60c. a

cost being S96,

He

sold the tea at 75c. a lb.


lb.

and gained

.$21.

How many

and coSee at and the of each did he buy ?


lb.

coffee at 35c.,

Three times the greater of two numbers exceeds twice the less 25. by 90, and twice the greater together with three times the less is 255. Find the numbers.
26.

The sum
Find the

of

two
If

fractions

respectively

is 2-9.

the numerators be interchanged the

whose denominators are 2 and 5 sum would


divided

be

4*1.

fractions.
is

27.

by

17

Divide 142 into two parts so that when the larger part and the other by 19 the sum of the quotients will be 8.

farm was rented for S650, part of it at S6 and the rest at S8 If the rates had been interchanged the rental would have been S750. How many acres were in the farm ?
28.

per acre.

29.

the

sum

^'s age 3 years ago was l^alf of fi's present age. In 7 years Find their present ages. of their ages will be 77 years.
travelled 240 miles in 4 days, diminishing his rate each
distance.
?

30.

A man

day by the same

The

first

two days he went 136

miles.

How

far did he

go each day

EXBRCISB
1.

44 (Review of Chapter VII)

Solve 2x+3i/ = 38, 3x + 22/ = 37.


I fire

2.

2, how many
3. 4.

20 shots at a target. hits did I make if

If

a hit counts 5 and a miss counts


score
is

Solve 7x 22/=13,

my net 2a; + 3y = 43.

51

65,

in

The average marks of those who passed an examination was If there were 1000 candidates and of those who failed was 25. all and their average was 53, how many passed ?
5.

Solve

2(a;

= 3(a;-4?/),
2/)

14(x

+ 2/)= ll(x + 8).


^'j

At an election A' a majority was 384, which weis 6. number of votes. How many votes did A receive ?
7.

of the

whole

Solve ^(a;+5)-5=(2/ + 2), ^(y +

8)-3 = J(a;-3).

Divide $5600 into two parts, so that the income from one part at 3% may be equal to the income on the other part at 4%.
8.

9.

Solve f 4- ?

3x

7j/

37

0.

86
10.
t of

ALGEBRA
Two numbers
If

the smaller.

differ by 11, and Find the numbers.

of the larger

is

more than

11.
q

= 2,

find x

px + qy is 74 when p = 5 and q3, and and y.

is

76

when p = 6 and
and 5% of ^'s

12. If 3% of ^'s salary plus 4% of B's salary plus 3% of B's is $111, find their salaries.

is

$93,

13.
14.

Solve

2l2/

+ 20x=165,

77i/-30x=295.
|

Divide 100 into two parts so that J of the less by 2.


15.
16.

of the greater part exceeds

Solve 5x2]/ = 7x +
If 3

2!/

= x+2/+ 11.
5

men and 4 boys earn $26, and what would 7 men and 3 boys earn ?
17.
18.

men and

2 boys earn $34,

Solve J(x+l)-J(!/ +
If

2)

= 3,

J(a;

+ 2) + i(]/ + 3) = 4.
that a = 3 and

3x 4 = ax + 6

when x = 2 and when x=5, show

b=-4.
19.
I

a profit of

I sold the horse at bought a horse and carriage for $400. 20% and the carriage at a loss of 4%, and on the whole

transaction I gained 5%.


20.

What
2x

did each cost

Solve

^2
is

27/

^= 2

7.

21.

A man
of bills

pays a debt
24,

of $52

in $5 bills

and $1
?

bills.

If

the

number
22.

how many

are there of each


140.

Solve 19x-21i/= 100,

21x- I9y=

A's wages are half as high again as B's, but A spends twice as If ^ saves $5 and B $10 per week, what are the wages of each per week ?
23.

much
24.

as B.

If

23x+15y = 91, and y

is

50% more than

x, find

x and

y.

married his age was I more than his wife's a 25. age. His age 8 years afterwards was } more than his wife's age. How old was he when he was married ?
26.
If

When

man was

3(5x-2i/) = 2(3x4-

6t/),

find x in terms of y.

27.
is is

A man has two farms rented at $5 per acre and the total rent WTien the rent of the first is reduced 20% and the second $1100. How many acres are there increased 20%, the total rent is $1120.
7

in aach

SIMULTANEOUS EQUATIONS
28.
If

87

+ i^ =

9,

find the value of

+|

Seven years ago B was three times as old as A, but in 5 years 29. he will be only twice as old. What are their present ages ?

31.

Solve?25

+ ?-', !^ = 3 + ??.

CHAPTEK
When

Vlll

TYPE PRODUCTS AND SIMPLE FACTORING


58.

Factor.

a quantity

is

more

quantities, each of the latter

is

the product of two or called a factor of the

given quantity.
Thus, the factors of 3hc are
3, b
is

and

c.

The product
.'.

of 6-f-c

and a

ab-^ac,

the factors of oA + oc are a and 6 + c,


ab-\-ac

or
Siroilarly,

= a{b + c).
by
a,

ab ac = a{b--c).
is

When

x-]-y-\-z

multiplied

the product ax-{-ay-\-az


recog-

contains the factor a in each term. If we wish to factor ax-\-ay+az,


nize that since a
Ls

we

a factor of each term, it ) ^-ry+^ x-\- y-{- z must be a factor of the whole expression. The remaining factor is the quotient found by dividing the

expression

by

a.

Then

ax^ay-\-az=a{x+y+z).

This is seen to be similar to the method in arithmetic. How do If we wish to factor 485, we see that 5 is a factor. we obtain the other factor ?

Ex.
18

Factor
On

4a2 6a6.
of

Here we see that 2 and a are factors


a factor.
Similarly,

each term and therefore 2o


3ft.

division the other factor is 2a .-. 4a*-6a6 = 2a(2a-3ft).


36a;

+ 6cx = 3x(
a(

ab--a*~a^^

).

The
and

result of the factoring

may
81

be verified by multiplication

this

may

usually be done mentally.

10.

90

ALGEBRA
The middle term in every case is seeu to be the of the two cross products, each taken with
r-f 5

sum

the proper sign.

x-Z

State the pn
1.

x+l x+2

TYPE PRODUCTS AND SIMPLE FACTORING


1

91

The last terms in the and 8 or 2 and 4.

factors

must be

factors of

8,

so they

must be

x+1 x+8
Which
of these of

x+2
a;+4
T

when

multiplied will give the proper middle term

What
The

are the factors of x*

6x+8

factors

x* 9x+14 must be
?

of

the form

(x

)(x

).

What

are the factors

Ex. 2.Factor x^-2z-l5.


Here the factors must be of the form (x must be the product of two numbers differing
combinations are
:

)(x-(-

),

since

15

in sign.

The

possible

x-15

X+I5

x+
Which

X-

x-5 x+3

x+5 x-3

of these sets of factors is the correct one ? In factoring a trinomial like x* 8x+15, we require two factors of 15 whose algebraic sum is 8. They are evidently 5 and 3.
.-.

x-8x+15 = (x-5)(x-3).
21 whose

In factoring x* 4x 21, we require two factors of algebraic sum is 4, and they are evidently 7 and 3.
.-.

x*-4x-21 = (x-7)(x + 3).

The
Thus,

pupil

is

below the expression he


X*

advised to write the factors under each other, is attempting to factor.


16 8 x*

6x

x*+nxy-4:2y*
X X

X X

+2
EXERCISE

+14?/ 3y

x-6x-16 = (x-8)(x+2).

+ Uxy-4:2y* = (x+Uy){x-3y).

47 (1-15. Oral)
3.
6.

Factor:
1.

x2+8x+7.
a^+22a+2\.
a^+3ab+2b^.

2.
5. 8.

x2+6x+5.
x2_^8x+12.

t/2+8y+15.

4.
7.

62+106+24.
2/2+40xt/+39x*.

m^+lmn+lOnK
x2 7x+6.
a^-Uab+28b^.

9.

10. 13.

x2-5x+6.
x*ixy+3y'.

11.

12. 15.

x2-12x+ll.

14.

m^ 7mn+12n.

92
16.
19. 22.

ALGEBRA
X2-X-20.
17.

y^y-30.

18.

a^+a30.
a:2_i0a;-24.
a;4_i0a;2+9.

x2 5x 14.
a^b^+8ab + 15.

20.
23. 26.

m2-6m-40.
x2y2_ii_j.j^_^30.

21. 04, 27.

25.

a2+6a + 9.

x^

14x+49.
_. ^y.

y*\2y'^+36.

Use factoring
-_ ^ ^o.

to simplify the following

a2^5a-4
-f

a2+4o-5
.

a-ri

a+5

m^ 5to+6 m^-7m+l2

^B

(x''+3x+2)(x-5)

3x2--6x

2x3-4x2

x2-5x-f-4
"^

x2-3x-10
32.

3x

2x2

x-1

What

factor

is

(1) (2)

common to X2-X-30 and x2-2x-35


a^+ab and a2+3a6+262
:

Find three factors of


33. 36.

2x2-10x4-12.
If

34.

3a2+3a-36.
has two
all

35.

x3-8x2+7x.
wi ?

the expression x2-L7ftx 6

binomial factors with

integral coefficients,

what are

the possible values of


factored

37.

Is the expression

x2 3x 10

when

it is

written in the

formx(x-3)-10?
62. Square Root of a Monomial. When a number is the product of two equal factors, each factor is called a square

root of the

number.

Thus, 16 = 4x4, therefore a square root of 16 is 4. But 16= 4x 4, therefore a square root of 16 is also 4. Similarly, the square root of 25 is -(-5 or 5, and the square root of 9a* is -j-3a or 3a.

Thus
It
is

it

is

seen that every


call
root.

number has two

square

roots

differing onl}' in sign.

customary to

the positive square root of a

number

the principal square


63.

Radical

Sign.
is

The symbol

-y/"

called

the

radical

sign

or root sign,
of a

used to indicate the principal square

root

number.

Thus,

^25 = 5, Va* = a,

V9x*y*==ixy.

T7PB PRODUCTS AND


When
considered, both signs

81 MPLB

FACTORING

93

both the positive and negative square roots are must precede the radical sign.

is

-3 not +3, but V9=3, and Thus, V'9 = 3 not -3; read " plus or minus the square root of 9 equals plus or minus 3."
Thus,

-V9=

but
If

\/4+V9 = 2 + 3 = 5, +\/iV'9=23=5or
represent the square root of 16 by
x,

1.

we

then 0:^=16.

To

solve this equation, take the square root of each side,


.".

x=4.
includes the four

We might have said statments


:

x= 4, which

+a;=-f4, ~\-x4:,

^=+4, x= 4.

If both terms of the last two be multiplied by 1, the statements become the same as the first two, which are represented by a;=4. We see then, that it is necessary to attach the double sign to the square root of only one side of the equation.

Ex.Solve (x+ 1)2=25.


Take the square
root of each side,
.'.

a;+l

= 5,
+ 5 1 = 5 I
or 6.

a;=

or

5 1,

=4
Show by

substitution that each root satisfies the given equation.

EXERCISE
State the two

48 (1-16. Oral)

94
20.
its side.

ALOEBRA
If

the area of a square

is

100 square inches, find the length ot

21.

If

is

the radius of

approximately.
radius, or

If the area of
is

what

a circle the area is nr^, where 7r=3^ a circle is 154 square inches, what is the the value of r, if S}r^154: ?
circle

22. 23.

Find the radius of a

whose area

is

616
its

sq.-in.

If r is the radius of a

sphere the area of

surface

is

given by
is

the formula, area =4:nr^.

If
?

the area of the surface of a sphere

164

sq. in.,

what

is

the radius

64.

Squares of

Binomials.
will

the result, which

If we multiply x-]-y by z-\-y, be the square of x-{-y, is x^-\-2xy-\-y^.

The diagram shows a geometrical


*'
tration of this identity.

illus-

The

first

and

last

terms in

x^-\-2xy-\-y'^

are the squares of the terms of X'\-y, and

the middle term

is

twice the product of x

and
the

y.

Therefore, the square of the sum of two numbers is equal to sum of the squares of the numbers, increased by twice their

product.

Also

{xyy=x^2xy-}-y^.

Therefore, the square of the difference of two numbers is equal sum of the squares of the numbers decreased by twice their product.
to the

In the square of a sum all the terms are positive, and in the square of a difference the middle term is negative.
Thus,

+ 2(3o){26) + (26), = 9a* + I2ab +46. (5x-3t/)' = (5x)-2(5x)(3!/) + (32/), = 25x* 30xy +9y*. (ix-42/) = (Jx)*-2(ix)(42/) + (42/), = Jx* 4xy -flGj/'.
(3a + 26)=(3o)*

TYPE PRODUCTS AND SIMPLE FACTORING


EXERCISE
What
1. 5. 9,

Sfi

49

(1-16, Oral)

are the squares of


2.
6.

a+l.
2a+l.

y+2.
l-3x,

3.
7.

1.

4.
8.

4.
2x+3.

p-g.
3x2y.
3a;
f.

2a 3.

10.

TO 2n.
2!/ J.

11. 15.

12. 16.

4x 3(x.

13.

a;.

14.

Simplify
17.*
19.

(x+l)2+(z-D2.
(2x-|-l)2+(a;-2)2.

18.

(a-6)2+(a+6)2. (a+6)2-(a-6)2.

20. 22.

21.
23. 25.
26.

(3TO-n)2 (2to+7i)2.
(a:+l)2+(a;+2)2+(x+3)2.

{3x+2y)^{2z-3y)^.

24.

[z-l)^+{x2)^-{z-3)^.

2{a+l)2+3(o-l)2-5(o-2)2.
Find the value of
Simplify
a^-f-fe^-j-c^

when a=xy, b=z-\-y, c=x2y.

27.

{x+l)^+{x-2)^+{x-Z)^-3{x-^)^

28.

From

the

sum

of the squares of

a;-l-2, a;-|-3, a;-f-4,

subtract the

sum

of the squares of

z 2, z 3,
differ

a; 4.

29. 30.

SimpUfy {2a-3b)^+{3a-{-2by-{2a+2b)K
If
is

two numbers

by

2,

show that the


+^ 2

difference of their

squares
31.

equal to twice their sum.


does the square of x

By how much
I

exceed the square of

2,
X
32.

Show

that the

sum

of the squares of three consecutive

numbers

is

greater
33.

by two than three times the square


of 1234
is

of the middle

number.

The square

1,522,756.

Find the square of 1235.

34.

30J, etc. that this


in J.

2^=2x3+^=^6^; the square of 5| = 5x6+J= In the same way find the squares of 6^, 8^, 20^. Prove method may be used to find the square of any number ending (Let the number be n+J.)
The square
of

65.

Square Roots of Trinomials.

Any
is

trinomial which

is

of the

form

a^-\-2ah-\-b^ or a^2ab-\rb^

In order that a trinomial

may

a perfect square. be a square, the first and

ALOEBRA
last terms must each be a square and the middle term must; be twice the product of the quantities which were squared to produce the first and last terms.

Thus, 9x* + 24x3/+ I6y*


(1)

-.a

square, because

the square of 3x, 16?/* is the square of iy, (2) 24Ty is twice the product of 3a; and 4y. (3) -. 9a;*+24xi/+16?/* = (3x + 42/). .. the square root of 9a;* + 24x1/4- 16?/* is 3x + 4?/. Is 4m* 12mn+9n* a perfect square ? What is it the square of What in its square root ?
9x*
is

Similarly.

25x*- 10x+

= (5x(

1)*,
)*,
)*.

36x* + 24x + 4 = (

a*6*-6a6 + 9 =

Why is a*4-5a6-f-256* not a square ? How would you change it so'that it would

it the square of o + 66 be a square ?

Is

ab
It
is

The square
is

root of a^-\-2ab-\-b^

is

a-\-b,

but

(a+6)

or

also a square root, since


.

(-ab)^=a^+2ab-\-b^.

customary, however, in stating the square root of a trinomial to give onl}' that one which has its first term
positive

EXERCISE
Express as squares
1.

50

(1-24. Oral)

a;2+2x2/+j/2.

2.
5. 8.

y^'-2y+l.
9a2-24a-fl6.

3.

4.
7.

4a2+20a-l-25.

a^b^-2ab+l.
a^b^c'^-2abc+l.
is

l-6y+9y^.

10.

11.

x^+x+J.
x^-^xy+^y^.

What
13.

the square root of


14.
17.

9a2+12a+4.
4a262_20a6 + 25.

16. 19.

'im^+2m+l.
912x4-4x2.
will

4-4a+a2.

20.

Supply the missing terms, so that the following


squares
:

be perfeci

22.
26,

a^

+ b^+9

23.

x-.

.+4y\
. .

24.
27=

x^^Gx
.
.

4m*.

26.

9a*+lSa,

. 4a;v-|-4t/

T7P10 PRODUCTS

AND SIMPLE FACTORING


is

97

28. If 16o ma-f4 is a perfect square, what Give two answers and verify eacL
29.

is

the value of

What

the square root of 9a;^+6a;+l

Check by putting

a;=10.
30.

Solve the equations and verify


(1)
(2)

Vx^+2z-}-l + Vx^+10z+25=U.

3Vx^Ax+'L2Vx'^+6x+9=2.

(3)

V9x^+6x+l + V&+Ix+l + Vx'^2x-\-l = 13.


that

31.

Show

2^02-60+9-^02 4o+4=3V'a2-2a+l-V4o2+4a+l.
66. a-\-b

Product of the

Sum and

Difference.

The product
a a

ot

and

ab
:.

is

a^b^,

{a-\-b){a-b)=a^-b\

+b b
-ab-b*

Here the two factors multiplied are the sum and difiference of the same two quantities a and b, and the product is the difference of the squares of a and b. Therefore, the product of the sum and difference
two quantities
Thus,
is

a*-\-ab

a*

-b*

equal

to the difference

of the of their squares.

same

{x+y){xy) = x*y*. (2a+36)(2o-36) = (2a)-(36) = 4o-96. (3a*-6)(3a*+6) = (3a)*-6* = 9o-6. (J + 3x)(J-3x) = (i)-(3x) = i-9x.
Factors of the Difference of

67.

Two

Squares.
I

a-b
-I

Since

a^b^~{a-{-b){ab),

the

factors of

the difference of

two squares are the

sum and

the difference of the quantities squared.

The diagram shows how

this identity

may
b

be illustrated geometrically.
Thus, 9a;*-252/*=(3x) (52/), which shows that is the difference of the squares of 3a; and 5y. Therefore one factor of 9x' 25^' is the sum of 3x

it

and 5y, and the other is the differeuoe of 3j; and 5v9j;-252/ = {3x + 52/)(3x-5i/,. That is, Similarly, ll6m-9-(47n) 3 = (4m+3)(4m-3>

98
If
is

ALGEBRA
we wish to factor Sx'^ 2y2^ we should recognize that 2 a factor of each term.
.-.

8x*-2i/ = 2(4x-2/) = 2(2a;+y)(2cc-y).

EXERCISE
State the products of
1.
:

51 (1-24, Oral)

m-{-n,

mn.

TYPE PRODUCTS AND SIMPLE FACTORINO


40.

99

Simplify {a^-b^){a^-5ab+6b^)-^{a'^-3ab+2b*). Simplify


a;2

41.

xy

yi
-]

3.2

j,2

3.2

and

iQ

3.2

g -

x-\-y

X 4

x+3

42.

Solve ?!i:l

x+l

+ ?!i:i^ = X

10

2(x-5)(a;+5)=15+(a;-l)(a;+l).

68.

Numerical Applications
this

of

Products and Factors.

In

Chapter

we have developed

certain

formulae

concerning products and factors.


(1) (2)

(a-fe)2

=a2-2aZ) + 62.

(a+6)2

=a2+2a6+62.

(3)

(a+&)(a-6) =a^-b^-.
for all values of the letters involved.

These formulae are true

By
see

substituting particular

numbers

for the

letters

we

will

how some
Since

arithmetical operations might be simplified.

(1)

(a-6)' = a*-2a6 + 6*, 99= (100- 1)= 10000-200+ 1 = 10001 -200 = 9801. 37= (40-3)*= 1600-240 + 9= 1609-240=1369. = = 998* = (1000-2)*= = = 89= (90-1)*=
{a

(2)

Since

(3)

+ b)* = a* + 2ab + b*, + 2)*= 8100 + 360+4= 8464. 121* = (120+1)*= 14400 + 240+ = 14641. = 75*= (70 + 5)*= = a*-6, Since (a + 6)(o-6) 92x88 = (90 + 2)(90-2) = 90*-2* = 8100- 4 = 8096. 65x75=(70-5)(70 + 5) = 70*-5* = 4900-25 = 4875. = = 27x23 = (25 + 2)(25-2)= = = )= 87x93 =
92*=
(90
1
(

)(

(4)

Since

a*-6*=(a + 6)(a-6), 53*- 52* = (53 + 52)(53-52) = 105x 1 = 105. 41*- 31 = (41+31)(41-31)= 72x10=720. = 727-627* = )= )( = 67- 33 = )= )(
( (

100
69.
(1)

ALOEBRA
Some
If a is

(leometrical AppliCwtons.

the length of t\u

^ide of

the large square and

b the side of the


IS

small square

,he area of the

shaded portion

evidently

a^ 6^.
If

a
(2)
.".

we wish

to find the area of the

snade" part when

a=77 and 6 = 23, we have


a

a-6* = 77-23 = (77 + 23)(77- 23)= 100x54 = 5400.


If

a = 225 and

6= 125,

find the difference in the areas

of the

two squares.
is

The

radius of the large circle

E
is

and

of the smail

circle is r.

The area
is ^-^-r^,

of the large circle

'^fR^

and

of the

small one

the area of the shaded part

is

"^^{R^r^).

If iJ

= 39

and r=31,

find the area of the ring.

The area
If i?

= 89

and

= V(-r) = V(39*-31) = V(39-f-31){39- 31) = Vx 70x8= 1760. = 82, show that the area of the ring is 3762.
In the right-angled triangle in the figure
in
it

(3)
'^

is

shown

geometry that

62-|-c^=a2 or
If

b^=a^c^

or c^^=a^

6^.

a = 41 and

If

a = 61 and

= 40, find the length of 6. 6 = a-c = 41-40=81xl=81, 6=-v/81 = 9. 6=11, show that c = 60.
.-.

SXERCISE3
Use short methods
1. 2.

52

in the following

Find the squares of 98, 999, 119, 58, 799.

Find the products of 91 X 89, 61 x 59, 47 X 53, 203 X 197.

3. Find the values of 522-48^, 79^-782, 2152-2052, 7252-2752, 6732-5732.

4.
5.

If
If

x2=62-c2,

find

X when

= 13,

c=12

when 6=25, c=24.

7x2=642-572,

find the value of x.

TIPS PRODUCTS AND SIMPLE FACTORING


6.
ii

101

Find the difference

of

the areas of

squares whose sides are

and b

for the following values

a=

41

13

102

ALGEBRA
12,

22." Find three factors of x^ x, 3x*


23.
24.

a^-3a* + 2a.

Solve (z-3)* = 25; 4(a;-i) = 9.


If a

= 7rr,

find r

Find the values of 997, 25. using algebraic methods.


Find four factors 26. a\x^-y>)-b*(x*-y*).
27. 28.
of

when a=12-56, = 314. 875*- 7o, 97x103, 81x81, 86x94,


7r

2x-32, a- 13a-f36, '27n'- 18m and

Simplify (6+
Simplify

92 If a 29. algebraic methods,


30.
31.
of

+ (6- l)* + (c-f l) + (c- 1). {x + y){x-y) + {x+2y)(x-y) + {x + y){x-2y). and 6 = 88, find the values of ab, a* b*, a* + b*,
l)*

using

Simplify (a-26)(a + 26) + (a-46)(o + 46)-2{a-36)*.

What

two factors with positive


32.
33.

are ail the possible values of b, if a;*+6x+42 integral coefficients ?

is

the product

Simplify ^ ^
If

?^^%"-i^V^^xZy xy x2y
is

the square of 426

181476, find the squares of 427 and 425.

CHAPTER IX
SIMPLE APPLICATIONS OF FACTORING
70.

Highest

Common
it

Factor.
is

When

a factor divides two


factor

or

more expressions

called a

common
12

of

those

expressions.
Thus,
4
is

and

a
in

is

a a

common factor of 8, common factor of a*,


the
highest

and

20,

2a and 3a6.
factor

As

arithmetic,
all

common

(H.C.F.)

is

the product of

the simple

common

factors.
3,

and

Thus, the simple common factors of 3a '6, 6ab* and 9abc are therefore the H.C.F. is 3o6.

and

b,

In the case of monomials the H.C.F.

may

be -m-itten

down

by

inspection.

Ex. 1. Find the H.C.F. of 6m^n,


(1) (2) (3)

12mH^ and 9mH^.

The H.C.F. of 6, 12 and 9 is 3. The highest power of m which is common is m*. The highest power of n which is conamon is n.
..

the H.C.F.

is

3xm*xn

or 3m*n.

If

the expressions are not monomials they must be factored


after

when possible, by inspection.

which the H.C.F.

may

be written down

Ex. 2.Find the H.C.F.


a*

of a^-{-ab, ab-\-b^, a^-\-3ab-\-2b^.

+ o6 = a(a + 6), ab + b* = b{a+b), o + 3o6 + 26* = (a+6)(o+26}, . the H.C.F. = a+fc.

104

ALGEBRA
E3XERCISE
64 (1-12. Oral)

Find the H.C.F. of

Thus.
.
.,

SIMPLE APPLICATIONS OF FACTORING ^=|=ij=|o = etc.


b

105

, Similarly,

am = = =
OCX
box
i

rbe

etO"

a*b*
a*bc

a6 abc

6 be

6
c

73.

Lowest Terms.

fraction

is

said to be

in

its

lowest

numerator and denominator have no common factor. If it is not in its lowest terms, it may be reduced by dividing both terms by all the common factors.
terms

when

its

EXAMPLBa.

18_18-f-6
42
15a6
2.

3
7-

42^6

25a6

_ "

\5a*b^5ab
25ab*-^5ab

_ ~

3o
*

66

x*y*
3.

x+y){xy)

^ xy
x+y

x*-\-2xy-\-y*

{x-\-y){,x-\-y)

x*
4.

+ 5xy-{-iy* ^

x*+3xy4y*

(x+y)(x+4y) (xy){x+4y)

^ x+y
xy

The attention
factors

of the pupil is drawn to the fact that it is and not terms which are cancelled from the numerator and

denominator,
Thus, in the fraction q-t-o
fraction
is

+ 2 ^6 cannot cancel the twos and say


is

that the

equal to

|,

for the value of the fraction


is

y\,

which does not


th.-

equal

J.

But

if

the fraction
is J,

7x2 -

we can now

cancel

twos and

the resulting fraction


Similarly,

= - after cancelling, or dividing by the common factor a.


not equal to
-.

But

a+c
is

is

It

thus seen,

that

no cancelling can be done until both

terms of the fraction are expressed as products.

108

ALGEBRA
KXBRCISE
55 (6-21 Oral)

Fill in

the blanks in the following

^15___30____6a:y
'

20~
ax bx

_
b

_ oc

~12~4a~ _ _ 5a _
bm

Zam

a^
b

^ g<P+g)
6(m+n)
Gfe""

6a'a;
*

a^x

_
(

a*a;

_
2x2

_
)

<*

12a2x2

a'-6'
*

K
(

a*-2a6-[-62

)a

^ ^.j

Reduce to lowest terms


6.

li. 21

7.

??. 6

8.

^. 25
1-^!^.
15to

?*. 6c

10.

?^.
00

11.

1^.
6x

12.

13.

^.
5a6

14

^"+^
6
'

15
'

3a+6a
12a
'

^^
'

4x

^^
'

2x2-8x'
^^^~y)
21
'

o(x-3) 6(x-3)
(~^) (a-b)^'
-

18

^(^-^)
(a;-l)2'

19

(^-l)(^-2

20

'

(x-2)(x-3)"
23
'

&a^

'

22*?!z:3 x^x
*

y'-y y^2y+l
a'-&'

24
'

^'-3^+2
x^ 4x+3

25

"t'+7m +12 TO+4m


;z:^.

2g
'

g?
'

3^'"%'
3x2+9xy+6t/'

'

o2-|-5a6+46*

28.

a*

2.

^^. 4a*

30.

?;^^
x'

74.

Multiplication and

Division of Fractions.

The methods
in

by which fractions aie multiplied and divided ^re the same as in arithmetic

algebra

SIMPLE APPLICATIONS OF FACTORING


Examples.
3 5

iO",

^
3

3><_5

15

10
^'

21

7_^_10 ~
15 21

15_25_
^ 4

"

a
b

c_axc_ac d~ bxd" hd
xhi
a* X*

ah

ab
xi/

x*y
o*

a
X*

xy

a
cd-\-d*

a*+os6

c*+cd ^ ab+b'

_ ~~

a{a+b)
c{c

d(c-\-d)

+ d)
56

b{a

_ + b) ~

ad
be

SXERCISE
Simplify
2.

1x^-^1. 15 7 14
xy

3.

5.

x |-be
25x2

6.

15^
7y

141/2

11

4a+6fe^
5x
^^

lOx

^,

2a+36

a2_3a_i_2

a2-7a+12

a'^-^a-\-Z

108

ALGEBRA
lowest
is

,-

The

common
is

multiple

(L.C.M.)' of

expressions
of factors

the expression containing the smallest

two or more number

which

a multiple of each of the given expressions.'*; .C^

Ex. 1. Find the L.C.M. of Qz-y, 9xy^ and I2xy^.


The numerical
of 6, 9
coefficient

of

the L.C.M.

is

evidently the L.C.M.


of

and 12 or 36. The highest power of x


so that the L.C.M.
.-.

is y^,

in any of the given expressions is x* and must contain the factors x* and y^. the L.C.M. = 36 xxX2/S = 36x*!/'.

Ex. 2. Find the L.C.M. of a^-b^ and a^-2ab+b^.


a-6 = {o-fe)(a + 6). a*~2ab + b* = {a-b)*.
:.

the L.C.M. =(a-6){a + 6).


7

Why

is

{a-b)(a + b) or {a-b)*{a + b)* not the L.C.M.

EXERCISE
Find
1.

67

(1-9,

Oral)

the L.C.M. of
3, 4, 5. a, ab, a^.

2. 5.
8.

10, 15, 20.


x^, xy, y^.

3. 6.

2a, 4a, 6o.

4.
7.

2ah, 3ac, 66c.


Ga^fc, 4ab^.

lOa^, 15a2, 5a.

3a^
3x,

2a^, 4a.

9.

10.* a2, a'^^a.

11. 14.

3x*+6x.
{x^-y)^.

12.

ab+ac, h^+hc.

^3.
16.

2X-I-2, a:"-!.

x^+xy,
a^-fe^,

15.

x^-\, x^-^x+2.
18.

a^-ab,ab-b^.
2x, 4a;+4,

17.

a2-2a6+62.

x^-x,x^-x.

^19.

2x^-2.

^.

y^-3y+2, y''-y-2, y^-\.

Show that the product of x^+x 2 and 21. the product of their H.C.F. and L.C.M.
76.

x^ x 6

is

equal to

Addition

and

Subtraction

of

Fractions.

If

we wish

add or subtract fractions we must reduce them to a common denominator. As in arithmetic, the lowest common denominator is the L.C.M. of the denominators.
to

SIMPLE APPLICATIONS OF FACTORING


Examples.

109

4
('^

35_2_9 1P_^_ 9+10-8 _ ''6 "12''" ~ "12*


11

12

12

12

.01
c

b
3

ac
be

ah

ac+ab
be
'

bc~
5

^ _ i^
a;(a;4-j/)

Soft
,

36-4a*+6qfe

aybx
x+x2/
2x
5.

a^+y

y{x+y)
2 ~ z+2 "

xy(x+y)

x 4

2 ~ x+2 ~

2 (x+2)(a;+2)

2z-2( z
(x

+ 2)(x-2) 2x-2z + 4
(x+2)(x-2)
Oral)

(x+2)(x-2)

BZERCISB
Reduce
1.

58

(1-8,

to fractions with the lowest


2.
^,

common denominator

2, 5. 3 9

.
46
r?
.

3.

1,1.
a
a^

4.

l.A.
3x

4
.

2x
36 -
a'^

^ O.

3a
-

43
,

4a -

b.

nm
, '

,16
7,

_, _.

2
-,

oc

S.

'

3a' 4a2' 2a3*


-?_
3t/2'

6'

c'

a*

'

'

'

36c

12

A.

13
*

^ ^
:

_L

14
*

+^
o
'

g 1 g+2
2a
'

3xy' 2x'

3aA' 26c' 4ac'

3o

"

Perform the operations indicated


15.

+ ?. 3^4
3 5

16.

^ + ^. 2^3
6

17.

18.

^^_?.
_1_
<

19.

oi
I'

"

l-x"^l+x
2

J_

22 *

^ "3 - i^

-^ + J-. x-l^x+1

20.

^+ 3^5 ^ ^%^. ^
5::^.
2

+ ^

23

^^ + ^i:^ - ^+^ "^


4

'

24

'^Z^-'^IZl

+ 3^4

'Lty.

26

-^

x-y

x+y

^.

no
26.

ALGEBRA
-JL + JL_ 8^. a+4 a 4 a* 16
27.

28.

-^__^. 3x+6 2x+4


s a*+3a+2

XX
'

^^4-?!:? +
o
x
9 a

ox

29.

a* aft

a6 6*

30.

+ a+5a+6 h-3^ ^^r-^ o+4a4-3


1

(Check when o

= l.)

31-

li-T o*+3a+2 q! 1 + ^T
Mixed Expressions.

a+a-2

77.

An
is

expression

which

is

partly

integral

and partly

fractional

called a mixed expression.

mixed expression
in arithmetic.

in algebra corresponds to

a mixed number

Thus, 31

i3

a mixed

number and a + c

is

a mixed expression.

Note that
omitted and

in
3r-

a mixed number the sign of addition is means 3+?. But in algebra the sign must
c

be inserted, as a - would mean

ax- and c

not a

-\

Since Reduction of a Mixed Expression to a Fraction. 78. every integral quantity may be written as a fraction whose

denominator is unity, it follows that the reduction of a mixed expression to a complete fraction is a problem in
addition or subtraction.

Examples.
2

15+2

17

c
a;

c
a;

a.

o y

5y = -= 5y-x

y
a(6

y
c)

ac
*

ac
b

ab+acae
b

~bTc^~b+c

+ c~

+c

'

_ "

ab

b+e

SIMPLE APPLICATIONS OF FACTORING


79.

111

Reduction

of

Fraction

to

Mixed

Expression.

To

separate

into

two fractions we merely reverse the

operation of addition.
rhus,

ab-{-bc

a
ab bdc

ab

be

a
bd
6

be
,

b
c

-\

a
,

and

ab 6 =T-

-6 = -'^-6'
(1-9,

EXERCISE
Reduce
1.

50

Oral)

to complete fractions
2.

2+|.

1+^.
x

3.

+ ?.

4.

aH ^4

5.

^3

6.

-.
c c

7.

x-^*.
n
a

8.

2x

+ ^. X
xy

9.

a6-^.
2a H
-.

10.

b+c

11.

x-\

^.

12.

ab

13.

x+l + l. 3z

14.

a-6-^. 2
:

15.

'

.-j/

+
i

"'+^
^_^y

Separate into fractions in their lowest terms

16

^+^
4
'

17
'

?^^.
ab
'

18

5x-8j/
10a
'

19

6a2-36*
3o6
*

20
'

^'^^"^
21a6
3oc
'

21
'

^iJi?
6xy
r
'

. ^^

3in 4n ^-^r 2n

_
*

6a6c 96c-|-c'

*.0

. ^*.

(a 6)*+x

EXERCISE
1.

60 (Review of Chapter IX)


factor

Define highest

common

and lowest common multiple.


8,

2.* Find the H.C.F. and L.C.M. of 3a;- 6, 4a;-

5x-10.

112
3.

ALGEBRA
Find the H.C.F. and L.C.M. of x' + xy, xy + y* and x*y + Qry*. Find
*.he

4.

H.C.F. and L.C.M. of

x 7x+10 and
is

Show

that the product of these expressions

x' + 2x--8. equal to the product

of their H.C.F.

and L.C.M.
:

6.

Reduce to lowest terms

a*+ab
a*
'

X* x*

xy' 8a6

abxbx* 6a' 9ab 126*' acx cx*

/6.

Multiply

5^. ;2' 2^'


2c*

,.,

3bd

6cd

Reduce

to lowest terms
a;

x*-2x x-5x+6'
J6.

2g -18 o*- 6 + 4x + 4 x + 5x + 6' 3x + 3x-r8* a*-2ab+b'


-i

Simplify

x~y , *,-, X
.

x+y K^~-^ X
,

2ax

xy y'
,

'/

0.

T^Divide

x+2 x o OX o-x by , ^r^ and - by ^ ,, ' x* x-j-l o* 4x* + 2ax


'

|'

Simplify

.,

x-3
3

x+4

~4

^^^

2x-l
3

, '

8-4x
g"
'

\/12.

Find the sum of Fi


T, From

xy
xy
t

yz
yz

zx
zx
,

/IB.

Zo

the

sum

36

of

+ 4a

and

6-6c

^t

^ ^ subtract
,

a+6c

^ 4.

Simplify

...

"^

x-3 x-5 -= ~K
3x

5x

and

3x-2/ xy

32-21/
yz
of

15.

Express

a*--b*
^r^-

as

the

difference

two frMtions
c*
c

in

their

lowest terms.

Do

the

same with

5_c*
,

o'c

and

a*
,

and

find the

sum

of the three results.


16.

By how much '

does

V 4 1 V5 exceed -= 4y 5j/

17.

Find the sum of

a-\-o

r-., a

11

and

2a
,

j-,-

a*

o"

SIMPLE APPLICATIONS OF FACTORING


3.

113

18.

By
?

wliat

must

3
_.

be multiplied to give

2;* -^

5a;

+6
b,b

n~~r'ity

the

product
19.

Find tne quotient when

+4
-j

is

divided by

x*4-a; 12
-j

2U. 21.

Solve (o-6)x = (o-6*)(o + 6).

Find the difference between


z ax + bx
,

cx
from

and
,

X X r ax h bx
,

cx

by '

first

subtracting

ax

ax

etc.

22.

Find the missing term

in the following identity

x*--5x+6 a!-3x-4 ^

x*

+ 5x.
a;-9

_ a;+2a;-8 ~ x*-x-12'

CHAPTER X
REVIEW OF THE SIMPLE RULES
furnish

In this Chapter will be found such exercises as will a review of the elementary rules. In it is also included matter which it was not thought advisable to present to beginners in the subject of algebra.
80. 81.

Use

of Brackets.

We

have already seen that

(1) a-^{b-\-c)=a-\-b-\-c, (2) (3)


(4)

a-{-{bc)^a-\-bc,

a{b-\-c)=abc, a{b c)=ab-\-c.


are preceded

by the negative sign, be removed if the signs of all terms within the brackets be changed ; but when they are preceded by the positive sign, as in (1) and (2), no change
That
is,

when brackets
(4),

as in (3)

and

the brackets

may

is

made in
In
(3)

the signs

when
of
b

the brackets are removed.

the

sign

in

a{b-\-c)
(

is

positive

as

the

expression might be written


are

a + 6 + c).

When

the brackets

removed we follow the rule and change -\-b into b. Sometimes we find more than one pair of brackets in the same expression.
Ex. 1.SimpHfy a-(3a-26) + (5a-4&).
Following the
rule, this expression

becomes
or 3a

a 3a + 26 + 5a 46

26.
it is
is

When
better

one pair of brackets appears within another,

to

advised to

remove the brackets singly, and remove the inner brackets first.

the pupil

REVIEW OF THE SIMPLE RULES


Ex. 2. SimpHfy 4:X-{2x-i3+x)}.
Removing the
inner brackets,

116

we

get

4x-{2x-3-x].
Removing the remaining
brackets,

we

get

4x-2x+3 + x
Ex.
The
3.

or 3x + 3.

Simplify

3a [a+6 {a 6 c (a+& c)}].

expression = 3a [a+b{a b c ab-\-c}],

= 3a [a+ba-\-b + c + a+b c], = 3aa b-{-a b c a b-{-c, = 2a-36.


After removing the
first

pair of brackets, the quantity

abcab+c
might have been changed into the simple form 26. problem again, simplifying at each step.

Work

the

When
removed.

brackets are used to indicate multiplication,

the
are

multiplication

must

be

performed

if

the

brackets

Ex. 4. Simplify ^x3{x2y)-\-2z^^y.


The expression

=ix (3i Gy) -\-2x^y, = 4x - 3x +62/ +2x - 4 y, = 3x+22/.

to

Note. When the pupil has had some practice he should be able remove the brackets and perform the multiplication in a single step.

EXERCISB
Remove
1.

61

(1-e,

Oral)

the brackets from

(a-fe)+(c-rf).

2. 4.
6.

(a-h)-{c-d).

3.
6.
7.

-(a-6)+(c-d).
a{hc){de).
a+{6+(c-d)}.
a-{h+{c-d)].

-{a-h)-{c-d).

a{b)-{c).
a+\b-{c-d)\.
a-{b-{c-d)\.

8.

9.

10.

11.

a-{-h-{c-d)\.

12.

-[a-\h-{c-d)\].
I

116
Simplify
13.
15.
:

ALQEBhA
4a-26-(2a-2fc).
2(7a;-3t/)-3(2a; 3j/)

14. 16.

3(a-6 + c)-2(a-^6-c).

2a- :3a4-2(a 26);-.

17.

3(a+6-c)-2(a-6+c)+5(6-c+a).

18.* 15a;-:4-[3-5x-(3x-7)];.

^.
20. 21.

Add 3(a+6) 5(/)4-?),


Add

2(a+6)+(p+?)
and

and 4(p+g).

1+z

2/,

1xy

1 z+y-

Add 3x-2(y-z), 3y-2{z-x), 3z-2{z-y).


the brackets and express in descending powers of x

Remove
.

22. 23. 24.

3{5x-3+2z2)_2(x2-5 + 3x)-3(4-5x-6x2).
2x(3x-2)-5(x-3)+6x(x-l)-2(x2-5x).
^(4x-3)-J(6-x2)+|(x2+8x-12).
:

Solve for X and verify


25.

4(x-3)-7(x 4)=6-x.
5x-[8x-3;i6-6x-(4-5x);]==6.

^JtQ.

^7.
28.

3(2x-7)-(x-14)-2{5x+17) = 6(5-8x) + 21z+149.


i(27-2x)==|-^(7x-54).
Simplify
S' '"^P lif

^9.
2(5

a [56 {a (3c-36)+2c-(a-26-c)n.
S{a-b+c)+2{b-c+a)-{c-a+b) 5{a-2b+c)-2{b-3c+2a)-{Uc-2a-nb)'

1/

'

31.

Solve (7jx-2J)-[4i-|(3J-5x)]=18J.
Insertion
of

82.

Quantities

in

Brackets.

The

trinomial

ab-\-c may be changed into a binomial by combining two This may be done in a number of its terms into a single term. of ways.
Thus.

a-6 + c = (a-6) + c = (a + c)-6 = a (6 c) = a-t-(c 6).


the brackets mentally and see that each of these
is

Remove
to

equal

o 6+c.

REVIEW OF THE SIMPLE RULES


Ex,
1.

117

Express

h-\-cd
may

as a binomial.
in

As we have

seen, this

be done

many ways

as a (6

c + rf),

(a-6) + (c-d), (a+c)-(6 + rf), (o-d)-(6-c), c-(6 + d-a).

Note. The pupil must exercise particular care when dealing with brackets which are preceded by the negative sign. The signs of all terms inserted in such brackets must, of course, be changed. He should, in every ^case, remove the brackets mentally to test the accuracy of the work.

last

Ex. 2. Express a-\-hc as a binomial by combining the two terms within brackets, preceded by the negative
a-\-b

sign.

c = a{ b-\-c), = o (c 6).

Either form is correct, but it is usual to make the first term within the brackets positive, so that the second form is preferable.
83.

Collecting

CoefiBcients.

Brackets are frequently used

for the
in

purpose of

collecting the coefficients of particular letters

an expression.
Thus,
ax-\-by cx dy = x{a c)-\-y(b d), mxny-\-px-\-qy = x{m-\-p)y(n q),

and

=x{m+p)-\-y{qn).
Verify these by removing the brackets,

BXERCISB
1.

62

Express 3a 26+ 4c as a binomial in three diff^ent ways and verify in each case.
2.
p^erify.

Express pqr-\-s as a trinomial in four different ways and Express

3.
verify.

xyzk

as a binomial in four different

ways and

Collect the coeflBcients of x


4.
6.

ax by ex

and y

dy.

mx ny px-{-qyax-\-by.
a(x-j/)+6(2j/-3x)H-c(5x+2t/).

6.
7.

x(c 6)+y(6 c) d(x+y).

8.

2ax 36y lOx 5j?+66x 4oy.


{a%)v~(^h)x-\-^y-\-2ax{Zx-\-h^),

118
10.

AWE BRA
Enclose a-

bcde-^f
;

in

alphabetical
tcrrriH in ea<;h.

order in

brackets,

with two terms in eich

with throe

Arrange
11. 12. 84.

in

descending pxiwcrs of x

a<z2+4-3z)-6(3a:-5x2)-c{l-4;c).
(ix'^-hx+c-(2jjx^-'iqx+r){ldx^^-'icx+f).
Multiplication

with Detached

Coefficients,

The method

of

multiplying two

bino.aiaLs has
is

Chapter V.

The same method

been shown in followed when the factorf?


already

are not binomials.

Ex. 1. Multiply
(3)

ar-3x+4 X -2

Check

x=\

+2
-1

the two methods are the same, with the exception that the letters are omitted in the second method and the coefficients only are used. When the second method is used the first coefficient in the yjroduct must h>e the coefficient of the product of x^ and X, that is, of x^. The next must be the coefficient of x^ and the next of x, as the product will evidently be in descending powers of x, as both factors multiplied are so written. In {'4) the check is shown as explained in art. 42.
coefficients.

The second method is called The proce.Hscs in

multiplication with detached

Ex. 2. Multiply ^x^~lx-\-2 by a;2-2x+3.


Here the term containing x* in the first exproHsion is rniBsing and a zero is Hupplie<^J
'

+ 07+

2
"

12 + 3
'14-0 "^ '

irj*

powers

of X

order to bring coefficientH of like ^ under each other in the partial

7-t-2 "t

HE VIEW OF THE SIMPLE RULES


Kx.
3.

119

Find

the coefficient of

.(''

in the

product of

5.1:3 6.1-2 +3.r- 2

and .r^ 2.r2

-3.r~f--t.
(lio

Horo the oomplete product


contains
;c*.

)s

not roqiiirod. but only

torin

which

The partial products wliicli will we obtain by multiplying 2 by


5x*
,-,

contain x' arc evidently those which .', 3.v. by 2.r*, tU^ by 3x and
f
1

by

4.

thp coefficient of x* =
4.

- 2-6

+ 20 -= 30.

Ex.

Multiply

ax~-\-b.v-{-c

by 7nxn.
-\-bx

ax*

Here the multiplication

+0
~\-cin.v

is

,,,^

performed in the usual way. In adding the partial [)roducts, the coetVicients of
tiie

(n/i.r'

biiix*
aii.r*

powers

of

.v

are collected.

amx^

-\-

-~ {bm an)x* + (cw bn )x ciu

biix

en

BXBROISfil 68
Multiply and check
1.

a;2-3a;+2, -2.

2.
4.
6. 8. 10.

2a;-5.c-3.

.'^.)-2.
h.

3.
5.
7.

x-a;+l,a;+l.

a^-\-ab+b\ a

x^-x+l, x^+xi-l.
3.:2-2a;-r>, .r2-f-;i--3.

a'-6a-2.
'Za^-^rnib
a;-''|
\

aM -l
\

:W\ 2a'
|

[wh-'Ah^

0.

a \-b-c,a-b-\-c.

2.r-'

4.r

S.

.r2-4.r+4.

11. 12.

b^-b+l,b^+b+l,b*-b^-\-l.
x'xy-j-y^-'rx-^-y-^-l, .r-fj/
1.

Use detached
13.

ooefVioionts to multiply

chcok the results

3x3-4xa+7x-3 by
r>a*-6<i3-2(i'-
I

a;2-2.r-

1.

Ai.
16. 16.

by

2f/-

:hi

|-2.

4x'-5a;-2 by 4j;a-3.c-l.
(x2-a;-2)(2x-a:

l)(3.r-2).

Simplify

17.* (a;-l)(x-2)

+ (x-2){j:-3)+(x-3)(x-l).

120
18.

AWEBRA
(a+x)(6-c)+(6+a;)(c-a)+(c+z)(o-b).
(a+6)(c+d)-(a-6)(c-(f).

19.

20. 21.

(o+6 c)(a 6) + (6+c a)(6-c) + (c+a-6)(c a>,


(x+l)(z4-2)(+3)-(a;-l)(x-2)(x-3).
:

Find the product of


22. 24.

(l_a;)(l+a;)(l+a;2)(l+a;*).

23. 25.

(x-l)(ar-2)(a;-3)(a;-4).

(x-l)(a;-3)(a;+l)(a;+3).
eoeffieient ofa;2 in the

(a-l)(a2+a+l)(a3+l).
:

Find the
26.

product of

3a:2_5x+ll and 5x2+6x-4.


a;3+4x2 5x+2 and x^ 2x-3.

27. 28.
29.

3x2-12x+15 and 2x2-7x-38.


Multiply

1+x+x^+x^ by

l4-2x+3x^4-4x', retaining no powers

higher than the third.


30.

Add

together

(x-l)(x+2), (x+2)(x-3), (x+3)(x+4)(x-l).

(x+4)(x2-2x+3) and 7-x2+3x. Check by putting x=2.


3*: 32.

Multiply

7x3 5x2y-x?/2-f 6?/' by 4x2+3x2/ 22/2.

Show

that the expression x(x+l)(x+2)(x+3)

is

equal to

^ (x+3x+l)2.
33.

Find the
2

first

four terms only in the product of:

+ 3x+4x2+5x3

and

l-2x+3x2 4x3.
product of

34.

Find the

coeflBcient of x* in the

X
36.
37.

l+4x+7x2-j-10x3+13x* and l+5x+9x2+13x'+17a;.


Solve and verify
:

(x-2)(t-4)(x-6)(x-10) = (x-1){x-5)(x-7)(x-9).
Collect the coefficieni^

Multiply ax^-\-bx-\-c by bx^cx-\-d. of X and write in descending powers.

Multiply px^qx-\-r by px-\-q, and {a\)x^-\-ax\ by ax+1.


Simplify

38.

(ay^-hy+c){ay+h)+{ay^^-hy-c){ay-b).

^^i^%.

Subtract the product of x2-|-x(p+l) 1 and

x 2p

from the

product of

X* x(j3 1)+2 and

x-f-j>.

REVIEW OF THE SIMPLE RULES


40.

121

Point out two obvious errors in each of the followin|^tatement8


(i)
(ii)
(iii)

ah{a^h){a^-]^b^)

(2x+3?/)'=6x='

= a*b^a^b-\-a^h^~ab*. + 36xy-54a;2/2+27/3.

x^%x^y^xy^+2y^={x2y){x^-Ax-\-y%

41. Use the formula (a+l)(6+l)=a6+(a+6) + l to find the product of 2146 and 3526, being given that the product of 2145 and 3525 is 7,561,125.

dividing

by a Compound Quantity. The method of by a monomial has already been shown in Chapter V. The method of dividing by a quantity containing two or more terms is in many ways similar to long division in
85.

Division

arithmetic.

Divide 672 by 32.


(1)

(2)

32)672(21

3. 10+2)6. 10* + 7. 10 + 2(2.

10+1

64 32 32

6ol0*+4.10
3.

10+2 3.10+2
form,

In
3
.

(2)

10+2 and
If

we

is expressed in the equivalent the dividend 6 102+ 7 10+2. substitute x for 10 the problem would be

the divisor

Divide Qx^-{-lx-\-2 by 3x+2.


The method here is so similar to the 3a; + 2)6x* + 7x + 2(2a;+l method in arithmetic, that little explana6x* + 4a; iion is necessary. The first term in the ix-\-Z quotient is obtained by dividing 3x into ^x+2 6a;. The product of 3a; +2 by 2x is then subtracted from the dividend and the remainder is 3x+2. The last term of the quotient is obtained by dividing the first term of the remainder (3a;) by the first term of the
divisor (3x),

the same as hers.

In more compUcated examples the method is precisely The division is continued until there is

122 J

J
Division.

ALGEBRA
is

no T-pmoinlbj^nr until a remainder nan^The degree tljan^he divisor


86. * Verifying

found which

is

of lower

The work may be


It
is

verified as in

arithmetic,
test

by

multiplication.

simpler,
for

by substituting a particular number

however, to each letter

involved.
Thus, in the preceding problem if we let x=l, the divisor is 5, the dividend is 15 and the quotient is 3, which shows that the result is very likely correct. If on substituting particular values for the letters involved, the divisor becomes zero, other values should be selected.
87.

Division with Detached Coefficients.

As

in multiplication

the method of detached coefficients

may

be used.

Ex. Divide
Check

\4.x^-x^-2^x^-\-\2 by 7a;2+3a;-6.

74.3- 6)14- 1-29 + 0+ 12(2- 1-2 14 + 6-12

x=l
4)-4(-l

-7-17 +

-7-

+6 -14-6+12 -14-6+12
3

Here the

first

term

in the quotient is 2x*, since 14a;*-f-7x*


is 2x''

= 2a;*.

The complete quotient

x 2.

Divide also by the usual method.

BXERCISB
a;2+3a;+2
1.

64

(1-6.

Oral)

State the quotients in the following divisions

'

x+1
a;2-4
,

a2-3a+2 a\
a2+2afe+62

'

a^

ab
3a;2-5x+2

'

x+2
:

5.
*

a+6
8.

3x-2
6x^+xy-l2y^hy3x-Ay.
9a^-\-6ab35b^hySa+lb.

Divide and verify


7.

6a;2+x-15by2a;-3.

9.

5x^Zlxy+&y^hyx&y.

10.

REVIEW OF THE SIMPLE RULES


11.

123
25a;-|-3.

7x3+96a;2-28xby7a;-2.

H^.

lOOx^-lSx^-^x by

yt3.

3+7x 6x2by3 2a;.


x^+13x^+54:x+'12hy x+G.

t^.

6a2+35 31a by 2a 7.

^^.
17.

^. 2a3+7a2+5a+100 by a+5.
18.

x'^+Sx^y+^xy^+y^hy x+y.
16to3 46wi2+39m 9 by

x^+ix^y3xy^+y^hyxy
,
^

19.

8m 3.

20.
j^^tf
22.

6x3 29x22/+18x2/H352/3 by 2x-7y.

a*+a3+4a2+3a+9 by a^- a+3. x< x3 6x2+15x 9by x2+2x 3.


by Sx^+llx+lL

t^. 5x*-4x'+3x2+22x+55

1^.

2x3-8x+x*-)- 12-7x2 by a.2_^2-3x.

^.
26.
27. 28.

30-12x2+x*-xby X-54-X2.
coefficients to divide
:

Use detached

x3 3x2+3x 1 byx2 2x+l.

6x*-x3-llx2-10x-2by 2x2-3x-l. oS-5o3+7a2+6a+l by a2+3a+l.

29.

4x2+9+x+3x+x3 by 2x+3+x2.
(x2-x-2)(2x2+a;-l) by 2x2-5x+2.

30.*
31. 32. 33.

^Ox^+llx*-2s:^-\lx^-x-\^ by 2x''+x-l.
Divide

a^ 1 by a 1 and o'+l by a+1.


x^-{-y^

Simplify

x^y^

x+y
02+0+l
-!

xy
"'+^ o2 O+l
-

34.

Simplify

^!l,+

__ 3o.

Q , Solve

6x2+x-2
2x-l

3x2+8x-3 ,, -J =11. 3x
find x.

i6j
37.

If

x(3a2-a + l)+2=-3a3-7a2+3a,
is

The dividend
divisor.

a^+ea^+Ga^- 9o4-2,

the quotient

is

a'+3o 1.

Find the
38.

Divide

x8+x*y*+^ by

x^x^y^-\-y* and divide the quotient

by x^xy-\-y^.

194
o11/

ALOSBRA
abx^x(ac+b^)-\-b
-^
,

acx^-\-x(nd+bc)+bd

bxc

cx+d

Without removing the brackets divide


40.
41.

ax^+{b+ac)x+bc by ax+b.

x^+{2p-l)x+p{pl) by x+p.
a^x^

42.
43.

2abx+b^c'^

by ax{bc).

aV+(2a'+a)y'+(a'+2a)2/+(a+l) by ai/2+ot/+l.

88.

Inexact Division.

As

arithmetic, the divisor

may
be a

not divide evenly into the dividend, and so there

may

remainder.
Thus, 34-^5 gives a quotient 6 and a remainder
/.
3ji

4,

= 6+4
is

or 34 = 6. 5

4.

Similarly,

when a*-r3a + 5
is

divided by

a+1, the quotient

is

o+2

and the remainder

3.

a-t-1

o+l

or

+ 3a + 5 = (a+l)(a + 2) + 3.

That
Ex.

is,

dividend=divisorxquotient+remainder.

Express
l

-j

as a

mixed expression.

Here the quotient

ia

+ x+x*+x'
is

1 a;)l ^-'

+x*{l+x + x* + x*

sind the

remainder
!

2x*.

+x

__ =
Divide

+ , + .. + .3+^_^
that
,

+ x*
+x* x'

1 x^by 1 + x and show


1

1+x

4- X*

+ X* 1+x
,

+^ +* + x x
2s

In such cases the division may, of


course, be continued to

any number

of

terms.

MVIEW

OF THE SIMPLE RULES


BXEROISB
66

i2

Find the remainder on dividing


1.* a:2-10a;+25
3. 6.

by a;-7.

2.

a2+20a+70 by a+6.

a:3_4^2_^53._^20by

X 1.

4.
6.

y^7y^+8y-\ hy y^y+l_
x^y^ by x+?/.

x^+y^ by

i/,

Express as mixed quantities


7

^^
a;4-l

8
'

^^^

9
'

^~^^

fe

a+b

10 '

Q^^+7a:-3
x-\-2

Find four terms in the quotient


11.
15.

of

l-^(l-x).

12.

l^(l+x).
is

13,

1^.

14.

l+a+2a' 1 a+a^
is

When
Divide

the dividend

a^ 3a + 7,

the quotient

a and the

I'emainder
16.

is 7.

Find the

divisor.

x^ 5x+a by x 2 and determine


is

for

what value

of a

the division will be exact.


17.
If

x^ mx+12
is

divisible

by

x 3,

what must the quotient

bfe

\nd what
18.

the value of

By

division

show that
-;

^a+b + ao a~o
'

_L^=a-64-
a+o
a-]-o

BXERCISB
1. 2.
3.
4. 5.

66 (Review of Chapter X)
3x

Add x^-2ax*+a*x+a^, Add

+ 3ax*,

2o ox* x.

^a }b,^a- lb, la+i'o.

Subtract 8a + 36 -5c from

Ua- 26 + 5c 3d.
zero.

Subtract Subtract

3a + 46 c

from

|a ^6 + Jc from fa+6 |c.


must be added
to

6.

How much
and
c

Jx iy+^z

to produce

xy+z\
sum
of

7.

Subtract the sumi of 3a + 26, 26 3c and 3c a from the

a 6, 6 c
8.

a.

Simplify

o-(36-4c)-(6+c-o)-2(ac).

im
Subtract 4x* 3x' 9. by substituting 2 for x.
10.

ALGEBRA
a;-|-2

from

7x' 6x* +

2x

and check

Find the value of o' + fe' + c' 3a6c when a=2, fc=3,
Simplify {a + b-c)-{b + c-a)-{c + a-b)-{a+b+c).

c= 5.

11,
,.,,-*2T

Multiply

l-4x-10x' by l-6x+3x*.

13.

Find the product of x+1,

x+ 2

and

X 3.
3x-2 by
3x + 4x 4 and

Divide the product of x + 2, 2x-3, %4. check when x= 1.


IfiT*

Multiply a*-'rb*-\-c* ab bc ca by a + b + c.

16.
-

Find the product of


Divide Divide

a- 2, a+ 2, a* + 4. o*+ 16.

17. 18.

x+64 by

'coefficients

by x* + 2x-3, using detached and verify by multiplication using the same method-.
of x' in the

+ 4x+8. x* + x* 24x* 35x + 57


x

Find the coefficient 19. and 2x + 5x+llx+4.


'

product of 3x' 2x* + 7x

20. The expression 44x* 83x' 74x*+89x + 56 is the product ^two expressions of which 4x' 5x 7 is one. Find the other.
,

of

Divide x* + 4x' + 6x* + 4x4v21 BUJMtituting x= 10.


22.
of

by

x* +

2x+l

and

check

by

powers of x
23.
24.

Subtract ax* + 6x + c from cx* + dx4-/. collecting the coefficients in the result.

Find the remainder on dividing x*+6 by x*

1.

Show that
(a

6)(x a 6) + (6 c)(x 6 c) + (c a)(x c a) = 0.


b

25.

Show that (a-6)(6-c)(c-a) = a{6*-c*) + fc(c-a*)+c(o-6*).

If a = x* + 2xy + 2y^, 26. value of ab c.

= x*-2xy + 2y^,

= x*-y\

find

the

27. 28.

Subtract 2x-3{y + 2z) from 32/-(Bz-3x).


If

= a + 6 + c,
in

find in terms of a,
a(3

b, c

the value of

a) + b{s b)-\-c{s c).


x,

29.

Arrange

descending powers of

c{ax b)x(a b)-\-bx(x*


30.

cx).

When

o = 5, find the value of

2a_i3a-(464-2oU + 5a-(46 o)..

RE'9'fEW

OF THE SIMPLE RULES


">

127

What quantity when divided by x* 2a;+3 gives x* + 2x 3 as 31. quotient and 9 as remainder

^.
33.
of:
30,

If

a = a;*-3x + 2,

6 = 3x-10x+8,

= 4x*-9a;+2,

find

the

value of (a+26-c)-^(x-2).

Arrange in descending order of magnitude and find the average -15, 27, 0, 3, -10, -2, 6, -8.

the

What number must be added to 5x* 13x'-|-2a; 1 so that sum may be divisible by x 2 ? Find the coefficient of x* when l+a; + x* is divided by 1 x x'. 35.
34.

y^.
37. 38. 39.

Divide 3p*-7p(l-j3)-(2+p) by (3j9+l)(p+l).


If X*

9x + c

is

divisible

by

x-|-4, find

c.

Find the sum

of the coefficients in the square of 2x*

x 3.

Find the product of x-\-a, x + 6, x + c. Collect the coefficients of the powers of x in the product. From the result, write down the product of x4- 1, a;-|-2, x + 3 and of x 1, x 3, x + 4.
JLii.

Divide

x-2x'+l by x*-2x+l.
a = 3, h = 2\, c = 2, find the value of

>1.

When

-+

\/7a6(2c-o6)

(2o-36).

/42.
43.

Prove that (I+x)(l+y>)-(I+x*)(l+2/) = 2(x-2/)(l-xi/).


If

p=X

and q

= x*

j,

show that

j)*(p*

+ 4) =

g*.

j^'^. Divide a' + 6' + c' 3a6c by a + 6 + c. of o'4-6 + c* 3a6c ? Compare with Ex. 15.
i^^ft5.

What

are the factors

Multiply x* + fex4-c by x*-\-px-\-q, arranging the product in descending powers of x.

^.
47.

Divide 9a*-46*-c* + 46c by


Multiply x* x(o 1)
I

3a-26 + c.

by x* + ax+l.

As.
49.
of X.

Divide o-166c* by a-2bc.

Arrange the product of x a, x b, x c

in

descending powers,

Divide a*6 + 6*c + c*a ofe* 6c* ca* by a b, and divide the 50. quotient by a c.
51.
of 2x*

What

expression will give a quotient of x*-f-l and a remainder

7x + 6whendividedby 3x lOx* 6 ? Divide x'-y^ + Bt/'- 12y+8 by x + 2. 62.


1/

CHAPTER XI
FACTORING
89.

[continued)

In Chapter VIII. we have already dealt with the This Chapter will of factoring in simple cases. furnish a review of the methods already used, and an extension of those methods to more difficult examples.
subject
90.

Type

I.

Factors

common

to every

Term.

When

every

term of an expression contains the same can be found by inspection (art. 58).
Thus, 2xy
is

factor, that factor

a factor of
.'.

4:X*y

6zy* + 2axy,
^

^x*y 6xy* + 2axy = 2xy{2x 3y -\- a),

is

a factor of aix-\-y)-\-b{x-\-y). divided hy x + y, the quotient is a + 6,


is /.

Also, x + y

When

this expression

aix+y) + b{x+y) = {x+y){a + b),

Type II. Factors by Grouping. When every term has common factor, if the number of terms be changed by grouping, we may sometimes obtain a common factor.
91.

not a

Ex.

1.

Factor mx-\-nx-\-my-\-ny.
(1)

(2)

mx + nx + my-{-ny^

= x{m + n) + y{m + n), = {m + n)(x + y)


factor in the

mx-\-nx+my-\-ny, = m(x+y) + n(x+y), = (x + 2/)(m + nV

Here we changed from four terms to two, and we found a conunon two terms. The other factor of the expression was then found by division. The two solutions show that different methods of grouping may be If the first method tried is not successful, try othsrs. Rtaployed.
12S

/
Usually those terms are grouped which contain a simple common In the example we should not expect to be successful by grouping mx with ny, as these terms have not a common factor.
factor.

Ex. 2.Factor x^+x^+2x-\-2.


Use two and a;* + 2.
different

methods

of

grouping and obtain the factors

x+l

Ex. 3. Factor {ah'fax-^-hz.


{a

b)* ax + bx = {a b)* {ax bx), = {a-b)-x{a-b), = {a b)(a b x).

Note. When quantities are enclosed in brackets, the pupil must not forget to verify by mentally removing the brackets.

EXBROISB
Factor:
1.
2. 3.

67

(1-9,

Oral)

3a;-2X

4.
5.

b^-5b.

7.

a{xi-y)-\-b{x+y).

2a-6.
a2-3a.

3a2_i5a6.
6x^yl2xy^.

8.

p{m-n)+(m-n).
x{ab)2y{ab).

6.

9.

Factor, using
multiplication
^ytO.
12. 14.
16. 18.
:

two

different

methods
11.

of

grouping and verify by

ax-{-bx+ay-\-by.

ambm-\-anbn,
bxax-j-abx^

x'^ax+bxab.

13.
15. 17.

2ac+^ad-2bc-3bd.

x^+x^+x+1.

a= a2 3a+3.
a;3+4a;2_3x-12.

xyxy+l.
a^- 7a 2 -40+28.

yt^.

20. Factor x^+x*x^x^+x+l by making three groups each containing two terms, also by making two groups each containing three terms.

21.* Find three factors of


22.

3x^6x^+3x6 and
and
ax-\-ay-\-bx-\-by,

of

axyayax-{-a.

Find a common factor of

am+bm+an+bn
and
oi

x^x^+x1 and

x^x^-\-2x2.

I0x^5zy6xz+3yz and a^b-\-a^bc3a^b^Sab^c.

130
24,

AIjOEBRA
Find a

common
it
is

and show that


25, 26,

factor of 2x^6x2 3a;+9 and a factor of their difference.

x^ 3x^+2a; 6,

Factor 48aa; 56a?/ 356!/+306x.


Factor (x+2/)2+4x+4!/ and 2(a-b)^a+h.

III. Complete Squares. We have already seen 64 how the square of a binomial may be written down. We have also seen in art. 65 how the square root of a trinomial may be found, when the trinomial is a perfect

92.

Type

in art.

square.
93.

Square of a Trinomial.

trinomial

may

be squared

by expressing it in the form by multiplication.


Thus,
(o

of a binomial or

.-.

+ 6 + c)=|a + (6 + c)|, = a + 2a(6 + c) + (6 + c)*, = o*+2a6+2ac + 6 + 26c + c, (rt + 6 + c) = a* + 6 + c+2rt6 + 2ac+26c.

results.

Multiply a + 6 + c by a-\-b-\-c in the ordinary way and compare the Examine the diagram and see that the same result is obtained.
Similarly,
(a

and
Complete these two

(a_6-|-c)* =
in

+ 6 c)= {a + (fe-c); (6-c)l*.


-|a

*,

a manner similar to the one worked in

full.

If we examine these products we see that they consist of two kinds of terms, squares {a"^, b^, c^) and double products

{2ab, lac, 26c).

We

The, square of

might express the result thus any expression is equal

to the

of each of its terms, together with twice the of each pair of terms.

sum of the squares sum of the products

In writing down the square, care must be taken to attach the proper sign to each double product.
Ex.1.
(2x-3?/ + 42)

= 4x4-9y+ 162*- 12xt/+ 16x2- 24j^2,

FACTORING
Ex.2.

131

(a-26+c-d)

= oS + 4fe + c + d*-4a6+2ac-2a<i-46c + 46d-2cd.


Ex.
This
3.
is

Factor

a;*

+ 42/* + 2* 4x1/ 2a;24-42/3.

Which

evidently the square of an expression of the form x+2t/ + 2. when squared will give the proper arrangement of signs ? Verify by writing down the square.
of these

EXERCISE

132

^ J

AL9SBRA
i

Complete the squares by suppijang the missmg terms


39.
41.

x^-

/.
.-

.+25.
,

40.

4a;2+.
.

+25y^.

a^+iab

:^\
*
.

42>/.'^'*r
.

12m+97i*.

43.

a2+962+.C

.-0a0-2ac
.

V .G'/^C

44.
45.

9x2+.y;.
Factor

.+.

...

.-6xy-l2xzi.^.C^Z-.

Find three factors

of 3x'^

+ 6a;+3
^
.

and of al -4-4026+406*.
->' ~

46.

c\-^ i^

"

(a+6)2+4c(a+6)+4c^ and \a+b)'2{a-\-b)(c+d)+{c+d)^.


47.
is less

Show that the square of the sum of any two consecutive integers than twice the sum of their squares by unity.
Divide the sum of the squares of a 26+c, 6 2c+a, of the squares of ab, bc, ca.
If

48.

c 2a+6

by the sum
49.
50.

i + X = 4,

find the value of x^

-\

i
-.

X*

Factor {axYby)'^+{bxay)^+c^(x^+y^).

51.
52.

Express a^x^+ft^^+a^^+ft^x^ as the sum of two squares

Find the

value

of

x2+2/^+22+2x2/+2xz+22/2,

when

x=a+26 3c, 2/=6+2c 3o, z=c+2a 36


94.

Type IV.

The Difference

ohe

sum and

difference of the

The product of Squares. same two quantities is equal


66)

of

to

the difference of their squares

(art.

Conversely, the difference of the squares of tveo quantities


is

equal to the product of their


Or, in symbols,

sum and

difference (art. 67).

and
95.

{a-\-h){ah)=a^h^, a'^-h'^={a+h){a-h).

The formula

may sometimes

for the product of the sum and difference be used to find the product of expressions of

more than two terms.


Ex. 1.Multiply
Here and

2a fe+c by 2ahc. 6 + c is the sum of 2a b and c, 2a 2o 6 c is the difference of 2a 6 and

FACTORING
is

133

therefore the difference of the squares of 2a


/.

rhey might be written {2a b'j-j-c and (2a b) c. 6 and c.

The product

{2a-b+c){2a-b-c) = (2a-b)*-c*, = 4a*-4a6+6* c*.


of

Ex.

2.

Find the product

2x-^yz and 2xy-\-z.

Here the first expression = 2a;+(?/ z), and the second =2x{yz), the product =(2x)*{y~z}*,
..

= 4x-(2/*-22/z+2),
= 4aj* 2/* + 21/2 z*.
Verify

by ordinary

multiplication.

Ex.
a and

3.

Multiply

ab-\-cd by a+bcd.

Note that the terms with the same signs in the two expressions are d These should be grouped to form the first term in each
a b+c d = {a d) {b c),
a-\-b
..

factor.

cd={a d)4-(bc),

the product = (a d)* (6 c)'.

Simplify this result and verify by multiplymg in the ordinary waj:

Ex.

4.

Factor p^4:pq-\-'iq^z^.
first
:

Here the be written

three terms form a square and the expression

may

[p*

^q-\-iq*) x* = {p 2q)* x*, = {p-2q+x)(p-2q-x).


and subtracted
to

What two
the factors
?

quantities were here added

obtam

Ex. 5. Factor a^-b^-i-2bc-c^.


Here the
square.
;.

last three

terms should be grouped to form the seconu

a-6* + 26c-c = a*-(6-26c + c),

= a-(6-c), = {a + (6-c)}{a-(6-c)}, = (o + 6 c)(a 6 + c).


Verify

by

multiplication.

i34

ALGEBRA

Ex. 6.Factor x^-{-y^a^-h2-\-2xy+2ab.


Evidently three of these terms form one square and the remaining three the other square.

The expression

= {x* + 2xy + y*)-{a*-2ab + b*), = (x + y)*-(a-b)*, = {x+yi-a-b){x + y-a + b).


EXERCISE
69 (1-10. 17-32, Oral)
:

Use the formula to obtain the following products


1. 3. 5. 7.

(2a+3)(2a-3). {xy+5){xy-5).

2.

(4x-l)(4x+l).
{abc){ab-{-c).

4.
6. 8.

(2m^+3n){2m^3n).
(x+J)(a;-i).

{abc-\-xy){abcxy).

(x2-y2)(x2+t/2).

9.

{x+y+z){x+y-z).
{a+bc){ab+c).

1.

(a-fe-c)(a-6+c).
(2x-|-3i/-5)(2x+3y+5).

11.*
13.
,

12.
14.

{p-2q+3r){p+2q~3r)

{\-z+z^)il+x+x').

15. 16.

{a+bc+d){ab-cd).
{a2b^c2d)(a-2b-c-^2d).
:

Factor and verify


17.

x2 9.
a%^-x^.
l-a262.

18.

20. 23.

26. 29.
32.

{x+y)^-'2o.

(a+6)2-(c-d)2. 30
a^

2a6-f-62_c2

35.

(4x+3)2-16x2.

^.

a^+b^+2ab-c^

FACTORING
45.

136
of

Find three factors

of

ba^\0ah+5b^-20c^ and

(a;-36)3 462x4-1263.
46.

Find four factors

of a^b^aH^bH'^-\-c'^d'^

and

of

(a2+62-c2)2-4a262,
47.

Factor

a'^x'^

b'^y'^ ^2aA^x-\-c'^

48.

Find the simplest factors of

w^ Gm^n^+w^ 2m. 3x^2^^30; +2 and of


and

a;* a;3_9x2+9x.
49. 50.

Simplify

(a 6+ c)(a 6 c) + (a+6-c)(a 6+c).

that x^y"^
51.
'

Arrange x^{x^a^)y\y'^a'^)-{-2xy{x^y'^) so as to show is a factor of it, and thus find the simplest factors.
Use. factoring to simplify
:

'^Af

'''^^''^^'-(1)
'
-

(a2-3a+l)2-(a2-3a)2.

(2) (a;-22/+32)2-(32-x+22/)''=.
(3) (4)

(a2-3a-4)2-(a2-4)2.
(5x2-2a;2/+?/2)2_(5x2+2a;y-2/2)2.

52.

Multiply aArb-\-c by a + 6 c and

a 6+c

by

abc

and use

the results to obtain the product of

(a+b-\-c){a-\-bc){ab-\-c){abc).
53.

Show

that x{y^--z^)-\-y{z^x^)-\-z(x^y'^)

is

equal to

{xy){y^z^)-(x^y^){yz)
and then
54.
find

the factors of this expression.


in the

Arrange a{b^c^)-^b(c^a^)+c{a^b^)

form

o(62-c2)-6c(6-c)-a2(6-c)
and thus obtain the factor bc.
Find the other two
factors.

96.

Type V.

Incomplete Squares.

We have already

factored

expressions which were seen to be the difference of two squares.

many

Sometimes the two squares

of

which an expression

is

the

difference are not so easily seen.

136

ALSEBRA

Ex. 1. Factor .r^+.r^/z^+y*


This expression would be the squar*^ oi
2x^y*.

We

will therefore

a-' + y* if the middle term were add x^y* to complete the square and also

subtract x^y* to preserve the value of the expression.

Then

.:

+ x^y* + y* = x*+2xh^* + y*-x*y*, = (x* + y*)*-{xy)', = {x* + y* + xy){x^ + y*-xy), x*+x*y* + y* = {x* + xy + y^){x*~xy + y*).
x*

seen that

In order that this method may be successful, it will be t1ie quantity we add to complete the square must itself be a square.
Thus, to change a* + ab + b* into a*-(-2a6 + 6* a6 is not an algebraic square.
is

of

no value as

06

Ex. 2. Factor a*+4t*This can be

made
,\

the square of a' + 26* by adding 4a*6*.


a

+ 46 = a + 4a*6-|-46*-4a6.

Ck>mplete the factoring and verify by multiplication.

Ex. 3. Factor
the factoring.

4m< IBw^n^+gn*.
to

What must be added


Try
to factor
it

make
it

it

the square of 2m* 3n*

Complete

by making

the square of 2m* + 3n*.

Ex. 4. Factor a*-\rb*-\-c*-2a%^-2b^c^-2c^a^.

How does this expression differ from the square of a*-f-6* c' T Express it in the form (a* + 6* c*)* 4a*6*, Write down the two factors and see if you can factor each of them again and thus obtain the result
(a

+ b + c)(a+b c){ab+c)iabc).

Factor and verify

FACTORING
10.

137

26a;* 89z^*+642/'=

li-

x*+y*\\x^y^.
and x^+z^+x.

i^^x^lx*+l.

13.* Find three factors of 2a;*+8


14.

Find four factors of 9a*l0a^b^+b^,

15.

Find three factors

of

x^+x^+lby

Find four factors of a*+b*+c*-2a^b^2b^c^2c^a^ completing the square of a^b^-\-c^r


16.
17.

Factor (a+l)*+(a2 l)2+(a 1)*.

97.

Type VI.

Trinomials.

We

have already dealt with

the factoring of expressions of the type x^-{-pz-\-q, where the


coefficient of the first

We now
where
98.

m is

term is unity (art. 61). wish to factor expressions of the t;^^e 'mz^-\-px-\-q, not necessarily unity
Method, by Cross Multiplication.

First

Ex. 1. Factor 2x^-\-lxy-{-3y\


The product
the
first

of the first terms of the factors is 2a;*, and therefore terms must be 2x and x similarly, the last terms must be
;

3y and y and the signs are evidently .'. the factors must be

all

positive.

2x+3y

z+
It
first
.'.

or

2x+ y x+Sy
in

is seen, by cross muitiplication, that the coefficient of xy product is 3 + 2=5, and in the second is l-i-6 = 7. the correct factors are (2x-\-y){x-ir3y).

the

Ex. 2. Factor 3x2 7a; 6.


Here the numerical coefficients of the first terms of the factors must be 3 and 1, and of the last terms may be 6 and 1 or 3 and 2. Since the third term is negative, the signs of the second terms of the factors must be different. The possible sets of factors, omitting the signs, are
:

3*

*2

3a;

2 3

3a;

6
I

3a;

138

ALOE BRA

Since the signs are different tor the last terms, when we cross multiply to find the ooefficien*^ of x in the product, the partial products must be subtracted It is easily seen that the second arrangement is the only one from which 7z can be obtained. Since the middle term is negative, the larger of the cross products must be negative.
.-,

the factors are


is

(3x+2)(x-3).

This method
pupil will find
in the work.

liable to

be found tedious

when the

coefficients

have a number

of pairs of factors, but in ordinary cases the

little difficulty

after he has

had some practice

99.

Second

Method,

by

Decomposition.

multiplying two binomials hke

2x+3 and

In the process of 3x-\-5, we have

(2a;+3)(3x+5)=3:c(2a;+3)+5(2a;+3),
=ex^-\-9x-\-l0x-\-15,

= 6x2+19x+15.
If

we wish
so

to

factor a trinomial like


process.

Gx'^+lQ^+lS, we

may do
Thus,

by reversing the

6x2+19a:+15=6a:2+9a;+10x+15,

= 3x(2.c+3) + 5(2a:+3), = (2x+3)(3x+5).


The only difficulty in this method is in finding the two terms into which the middle term, 19a:, should be decomThis difficulty may be overcome in the following way posed.
:

{ax-\-b){cx-{-d)acx^-\-x{ad-\-bc)-\-bd.

of z,

Note that the product of the two terms ad and be, is the same as the product of x^, ac and the absolute term, bd.
15

in the coefficient of the coefficient

In the trinomial 6z2-|-i9x-|-15 above, the product of 6 and is 90 and the two factors of 90 whose sum is 19 are 9 and 10, which shows that the middle term, 19a;, should be decomposed into 9x4- 10a;.

FACTORING
Ex. 1. Factor 6.c2+13a:+6.
The produc*^ The two factors
.-.

139

of the coefficient of x* of 36

whose sum

is

13 are 4

and the absolute term and 9.

is

3&

6x*+13x+6 = 6x + 4a; + 9x+6,

= 2x(3x+2) + 3(3x + 2), = (3x+2)(2a;+3).


Ex. 2. Factor \2x^-\lx-5.
Here we require two factors are evidently 20 and 3.
.-.

of

60 whose sum

is

17, and thej

12x*-17x-5=12a;-20x+3x-5,

= 4a:(3x-5) + (3x-5), = (3a;-5)(4x+l).


BXBRCISn
Factor and verify
1.
:

71 (1-18. Oral)

a:2^4x+3.

2. 5.
8.

a2+lla4-30.

3.

4.
7.

a2-lla+18.
x2_i5a.^l4_

x2-14a:+48.
a'^b'^-^ab+Q.

10. 13.
16.

l-21x+38x2,
x2--4x-5.
2/2_4y_2i.

11. 14. 17.

x2-6x2/+8i/2.

a2-9a 22.
l-2a-loa2.
4x2 +8x2/ +32/2.

19.
.^.

2x2+5x+3.

20.
23.

^^^0^2.
25.

8x2+x-g.

3x2-35-2,

4x^+x 5.
1062-896
9.

26.

1562-196-8.

28.

29.

9x2 31xi/+12i/2

140
41.

ALGEBRA
Show
If

divisible

that the product of 6x2- 13a; -(-6 and 2x^-lx + 5 by 3x^ 5a; + 2 and tind the quotient.

is

42.

3x^-\-ax\'l

is

the product of two binomials with integral

coefficients, find all the different values that a

may

have.
of

43.

By

factoring, find the quotient

when the product

6a^ + lab-20b^ and


is

22a^-l^ab-Ub^

divided by
44. 45. 46.

40^ 4a6 356^


x'^-{-5xy-\-^y^-\-x-\-y.

Factor

Factor Za^ab2b--\-%a+4:b. Divide the product of x^-\-Zx+2 and a;* 1 by the product

of

x2+2a;+l and x^+x 2.


100.

Type

VII.

Sum and
x-\-y,

Difference of Cubes.

Divide x^-^y^ by

x4-y)x^

+y^{x*-xy-\-y*

and
.-.

x^y^ by xy.
a53+i/3=(a5+i/)(a52-aci/+i/2),

+^

and x^y^={xy){x^+xy-\-y^).

-^v ^v _ ^

Examine
factors.

carefully the signs in these

+^

+y "^'^y

It
is

is

thus seen, that

the

sum

of the cubes of two quantities

divisible

by their sum, and the difference of the

cubes

iif

divisible

by their difference. The quotient in each case consists of the square, product and square of the terms of the divisor., with the proper algebraic signs

Ex. 1. Factor 8a^+21b^.


Here
.'. .'.

8a* = (2a)

and 276 = (36),


'

the expression the first factor

may
is

be written (2a)' + (36)', 2a + 36 and the second is

.-.

(2a)-(2a)(36) + (36)* or 4a*-6a6 + 96>. 8a3 + 2768 = (2a + 36)(4a-6a6 + 96).

Ex.

2.

Factor

a^x^64y'^.

oa;-64y = (az)-(42/*),

= (a!e-4v*Xo*a; +

4<ftB|f

s+ !8y>

FACTORING
Ex. 3. Factor
This
cubes.
.-.

141

.r-?/8.

may

be expressed as the difference of two squares or of two

x8-2/ = (a;)-(?/3), or (a;*)-(2/)3, = {x^ + y^){x^-y^), or {x*-y^){x^ + x^y'^^y'^).


will

use,

Complete the factoring by each method and decide wliich you if you have the choice, as here.

Itt
If

ALGEbBA
one of the factors of a product be zero, the product
also be zero.

must
If If

what can we infer a6=0, it follows that either a=0 or 6=0. If (x 3)(x 4)=0, then either x 3=0 or a:-4=0. (x-2)(x2-7x+12)=2;3 9a;2+26j;-24, Since x^ 9x2+26x 24 must be equal to zero when x=2, for then one of its factors, x2, is zero.
1

the product of two numbers be zero,

.'.

If

we

substitute 2 for x,

we

see that this


.

is
.

true.

a;3-9x +

26x-24=2-9

= 8-36

+ 26 2-24, + 52 -24 = 0.

Conversely, when any expression becomes zero when x=a, then xa is a factor of it.
Substitute

x=3

3-6
a;

Divide

it

by
.-.

&x*-^\\x Q and it becomes 3-6 = 27-54 + 33-6 = 0, 3 is a factor of x' 6a;* + llx 6. x 3 and the other factor is x* 3x + 2. x-6x*+llx-6 = (x-3)(x*-3x+2), = (x-3)(x-2){x-l).
in x^
.

3+ll

If x-|-

is

a factor of an expression, the expression must be equal to


since

zero

when

Thus,

x= 1, for then x+ 1 = 0. x+1 is a factor of x' x* lOx 8, x+ 1


and complete the
is divisible

(-!)-(- l)-t0(-l)-8=- 1-1 + 10-8 = 0.


Divide by
factoring.

Any
zero)

expression

by

xa,

if

it

vanishes {becomes

when a
is

is substituted

for x.

This

called the factor theorem.


is

Show Show

that x a that x + a

is

a factor of x' 7ax*+ 10a*x 4a'. a factor of 5x' + 6x*o+ llxa*+10a*.

Ex.Factor x^ 9x+10.
If it

has a binomial factor

xl,
Testing for these factors
.-.

it must be of the form x2, x5 or x+10.

we

find that

x 2

is

a factor,

The

factoring

is

x-9x+10 = (x-2)(x* + 2x-5). complete as x* + 2x 5 has no simple factora^

FACTORING
102.
of
Special Case.

148

It

is

easy to see when


1

x 1
x,

is

a factor

any expression,
Thus, ifx=l

for

when

is

substituted for

the value

of the expression

becomes equal to the sum


x-2x-19x + 20,

of its coefficients.

=
.'.

-2 -19 +20 = 0,
factor
is

X 1

is

a factor. xa

Complete the factoring.


is

Similarly,

of

3x'

16a;*a

7a;a* + 20o',

since

3-16-7 + 20 = 0, 1-6 + 3 + 2 = 0.

and

a-b

a factor of a-6o*6 + 3o6* + 26, sine?

EXERCISE
Each
all

73

of these expressions

is

divisible

by x 1,

x2

or x

3.

Find

the factors of each and verify.


1.

z'-I0a;2+29a;-20.

2. 4,
6.

x^-Zx'^-\2x+\i:,

3.
5.

x3+5a;2 2x 24.

x-^x^-\-x+^.

2a;3-7x2+7x-2.

4x^-9x2- lOx+3.

Factor

^
(^

2x3-llx2+5x+4.

,8.

x'-2x2-x+2.
x3-19x+30.

x3-7x+6.

^^.
x+2 x+o
is is
is

11.
13.
14.

o3+a2-10a+8.
Show
Show
that that that

^f2^ a'^-'iab^-2b^.
a factor of a factor of

x' x* x+10. x'+7x%+9xa^ + 3a^


are the factors of
find a.
all

16.
16.
17.

Show
If

x+3, x+4 and


divisible
is

x 7

x' 37x 84.

x^ lOx+a
that

by x+2,

Show

ab

a factor of a^+'ia'^b+ab'^-%^, and find

the factors.
18.
is

Noting that x^- 2x 3=(x+l)(x 3), a factor of x* 4x3+2x2+4x 3.

show that x^ 2x

19.

Show

that

ab, bc and ca

are factors of

a(62_c2)+6(c2-a2)+c(a2-62).

and x2 are factors of x^ Sx^+ax+ft, find px3-3x2+gx-10 and qx^+2x^-\lx+p are both by x2, find p and q.
20.
If If

X 1

a and

b.

21.

divisible

144
103.

ALOBSBA
Equations Solved by Factoring.

We

have seen that

if

then

z 3 =

or

x 4=0.
is

Thus the equation (x 3)(x 4)=0


simple equations a: 3=0 and

equivalent to the two

But and if
.'.

if

x4=0. 23=0, x=3,


x

4=0,

a;=4,
are
3,

the roots of the equation

(x 3)(x 4) =

4.

The truth
If

of this

a;=3,

Ifa;

= 4,

may be .seen by substitution. (x-3)(x-4) = (3-3)(3-4) = 0x -1=0. (x-3)(x-4) = (4-3)(4-4)=lx = 0.


(a;-3)(x-4) = x*-7x+
12,

Since
the given equation

may

be written

x-7x+12 = 0.
equation which contains higher power is called a quadratic equation or an equation of the second degree. The preceding shows that if we wish to solve a quadratic equation we may do so by finding, by factoring, the simple
104.

Quadratic

Equation.

Any

the square of the

unknown and no

equations of which it is composed. The simple equations when solved will give the roots of the given quadratic equation.

Ex. 1.Solve
Factoring.
.-.

x2-6a:-7=0.
(x

7)(x+l) = 0,

Verification

or x+i=0, x=7 or 1. if x= 7, x*-6x-7 = 49-42-7 = 0, if x=-l, x*-6x-7= 1+ 6-7 = 0.

X7=

Ex. 2.Solve
previous problem

3x^-\-lx=6.

Transpose the 6 so aa to make the right-hand side zero, as in the

3x-f7x-6 = 0,
.-.

(3x-2)(x-l-3) = 0, 3x-2 = or x + 3 = 0, x=f or ^3.

Verify both of these roots.

FACTORING
Ex.
3.

145

Form
is

the equation whose roots are 2 and 5.


is

The required equation


two simple equations

at once seen to be a combination of the

a;--2

and
x

x-t-5

= 9,

and therefore
or

{x2){x-\ 5) = 0,

+ 3a; 10 = 0.

Ex.

4.

is

If

is

a root of the equation

x3

+ 3a;2-16x+12 = 0,
1
is

find the other roots.


Since 2

a root, then x

a factor of x^-\-^x'^
is

\Gx-\-\'i

and

the other factor, found by division,


.-.

x^-f-Sx 6.

.'.

the other

+ 3a;-16x+12 = (a;-2)(a;-l)(x+6) = 0, X 2 = or X 1 = or x+6 = 0, x=2 or 1 or 6. roots are 1 and 6.


a;

BXBIROISEI 74

(1-16,

Oral)
is

To what equations
aquivalent
i.

of

the

first

degree

each of

the following

(x-l)(x-2)=0.

146
27. 28.
If 2 is 3

ALQEBitA
root of j-'- 19j-+30 =0, find the other roots.

Solve

z3 6x2+llx 6=0 and ix^ 12x=^4-llx-3=0


is

(Note that the sum of the coefficients


29.
30.
P'ind

zero.)

The sum The sum


them.

of a
of

number and

its

square

is

42.

Find the number.


is

the squares of two consecutive numbers

61.

31.

The

sides of a right-angled triangle are x, x-\-

and x+2. Find

x.

105.
is

Notes concerning Factoring.

The subject

of factoring

one of the important parts of algebra, as it enters into so many other processes. We have already had examples of its use in solving equations and in performing operations on
fractions.

to

In the preceding exercises, in this Chapter, the expressions In the be factored have been classified for the pupil. practical use of factoring, however, he must determine for
himself the particular

method

This
to

is

usually

done by

to be used. determining

the

type

or
in

form
the

which

the

expression

belongs.

The examples

review exercise which follows will give the required practice. The types which have been discussed in this Chapter are
here collected for reference
I.
:

ax-\-a(j.

(Common

factor in every term.)

II.

ax-\-ay-\-hx-\-hy.

(Factored by grouping.)

III.

x^2xy-\-y'^.

(Complete squares.)

IV.

a^h^.

(Difference of two squares.)

V.
VI.'

x*'-\-x^y^+y'^.
ax'^-\-bx-\-c.

(Incomplete squares.)
(Trinomials.)
or difference of cubes.)

VII.
VIII.

X^

y^.

(Sum

Factored by the factor theorem.

FACTORING
BXERCISE:
1.

147

76 (Review of Chapter XI)

State the squares of a-\-b, a b,

x3y, 2x\, 3x 5.

5a-\ 2b

!ix4:y,7aZ,a*l,a-\
2. 3.

State the squares of o + 6 + c, x-\-y z,

ab c.
a(o 6 + c), {x-\-l)(x-\^

Writedown the products

of

x(a b),

[x-3)(x-5), (2a;-3)(2x+3), (ax-6)(ax+7).

Use short methods to


4.* {x + a + b){x
5. 6.
7.

find, in the simplest form, the

value of

+ a-b) + {x a-b){x-a + b).

+ x+l)(x* + x-l)-(x*-a;+l)(x*-x-l). + b + c)* + {a + b-cy + {a-b + c)^ + {a-b-c)K (2a+36-c) + (3a + 6-2c) + (a-26-|-3c)*.
(x* (a

8.

9999-9998*.

9.

57432-4257*.
(a
:

10.

503x497-502x498.

11.

+ 99)-(a+98)2.

_^.

Find the simplest factors and verify 12-29


1^12.

^^

x*-x-42.
x'

^3.

^^\.
y-^Q.
i.,^8.

4x.

X* a x
1

+ ax X.

+ 5x-4x-20. 4tx*.

^^20.
22. 24. 26. 28.

5a* + 32a + 9.

x*

(x-3)* + (x-3)(x + 4).


l

abc*

+ 2a6-a-6*. + a*cd + abd^ + b^cd.

>80.
32. 34.
36.
38.

4(x-2)*-x+2.
(a+26-3c)*-(3a + 26-c)
108a-500.

x-7x*-18.
X--X2/-1322/*.
x^

40.
42.
<i4.

+ y^ + 3xy{x + y).

o*-46-3a-66.
a* + 2ab + b*-\-ac

+ bc.

45.

o*-2o6 + 6*-a+6. L 2

148
46. 48.
x*-\-x*

ALGEBRA
+ x-y* y*-y.
47.

ia*-25b* + 2a + 5b.
x*

49.
61. 63. 65.
is

,60.

+ y*-18x*y*.

- a*b' - ab* + ab*. 8(a + 6)-(2a-6). a*-a-9-2a6 + 6*+6a.


a*b

^62.
64.

x-Ux*+7x+3.
xc-c + x*-l.

3a-5a*-8a+10.
a'-a'' + 8a-8.

66.

Show

that a b + c

a factor of

(2a-36 + 4c) + (2o-6).


Factor 4a 37a*6* + 96, (1) by cross multiplication, (2) by 67. completing the square of 2o* 36*, (3) by completing the square of
2a

+ 36.
Without multiplication show that
(a;-4a;+3)(z-12x + 35) = (x*-6x + 5)(x-10x + 21).

68.

69. 60. 61.

Make a diagram
Find the factors when x = 2.

to

show the square

of a

+ ft-f-c + d.
being given
that
it

Factor (a-b)(b*-c*)-{b-c)(a*-b*).
of 6x'

7x* 16x-f- 12,

vanishes
62.

Find four factors of (x*-5x)+ 10(x* 5x)-|-24 and

of

(x*-6)*-4x(x*-6)-5x.
63.

Use the factor theorem

to solve

x*-31x + 30 =
64.
If
is

and x-43x + 42x=0.


by
6,

two numbers

differ

show that the


of

difference of their

squares
65.
x*

equal to six times their sum.


x*

(c a)x ca
66.
67.

Find the quotient when the product is divided by x' + (a 6)x a6.
Multiply
a*-b*-c + 2b by +^ +

{b c)x bc

and

a+bc

If x*

+ x^

-\-

ax* -\- bx 3
factor.

is

divisible

by

and x+3,

find a

and

and the remaining


68.

Factor 2x* ax + bx ab a*.

69.
in

Express a*b*-^-c*d* a*c* b*d* as the difference of two squares


different ways.

two

Factor a*-{-b* + c*2a*b* 2b*c* 2c*a* 70. square of a* b* c*.


71.

by

completing

the

Find four factors of {a*-b* c* + d*)* 4{ad-bc)*.

CHAPTER

XII
{continued)

SIMULTANEOUS EQUATIONS
106.

In Chapter VII. the solution of simple examples two unknowns has been considered. The method there followed was to make the coefficients of one of the unknowns numerically equal by multiplication, and then that unknown was eliminated by addition or
of equations in

subtraction.

Other methods of eliminating one of the unknowns are


useful in certain cases.
107.

Elimination by Substitution.

Ex. Solve

x2y=

2,

(1)

5z-^ly=18.
Froin(l), x = 2 + 2y. Substituting this value of x in (2),
5(2
.-.

(2)
(3)

+ 22/) + 72/ = 78,


172/

10+102/ + 72/ = 78,

= 68,

Substituting

2/

= 4in(3),

2/= 4. a;=10.

Here we eliminated x by finding the value of z in terms from (1) and substituting that value in (2). We thus obtained an equation which contained only the unknown y. This is called the method of elimination by substitution.
of y

We might
Thus from

take the value of x from


(2),

(2)
.'.

and substitute

in (1).

5x = 18-ly,
(1),

x=
2y

^^T^^
2.

'

Substituting in

^^~S^
140

Complete the solution and verify the

roots,

150

ALOE BRA

The value of y might have been found from either equation and substituted in the other.
Thus from
(1),

2y=-x-2,
(2),

:.

j/=
=
78.

_,

Substituting in

5x

+
(2)

"^^^s^^^

Complete the solution.


Solve also by finding y from
If

and substituting
it

in (1).

this problem, the first

the four solutions be compared is the simplest.


solving

will

be seen that, in

In

should

equations with two unknowns, the pupil examine them carefully and choose the unknown
will

which he thinks
108.

be the simpler to deal with.

Elimination by Comparison.

E^-Solve

2a;- 3?/=

7,

(1)
(2)

3x-\-5y=39.

+ 3y _
2

39-52/
3
'

.-.

3(7

+ 32/) =

2(39-51/).

Complete the solution and verify

theNi;Dots.

Here we effected the ehmination


values of x from the two equations.

of

xM^
of

comparing the

This

is

called the

method

of elimination by coln^arison.

might have compared the values Solve it that way. the two equations.
109.

We

y obtained from

Three

Methods
of

of

Elimination.

We

have

illustrated

three

methods

elimination,

by addition or subtraction,

by substitution and by comparison. When no particular method is specified, the pupil is advised to use the first method as no fractions appear in it.

SIMULTANEOUS EQUATIONS
EXERCISE
State the value of eaoh
1.

16i

78
in

(1-6,

Oral)

unknown
2.

terms of the other in


3.
6.

4,

x+y^5. Zxy^%.

X y=3.
2a;+3;/=12
:

x+2y-ll.
5a; 4?/=
19.

5.

Solve by substitution and verify


7

x+2y=\%, 2x+oy=i\.

/S.
11.

Zx+ y^

7,

4x+3y=ll.
3a; 4i/=10,
12.

2z t/=19, 5a;-32/=46.
8a;+ 9?/=
7,

10.

2z 32/=14,

\y

x-5y=
x+3y=
x-\-by--

0.

u^

2x+&y=\\.
:

10x+21?/=12.

Solve by comparison and verify


13.
10,
14.

14.

2x-rJ/= 26,
=

15.

3a;+4!/=10,

3x !/=

14.

4a;-3i/=

5.

Solve by any method and verify


16.

\x-\y=2.

,17.

Zx^2y,

18.

2/=

-\x-\-&.

hx=ly-2.
19.
20.

fa;-

3'

^3
sy.
11|.

2^

3y~x=x,

!='
22.
3a;

a;+

3x-l

1.

+ 2 y+7 .11

2y

+ ?i^ =

10.

23
24.

x-52/+3=2a;-82/+3=7a;- lOy+16.
(x-l)(2/-2)-(y-3)(x+l) = 17,
(x-3)(2/-5)-(x-5)(2/-3) = -22.

25. 26.

lx+-21y+-52=01a;+01?/+3-0.

x+5 = 3(?/-3),
4x-

5x- 4
11

2x
9

19.

27.

-3y
4

_ lx-2y- 23
of

2x- 9y

28.

If

the

sum

two numbers
is

is

of the greater

number, the

diSerence of the numbers

how many times

the less

162
110.

ALGEBRA
Eqiy^iops with three

Un]^wns.
(1)
(2) (3)

Ex.Solve

2x-\-3y4z=12,

3x- y+22=15,
4a;+

*
'^

y-3z=19.

This system of equations differs from the preceding by containing three unknown quantities.
If

we can obtain from


the

these three equations, two equations

same two unknowns, the solution can be efiFected by preceding methods. How can we obtain from (1) and (2) an equation containing X and z only ? How can we obtain another equation from (2) and (3) containing x and z only ? Perform these two eliminations and find x and z from the
containing
resulting equations.

Now find y by substituting in any one of the given equations and verify by showing that the values you have found for x, y and z will satisfy all of the given equations.
The
solution might be written in the following form
(1)
:

Eliminate y from

and

(2),

SIMULTANEOUS EQUATIONS

163

EXERCISE
1.

77

(1-4,

Oral)

What
(1)

operation will eliminate both x and

a;+?/-|-z=35

(1)

y from
of

and (2)?

What

is

the value of 2?

x+yl
2.

x-\-yz=25 x-y-\-z=\5

(2)
(3)
?

What
is

What
3.
is

operation will eliminate both y and z from (2) and (3) the value of a; ? of yz ?

How

can you eliminate both x and


t

from

(1)

and

(3)

What

the value oiy


4.

of x-\-z

In No. 5 below, which letter


?

is

simplest to eliminate from two


?

pairs of the equations

Which

in

No. 6

Which

in

No. 7

Solve and verify


6.

^\

x+2y+ x+Zy+

3z=16, 4z=24, a; -t-42/-|- 102=41.

6.

2a: i/+3z=-

7,

3a;+s/ 42= 7> 6x !/+52=21.

7.

4x-3i/+ 2=10,

8v
-'

x+y
xy-{-

2=16,

^
9.

6x 5?/+22=17, a;+ i/+ 2= 8.


x+22/+3z=32,
4a; 52/-l-6z=27,

z=

4,

a;+y+22=22.
10.

a;+?/=25,

7x+8y-92=14.

2/+z=75, 2+a;=70.

.^
J 13.
'

a;+22/=12,

^.

3(2-1)
4(2/+x)

=^2{y-l),

32/+42=

2,

5z-2x=-21.
3+x=5+4!/,
2+x=3i/, 7?/=z+2.
2

=92 4, 7(5x-32)=2y -9.


i2/+ Jz =36, lx+^\y+.}QZ =10, ira;+ i2/+Az =43.
Ja;-f

^. ^
\

^^^^

+ 3+4-3 + 4 + 5-4 + 5 + 6-^xy-\-z=9,


ax^hx-\-c
c.

'r
and
x+y-\-z-\-w=l2,

16.* If x-\-2y=25, ?/+32=55, z+4a:=35, find the value of x+y-\-z.


17.
find
i.

If

2x-\-y=8,

1/ 42=5

18.

If

is

when x=l, 8 when

a;=2, and 10

when

a;=3,

find a, 6

and

V.f.if

154
19.
If

ALGEBRA
ax^+bx+c
find its value
is 9 when x=l, when x = 3.

3 when x= 1,

18

when

z=2,
20.

Determine three numbers whose sum is 9, such that the sum twice the second and three times the third is 22, and the sum of the first, four times the second and nine times the third is 58.
of the
first,

21.

If

+ 6=12,

6+c=15, c+d=l9,

find

a+d.

111.
first

Two equations of the Special Forms of Equations. degree in x and y will usually determine the values of
:

X and y. Consider the following sets of equations


(1)

2x-3^=10,
4j-+5j/=42.
(I),
if

(2)

2x-3y=l0,
4x-6.y==20.

(3)

2x-3y=l0,
4x-6!/ = 30.

In

the two equations are solved in the usual

way

we

find

that

x=8, y=2
of x

will

satisfy both of the equations,


will satisfy

and no other values

and y

them.

We
is,

therefore say that these equations are determinate, that

they determine the values of x and y. In this ca.se the second equation can not be deduced from We therefore say the first, nor the first from the second.
that the equations are independent.

In

(2),

the second equation


2.

by multiplying by

may be deduced from the first These equations are dependent and

not independent as in (1). Any number of values of x and y will satisfy both equation^ because any values which will satisfy the first will also satisfy These equations are therefore indeterminate. the second. In (3), if the first equation is true, the second can not be
true.

They are therefore said to be inconsistent or impossible, and no values of x and y can be found to satisf}' both of them. We thus see that tioo equations in two unknovms can have a definite solution only when the equMions are independent and consistent. Zx-\-2y z= 5, In this set of equations, the third may be 4x y-\-Zz = 2Q, obtained by adding the other two. They are
therefore dependent equations

and consequently

lx-\-

y + 2z = 25.

indeterminate

SIMULTANEOUS EQUATIONS

156

EXERCISE
1.

78

Find three pairs of values of x and y which satisfy the equation

2x-Zy=\2.
2.

Solve 2x+3?/=13, bxy=24^.

Is it possible that

2x+3!/=13,

5xy=24: and 4x+5?/=19 can be


3.

true at the

same time

What
8a;

is

peculiar about the equations i^x-\-y=\l, %x-\-2y--^5


?

Also about
4.

+12^ =60, 6x+9i/=45

Show

that the equations

x+z+4=32/, 2x+z=2y+%, 2x+t/=10,


are indeterminate.
5.
If

2=5, solve the equations


of the simultaneous equations

Find two solutions

x+z/+2=10, ^x2yz=l.
For what values
6.*
of a will the following sets of equations be consistent
7.

3x y= 5, x+2y=25,

3x+2/=
I0x-'iy=

7,
2,

8.

9a; ay=Q,

3x-

y=2,

x+4y=
9.

a.

3x+ay=ll.

ox^y=^.

Show

that these equations are inconsistent

2x+3y-3z=20, 3x+7y2z=5, x+2yz=6.


112.
Special Fractional Equations.

Ex.Solve

--- =
y
2

11,

(1)

+ - = 21. y

(2)

Here we could obtain the solution in the usual way by removing See if you the fractional forms, by multiplying each equation by xy. can complete the solution by this method. It is simpler, however, to eliminate v from the equations as they dtand.

166

ALOE BRA
(2)

Thus, multiplying
'^
1

by
95,

and adding
95x

=
8

19,

Substitute x

- in (1)

and 15

11.

y
2/

2.

The

solution, therefore,

is

x = l, y-

Verify this result.

EXERCISE
Solve and verify
:

79

2,

^ X

y
1/

29,

^
5

24_21_j

y 6
2.

1^

X
X
!

y
2/

7_9 = 3. 1 - -

yi
*

_4
y
101.

+ 2t/ =

15,

+ ?=!..
1/

ii+i
y

--3y = 6.

?-? = 2.
y
2

^-1 =
3y5x=xy, 2y+3x=2exy.
9.

17.

x^22/

= i^ + l =
2x^3y

5^3^13575^3^
X
J/

J/

10.

3x

+?
y

12x

+-+
y
u

14

= - - 2x y
82

14.

11.

4.^1 + '
X

122

+ 17.

?/

Problems leading 113. Simultaneous to Equations. In Chapter VII. we have had illustrations of problems which were solved by using equations of two unknowns. We now give some further examples on special subjects which ^ere not then considered.

SIMULTANEOUS EQUATIONS
.

167

The number 47 might be written 4 10 + 7. What is the sum of the number ? What number would be formed by reversing the digits ? What is the sum of the number and the reversed number ? What is the sum of the digits of the reversed number ?
digits of this

Ex.

1.

the digits are reversed.

number has two digits. If 18 is added to it The sum of the two numbers is 88.
digit,

Find the number.


Let a;=the units digit and y the tens
.".

the

number
:.

and the reversed number

=10y + x, =lOx+y.
10i/ + a;+18=10x+2/, + a;+10x + = 88. 9a; 9t/= 18 or x = 2, lla;+ll2/ = 88 or x + = 8. z=5, y 3.
2/
1/

(1) (2)

and
Simplifying
(1), (2),

102/

2/

Solving
.'.

the required
:

number

is

35.

Verification

35+18 = 53,

35 + 53 = 88.

Ex. 2. If 4 be added to the numerator of a fraction and 3 to the denominator, the fraction becomes |. If 2 had been subtracted from the numerator and 5 from the denominator the result would have been ^. Find the
fraction.

Let

the fraction,

x+4
2/+3

168

ALGEBRA
better way, however, would be to represent the larger
Ix,

A
by

and then the smaller would be

6.r.

By

doing so we

get rid of the use of fractions.

Ex.
2,

3.

The
S3a;

incomes of

and

are in the ratio of 3 to

and

their expenses in the ratio of 5 to 3.

Each saves

a year.
Let
Let
Solving
.-.

Find their incomes and expenses.

= yl's

income, then 82x=B's income.

$5y = A's expenses, then $3y = B's expenses.


.-.

3x-52/ = 400 and 2x-3?/ = 400.


X = 800 and y = 400.

^'s income

= $3x= $2400

and B's

.-.

^'s expenses = $5i/= $2000 and B's

= $1600. = $1200.

Note.

In solving the problems

in the exercise following, the pupil

he frequently has the choice of using one, two or more unknowns. Except in special cases, he is advised to use as small a number of unknowns as possible. In each case the results should be
will find that

verified.

BXERCISB
1.
If

80
10 ^>oys, earn $96 in a

10

men and

4 boys, or 7

men and

day, find a man's wages per day.


2.
10.

Two numbers are in What are the numbers


The sum
of three

the ratio of 5 to 7 and their difiference


?

ia

3.
is

numbers

is

370.

The sum of the


first is

first

two

70 more than the third, and six times the Find the numbers. the third.
4.

equal to four times

Find three numbers such that the results

of

adding them two

at a time are 29, 33, 36.


5.

by

7,

the second by 4 and the third by 2 will

Divide 429 into three parts so that the quotient of the all be equal.

first

6. A workman can save $200 a year. He goes to another town where his wages are 10% greater and his expenses are 5% less, and he can now save $310 a year. What are his wages now ?
7.
If

The denominator
is

of a fraction exceeds the numerator by 3. subtracted from each term, the fraction reduces to |. Find

the fraction.

SIMULTANEOUS EQUATIONS

15

8. Divide 120 into three parts, so that J of the first part is greater than the second by 5 and J of the second part is greater than the third by 10.

9.

If 6

earn $57 in

men and 2 boys earn $56 in 2 days and 7 men and 5 boys J days, how long will it take 3 men and 4 boys to earn $60 ?
is

>

^10.

A number between 10 and 100 and if 45 be subtracted from it, the number. ^"V

8 times the

sum

of its digits,

digits are reversed.

Find the

^^1. The difference of the two digits of a number is 4. The sum the number and the reversed number is 110. Find the number. 7iJ
yf2.

of

The sum

of the

two

digits of a

number

added to the number the

digits are reversed.

is 14, and when 18 Find the number,

is

^f

When 1 is added to both terms of a fraction the result is \. If had been subtracted from the denominator only the result would have been \. Find the fraction.

^/

Ai
oithe

A number

consists of

digits be reversed, the

than twice the original

digits whose sum is 11. number thus obtained is number. What is the number ?

two

If

the order

greater
i-

by

p^
Snow
16.

The

difference

between the
is

digits of a

that the difference between the


digits

number less than 100 is 6. number and the number formed


two numbers

by reversing the

always 54.
is

The sum

of the reciprocals of
first is

^.

Six times

the reciprocal of the


the second

greater than five times the reciprocal of

by

\.

'

Find the numbers.

(The reciprocal of x

is

-)

X
Divide 150 into two parts such that the quotient obtained by dividing the greater by the less is 3 and the remainder is 2.
18.

n?/

tea worth 30c. per

must
19.

wish to obtain 100 lb. of tea worth 34c. per lb. by mixing lb. with tea worth 40c. per lb. How much of each take ?
I

Three pounds of tea and 10


Find the price of each per
lb.

of sugar cost

$2-40.

If

tea

is

increased 10^/ in price and sugar decreased


$2-52.

10%, they would cost

20.
is

Two numbers
I

are

in

the ratio

of
is

7 to

5.

What

quotient

obtained when three times their

sum

divided by six times their

difference

100
21.

ALGEBRA
Show
that the

sum

of

any number
is

of

two

digits

and the number

formed by reversing the difference is divisible by

digits
9.

always divisible by 11 and that the

^2^ A number
and
/

has three digits, the middle one being

be added the digits are reversed.


five

The
is

difference

0. If 396 between the number


is

times the

sum

of the digits

257.

What

the

number

Divide 126 into four parts, so that if 2 be added to the first, 2~be subtracted from the second, the third be multiplied by 2, and the fourth be divided b}^ 2, the results will all be equal. (Let the result=x.)
23,
^24.

There are three numbers such that when each

is

t<Ihe^sum of the remaining

two the

results are 44, 42, 39.

added to twice Find the

numbers.
25? The sum of the three digits of a number is 12. If the units and tens digits be interchanged the number is increased by 36, ariS if Find the hundreds and units be interchanged it is increased by 198. the number.
,-

Find three numbers such that the first with \ of the sum of / 26. the other two, the second with J of the others, and the third with \ of tb%others, shall each be 25.

work can be done by A working 6 days and B\2\ working 8 days and B 18 days. In what time could each of them complete it alone ?
27.
piece of

days, or by

28.

2 to 4 to
^

3,
5.

the second to the third as 3 to

Divide 84 into four parts, so that the first is to the second as 4, and the third to the fourth as

29.

^oi the

first

Of what three numbers is it true that the sum of the reciprocals and second is |, of the first and third is \ and of the second
is

and third
30.
order.

Two numbers
The sum
15

consist of the

of the

numbers

is

same three digits but in inverted 1029. The sum of the digits of
is 5.

each

is

and the

difference of the units digits

Find the numbers.

.31.

Iflistance

stream flows at 2 miles per hour. A man rows a certain in 5 hours and returns in 1| hours. How many miles per hour could he row in still water ?

up stream

32.

balance at 150 a head.


first

rancher sold 50 head of horses, part at $125 a head and the After spending $50 he was able to make the payment of J of the purchase price of 1200 acres of land at $18

per acre.

How many

horses did he

sell

at $125 a head

SIMULTANEOUS EQUATIONS
33.

161
digit,

A number
2.

consists of a units digit

and a tenths
is

the units

digit being the greater

by

1.

The sum

of the digits

less

than twice

the

number by

Find the number.

34.

grocer spent S120 in buying tea at 60c. a

of coffee.
at

He

sold the tea at an


of

advance

of

an advance

20%.

The

total selling

and 100 lb. and the coffee price was $148. Find the
lb.,

25% on

cost

number
35.
is

of lb. of tea purchased.-

When
\.

is

equal to

Show
if

subtracted from each term of a fraction the result that the result would have

been the same

had been subtracted from


geometry that the two

the numerator only.


36.
It
is

shown

in

tangents drawn to a circle from a point are


equal.

Thus, in the figure


(7.4

AD=AE,

etc.
z.

If

AB=15, BC=U,
37.
19, If

= 13,

find x,

y and

the sides of a triangle are 10, 15 and


will

where
?

the inscribed

circle

touch the

sides

(See figure of preceding example.)

38.
in

If A can do a piece of work in m days and what time can they do it working together ? If a; is the number of days required,

can do

it

in

n days,

then X

111 =
m
|-

m-\-n

Use the preceding together can do a piece


39.

separately in
(1)

A and B working work which could be done by A and B the following number of days
result to find in

what time
:

of

in 10, fi in 15.

(2)

in 20,

in 5.

(3)

in J, fi in IJ.

BXBRCISE
Solve and verify
:

81

(Review

of

Chapter XII)

/^~x

y. 7x-8i/=10,
^
Zx-2y=\Q.

2.

4a;+72/=-l, 3a;- y = 3.

-^

73x+2/=

76,

z+732/=147.

y
7

25
2y

32/

+ 4x:=2xy.

ALGHBEA

^
^'
3:r-^=6, /fit 5 o^ "4y
"'

/fg)

?i _ 1 - 2 y + 2- 2y+l~'^'
""3"
3(a;

- ^T~* + y) = 8.
^-5

^' =

12.

^
/^/
__

^-3 .rffc^^-1 ap^ x+j,+^=33. '


2x+3y2

24-6
= 5,

x+ y+ z=-3. 2 = 0, 3x4- y + 62 = 0.
X + 22/+ y

\_y^ 3x-4y + 22=l, 4x6y + 5z = 7.

a:+y + 2 = 24.

17

18.

_ x+2 ~ ~60"' (x-li)(y-li) = x2/-5. ix-|y+z+l = 3(x-i/) + 52 + 4=x+6i/-22-9 = 0.


~3
3/-f5 12"

X 1

^
to

2^^102+ix^g
x + y = 5,
?/-)-z

20.

= 3, 2 + 1^= 1, X w = 3.
(Add
and
subtract
th.

31x4-282/ = 146, 28x4- 31y= 149. 21. equationa and remove common factors.)
22.
23.

97x-592/ = 329, 59x-972/=139.

What
?

values of x and y will

make

X 2?y ^

and

X 4* V

=--

each equal

x-10
24

Show

that x + y + z=\2, 3x4-

4?/

-52= -22,

10x4- 12?/-6z = 4,

are indeterminate.

Divide unity into two parts so that 18 times the

first

part

mav

e^eed
26,

12 times the second

by

13.
its

A number
18 be
?

and
the

if

of two digits is four times the sum of added to the number the digits are reversed.

digits,
is

What

number

O^aT^.^

Wh&i> the The tens digit of a number is twice tiio units digit oumbr i^ dividend by tbAmjDo oi the digits what muBt thequoUeat be ^
t

SIMULTANEOUS EQUATIONS
28.

16S

exceeds

Find a fraction equal % of the numerator by

to
8.

such that

of the

denominator

/29.
if

'

Two

persons

who

iiTliey

walk in opposite directions, but are not together they walk in the same direction. What are their rates ?
JO.
'

are 30 miles apart are together after 5 hours for 15 hours

A'a age

is

ago

was twice

as old as B.

equal to the combined ages of B and C. Ten years Show that 10 years hente A will be twice

as old as C.

Qi^ A bill of $19-50 was paid in half-dollars and quarters and four times the number of quarters exceeded twice the number of half-dollars by 12. How many of each were used ?
f

32j
lb.

If 5 lb. of tea

lO%''^in price

and 8 lb. of coffee cost $580, and coffee advances and tea 15% and they now cost $6o3, find the prices

per

of each before the advance.

refceive

sum at 4% and another sum at 6% and $42 interest. If the sums had been interchanged I would have received $850 more. What were the sums ?
/dS.y I invest a certain

If each side of a rectangle is increased by 5 feet the area is increased by 275 square feet. If each side is decreased by 5 feet the Show that the sides can not area is decreased by 225 square feet. be determined from these conditions.

35. 36.
40c. to

Solve

^-^i^^=^ = 4x-^=l.
mix tea worth
601b.

grocer wishes to
?

30c. a lb.
36c.

with tea worth

make a mixture weighing


must he use
If

worth

lb.

How many

lb.

of each

37.

thata = 3,

3x* 2a;-f 5 = ax* + 6a; + c, when x=\ 6= -2, c = 5.

or x='2 or

x=Z, show
3.

The tens digit of a number exceeds the units digit by 38. how much is the number decreased by inverting the digits ?

By

A train 39. miles per hour


hour.

is

27 minutes late
is
?

when

it

makes
it

and

42 minutes late when

its usual trip at 28 runs at 27 miles per

What

is

the distance

40.
or

by

A piece of work can be done by A working 6 days and B 16 A working 9 days and iS 14 days. How long would it take
it ?

days each

alone to do
41.

A number has

of the hundreds,

^f 396 be subtracted the digits a

three digits, the units being J of the tens versed.

and J Find

the number.

y^

v_i

Wo^

184

ALGEBRA

When the greater of two numbers is divided by the less the 42. quotient is 5 and the remainder is 2. When 12 times the less is divided by the greater the quotient is 2 and the remainder is 12. Find the
numbers.
le

sum

Find four numbers such that when each of the remaining three, the results are

is

46,

added to twice 43, 41 and 38

respectively.

H|^M^''^*-'^>

44.

If the
is

sum

of

difference

b times the smaller,

two numbers is a times the greater and the show that a 6 + a6 = 2.

CHAPTER

XIII

GEOMETRICAL REPRESENTATION OF NUMBER


114.

Function

of

x.

The value
x.

of

the expression 3a;

depends upon the vahie of


Thus, when

x=

4, 3, 2, 1,

3x-2=10,

7, 4, 1,

-3, 4, 0, 1,-2, -2, -5, -8, -11, -14.

When
Thus,

the value of an expression depends


is

upon the value

of X, the expression

called a function of x.

2x 3,
is

What
-1,

5x, \x-\-\, are functions of x. the value of each of these functions

when x = 2,

1,

0,

-2?

Instead of repeating the words " the expression " or " the function," we might represent the function by a symbol, say y.
Thus,
If
7/

-1,

= 5a;+ 1, when x= 1, = 6 x=3, y= 16. if = |x+4, what are the values of y when x has the -8 ?
2/ 3/
;

values

6, 3, 0,

115. Variables and Constants. A quantity that has nor always the same value is called a variable, while a quantity whose value does not change is called a constant.

variables, while the

Thus, the population of a city and tha height of the barometer are number of days in a week and the length denoted

by an inch are constants.


Note. To do the work of this chapter properly, pupils should be supplied with squared paper. Paper ruled in tenths or eighths of an

mch

will

be found most satisfactory.


165

106

ALOE BRA

Two variables may be so 116. Connected Variables. connected that for every change in the value of one there is a corresponding change in the value of the other.
Thus, value of
if

y.

y = 2x-\-5, for each value of x there is a corresponding Here x and y are variable quantities, but 5 is a constant.

20 and 21 we have shown how the changes in two may be represented by a diagram. Those diagrams show that for each variation in time there is a corresponding variation in temperature.
In
arts.

variable quantities

A line so drawn as to exhibit the nature of Graph. 117. the relation of two variables is called a graph.
118.
iji

Arithmetical Graphs.

The

solution of

many problems
:

arithmetic might be represented graphically as follows

Ex.
mile.

Thb passenger rate on a railway Lg 3 cents per Represent graphically the amount charged for any number of miles from 1 to 10.
1.

In the diagram each unit on the horizontal line OX represents 1 mile and each unit on the vertical line represents

OY

3 cents.
;
1

The point A shows that the


4 miles
is

Hj 12

12 cents.

What does

cost for the point

show ? The point C ? Read from the figurj the


5 miles, 9 miles.

cost for 2

miles,

far can 1 travel for 9 cents, 21 cents, 27 cents T

How

OEOMETRICAL REPRESENTATION OF NUMBER


peculiarity do
interest

167

What Place a ruler on the points marked A, B, C, D, E, F. you notice ? Make a similar diagram, on squared paper, which will give the
4%
1

on $200 at ber of years from


your diagram
will

for
7.

any num-

to

not

much
of $1.

unit on

space vertically, OY to represent $4 instead


in-

So that occupy too suppose each

CJ 8

C
for 3 years,
is
?

Read from your diagram the


terest for

5 years.

In

how many

years $24, $4, $12, $44

the interest $8,

108

ALOEBEA

They meet at at about 3.15 p.m. and at a distance from P ol about 26 miles. How far are they apart at the end of each hour from 1 1 to 5 ? Wken
did

reach

We

might solve the problem

algebraically.

Suppose they are together x hours after 9 a.m. Then A has travelled x hours at 5 miles per hour, and B X 4 hours at 15 miles per
hour,
.-.

5(x-l)+15(x-4) = 60.

Solve and compare with the

io

11

12

resvdts obtained graphically.

119.

Graphical Results only Approximate.

The

last

problem

that the results obtained by graphical methods are approximate only. When the problem is solved algebraically we find that they will meet 26 J miles from P
illustrates

the fact

at 3.15 P.M.

GEGMETEWAL REPRESENTATION OF NUMBER


6.
If

169

8 kilometres equal 5 miles, construct a graph which will

enable you to change into miles any number of kilometres up to 20. Read the approximate number of miles in 3, 5, 11, 13, 16, 19, 20
kilometres.
6.*

A starts from Toronto at 12 miles per hour to motor to Hamilton,


An
hour and a half later

a distance of 40 miles.

starts

from Hamilton

to drive to Toronto at 8 miles per hour.

By means

of a graph, find

when and where they


7.

will meet.

The distance from A

to

B E

is

10 miles,

to

8,

to Z> 8, i) to

10 miles.

mail train, which leaves

at 10 a.m., arrives at

at

10.24,

at 10.48,

at 11.12,

at 11.40.

An

express train leaves


If the

and without stopping reaches A at 11.28. stops 4 minutes at each station, show graphically
at 10.24
(a)
(6)
(c)

mail train

when and

at

what point they pass each


and at

other,
11.12,

how

far they are apart at 10.30

when

the express passes through B.

120.

The
call

the axis of

We will
OX,
X,

In the diagram the and Y the axis of y. the measurement along


Axes.
-7-

line

OX

is

called

and along OY,

y.

For the point A the x measureis 3, and the y measurement is 1. What are the x and y measurements for the points B, C, D, El. Examine the x and y measurements for each point marked on the line OP. What equation connects the values of x and y for each point on the line OP ? For each point on OQ, OR,

ment

OSl
of
is the graph of the equation y=2x, OQ of i/=4x, OR y=x, and OS of y'=lx. The X and y measurements of every point on the Une OP satisfy the equation y2x. This equation is not satisfied

OP

170

ALGEBRA
line

by the x and y of any point not on the by the x and y of the points A, C, D,
121.
satisfied

OP.

Is

it

satisfied

Equation

of

OP

by the values and by no other


of the line
is

of x

equation y2x is each point on the line points, the equation y=2x is called
Line.

Since

the

and y

for

the equation

OP.
OQ
?

What
122.

the equation of

of Oi?

of

OS

To Construct the Graph

of a given

Equation,

Ex.

Construct the

graph of y=lx.
Here when

GEOMETRICAL REPRESENTATION OF NUMBER

171

172
to tlic

ALGEBRA
or

OXy

two points and transferring the compasses to the line any other line, and reading off the distance. Plot the points (3, 5) and (6, 1), and see if the distance between them is 5.

BXBRCISEI
1.

84
(3,

In what quadrants are the points

4),

(4,

1), (5,

3),

(-1, -2)?
2.
3.

Plot the points Plot


the
?

(1, 2), (4, (5,

-6), (-3,

7),

(-5, -2).
is

points

0)

and

3,0). What
(-5,

the distance

between them
4.

Where

are the points (0, 0), (0, 2),


is

0), (4, 0)

situated

5.*
0,

What

the distance between the points


is

(6, 4), (1,

8)

What What
?

kind of figure
order
?

formed by joining the points


is its

(0,

0),

(4, 0), (4, 4), (0, 4) in

What
is
?

area

7.

kind of a triangle

formed by joining the points

(0, 2),

(2, 6), (2, 2)

What

is its

area

8.

Plot the points

(1, 1), (1, 3),

(2,

1), (3,

3), (3, 1).

Join them

in order. 9.

What
is

letter is

formed

What

the area of the figure formed by joining


6), (1, 6) in

(1,

3),

(-5, -3), (-5,

order

10. The angular points of a triangle are (6, 0), (3, 4), ( 2, 0). Construct the triangle and find its area. Measure or calculate the

lengths of the sides.


11.

What

is

the length of the perpendicular from the point


?

(5, 8)

to the line joining (3, 2) and (7, 2)

127.

Complete

Graphs.

GEOMETRICAL REPRESENTATION OF NUMBER


(2,-1),

173

(4,-2) and (6,-3).

This

is

as

we would

expect because

x=2,y= l; x=4:,y=2; x= 6, y= 3,
all

satisfy the equation y^=^x.

128.

Linear

Equation.

It

is

seen that the

graphs of

all

the equations so far constructed have been straight lines. This is true concerning all equations of the first degree. For this reason an equation of the first degree is sometimes called
a linear equation.

Since a straight fine


points on
of
it

is

fixed or determined

when any two

are fixed,

it

follows that to construct the graph

an equation of
it.

the first degree,

we need

to

determine only two

points on
129.
of the

Lines not passing through the Origin.

Every equation

form

y=mz

represents a straight line passing through

the origin, because the equation is satisfied by x=0, y=0. If the equation contains a term independent of x and y, it represents a straight line which does not pass through the
origin.

Thus, 2/ = 2x4- 1 represents a straight line which does not pass through the origin, because this equation is not satisfied by x = 0, y = 0.

Ex.
are

1.

Construct the graph of y==2x-\-\.


This
is

The coordinates of two points on the line x=0, y = \ and a;=l, 2/ = 3. Locate these two points and draw the un-

limited straight line whicli joins them. the required graph.

it also passes through points (2, 5), (1, 1), (-2, 3), (-3, 5). Do the coordinates of these points satisfy the equation ?

The diagram shows that

the

In constructing the graph of an equaby locating two points on it, the pupil should try and determine two points whose coordinates
tion

are inteaers.

174

A LOB BRA
2.

Ex.

Construct the graph of 3x+4y=15.


= 15-3x
-

Here

J and when
,

/x=l, w = 3
-

^ join

Plot the points graph.

(1, 3)

and

(5, 0)

and

them giving the required

We might have found the points at which the graph cuts the axes. Thus, when x = 0, 2/ = 3J and when 2/ = 0, x = 5. The required line is then found by joining the points (0, 3|) and (5, 0).
If the latter

method

is

followed and fractions appear in

the coordinates of either of the points found, the unit of measurement should ijo changed, in this case, by taking /ottr

spaces as the unit instead of

07ie.

not one space, it should be clearly shown on the diagram what the selected unit is
the unit
is

When

EIXBROISE

86

Find two pairs of values of x and y which satisfy x-{-y=6. 1. Plot the points whose coordinates are the values found and construct
the graph of the equation z-\-y-=6.
2.

What

are the coordinates of the points at which the graph in


1

Ex.

cuts the axis of x, the axis of y

Construct the graphs of the following equations


3. 6. 9.

y=x+3.
y=3x2.
2x + 3y=\2.

4.
7.

y=x3.
x+2y=l.
3x-4y=\6.

5. 8.

y=2x-3. x-2y=l.

10.

11.

5x+6y=n.

12. Construct the graph which cuts off 4 units from the axis of x and 6 units from the axis of y. Find the area of the triangle which this
line

forms with the axes.

13.
i;

On
?

the

4-

2^=7.

/ntersect

same sheet construct the graphs of xy=^lO and Wliat are the coordinates of the point at which they Do the coordmates of this point satisfy both equations ?

GEOMETRICAL REPRESENTATION OF NUMBER


14,

175

Will the point

(3, 4) lie

Which
(5, 2),
(

of the following points lie

on the graph of the equation 4x+3?/=24 ? on it: (2, 6), (0, 8), (6, 0), (9, -4),

1,
By

9)

V^erify

by constructing the graph.

15,

positive integral values of x


16,
of X

constructing the graph of 2a;-f 3t/=24, find three sets of and y which satisfy this equation,
is

Why

there an unlimited
will satisfy
2a; +3?/

number

of positive integral values

and y which which wiU satisfy


130.

2x3y=2'i, but only a limited number


?

=24

Graphical Solution of Simultaneous Equations.

In this

diagram are shown the graphs of the equations

x+y = 5

and

2x3y=lo.
The coordinates
which the lines both equations.
.*.

of the point P, at

intersect,

must

satisfy

The coordinates of P are (6,-1). x=6, ?/= 1, must be the values of X and y which satisfy both equations.

We

the

sohition

have therefore obtained of these two equations


evident that two straight

Graphically.

Since
lines
it

it is

can

inter.sect at

only one
there
is

point,

must follow that

only one

pair of roots of

tivo

simultaneous equations of the first degree. In this diagram are shown the graphs of
(a)

x-y=3,

(6)

2x-y=7,

(c)

iJa;+?/=-7.

At what point do the graphs of (a) and (6) intersect ? (a) and (c) ? (6) and (c) ? Is there any point which is common to Are there any values the three lines ? of X and y which will satisfy these three equations at the same time ? When three equations in x and y are all satisfied by the same values of x and y, what peculiarity will appear in their
graphs
?

176

ALQEBHA

GEOMETRIC AL EEPRESEN^^TION OF NUMBER


4.
of X.
If

177

3x2y=6,

express x as a function of y and y as a function

Solve graphically and verify


5.

x+'Sy=9, 2x+ y^8.

6.

x+ y=
2?/== x,

8,

7.

x 2j/=

6,

3xiy=lO.
6,

2x 3t/=ll.
10.

8.

2z+3y=

9,

x+iy=9,
3x-8i/=-3.
12,

3x-\-2y=14:.
11.

10jr=4x-2.
the

3x2y=5
12.
sistent.

Show by graphs that have a common pair


Show Show
graphically that
is

equations x-\-y=5, 2x-r3y

of roots

and

find them.

2x+3y=13 and \x-\-%y=2

are incon?

What

peculiar about the graphs of these equations

13.

the equations x-{-y=4:,


.he

graphically that no values of x and y will satisfy all of 2xy=ll, 4x+2/=13. What values satisfy
first

first

and second, the

and

third, the second

and third

14.

Show

the coordinates of the points where the graph of y=2x-\-3

cuts (1) the axis of x, (2) the axis of y, (3) the graph of
15.

?/=6 x.
3x4:y=0 and

Show by graphs

that the values of x and y which satisfy


will

2x 32/4-1=0 and 5x-2?/ 14=0 x2y+2=0.

also

satisfy

BXEIRCISB
1.

87 (Review of Chapter

XUI)

At what point does the graph


?

The

axis of y

the graph of x
2.
first

of x-{-y = 5 cut the axis of x 7 Construct the graph. In the same way construct 4!/= 4. At what point do they intersect ?

it appear geometrically that two equations of the degree can have only one set of roots ?

How

does

3.* Plot the points (0, 0), (-3, 4), (3, 12), (-2, 0). distance between each conscoutive pair of these points ?

What

is

the

4. From a certain point a man walks 5 miles E., then 4N.; than 2W., then 3N., then 3E., then 4S. Using squared paper, determine by measurement how far he is now from the starting point.
5.

A man

walks 8 miles

W. and then

5S.

Find by calculatioo

how

far he

must now walk

to reach a point 4 miles E. of his starting

point.

179

ALOlCBiiA

If 11 lb. equal 5 kilogrammes, make a graph from which you 6. can express any number of kilogrammes in lb. or lb. in kilogrammes. Read from the graph 3J kilogrammes in lb. and 8J lb. in kilogrammes.
7.

What

is

the perimeter and area of the triangle whose angular

points are
8.
of 5x
(3,

(0, 0), (5, 0), (0, 12) ?

How do you show that 2y=l9 Which of the


1

the point following


?

(3,
lie

2)
on

lies

on the graph,
5), (1,

it:

(6,

7)i
(5, 9),

-17),

(4, 1),

(-2, -12),

(5, 3)

9.
(8.

Find the area of the triangle formed by joining the points -6). (-7. -6).

find its area.

the triangle whose vertices are (2, 0), (10, 0), (5, 6) and Why do the points (2, 0), (10, 0), (8, 6) determine a triangle of the same area ?
10.

Draw

Solve graphically and verify


11.

x+2y=l2,

12.

3x-'iy=

0,

16.

t/-x = 4,
x='2.

14.

x~2j= 2. y-2x=-3,
x+2y=li. What is the
x = 4,

4x-3i/ = U.
15.

2x+7y = 52,
3x-5!/=16.

16.

j,'

!/

= Jx + 4. = Jx+5.

17.

area of the figure formed by the lines whose

equations are:

x= 2, y = 3, y=
?

What 18. the points (2, 3) and


19.

are the coordinates of the middle point of the line joining


(6, 5)

On

the

same sheet draw the graphs


y = x+4,
2/

of the equations

= 4x-2, y = 2x+2.
?

What peculiarity is presented by the graphs do you draw concerning these equations ?

What

conclusion

Draw the graphs of 2x-f-3y = 20, 4x-|-6!/ = 35 on the same 20. What do you conclude as to the "solution of these equations ?
Determine graphically whether these
or inconsistent
:

sheet.

sets of equations are consistent

21.

4x4-

X- y= 4, = 26,
2/

22.

x+2j/=10,

23

-x+

y=

8,

2x-5y=

2.

3x- y= 2x- y=

9,
1.

3x4-2?/=

13,
9.

5x-3i/=

.vhose sides are represented by the Describe the crianglt 24. 14, x-!- 102/= 14. W^hat are the equations: 3x-r2y=14, ox coordinates of its vertices 7 (Verify by solving the equatiorw *> oairs.)

%=

GEOMETRICAL REPRESENTATION OF NUMBER


25.
intersect

179

At what point do the graphs of 2x+Zy=\2, ? At what angle do they seem to intersect ?

3x 2y = 5

A teacher's salary is increased by $50 each year. His salary Construct a graph from which you can the first year is $750. What is his salary for the 8th year ? read off his salary for any year. In what year would his salary be $1300 ?
26.
for

27.

In the process of solving 2x 3y=l, 3x + 22/ = 8, by eliminating

we have

2x-3y=l, 3x+2y = S.

4:x-&y=
I I

2,

x=2,
j

x=
|

2,

9x + 62/ = 24.

2a;-3y=l.

y=l.

On the same sheet show the graphs of each of these sets of equations, and thus show that they all determine the same point and that the
four sets are therefore equivalent

CHAPTER XIV
HIGHEST COMMON

FACTOR AND LOWEST COMMON MULTIPLE

In Chapter IX. we defined the terms highest common and lowest common multiple, and showed how they were found in simple cases. When the expressions under consideration can be factored, the H.C.F. and L.C.M. can at once be written down from
132.

factor

the factored results.

A few examples are here given of a more than those previously considered.
Ex. 1. Find the H.C.F. and L.C.M.
x^y-{-lxy^-\-12y^
of

difficult character

and x^yx^y^-~l2xy^.

x*y-\-lxy*+l2y^ =y{x*^lxy+\2y^) = y{x^-'^y){x + ^y).


x'y x*y* \2xy^ = xy(x*~xy~ l2y*) = xy{x 4y){x-\-Sy).

Here the common factors are y and x+Zy, and since the H.C.F.
is

the product of

all

the
.-.

common

factors,

the H.C.F.=2/(x+3!/).

The L.C.M.
will

is

include

all

the expression with the lowest number of factors which the factors of each expression, .-. the L..C.M. = xy{x + 'iy){x + 2y)(x-iy).

Ex. 2.Find the L.C.M. of


a;2
1,

x^+l, x*x and

x^+x^+L

x*-l = (x+l)(x-]). x^+l = {x+l){x*-x+l).

x*x=x{x' l) = x(x\)(x*-{-x-\- 1).


x* +
:.

x*+\ = {x*+l)*-x' = {x* + x+l){x*-x+l),

the L.C.M.

= a;(x+l)(a;-l)(a; + a;-|-l){x-x+l).

H.C.F.
If

AND

L.C.M.

181

the multiplications be performed the L.C.M. will be found be x' x. It is customary, however, to leave the result in the factored form, as it is in this form that it is usually made use of.
to

BXBRCISB
Find the H.C.F. and L.G.M. of
1.* ^x'^y^z, 8xy*z% \2axyH.
2. 3.

88

x^y^, xyy'^,

x'^'xy.

a^~h\ o6+62,

a'^+2ab-\-b^.

4.

x^-lx-\-\2, x^-\-2x-\5, a;2-9.

^.
Jo.
[/I.

a2+8a+15, a^-2a-25, a'^^Za~\Q.


3x2- 12x4- 12, 3x2-12,
x'^xy-\-xzyz, xyy"^.
3x2

^3x-6.

^. m3-8,
j^.

m*n^-47n^h^, 4i2-16ffi+16.

6a3-663, 2a^+2a^b+2abK

f^.
^12.

a^+ab-ac, a^-i-b^-c^+2ab.

[kU. a2-62-c2-26c, b^-c'-a^-2ca, c^-a^-b^-2ab.


x3+2/3,

x*+x^y^+y*.

- )^
14. 15.
j(^16.

3x2+7x-6, 3x2-llx+6, 6x2-13x+6.


10ax-2a+15cx-3c, 25x2-1, 25x2-10x+l.
x3-5x2+6x, x3-3x2+5x-15.
u*v*, u^v^,

u^v\ uv.

1^

xH2x2-8x 16, x3+3x2-8x-24.


Show
that the product of

18.

x* 8x+15 and x^-\-x\2


a'^

is

equal

to the product of their H.C.F.

and L.C.M.

The L.C.M. of a2-5a+6 and 19. Supply the missing term.


20.
is

-6
is

is

a3-3a2_4a4-i2.

Find two trinomials whose H.C.F.

x2y and whose

L.C.M,

z' 7xy'+6v*.

182

ALGEBRA

Ex. 1. Find the H.C.F. and L.C.M. of


a;2-|-2.r-3

and .r^-S.r+S.
factored,

Here
tlie

ar*

+ 2x 3
is,

is

readily

but

none

of

<reviously given will apply in factoring

x* 8x + 3, except by
:

the methods using

factor theorem of art. 101.


difficulty

The
If

however, easily overcome thus


x

+ 2x-3 = (x-l)(x+3).

X 1

the expressions have a common factor it must evidently be either or x+3. By using the factor theorem, find if x 1 or x + 3 is a factor of

x-8x+3.

When x-l=0

or

x= 1,
.".

x3-8x-(-3=
1

-8-f3= - 4,

X is not a factor. When x+3-0 or x=-3, x-8x + 3= -27 + 24 + 3 = 0, x+3 is a factor. How can we obtain the other factor of x* 8x+3 ? x'' + 2x-3 = (x- l){x + 3), We now have x*-8x + 3 = (x+3){x*-3x+l). and
/.

the H.C.F.
the L.C.M.

and

= x-l-3, = (x+3){x- l)(x2-3x+i),

Ex. 2. Find the H.C.F. and L.C.M. of


a;2_7a;^10 and a:^ 6a;2+lla;-6.
The
Here
since
Is

factors of
it is

x*-7x+10

evident that

x 5

are (x-5)(x-2). is not a factor of the second expression,


5.

term is 6, which is not divisible by a factor of x^ 6x'+llx 6 ? Complete the solution.


its last

X 2

SiXBROISB
Find the H.C.F. and L.C.M. of
:

89

^*
2.

x2-3x+2, xs-6x2-f 8x-3.


a'-6a+5, a^-lQa^-f I7a+1.
x8-2x2+4a:-8, 2xS-7x2+12.

3.

/f.

yt

a-o2+a-l, 3a3-2a2+5a-6. a:8+3a;2_4a;. x3-7x+6

LC-.F.

AND

L.C.M.

VSa

6^^If X 2

is

a common factor of xHSa:^ 9a; 2 and x^ 4x2+3x+2,

find their L.C.M.

yf.

Reduce

to lowest terms

x^2x^'ix a2 3a6+262 and 2x*Ux^12x a3-19a62+3063


8.
is

a;2_5a;_j_6

Find two expressions of the third degree in x, whose H.C.F. and whose L.C.M. is x* lOx^+Soz^ oOx+24.

133.

Method

of

finding the

H.C.F.

of

two expressions which

can not be factored by the usual methods.


it

From

the preceding

seen that Lhe chief difficulty in finding the H.C.F. of two expressions is in factoring the given expressions. If neither of the expressions can be factored by the usual
is

methods, another method may be used which depends upon the same principle as that of finding the G.C.M. of two numbers in arithmetic.
134.

Fundamental Theorem.

This method of finding H.C.F.


:

depends upon the following theorem


If 0^ is

factor of

common the| sum or


a
is

factor of

any two

quantities, then oc

is

also a

difference of

any multiples

of those quantities.

Then mx~nx, mxnx, pmx-\-qnx, rmx snx, sum or difference of multiples of mx and nx.

Thus, X

common

factor of

mx and

nx.

are each the

It is evident that each of these is divisible by quotient in each case being found by division, thus
:

x,

the

x)mx-\-nx

184

ALQEBRA

It is evident that x 3 is not a factor of either expression, since their terms are all positive. Therefore if they have a common factor it must be x+3. By applying the factor theorem, or by division, we find that x+3 is a factor of each, and since it is the only common factor, it must be he H.C.F.

Ex. 2. Find the H.C.F. of

3a;3-17x2-5a;+10 and 3a;3-23a:2+23a:-6.


TheirdifEerence = 6x*-28x+16 2(a;-4)(3x-2). Now 2 is not a factor of either and may be discard_ed, also x 4 not a factor, since 4 is not a factor of 10 nor of 6. Therefore if there a common factor it must be 3x 2. Divide 3x 2 into one of them and see if it divides evenly. If

is is

it

does not there is no common factor but unity. If it does divide evenly into one of them, it is not necessary to divide it into the other, for if it is a factor of one of them and also of their difference it must be a factor of the other.

Ex. 3. Find the H.C.F. of

3x3-13a;2+23a;-21 and Qx^-\-x^-4Ax+2\.


Multiply the
first

by

and subtract the product from the second

and we get

27x-90x + 63 = 9(x-l)(3x-7).

Now since 9(x l)(3x 7) is the difference of two multiples of the Which given expressions, it must contain all their common factors. Complete the solution. of these factors may be discarded ? We might have obtained the H.C.F. thus The sum of the expressions is
:

9x-12x*-21x = 3x(x+l)(3x-7).
This expression contains
expressions.
all

the

common

factors

of

the

given

Complete the solution by

this

method.
is

The object
expressions

in

each

case

to
the

obtain
second

from the given


degree.
If

an

expression

of

this

not be factored, it must be the H.C.F., if there is any common factor other than unity. If it can be factored the H.C.F. can then be found either by the factor theorem or by ordinary division.
expression 'can

H.C.F.

AND

L.C.M.

185

In obtaining the expression of the second degree, the last problem shows that it is sometimes easier to eliminate the last terms than the first terms.

Ex. 4. Find the H.C.F. and L.C.M. of

6x3 5a;2 8a;+3 and 4^3 8a;2+a:+3.


Eliminate the absolute terms and show that 2x 3 is the H.C.F. Since 2x 3 is a factor of each, the other factors may be found by division, then 6a;-5a;-8a;+3 = ((2a;-3)(3x* + 2a;-l),

4a;-8x*+x+3 = (2a:-3)(2a;-a;.-.

1),

the L.C.M. = (2a;-3)(3a; + 2a;-l)(2a;-a;-l).

Why

is it

unnecessary to factor 3x* + 2x

and 2x* x

Ex. 5. Find the H.C.F. of

and
Subtract
Multiply
terms.
(1)

a;* 4a:3+10a;2 llx+10, a;* a;3 4a;2+19a: 15.

(1)
(2)

from

(2),

and we get
3a;-14a;

+ 30a;-25.

(3)

by 2 and add to eliminate the absolute Remove the factor x and we obtain 5x3-14x + 22a; + 5. (4)
(1)

by

and

(2)

The common factor we are seeking must be a factor of both (3) and (4). Eliminate the absolute terms from (3) and (4) and show that the
H.C.F.
is

x-3x+5.

Find also the L.C.M.

Ex. 6.Find the H.C.F. of


8a:*+4a;3+4a:2 4a; and 6a;*+2a;3+2a;2 4a;.
a factor of the first expression and 2x of the second, and a common factor. Remove these sioEiple factors and find the H.C.F. of the quotients, and show that the H.C.F. is
is

Here 4x

therefore
^x(x

3a;

is

+ a;+l).
Product of the H.C.F. and L.C.M.
of

135.

Suppose that x is the H.C.F. n have no common factor.

mx and

nx, so that

and

186

ALGEBRA
tlie

Then
But

L.C.M. of

mx and
of

nx

is

mnx.

xxmnx= mx x nx,

any two quantities is equal to the product of their H.C.F. and L.C.M. Is a similar theorem true concerning any three quantities
therefore the jiroducl

mx, nx and px ? If the H.C.F. of two quantities has been found, we might therefore find their L.C.M. by dividing their product by the H.C.F.

BZBROISB 90
Find the H.C.F. of:

@l* x3-7x2 + 13x-15, x3-6a;2+x+20. a3-10a2+33a-36, a^-2a--2'ia+m.


.

6x3

+ 10a;2+8x+4,

6x32x2-4.

1/4.

2x3-5x2-20x+9, 2x3+x2-43x 9.
263

+ 562-86-1.5, 463-462-96+5. ^. *3x3 + 17x2j/-44xi/2_28)/3, 6x3-5x2^-33x?/2+28j/'. ij^ 2a3-3a2-4a+4, 3a*-4a3-10a+4. ^. 2x^-12x3 + 19x2 -6x+ 9, 4x3- 18x2+ 19x-3.

t^

9.

18^55_3a4ft_i2a36_3a26, 12a5c-6a*c-9a3c+3a2c.

10.

x3 x2-2x+2, X* 3x3+2x2+x 1.
:

Find the L.C.M. of


A.1.

x3-7x-6, x3-4x2+4x 3.'

j^2.

x3+6x2+llx+6, x3+7x2+14x+8, x3+8x2+19x+12.

/3.
^4.
]jCb.

2x3+9x2+7x 3, 3x3+5x2- 15x+4.


x3-6x2 + llx-6, x3 7x2+14x 8.

20x*+x2-l, 25x*-10x2 + l, 25x*+5x3-x-l.


Find
a
value
^acti

16.

of

x which
0.
is

will

make

'x3

13x+12
ta 7.

and

x' 6x2 x+30


17.
of the

equal to

The L.C.M. of two numbers numbers is 14, find the other

70 and the H.C.F.

If

ooa

H.C.F.
18.
If

AND

L.C.M.
is

187

The H.C.F. of two expressions is x 2, the L.C.M. one of the expressions is x^lx+10, find the other.

x^ 39x+70.
factor,

19.

Two

integers differ

by
it

11.

If

they have a

common

other than unity, what must

be

EXERCISE

91

(Review of Chapter XIV)


:

Find the H.C.F. and L.C.M. of


jt!*'

x*-20x+99, x*-24x+143, a;*-21a;+110.


a;*-15x+36, x'-27, x3-3x-2a;-f 6.
.

itf

''-

a*-b*,a*-2ab + b*,a^-b\ A. x3-2x*-15x, x^ + x* 14X-24. 4a'-12a-a + 3, 2a3 + a*-18a-9. y<^ 6. X* ax 6x-f-a6, x^ bxcx-\-bc. x 6x+llx-6, x* + 4x + x-6. 7. x + 3x + 3x* + 5x-12, x*-4x'- 19x+ IOx+ 12. 8. 2a+15a' + 39a + 40a+12, 2a + 9a-2a*-39a- 18. fin. x*6x*2/+13x2/*- 12x1/3 + 42/*, x* + 2x3j/ 3x*i/* 4xt/3 + 4V*. </ll. x* + x*2/* + 2/S X* 2x3?/ + 3x2^ 2x2/' + y*-

^.

'

'

12.

Show that two consecutive

integers

can have no integral

common factor except unity. Two odd integers which differ by 13. other than unity. What must it be ?
14.
15.

have a common factor

Find the H.C.F. of x^ + a' and


If the

x^

+ x*a* + a^.
is -=

H.C.F. of a and b

is

d,

show that the L.C.M.

If a is the H.C.F. and b is the L.C.M. of three quantities, show 16. that the product of the quantities is a^b.

17.

For what
?

x'

common values of x will 3x*-x+3 and x*-4x+12x


in x,

both vanish
18.
is

x-1
10, 19

Find two expressions of the second degree and L.C.M. is x^-Sx-^ + 17x 10.

whose H.C.F.

D J Reduce

18x-3x +-2x+8 ^ , . ^^ to lowest , terms. ^ , 12x+8x* 7x+12


,

-,.

^pr ^''

^^

CHAPTER XV
FRACTIONS
In Chapter IX. fractions were introduced, and simple ^ examples of operations upon fractions. In this Chapter the subject is extended and applications made to more complicated forms.
in the Form of a Fraction. Both terms of a be multiplied or divided by the same quantity without altering the value of the fraction. As previously stated, the only exception to this rule is, that the quantity by which we multiply or divide must not be zero. The rule might be stated in the symbolic form

136.

Changes

fraction

may

a_
b

ma ~~
mb

na _ a
nb
~~

The case
by

in

which the terms are multiplied or divided

deserves special attention.


the rule of signs for division

From
same
as

is

seen to be the

a -

, ,

so also

is

FRACTIONS
Since

189
it is

(a 6)
differ

/;(

!) = a-f-ft
1.

or h

a,

seen that
is

ab

and

ha
is,

only in sign, or that each one

equal to the

other multipHed

That

{ha) ah =
by
(

and

ba= {ab).
ab _ dc ~

ab _ (g 6)x( 1) _ ba _ ~ (c d)x( 1) ~~ dc ~ cd
Also, since
it
(
(

ba
cd'

follows that

m
(a-6)(c-d)

a) x( 6) = + a) x + 6) = a6, (o 6)(c d) = {6 a)(d c), m


{b-a){d-c) (a-6)(d-c)

m
(6-o)(c-d)'

(a-a:)(b-y)

{ba){ay)

( x-a){y-b) (xb){ya)

a-x){y-b) (bx){ya)
(

^^^

E3XERCISE! 92

(1-29, Oral)

Express these fractions in their simplest forms with no negative signs in either term
:

-2
4

-6 9

-3a

a
2/

4o6
2b

m
axb
10.

a ax bx
^"

9.

3x 5 =
7
b

a.b
11.

x.y ^

J-b

c
ba xy

Express with the numerator a b


13.

3
:i

a
:r

14.

0 r

15.

Express with the denominator


17.

cd

5 a
-4
7

18.

x. y J a
4

a
19.

:i a

Express with the positive sign before the fraction

21.-=-25. 29.

22.-;:.

23.

-T6

ab What is

26.

a+b
ab
,

27.

x2 xy
.

28.

, cd

c+o

the relation between

a , and icy

a
.

yx

a+b ab
;

and

b+a r ba

ba s ^ and 3
a 3

190
30.
rrr

ALGEBRA
Write

(?
.

?){? _
..

')
.

in four equivalent forms, with the positive

sign before the fraction.


31,

Which

of the following are equal in value

(a-b){b-c){c-a), {b-a){c-b){a-c), {a-b){c-b){a-c), (ab){bc){ac), (ba){bc)(a-c)


137.
?

Reduction

of

Fractions to Lowest Terms.


its

The formula
lowest terms.

ax
,

ox

a
7

may
^

be used to reduce a fraction to


all

by dividing both terms by

the

common

factors.

Ex. 1. Reduce
x'

"^^

x* + x*y* + y* = {x*
.'.

+ y^ = (x-\-y)(x^~xy+y*). + y*)* -x^y^ = {x' + xy + y*}{x*-xy + y*),


the fraction

x^+xy+y*

Ex. 2.-Reduce

._,.^^,^,^ a + 6*-c + 2a6 = (a + fe;*-c* = (a + 6 + c)(a4-6-c).


x2-lla:+28

Complete the reduction.


o T. J Ex. 3.-Reduce
1-

,^3_6,2+7,_60 a;-llx+28 = (x-4)(x-7).


can not divide into the

"

Which

of

these factors

denominator

Complete the reduction.


T^ J Ex. ^.-Reduce T.
.

3x3- 15x2- 19a;+6

e.3+3.2-5.+ l

"

common

Here the factors of neither term can be readily obtained, 30 the factor must be found by the method of art. 134. Eliminate the x' and we obtain
33x* + 33x-ll or
1

l(3x

+ 3x- 1).

This expression must contain any common factor of both terms. Since 3x* + 3z 1 can not be factored, what conclusion can be drawn ? Complete the reduction.

FRACTIONS
EXBRCISB
Reduce to lowest terms
98

191

a2+3a+2
6a;2+a;

/2.
4.

x^-\-5xy

4x2+8x+3*

24i/2 x^-{-2xy-\-y^ z* y^2yz a*


a;2
*

2-3j/-2y2 4_5y_62/2

7*

og+2a+l a3+2o2+2a+l

a;3
'

x^2x
*

8.

a;3-3a;2+4

a^-4a+3
4a3-9a2-15a+18'
11.

10.

_3x2 3x 18_
6a;6-12x*-18a;3"

2a:3 a:^+2x

12.

3x3+4x2 6a;
36a;3+27a;2_40a;-16'
2a: 4a;3 2a;2 12a;

2a;3+3a;2+4a;+3
13.

gg+gg 3g
g5irct*^a3+2g2 3a
Addition

14.

4a;*+2x3+6a:2 4x~

138.

and

Subtraction.

In adding or subtracting

fractions

we should be

careful to note

whether any of the

given fractions can be reduced to lower terms. result is obtained we should examine it to see
reduced.

When
if

the

it

can be

Ex. 1.Simplify
The expression

2y

xy'^+y^

x^xy^
y'{x+y) x{xy){x+y)

= xy + 2y X xy

^xy
^ _ ~

^P~..~ xy
"""

x{xy^
^

{xy)*+2xy-y*
x(xy)
x*

x(xy)

_ ~ xy'
a;

The form of the last fraction in the given expression should prompt the Dupil to examine whether it can be reduced.

192

ALGEBRA
2.

Ex.

Simplify
x-2
'

x2 +

x^3x-\-2

a;2 4a;+3

The expression
{x-l){x-2)
(a;-3)(x-l)'

_ _

(a-l)(a;-3)+x-3-2(a:-2) {x-2){x-l){x-Z)

g* 5x+4
(x-2){x-l){x-3)

(x4)(x~l) {x-2){x-l){x-3)

xi
{x-2){x-3)'

BXERCISB
Simplify

94

i.*_?_^__L. a+ft a
a 4 a 2
yf
2x'

2.
'

.
t/

3.
'

x-\-y

^+y xy
a'

-y
x-\-y

o a
2x2

2a2

2a

o* 6^

o+6
1

aa^
x'-4y' x*+2xy
3y
I

1+a'

x* 2/2

x^+xt/

^
'

^_y
x^ y2

^
t/
*

x-2y
x

2x

^JC x*+9x+20

,<ft^

__i

.j^
-TT;

x2+12x4-35 lAll5 jj+y

^
y

^'+y'

x-y'^ x^-y*
2x

^^

x2-3x-10 x2-8x+15

x2+2x-3 x2-3x+2'
^

i^
Cj-'
'

x-y

x^
izy

x'+x'y x^y-y^'

a'ab+b^ _ a^+ab+b^

ab

0+6
a+6

V^* xy
Af^"

^ x+y
3x^-8 x^-l

_ yx ^. - %^
x-\-y

x^y^

^'
\\jf

a-fe
I

2(o+6)

^ 2(a-6)

o^+fc" o^-ft^

5x4-7
x^+x-j-l

"""x-r

^_+i^ + _L_. i^ J___1_ + _L., ^' 2a-36^4a-962^2a+36 k^ x-1 x-2^x-3

-.-d=.* x2-7x+10 x-3x+2

x2-6x+5
a 6

(o4-6)(6+c)(c+a)
a6c

_ a+fe _ 6+c _ c-fa


c

FEACTIOHB
a 6*+26c c

aj

cg+2ca+o^

^.
'

az-{b-c)^
(o4-c)2-62
o*

b^-{c-a)'
(6+a)2-c2
3a
"^

c^-ia-hY' (6+c)2-o2

2a
x+y
1

5a

oa-a-2
25.

6a-4
and

6o2+2a-4

xy
x-t/
1

x-y
?^

x+y

x^+y*

^
27.
^28

:6;I_i

i^.
x*+y*

a;+?/
1

x^+y^
2x
Q+a;^
2a=

3
,

a;

3+x

"^
I

4a'6

>

a-6
1

a+6 "^ a^+fc^ "^ a*-b*


1

4 4x
'"

4+4x

2+22

1+z*

^"^"^

x^lk+6+ x^+lt+U
Types
in

x^+ll+2l

= l'

<^^^y-^

139.

Special

Addition

and

Subtraction.

We

have already seen that

6 a= (a 6) and ab= {ba),


cr

(a 6)-i-(6 a)= 1.

Df

oecur in the factored denonainatoM which are to be combined, it is not leeessary to include both of them in the L.C.D.
different fractions,

When a 6 and 6 a

Ex.

1.

Simplify ^
-^

a
r ah
is

+? ba
it

Here only one faetsr


either

required in the L.C.O. ftad


is

we may uie

ab

or b

a.
vise

a
,

we

de<nde to

a b, then
6

better to change the second

fraction into the

form
a
i

ab
b

Then

a_|,

r + ba = ab

ab

= ab -= r
ab

I.

194
t:.

ALGEBRA
4
3
,

o o ir Ex.2.-S.mphfy--^-^j +

X^S
j-^,.
be changed to a;*
1,

The denominator

BO as to be the product of
rru expression The '^

.43
x 1

of the last fraction should

and

x-\- I.

xl

x+l

x-3
x*l
10
z

4(a;+l)-3(a;-l)-(a;-3)

(x-lKx+l) Ex. 3.Simplify

(x-l)(x+l)

{x-l){x-2)
L.C.D.

1^2
'

{x-2){3-x)

{x-S){lx)

Here there are only three factors xl,

x 2,

x 3, required

in

the

therefore change the second and third fractions so that the given expression
1
9.

We

(x-l)(x-2)
Complete the
140.

(x-2)(x-3)

+
'

(x-3)(x-l)

simplification.

Cyclic Order.

Suppose we wish to simplify


c-\-a
, '

6+c
{ab){a-c)

a-i-b
'

{b-c){b-a)

(c-a)(c 6)

The L.C.D.

in this case will contain three factors


dififerent

and

it

might be written in

forms as
etc.

{ab){bc){ca), {ab){ac){b c),

The

pupil

is

advised

to

write the

factors

in

what

is

called cyclic order


It we arrange the letters on the circumference of a circle, as in the diagram, and follow the direction of the arrows we see that a is followed by 6, b by c, and c by a.

Thus, if we write a b as the first factor, then changing a to 6, 6 to c, and c to a, we write the second
factor b c and the third c a. write the L.C.D. as (a 6)(6 c)(c a), we should change the fractions so that these factors appear in the denominators.
If

we

FRACTIONS
'Rie given expression then
b+c c+a a+b _ ~ ~ (ar-'b)(c -"7) ~~ {b-c){a-b) ~ (c-a){b-c) _ (6+c)(b-c -(c + a)(c g) (a+6)(a 6) ~ (a b){b c){ca) b* + c* c* + a^ a' + b* = 0.
)

195

(ab){bc){ca)
be

Ex.

Simplify
oivc

j {a-b){a-c)

+
,

ca
-j-

r-

{b-c){b-a)

+
'

at>

{c^cjjic-b)
result

Proceed as in the preceding example and you should get the

6*c+6c* c*a+ca* a*6+a6*


(a=^(6-c)(c-a)
This fraction is equal to uniiy, for the numerator denominator. Prove that this is true.
is

equal to the

EXSRCisE
Simplify
1.*
1

gs

a^+ax

x'+ax

x+3_x3
x-2
a^ab

r'
5.

2^'
b^ab
3

_2

x2--3

Xl

X+1

1 X2
y

/^

xy y

x+y + y^x^ x-\-

a;+4

x 4

16 x*

^2 */ j3

3a+2x 3a-2x
^^

_ 3a- 2x
4

16x2_

3a+2x'^ ix^-9a^

_8_

3a:+7

j'

x-1

1-x

i+x^x2-r
>

IM
14

ALOEBRJi
-J^
i

'a-h){c-a)
16.
,

'

(6-a)fc-6)
*

(x-oXa-fi)
17.

f^
,
..,

(x-6)(6-o)
2

-^ +
3-x
l-3x
2

a
20
*

3+x
l+3x
'

l-16x
9x2-1
2

a;2-8x+15
<^+^
1

x2-4x+3
fe+c
"^

Gx-x^-S
J
"^

f +a
(a-6)(6 -!

(6-c)(c-a)
1

(c-a)(a-6)

21

(a-b){a-c)
n.^
?^

1 L ^ (b-cyh-a) ^ {c-a){c-b)
,
I

/22.

7)2

(a-6)(a-c)

(6-c)(6-a)

^ + ^ {c-a){c-b)

'

{x-y){x-z)

(y-z){y-x)
~r 7T

{z-x){z-y)

ax be
(a-6)(a-c)
a*

(6-c)(6-a)
62
"^

bxca
TTTT
,

+ (c-a)(c-6)
g "^ (c^TT^sj/^lTj;^

cxab

(/^^

(a2-62)(a2-c2)

(62_c2)(62_a2)

/
27

(a-b){a-c)~^ {b-c){b-a)'^ {c-a){c-b)'


1

Q+36

463

0-6

2(b+a)

2{a^'+b^)^ b*-a'-'

Va;-6^x+5/
29,

Va;+3^x-3
a+1
x+l

+4
d

a+3 z+3

^4--^ a+2
1

(Check when. a- i.)


^

30.

_L-|-_L

X 1

L.

(Checiwhena;=2.)

FRACTIONS
141.
Multiplication

197

and Division.

The ordinary

cases

in

multiplication
art. 74.

have been treated in Some special forms which appear are illustrated
of fractions

and division

in the following examples.

Ex.

1.

Multiply
,

-\

by a

Here the mixed expressions should be reduced to the fractional form before multiplying.
_,

The product

a*ax-\-ax
'

a*+axax

o*
a*

a+z
!

x'

Ex. 2.Multiply ?

+-+ a

by

J+--1. a

Multiply thia in the ordinary way, by multiplying each term of th

one by each term of the other.

We
and
1

should recognize that the

first

expression

is

the

sum

of

t H

and the second

is

the difference.
1

Th produce
Or,

(I

1)'-

?,
1,

^^:

_ -! + +

^^:.

we might proceed
a*j^b* + ab ab
a*

as in Ex.

thus

+ b*-ah ^
ab

{a*

+ b*)*-a*b* _ ~
o*6'

a*+a*b*+b*
a*b*
6*
j,

This result

is

seen to be the same aa

a*
i-i

0'

a'

and the answer

may

be given in either form.

Ex. 3.Divide
The dividend
:.

^+X y

by

^-- + -. ^
y

= ^^^^, =

the divisor

= ^*-^+y* =
^1^ y

the quotient

"^
xy'

, , x'xy+y^

'^'^

Divide in the ordinary

way and

get the quotient -

-f-

1.

We
the

must not make the


or

error of tliinking that


reciprocal
of
it,

we can

invert

divisor,

take the

by inverting

198

ALGEBRA

each term of it, and change the problem to one in multiphcation, thus
:

The
a

reciprocal

of

,,

is

a-\-b,

but the

reciprocal

of

+r
_,

is

not
^ b

a-\-b.

or

1,1 = a

a-\-b

r and ab

.^

ab
=-

its

reciprocal

is

a-{-b-

EXERCISE
Multiply

96
a'^+b^ ab-b^ a a^ab' a*b*' b

^^ x^x6
'

x^2x 8
2.

'

a:2+4x+4' a:^ 7x4-12"

M-2x-15
'

x^+lx- U

r*-b* a'
4.

a-b

x^+8x33' x^-{-9x+20'

a2 2a6+62' ^2^06*

^^

a+b

ab
x2
I

(Py + ^,!j I/
a;

^
a;+y'
-

y^

y *-" x*

a;2+l+-,
x^
9.
6

+ -2
6a;

^
12,

a^

+ 2 + a" ^2,

a2

-2+
-

~2

+ ^^,

a+a;'

a^-b^ ajrb a^-ab+b^ J3+P' a-b' o2+a6+62'

Divide
11
a;^-lla;+30
,

o^5x
"^

o2._62

a;2_6x+9"

a;2-3a;
,

x^xy

by

a^-\-ab
I M* xyy

^.
b
'

o2+62_g24.2a6 c2 a2 62+2a6

a-\-b-^c

6+c
y

i+rby? + ^,
y3 x'

.<9. ^^^

x*y*

^-^->y^'-^' xv xy
gg 4 o2+5o
a''+2g

iplify:

0+1
a^2a

0+2
a^a
'

o*-4
a*a^

'^'

o2 25'

FRACTIONS

199

XJ^ ^'

2x2+x-l

2x^-5a;+3

2x^-7x+6
3a;2-7a;-6*

a;2-4a;+3 ^ 6a;2+a;-2

>. ( -^ + ^.)-(+-)^-

^ ^'
23
'

(a+6)^-(c+t^)^

^ (g-c)2-(rf-6)'^

(a+c)2-(6+d)2
a''-64

(a_fe)2_(d_c)2-

a2+24o+128
A

02+120-64 0^-64
\

^ a2-16a+64
"

'

a2+4a+16
26-a \ 26-3a/

24

^ _
Complex

26

2a

V0-6A

3a-26/

U+26

142.

Fractions.

complex

fraction

which contains fractional forms denominator or both.

in either the

is one numerator or

Thus, -

is

a complex fraction

and

is,

of course, onky another

way

of

writing r

-f-

It
,

is

simplified in the usual

way by changing

it

into

X be which
,-

d
-

equals r^
be

ad

A complex fraction may sometimes be easily by multiplying both terms by the same quantity.
Thus,

simplified

=
r

TT on multiplying each term by


a

4.

ii ^-

a+2b
a+b
"^

6(a

a+2b
b

a_

(a+2b){a-\-b)

+ 26) + a(a + 6) _ ab

a*+2ab + 2b* a*+2ab + 2b*

~ a+b

Here both terms were multiplied by 6(0+6).

500
If

ALOE BR
th

L.C.D.
it is

is

not the same for

both terms

of

the

fraction,

usually better to simpKfy tko terms separately.


a;-|-l

Ex.

Simplify z2

X-f2 x-{-2~

xl X2
x+3'

z^
2x
(x+2){x-2)
-10a;
(a;+2){a;-3)

The numerator The denominator


..

= {x+l){x-2)-{x-\){x+2) (x+2){x-2)
= =
(a;-2)(z-3)-(x+2)(x+3) (x+2)(x-3)

-2x
(x+2)(x-2)

(a;+2)(a;-3)

the fraction

x-S
6(-2)

-lOx

BXBROISBl
Simplify
:

97
1

1.*

60 66
2.

11
ft

60
3.

12c

g a+6

a;

4.

xy
x^+y*
a;4-y

12c
1

1+*
6. 6.

0+6
1

06
7.
]

a;

+y
+y

1o

2zy_
x-\-y

l+x

a-j-6

a
g

2 a^5a
a
(a;+3y)g-(a:-3y)'
a

+ a
IV

(Zx+y)^-{Zx-y)*

2
10.

+ xy + m xhj[^ 1
t

+ 6-

a
6_'
c-H

12
\

_a
t-fc

13^

FRACTIONS
a
1*.
b

20%

a+b
I

a+b
I

16

"

a^
,

ab^ b^

16.

Find the value

of

17.

Find the value of

-^^ when x= -^ y= ^ a-\-b ab x+y ^~^^+? when a - ^11^, 6 = fc^' (a:+2/)2 x+2/ l+2a+6
98 (Review of Chapter

EXERCISE
Simplify
^
:

XV)

a a
a
2a*

a
c

/^'

_
a;+3y

(a?-!/)*

Sy-a:

a:-9i/'

l_lj_!/
X y
1

ac_

^_x
y

/K

^_ ^ _i_
/i
/
.

4.

^_
^

4-

^
fi

^_ _
(l-a;)3
(l

L-

("-fe)' )

fe'
1
I

r"^

4a6
26c

V
,

2o&
26c

+ Q* \ /
V,
1

J,

+ (l+x)=' + a;)-(l-a;)'
6*

\(,

6c-c\

_1

x+1
a*ab a'-b
t/^
'

a;+3x+2"^a;3+6a;+lla;+6'
ab + b*
'

2a6
'

+ 68

a*+o6+6

a+6 o 6 a* + 6* a+2a6 + 6 ^ o^^ ^ o*+o*6+6*

v'o

a+x i_+
+oa;+a;*

ax - a* aa;+a;*

2x' zz

a*+a*a;*+x*

yi4.

(l-a)

|(l-a)

(a>-l)

(a

+ ^)}.
*

6+c
W^*

(o-6)(o-c)

c+a
(6-c)(6-o)
'"

0+6
(c-o){c-6)

202
^*

ALGEBRA
ix-y)ix-z)

'16.

y'

''

{y-z){y-x)

^
1

{z-x){z-y}

47 '
18.

^+6
6z

3x-9
2a;-3x-9

+ 6x-6"^

ISx-lO 9x-12x+4"

Express the product of

8-6x + a; ^ and l+x

(a;_i)(2-a;)

(x-2)(4-x)

(a;-4)(l-T)

as a fraction in its lowest terms.

19.

How

can you show mentally that 3 a

x+a

XXX
x+b
-\

is

the
,

sum
b
I

of
'

x+c ^^
x*y*
y*_ ^^

x+a~^x+6
-

x+o

4^'

2x_ Divide -=Px+w ^.


.,

+y

xy

_^ ^

by "^
'
,

^^^

x+y

f'

x-t/* x

a*-(6 + c)*

6'-(c-a)

(a-fe-c)

f' '22.
/

Divide

^
a
7.

+ '-^ + b

^-:::i-i by
c

2-fl+UlY cl \a b
^

0/f,

Show that ,/'^+^^^^-^^


Tf lfa = ;r

x(l-y) + y(l-x')
(l_a;)(l-2/)-4x2/

(l-X3/)-(x+t/)
c=i5

o. 24.
25.

2,2 o=r
2

5. 2 d o=s X 2

2,2

prove a = x. ^

Find the product of


J y-^, and 1+X+
,

l+a:'

'^^'
" 10 26. Subtract . -^ from t- and determine which fraction o-\- 10 greater if a is greater than b and if both a and 6 are positive.

is

the

y
28. 29.

Add

(^+^)(l+g ) (l+bc)(l+&a) (a-fe)(a-c) (6-c)(6-a)


'

(l
'

+ cq)(l+c6)
when x = a-\-b
as
it

(c-a){c-6)

Show that

xa

xb

has the same value

has when

x=

j a+b

Bum

Prove that the product of any two quantities divided by the sum of their reciprocals.

is

equal to their

*,><'/
FRACTIONS
30.
If

203

c
,

y = ;

a 2= ab
,

^, ^ prove that

xyz + x + y-\-z = 0.
31.
If

a=^^^,

x+y

= ^-^,
y+z

c=^^=^, show that

z+x

(l_a)(l_6)(l_c) = (l+a)(l+6)(l+c).
32.
If

a and b are positive, which

is

the greater

a+3 b
a+26
y33.
Simplify
-

a + 26
^"^

a+6

-6

^^
2(a+6)

a+3b
2(a*

4b

+ 6*)
zx

o*-6*'

x~y
Simplify ^ ^

yz
i/*

2* (a; 2/)*' a;* (y z)*'


4
3
r

35.

xt

X ^ 3
,

-|

X 1

12
,

(z a;)*

^. X

36

When x=

a+c

find the value of

x2a _ x+2a X 2c x+2c


r
,

iac

X* 4c*
^
I,

a;

y x
^

37.

Simplify J
j

_i^-^;-^ x(y-x)
^

-,(^-?\
\a; />

2/(y-a;)

CHAPTER XVI
FRACTIONAL EQUATIONS
If an equation involves fractions, the fractions may 143. be removed by multiplying every term by the same quantity. In Chapter VI. simple examples of fractional equations

were given.

The case
attention.

in

which two fractions are equal deserves


c

special

Thus,

if

3 and each side a

is

multipUed by bd we have
bd.

- X bd

^ X a

ad=bc.
144.
Cross
Multiplication.

It

is

fractions are equal,

we can remove

the fractions

thus seen that when two by multiplying

the numerator of each fraction

other and equating the results.


called cross multiplication.

by the denominator of the This operation is sometimes"

Ex. 1.Solve
Cross multiply,
.-.

z5 x1
a;

x-{-3

a;+9

(a;-5)(a;+9) = (x4-3)(x 7),

+ 4aj-45 = x-4x-21,
8a;=24,

x=3.
Verify

by

substitution.

This method is applicable only when a single appears on each side of the equation.

fractioD

FRACTIONAL EQUATIONS
r. o Ex. 2. Solve
,

206

4a:+17
x4-3

r-

7 =^

3a:- 10

Simplify the right-hand

member and we have


4a;+17
a;-f3

_ ~

4a; 18 a; 4

'

Now

cro8S multiply, complete the solution

and

verify.

SXBRCISE]
Solve and verify
^
:

99

3a;+l
2

_ a;+12 3~'
=
a;

2
*

3a:-5

3a;-l

io. r

a;

a;
11

-- 9.

4.

5x-Z~ 5x-\' x 2 a; 6 = ^
a;-3

x-5
3

0.

x4-2_a; 2 ~l 2
7a;-3
,

a;

1~'

x+1
~2

_ x 3 _ a;+30
13"

2x+5
3

a;2_^7x-6

/
9.

x2+5x-10
10.

X 1

^
x+2

x+1 x-1
x-(-l

= 2-^.
X
X 2
x-3
^

X 1

J_ + l = ^.
X

11.

J_ + J_^^.
2x+38 x+12
2x4-7
3

12.

3x 2

^i:i_2^ =
6x

o.

13.

_ 6x+8
2x+l
8x+19
12
,

^^
^^
.

y-S
2/2-87/+15

^.

y-l2
2/2-122/+30

10.

^^

= 5x+ll
7x+9

lb.

X 1 X 2
2x+7

x-5 = x-3
,

17.

._

6-8x

3 X

= 8.

/
x+1

^ /IS.

x+1

3x-5 5x+9

x+2

x+3

49.

4x3+4x2+8x+l 2x2+2x+3

_ 2xg+2x+l

X 3
/21
*

X
J-

^-^-^
1-6

"^1-25

_^ _

^=
1-8

2x-3
9
"^

"

206
oo 22.
to a
cbolve
1

ALOEBRA
3a; 4a;+5

3x

= 4a; 5a;+7 4x

by

-,

farst

j u reducing each iraction

mixed expression.
Find three consecutive numbers so that the sum of the second anu } of the third may be 30.
Divide 300 into two parts so that
7,
if

23.
first,

of ^ of the

24.

the other by
answers.
25.

the difference of the quotients will be 18.

one be divided by 5 and Give two

How much
make
a

water must be added to 100

lb.

of a

4%

solution

of salt to

3%

solution

26.

pupil was told to add 3 to a

number and
the

to divide the result

by

5.

Instead of doing so he subtracted 3 and multiplied by 5 and

obtained the correct answer.


27.

What was
of tea

number
lb.

A man

bought 180

lb.

and 560

of coffee, the coffee

costing 1^ as much as the tea per lb. He sold the tea at a loss of 25% and the coffee at a gain of 50%, and gained $62-60 on the whole. What

did the tea cost per


28.
for
I sold

lb. ?

1 lb. less,

some butter at 25c. a lb. If I had received 5c. more I would have received 2c. more per lb. How many lb.

did I
29.

sell ?

If I walk to the station at the rate of 11 yards in 5 seconds have 7 minutes to spare yards in 6 if I walk at the rate of 10 seconds I am 3 minutes late. How far is it to the station ?
I
;

145.

Fractions with similar Denominators.


..

CI

^+6

2x-18

2z-\-3

16

3a;+4

Here we might multiply each term by the L.C.D., which is 132. be found simpler, however, to remove all the fractions but the first, to the same side of the equation, as they are easily reduced to a
It will

common

denominator.

g+6 _

2a; 18

2a:+3

16

3x+4

n "
U ~
a;

x+6 ^

"^ 3 + 3 4 12 4(2a;- 18j-,3(2a;+3)-l:64+3a;-|-4 12

Now
ettrrect

simplify,

cross

multiply

and complete the

solution.

The

answer

is

=6.

FKACTWNAL EQUATION
This

if

207
of

problem

shows

that

the

denominators

certain

fractions are such that these fractions can be conveniently

combined when they are grouped on one


T. o EX. o 2.-Solve
,

side of the equation.

2a:+3

4a:+5 __=.__ + 5a;+4 __^.


,

Since 4x+4 = 4(x+l), second fraction with the

it

is

seen that

it

is

simpler to combine th^

first

than with the third.

2x+3
Subtract the
the result.
T^
first

4a;+5 _ x+l ~ 4{x+\) ~


fractions,

5a:+4
*

5x-{- 1

two

complete the solution and verity

Ex. 3. Solve
Here
it
is

xl

X 2

x2
o a: 3

x4:
K xo

+ x5 = 0
,

xo

laborious to multiply all the fractions by the found easier to change the equation so as to have two fractions on each side, then simplify each side and cross multiply. Solve by transposing the last two fractions, also by transposing the second and fourth, and compare the results.

too

L.C.D.

It will be

Ex. 4.-Solve
AA^Addmg
u on each
-J

-1^

-1_

= ^-1-^ +

-1-g
,.
'

side,

2x+7 x*+7x+10
'

2x+l 2a;+7 ^Tpf^+To = ^^+T^c+12 2x+7 _^j^ x^+lx+V2

^^>

^^"'^ '^

(x^+lx+lO ~ x^+lx+u)

= ^'

x=-S^

or x^

+ lx+lQ = x' + 7x+\2.


the

Since the equation x* 7a;+ 10 = x^+7a;+ 12 is impossible, only root of the given equation is x= 3^. (Verify this root.)

If in line (1) we divide each side of the equation by 2x-\-'i an impossible equation will result. It is not allowable to divide both sides of an equation by a common factor unless we know that the factor is not zero. Here 2x-\-l might be equal

208

aW:BRA

to zero, and, in fact, would be if x= If x=Z\ the 3^. equation in line (1) is satisfied as each side becomes zero. Solve the equation by writing it in the form

ji
X-+5

a;+4~z+3
EZBRCISB
100

L--J

L
x-\-2'

Solve and verify

2x+l
5

_ 6x _ 3a:
~T5
3a:+5
6a;+3'

^
'

2a:+3
4
6x4- 1

6x^ ~
3x-l
2

x\

_ x-\-2
2

a;+3
*

"t"
*
I

6x+2
3

_ 2a:4-l ~ 14
29
,

2x-l
=ft.

4 ^

3a;-2

x-8
_5
"*"

2a;-16
5a;

24

3x-24

12

12x+8

_ 6x4-7 ~ 9x+6
_ 7x-14 _ 23x-88 f2~ ~ 17X-66

13x 10
36

4x+9
~T8
1

3x-4 6x-9
5X-17 18-4x
1
,

12

6x-5 8x^12
23

2x-ll
14

3x-7
21

42

5x-7
lOx-5

10

4x-3 4x-2
1^

Jl

X 1

X 10. X 5
r^.

1111
X 2 X 3
X

l___l

X 7

X
x f6

-i^+
3x-fl2
3

'

'

6x+24

2x4-10

14.

5__
, 1

_8

1_
,,
,
.

15.

X 8
?; 10

X 7 7 X 5 X 4 = X 5 X 7 X 6 X 7 X
, 1

(transpose terms)

FRACTIONAL EQUATIONS
,^
lb.

209

2x-21

x-14

x-1 = x-12 x-8 x-13

2x-n
z-9

17.

Solve the preceding example by changing

2x 27 x-14

'

into 2

+ x-14
'

and making similar changes


._

in the other fractions.

4x-17

lOx-13

X
19.

2x 3
2x-ll

8x-30 2x 7
4X-55

5x-4

X
x-6

5X-64

X 13

X 6

X 14

x+l^x-2^x-l
21.
22.

Solve
b-\-c
If
is

1
\

c+a

a+b
it

3.

a^b^ab,

does

follow that a
?

must be equal to b)

What

the alternative conclusion

Unknown. Equations often which the known quantities are represented bj" letters instead of numbers. These are called literal equations. The same methods are used in solving them as were used
146.
Literal Equations with one

occur

in

in solving

equations with numerical coefficients.

Ex.

1.

Solve az=bx-\-c.
ax~bx = c,
.*.

Solve
.-.

8a;

= 3>c+20
4.

8x-3x = 20,
5x=20,
c
.-.

x(db) = c,

x=^ =

ab

Here the letters a, b, c represent some known numbers whose values, however, are not stated, while x represents the unknown whose value is to be found in terms of a, 6 and c.
represent
represent

Usually the earlier letters of the alphabet are used to known quantities, and the later ones x, y, z to
are practically
coefficients

unknown ones. Compare the two solutions given. They identical. When we work with numerical
result

the

can usually be expressed in a simpler form.

210

ALGEBRA

Note.

The

pupil

must not make the mistake


true,

of giving

x=

as a sohition of ax

= bx-\-c.
but
it is

This statement

is

give the value of the

umknown

in

not a solution, since it does not terms of known quantities only.

Ex. 2. Solve
Removing
brackets,

Transposing,

a{x2)-h = a2x. ax2a b = a2x. a^-[-2x = a+2a + 6,


.-.

a;(a

+ 2) = 3a-t-6,
_3a + 6 "^"0+2'

The result should be verified by substitution, but this will frequently be found more troublesome than the solution. When it is not verified in the usual way, tue pupil should review his work to ensure accuracy.
T^

Ex. 3. Solve
Cross multiply,

r.

^^ ax

xa hx

bxb^ x^-\-bx = axa*x^-\-ax, :. 2bx-2ax = b'-a', _ ^' ' _ ^+a 2" ^~2{b-a)

Verify by substitution.

EXERCISE
Solve for
1.
x,

101

verify 1-12:
2.

mx-\-a=b.
x-{-a

ax=bx-\-2.

3.

a-\-h

c.

_7
3 6
_
'
"

xa_a
x-\-b

_
'

xc_ab
x-{-c

xa

b
.

a-\-b

a_
X
..

xa_xb
xb

xaxh_.
*
'

xa+b.
c

xa
** 11.
I

26
i

2a
L\
c\

ax

10.

=b
a
1

dx
.

o(a; o)4-6(a; o)=0.

Ti

d
12.

_ 1l_ 1 - 1 = 1_
X X
b

13.* (o+a;)(6+a;)=(c+a;)(d4-a;)-

FRACTIONAL EQUATIONS
14.

211

{ax-b)(bx+a)=a(bx^-a).
a
b

15.

"J^^ - ^-^"^ =x.

16.

_ ab
xc

xa

zb

17.

x{xa)+x{xb)=2{xa){xb).
{x-a){x-b) = {x-a-b)^.
Xa

18.
19.

2a

'-^^
x

3a

x 4a

'-.

20
.21.

{xa)(xb)-(x+a){z+b) = {a+b)^.

{a+x){b+x)-a(b+c)^'^ + x^
b

22. 23.

(a4-z)(6-x)+x2=fc(a+x)-

a
a
is

a'^x^b^+abx=a^bH.
The excess
of a

24.

number over

three times

its

excess over

b.

Find the number.


25.

Divide the number a into two parts so that one part


c.

may

contain b as often as the other will contain


26.

Divide a into two parts so that


b.

times the greater

may exceed

n times the less by


27.

A
If

rectangle

is

square of the same area.


28.

feet longer and b feet narrower than a Find the side of the square.
a,

a number be divided by
of the
sells

the

sum

of the divisor, quotient

and one-third
29.

number

is 6.

Find the number.

A man
?

a acres more than the


left.

mth part

of his

farm and

has b acres more than the nth part

How many

acres were in the

farm
30.

Solve {ax){bx)={cx){dx).

Check by putting a=l, 6=6, c=2, d=Z.


31.
If 5

= - (a+i),
=
r
,

solve for w

for a

fori.

32

If 5

solve for a

for

I ;

for

r.

33.

If 8=at-\-^gt^, solve for

for g.

V 2

212

ALGEBRA

147. Literal Equations with two Unknowns. Every simple equation in x and y may be reduced to the form ax-\-by=c, where a, b, and c represent known quantities. If two equations in x and y with literal coefficients be given, the equations may be solved by the same methods as were used with equations with numerical coefficients.

Ex.

1.

Solve

ax-j-byc,

axby=d.
Adding,
2ax=c-\-d,
.".

-x

Subtracting,

2by=c-d,

cd
,".

26

Verify in the usual way.

Ex.

2.

Solve
(1) (2)

ax-\-by=c,

(1) (2)

'mx->rny=k.
Multiply
Multiply

by by

n,
b,

nax-{-'nby=cn.

binx+bny=kb.
x(na 6m) = cn ^6,

Subtracting,

cn kb

nabtn

We

might substitute
it is

to find y, but for X.

this value of x in either of the given equations simpler to solve for y in the same way as we did

Eliminate x from the two equations and find y

bman

or

anbm

Ex.

3.

Solve

a^x-^b^y=c^, a^x+h^y=c.^.

Here the symbols


tities.

They

a,, a.,, etc., are used to represent are read " a one, a two, b one, etc." There

known quanis

no relation

in

value between a^ and a.,, nor a, and fej. The notation is used to obviate the necessity of employing many different letter forms. Solve these equations as in the preceding example and obtain

62Cj-6iC^

ggCi-aiCg
ftjOa-ft^Oi

016,-026,'

FRACTIONAL EQUATIONS
EXERCISE
Solve for
1.
.r

213

102

and

y, verify 1-12:

mx-\-ny^=a,

2.
'

lx-\~my=m,
mx-\-ly=l.

3.

px-\-qy=r,
x-|-j/=0.
^'

mxny=b.
4.

ax-\-hy=a^-{-h'^,

5.

ax-}-by=2ab,

yji.

x-\-y=a-\-b.

bxay=b^a^.
8.

ax^hy=2, ahcb^y=ab.
^

J.
9.

axby=2a^+3b% bx+ay=ab.
a^x+bh/^a^ab+b^,
ax by=a b.

axby=2ab, 2bx+2ay=3b^-a^.

g^

10.

"^

a X a
13.*

+ 1 = 3, n b
y
b
,

^1. ^^ ^

n. a

+ f = 2, h b

12.

a^x-b^y=a-'+b^
r

axby=a^b^.

= 2. = _ ^,
^,
'

a2x+62y=a3-63,

14.

a^x+biy=Ci,

16.

3a

46
y

X
16.

(a+6)a;-(a-6)l/=^a2+6^

17.

^ + ^=c
X
y

xy=ab

X
18.
a:

+ a

^=^ +

2/

= 6.

19.

a-\-b

l:^ +
X
5a
a;

^==22.
y
'356
2/

_rt_

20.

If
If If
z.

ax-\-by=c and

x 2/=l,

prove that x{ca)=y{b-\-c).


that y{qbp)=x{aq

21.
22.
x,

y=ax-\-b and

x=pyq, prove

6).

bx-\-ay-\-cz=ac-{-bc,

ax-{-by=ac,

cy-\-dz=ad,

solve

for

y and
23.

What

is

the value of m, in terms of a and

b,

if

the following

equations are consistent

ax+362/=a2+362,

3x+i/=3a+6,

4tx~Zy=m

14

ALGEBRA
BXE3RCISE;(39ii^

mevlew

of

Chapter XVI)

Solve and vnrify 1-24

5,7-.,-i.-.)=,(-8).
3
'

-xy_^xy_^ 3"^ ^ 2"^


2

A- ifEi-:f;=i2x+] _ 9a;- _ x-U ^~ 2 7 11


^'

'

/2x-3
x-4
"^

3x-2 _ 5x-29a;-4 x-8 " x*-12x+32'

x+1 _
~^

x~3

5-x
6~
*

5+3x _ 4x-7 _ 16x-27 _ x+3


2
3 1^.

2x-3 3x-7 _ ~ 21 5 V"- 2x+l + 3x+5~ a(x a) 6(x 6) = (o + 6;(x o 6). 12. ax+6 = 6x+a.

^,
,/^-

8x + 5 9x-3 4x-3 -To"- 7^+2= -5--

^,

4-x
"13

/*
>6. -^

+ -7

4x

^-^-

16.

+ f=l.

f-f,= 40
3a

5.

^J = ^. axb ex
^

17

A4-l-7l + -i--3
ax + 6i/ = 2o6, aw-6x=o-6. ^ ^
'

^18
tf

1
I

^^
b

19. y^

20.

X r , x+6
^

=1
^
I '

-r x+6
^

21

x+3

^^_^iJ_^+^_^^.
x+4

1^22.

xH-6

x+ 7

x+10

x-10

x-2

^Hi _ ^iJ _ ?Z:5 x-7 V" x-2 x-4 x-6'^x-8


oq.

"

t/ ^^'

8x-3 x+1 _ 1 6'^15x-20 3x-4

(i^5.* (3a-x)(o-6.) + 2ox=^46(a+x).


'\x

A/26. ^

a
2x

\-2b(a-c)

X +r=

c(a

+ b)-\

2x
c

27.

+ ^=^--3+7.?^-^ = 24i-4j,.

28.

1+2^-3x^3 7x-3y_^^^^
-

2.

+ -=

-7,

3x-2y=

-10a:y.

FRACTIONAL EQUATIONS
30.
31.

216

^ + ^ = 2lxy,
What

5y-2x = 24x7/.
make
equal to
,
,

value of y will

x+5

-^+V
,

yl

xl

?^? -^ + y+11
.

Two sums of money are together equal to $1000, and 5J% of 32. the larger exceeds 6J% of the smaller by 16 cents. Find the sums.
Find a fraction such that if 4 be added to its numerator 33. becomes equal to J, but if 4 be added to its denominator it becomes f
34.
If
it

ax + b = cx + d, give the argument which leads to the con-

clusion that X

= a c

indicating at

what point

it

is

assumed that a

and

c are

vmequal.
fractions whose sum is unity. Add between their squares. Show that the result

35.
is

Take any two proper

'onity to the difference

always twice the greater fraction.


36.

A man

He

receives $65 per

the latter.

has 30,000 invested, part at 4|% and the rest at 5J%. annum more income from the former than from How much is invested at each rate ?

37. The sum of three numbers a, b, c is 3036; a is the same multiple of 7 that b is of 4, and also the same multiple of 5 that Find the numbers. c is of 2.

38.
39.

If 5 If
b.

(2a

+ nd d),

solve for a

for d.
a;*

ax by = a*-{-b*, x y = 2b and

+ 2/* = c,
hill

find c in terms of

a and
40.

A man
hill.

can walk 2^ miles an hour up

and 3J

nailes

per hour
of

He walks 56 miles in 20 hours on a road no part is level. How much of it is up hill ? A farm cost 3| times as much as a house. By selling 41.
Jown
at

which

7|% gain and the house at 10%

loss,

$2754 was received.

the farm Find

the cost of each.

In 10 years the total population of a city increased 11%. 42. The foreign population, which was originally ^j of the total, decreased by 1160 and the native population increased by 12%. Find the total

population at the end of the period.

CHAPTER XVn
EXTRACTION OF ROOTS
In art. 65 we have seen any trinomial, which is a perfect square, may be written down by inspection. We have also seen that every quantity has two square
148.

Square Roots by Inspection.

that the square root of

roots differing only in sign.

and

Thus, the square root of a^-]-2ab-\~b^ a^2ab-\-b^ of

is
is

{a-\-b),

{ab).
or

{a-'rb)=a-\-b or
If

ab; (ab)=ab
in
is

ba.

form a root. It is usual, however, to give only the square root which has its first term positive, and we say that the square root of a^-\-2ab-\-b~ is a-\-b and of a^2ab-{-b^ is a 6 or ba.
its

we had

written a^2ab-{-b^
is

equivalent

b^2ab-\-a^,

it

seen

that

ba

EXERCISE
State the square

104 (Oral)

EXTRACTION OF ROOTS

WH

149. Formal Method of Finding Square Root. When the square root of an expression of more than three terms ia required, it is not always possible to write down the square

root

by

inspection.

Thus, to find the square root of

9x4 12a;34- 10x2 4a;+l.


Here we could say that the first term in the square root and that the last term is either +1 or 1, but it evident that there must be another term as well.
3x2,
is is

Let us again examine the square of the binomial a -[-6, which is a2-(-2a6+62 The first term of the square root is a, which is the square root of a2. The second term of the square root, h, may be obtained in two different ways, either from the last term, h^, or from the middle term, 2ab. Let us now see how we could obtain the second term in the square root from the middle term 2ab. This term is twice the product of a which is already found, and of the last term of the square root which is still to be found. If twice the product of a and the last term is 2a6, then we can find the last term of the root by dividing 2ah by 2a, which
gives
b.

The quantity 2a which we use


the square root
divisor is 2a-\-h,
is

to find the second

term

in

called the

trial divisor.

Since a'^-\-2ah-\-'b'^=a^-\-h{2aArh),

we

see that the complete

that

is,

the

trial divisor
it.

with the second term

in the

square root added to


:

The
(1)

steps in the process are

a*-\-2ab-\-b* \o.-{-h
is

The square

root

of

a*

a.

The

a*

square of a is subtracted from the expression leaving 2a6 + 6*.


(2)

2a + 6

+2a6 + 6
2ab + b*

The
2a

trial

divisor for obtaining the


6,

second

term
is

When
root.
(3)

the square root is 2a. divided into 2ab the quotient is


in

the second term in the


is

The complete

divisor

is

2a +6, and when this

multiplied by b

21

ALGEBRA
2ab-{-b^ there
is

and the product subtracted from


square root
is

no remainder.

Tlie

thnn

17

+ 6.

It might be thought that step (3) is unnecessarj-, as the root has already been found in (1) and (2). It is unnecessary if we take for granted that the expression is a perfect square. If you attempt to find the square root of a* + 2a6 + 46* and do not go beyond steps (1) and (2), you would get the result a-\-b, aa before. This, however, is not the correct result. Why ?

We can now extend the method to find the square root of a quantity of more than three terms.
9x*- 12x'+ 10x-4x+
9x*
1
|

3x-2a;+

6z^-2x -12x3+I0x2-4x+l
\

-12x+

4x

6x 4x+I |6x'-4x+l

6x-4x+l
After finding the first, two terms in the root, as in the previous example, the 3x^ 2x is treated as a single quantity and tlie second trial divisor is twice 3x^- 2x or 6x^ 4x. The square root is 3x' 2x+ 1.

150.
in

Verifying

Square

Root.

We

might verify the result

the preceding example by

writing

down

the square ol

3:c- 2x-4-l.

Verify in this way.


is

simple method of checking


for x.

to substitute a particular

number
^nd

When x=l, 9x<- 12r+ 10x- 4x+ 1 = 9- 12+ 10-4+ 1 = 4, =2. 3x2- 2x+ 1 = 3-2+1
Since the square root of 4
is 2,

we presume

the work

is

correct.

EXERCISE
1.

106

Find the square root, by the formal method, and verify the results

x2+12x+36.
9xH24x!/+16;/2.

?. 4. 6. 8.

9a2-6a+l.
25x'^\0zy+t/-.

3.

5.

\-\Sab + Hla'b'.

49a*-28a^b^+ib*.

7.

aH2a^-3a* 4a4-l

4x*+4x'+5a;+2a;+l.

EXTRACTION OF ROOTS
9.

21

z-6a;3+17a;2-24x+16.
a*-4a36+6o262_4Q/,3^ft4^
13. 14.

10.

9a*-12a3+34a2fiOa+25
a^-^a'^+^a^^.

11.

^12.

9a*

+ 12a36+34a262_(_20a63+256*.

x-4x5+6x3+8x2+4x+l.
a;*-2x3+2x2-x+i.
a*

15.

4a3
fc"

6*^
,

2a^_4a
fe2
ft

^
its

17.

o2_4a6+6ac+462 126c+9c2.
square root.

18.* Simplify a(a+l)(a+2)(a+3)4-l, and find


19.

By
If
is

extracting the square root of

value of X which will


20.
>vhat

make

it

x*+4x'+6x^+3x+7, find a a perfect square. (Verify by substitution.)


x* 8x^+30x2 56x+49 be x^+mx+T

the square root of

the value of

21.

Using factors,

find the square root of

(x2+3x+2)(x2+5x+6)(x2+4x+3).
22.
of

Find the

first

three terms in the square root of

2x Sx^

and

4-12x.
23.

When x 10,

the

number 44,944 may be written


4x+4x3+9a:^+4x+4.

Find the square root of the


44,944.

latter

and thus deduce the square root

of

In algebra, an expression of which the square root is is usually a perfect square. When such is the case the formal method may be greatly abbreviated.
151.

required

Ex.

1.

Find the square root of


-r4-4a;3+10a;2 12x+9.
term
is

The The
.'.

first

x'

and the
is

last is

+3

or

3.
is

trial divisor for

obtaining the second term of the root

2x',

therefore the second term

the square root

is

4x*-i-2x' or 2x. x* 2x + 3 or x* 2x 3.

22t

ALGEBRA

If -(v* square ar* 2x + 3, the term containing x will be twice the product of 2x and 3 or - 12x. If we square x* 2x 3, the term containing x will be +12x. We thus see that if the expression is a perfect square, the square

root

is

X*

2x+3.
by putting

Check

this

x= 1.

Ex.

2.

Find

the square root of

4x4+20x3+ 13x2-30a:+9.
What is the first term in the square root ? What is the trial divisor 7 What is the second term in the root ? What may the last term be ? What is the square root ? (Verify your answer.)

Ex.

3.

Find the square root of


-3a3+-V-+a*-5a+fla2.
in

Write the erpression

descending powers of

a.

The
.".

first

term
is

in the root is o*.


is

The

trial divisor is 2a*, therefore

the second term

3a'-^2a*

or 3a.

the root
is it ?

a* ^a-fj ora* 5a .
(Verify

Which

by squaring.)

Ex.

4.

Find

the square root of


4

Here the terms are already arranged


the term

in

descending powers of

o,

+9
2

coming between a and


in the root
is

-.

The
la^t 13

first

term

2a, the

second

is

4a -r 4a or 1,

and the

+-.

~a

Complete and

verify.

It will be recognized that

cases that
It
is

it is only in the most complicated necessary to use the formal method in full. advisable to use the contracted method whenever it is

possible.

EXTRACTION OF ROOTS
BXBRCISB
Find the square
results.

221

106
Verify the

root,

using any mt^thod you prefer.

1.
3.
5. 7.

x*+2x^-^x^-2x+l.

2.

x*-4kx^+6x^-4x+l.

o* 6084.502+ 12a+4.
9a*-6a3+13a2-4a+4.
4a;*+20a;3_3x2-70x+49.
67a;2+49+9a;*-70z-30x3.
a:+2x3-a;+].

4.
6.

x*+8x^+12x^lQx+^.
x*

+ 6xhj+7x^y^6xy^+y*

x^' l-l0z+21x^-l0x^+x\
10. 12.

9.

ai2_8a9+18a-8a34-l.

11. 13.

x^-2x^+?,x^-lx+^^.

a-6a2

+ ll --, + 1 a^ a*
i^a;2_^a;^i.

14.

15.

- - 3x3 ^
4

17.

54-

1
X^

!_2x+x2.
X

19.* (x+i/)-4(x+2/)3
20.

+ 6(x+2/)2-4(x+2/) +

l.

x2(x-5a)(x a)+a2(3x-a)2-3a2x2.
(a-6)2Ka-6)'-2(a2+62)f + 2(a*+6*).
(a4-6)*-2(a2462)(o4fe)242(a*+6*)(..
If

21. 22. 23.


24.
of

+ J.) + 4(.'+i^) + 6.
is

a;*+6x'+ 7x^^4-0x41

a perfect square,

what

is

the 7aiue

25.

If the

sum

of the squares of
of their product,

any two consecutive integers be


prove that the result
will

added to the square


square.

be a

26.

If

4cX*+l2x^y+kx'^y^-{-6xy^-\-y*

is

a perfect square, find

k.

27

If

m=X

and n=^ y

-,
y

show fjat

mn4V(m2+4)(n24-4)=2x2/428.

xy

Find the square root of 4x*48x348x244x41.

Check when

a:=10.

222
152.

ALGEBRA
Cube
of

Monomial.

When
is

three equal factors are

multiphefl together, the product

called the cube of each of

the factors.
Thus, the cube of 2a or (2a)3=2a 2a 2a = 8a', the cube of a* or (a*)' =a* a* a* = a^, and the cube of Za^ or (Sa')' = ^^a^ Sa' 3a= 27a*.
. .
. . .
.

The cube of a monomial


each factor of
it.

is

found by writing down

the cube of

Thus, the cube of 5ab*x

is

125a'6'x*.

153.

Cube
a-\-b

of

Binomial.
its

cube of
a-\-b.

by multiplying
also the

Find the square by

2a6 "^^
a^

+ 6

Find

cube of ab.

{a+bY=(i^+'^a''h + Zah-'-\-b\

+ 2a^b+ ab* + a^by,ab^+b^

Note that in each case the cube contains four terms, in descending powers of a and ascending powers of b, and the numerical coefficients are 1, 3, 3, 1. The cube of ab is the same as the cube of a-\-b, except that the signs are alternately plus and minus. From the forms of these two cubes, the cubes of other expressions may be written down.
Ex.
1.

Ex.

2.

Ex.

3.

+ 2yy = x^ + 3x^{2y) + 3x{2y)^ + (2y)\ = x^ + 6x*y+l2xy^ + 8y^. (32r-2i/)3 = {3x)3-3(3x)'(2i/) + 3(3x)(2i/)-{2i/)3, = 21x^-54x^y+36xy^-Sy^. (a^-6 + c)3 = (a-^-fe + c)^
{x

= (a + 6)3 + 3(a + 6)c + 3(a + 6)c + c3, = a + 3a*6 + 3a6 + 63 + 3aSc + 6a6c + 36c + 3oc* + 36c + c', =a* + b* + c' + Z{a*b + ab* + b*c + bc* + c*a + ca*) + 6abc.

EXTRACTION OF ROOTS

223

224
155.

ALGEBRA
Cube Root
of

Compound

Expression.
is is

The cube and or

root of a^-{-2a^b

+ 3ab^^b^

a-\-b,

a^2a~b^3nb^-b^
when an expression
its

ab.

Therefore,

a perfect cube,

of four terms is known to be cube root can at once be written down


of its first

by finding the cube roots

and

last terms.
is

Ex. 1. The cube root of x^--6x*y-\-\2xy^ 8y^ cube root of x' is z and of Sy^ is 2y. Ex. 2. The cube root
of

x2y,

since the

a^-2a*b + lab*-

^^b^

is

evidently a-^b.

In the cube of a-\-b, the second term is 3a^b. After finding first term a of the cube root, we might have found the second term of the root by dividing Sa^b by Sa^, that is, by three times the square of the term already found.
the
Thus, the second term of the cube root in Ex.
1

is

and

in

Ex.

is

6x*?/-^3x* 2a*6H-3a'

or

2y,

or -6.

Here
trial

three times the square of the first term of the root is the

divisor,

corresponding to twice

the first

term in finding

the square root.

Ex.

3.

Find

the cube root of

The The

first

8x+12x5-30x4 35a;3+45x2+27a; 27. term in the root is 2x* and the last is 3.
the second term of the root
is is

trial divisor for finding

3(2x*^

or I2x^ .". the second term of the root .". the cube root is 2x*+s; 3.

12x^-1- 12x* or x.

It is thus seen that it is easier to find cube root by inspection than to find square root, as in finding cube root there is no ambiguity as to the sign of the last term in the root. 156.

root

Higher Roots. Since (x^)^=x*, we may find the fourth by taking the square root and then the square root of

the result.

EXTRACTTON OF ROOTS
Also, since {x^)^=z^

225

root

and {x^)^=x^, we can find the sixth by taking the square root of the cube root, or the cube
the

root of the square loot.


Thus,
square root of x*+8x^ + 24:X* +
is

32x+16

is

x*

+ 4x + 4,

therefore the fourth root

x-\-2.
is

The cube root

of

x^-Qx^+\5x* 20x^+l5x* Qx+\


is

x*-2z+l,

therefore the sixth root

x 1.

BXE3RCISE
State the cube root of
1.
:

108

(1-15,

Orali

-64.

2.

27a3.

3. 6. 8.

-\25a%^.

4.

-8(a-6)3

5. 7.

x3+3a;24-3a;+l.

x^-'^xh^+Zxy'^-y^.

a3+6a2+12a+8.

9.x^-\2x^+Qx-\.

9.

a;V+3xV+3x2/+l.
125x3-75x2+ 15x-l.

10.
12.

64a3-144a2+108a-27.
27x3-27x22/+9x?/2-j/3.

11.

15.

m3

- 9w + -
first

'

"^

16.

:r-

- 6x< +

12x2m3

_ gw*.
What

'

f.

17.* In finding the cube root of

x+3x5+6x*+7x3+6x2-t-3x+l,

what
the

is

the

term

in the root

What

is
?

the last term

is
?

trial divisor for

finding the second term

What

is

the cube root


^

Check by substituting x=l.


Find the cube root and check
v^lS.
:

'

t.

^
27
27

^*l

u^^Sim

l-6x+21x2-44x3+63x*-54x5+27x.
^ 2-,-3+2x-7 + ---. + ^- -f
a;3
,

I-Z^^-Ja..*

**

-n a9.
20. 21.
22. 23.

x2

18

.^

-/

^l
^-iZ

Ik^im

^^

t^

27a-108a + 171a*-136a=+57a'=-12a+l.
(I

^q^"

VQ.

-/
?

+ 3x2)2-x2(3+x2)2.
will

^^''^itOiay
x3+3cx2+2c2x+5c3 be a
perfect cube

For what value of x

Find the fourth root of

X* 4x^+6x2 4x+l,

'^i.i^-.'i.'n
(f)i

'^

226
24.
25.

ALGEBRA
Find the fourth root of a 12a3-)-54a* 108a Find the sixth root of

+ 81

x-12x* + 60x*- 160^3+240x2- 192x4-64.

EXERCISE
Find the square root of
1.* 9z*-24xt/
2.
3.

109 (Review of Chapter XVII)


:

+ + 4xS-2x-10x'+I3x*-6x+l. xi + 6xi'' + 5x*-8x+16x*-8x* + 4.


28x*2/-16xt/4-4t/.
x

4.
6. 6. 7.

Jx-Jx+Vx*-5x+l.
12a3x-26a*x* + 25x + 9a-20ax3.

+ 3a) + (3a+7). (x* + 5x + 6)(x + 7x+12)(x* + 6x+8).


4x(7-f x*

8.
9.

(2x'-x-3)(x*-4x-5)(2x*-13x+15).
4x
is

20x

33x

32x

34

+ -^ XX*
^_^
*

What
10.
11.

the cube root of

27-136x+225x-125x.
8x-12x'>+]8x-13x-f 9x-3x+l.
(a-fe)3

t,

'^*

'I**

"*"

12.

+ 36(fc-)*-f 3fe(-*) + i.
:

13.

Find to three terms, the square roots of

l-2x, 1-a,

Find the value of y for which 14. square and prove by trial that your result
16.
last

+ x. x* 2(a y)x + y*
is

is

a complete

correct.

The first two terms of a perfect square are 49x* 28x*, and the two are -|-6x+|. What must the square root be ?
Prove that the product of any by unity is a -perfect square.
four

16.

consecutive

integers

increased

Find the square root of a* + 4a*+6o* + 4a+ 17. square root of 14,641.
18.

and deduce the

By

finding the cube root, simplify


{a

t.

If a

+ b)' + 3(a + b)*(a-b} + 3(a + b){a-b)' + {a-b)\ = fc4- 1, show tiiat a> fc- = 3a6.
l

3;

EXTRACTION OF ROOTS

227

Show that the product of any four consecutive even integers 20. How might the result be deduced increased by 16 is a perfect square.
from No. 16
21.
?

By

inspection, find the values of

(a-6)*+(ft-c)* + (c-a)* + 2(a-6)(fe-c) + 2(6-c)(c-a) + 2(c-a)(a-6),

{2x-y)^-Z{2x-y)^{2x + y) + Z(2x-y){2x+y)*-{2x + y)\


22.

To the square

of the

integers,

add the square

of tlieir

double product of any two consecutive sum. Prove that the result is always

a perfect square. 23.

Express in symbols

The
of

difference of the cubes of

numbers exceeds the cube product multiplied by their


24.

their difference

any two by three times theit


this
is

difference.

Prove that

true.

The expression 8x9-36a:8 + 66x'-87a;+105x*-87x* + 61x-42x+12x-8 Find its cube root by getting two terms from the is a perfect cube. first two terms of the expression and the other two from the last two terms. Check when x=I.

What number must be added to the product of any four 25. consecutive odd integers so that the sum may be a perfect square ?
Show that the sum of the cubes of three consecutive integers 26. exceeds three times their product by nine times the middle integer.
27.

Find the cube root of

(4x-l) + (2x-3)3 + 6(4x-l)(2x-3)(3x-2).


[Note that
28.
If
it is

of the

form a^ ^b^ -\-Zab{a-\-b).'\


first

4x*+

12x^4- 5x*2x* are the

four terms of an exact

square, find the remaining three terms.

^y^<^-^

Q 2

CHAPTER

XVIII

QUADRATIC SURDS
157.

Surd.

When
is

the root of a

number cannot be

exactly

found, that root

called a surd.

equal to

Thus, we cannot find exactly the number whose square is 2, and we represent the number by the symbol a/2
it is

and we call V2 a surd. If no surd appears in any quantity,


quantity.

called a

rational

By the arithmetical process of extracting the square root of 2, we can obtain the value of V 2 to as many decimal places as we pleaae, but its exact value can not be found. To four decimal places the value of V2 is 1-4142. Find the square of 1-4142 by multiplication and see how closely it approximates to 2. We can find geometrically a line whose length is V2 units. In this square, whose side is 1 unit, draw the diagonal BD. Then, from geometry, we know that

:.

:.

BD*=l*+\* = 2, BD =V2.

On squared paper mark


whose
side
is

10 \inits.

the corners of a square Measure the diagonal and

thus estimate as closely as you can the value of V 2. Make a diagram like this to show how to represent
graphically lines whose lengths are V^, VS, etc. Take the unit line 1 inch in length.

vi, V5,

What

test

have you of the accuracy


158.

of

your drawing

Quadratic surd.
to be

surd hke

V2

in

root

is

found

is

called a quadratic surd.

which the square In this Chapter

quadratic surds only are considered.

QUADRATIC 3URD9
159.

228'

Multiplication of Simple Surds.


is 2,

Since \/2 represents a quantity whose square


.-.

\/2x V'2=2=V4,

also

^4x^9 = ^/36,

because

2x3=6.
this

Similarly,
is

true

we might expect that -v/2x V3 = V6. That may be shown by finding the square of y/2 x V3.

(\/2xv'3)2=V2xV3x\/2xV3,

[Just as {ab)'^=a .b .a .b.]

= \/2xV2x VSxVS,
=2x3=6.
.-.

a/2x\/3=\/6.
A/3xV'5=Vi5,
y/a x

Similarly,

and

Vb= Voft.
is

Therefore, the product of the square roots of two numbers equal to the square root of the product of the numbers.

Since

Vab=VaxVb,
;

.'.

\/f2=V4x a/3=2a/3,

and

V'50=\/25x V2=5\/2
Thus,

VT8a^=V9a^xV2a=3aV2a.

see that if there is a square factor under the radical sign, that factor may be removed if its square root be taken.
Conversely,

we

5^3= a/25 x V3= VTS, aVb=Va^xVb=Va*b,


axViny= Va*x* x

Vmy= Va'x*'my.
quantity
it is is is

Mixed and Entire Surds. When a surd 160. the product of a rational quantity and a surd, mixed surd. If there is no rational factor it
entire surd.

called a

called an

Thus, 5V3, aVb,

{a

b)Vx y

are

mixed

surds,

and V3, V50,

Vax-\-b are entire surds.

In the preceding article we have shown that a mixed surd can always be changed into an entire surd, and an entire surd can sometimes be changed into a mixed surd.

230

ALGEBRA
surd
is

said to be in

its
is

simplest form
integral

when the quantity

under the radical sign


factor.

and contains no squar*

Thus, the simplest form of

V50

is

5\/2.

EXERCISE
Find the product
1. of
:

110 (1-29. Oral)

V2, V3.

QUADRATIC SURDS
f

231

^4.
radius
35.

If the area of
(7r

circle is

66 square inches, find the length of the

= 3^).
of the squares of

The sum
is

two surds, one

of

which

is

double

the other,
36.

40.

Find the surds.


of the diagonal of a square
is

The length

10 inches.

Find

the length of the side.


37.

One

side of a rectangle

is

three times the other

and the area

is

96 square inches.
161.

Find the

sides.

rational factor

In the surd quantity 5\/3, 5 is a is called a surd factor. When aurds, in their simplest form, haye the same surd factor, they are called like surds or similar surds, otherwise they are unlike surds.
Like
Surds.

and

VS

Thus,

3V2, 5^2,
2\/3,

and
162.

aV2 3\/2, jV5

are like surds.

" "^

are unlike surds.

Addition and Subtraction of Like Surds. Like surds be added or subtracted, the result being expressed in the form of a surd.

may

Thus,

3V'2 + 5V'2 = 8-\/2, just as 3o+5o=8a.

7V'3-4\/3 = 3\/3, just as 7a-4a=3a. a/76-2v'3 = 5\/3-2v/3 = 3V3.


V'50 +

V32-\/r8 = 5V2 + 4V2-3\/2 = 6\/2.

Che

sum

or difference of unlike surds can only be indicated.

Thus, V2-\-V^ can not be combined into a single surd, but the approximate values oi V2 and V3 may be found and added. Show that V2+v'3 = \/5 is not true, by finding the square roots of 2, 3 and 5 each to two decimals.
Is it true that V'i

+ VQ=VYZ
BXBRCISE

111

(1-8,

Oral)

Express as a single surd


1.

3^2+5^2.

2. 6.

5-\/7-3V7.

3.
6.

2Va+3Va.

4.

2Vx+5Vx-Vx.

VS+V2.

^12+^3.

232
7.

ALGEBRA
Vis-Vs.
V75+Vr2+3\/3.
8.

V4a+V9a.
2V18+3a/8-5\/2.
2\/63-5\/28+a/7.

"^
11
13.

10.
12.
14.

Vi5-V2b+V80.
4\/f28+4\/56-5Vr62.

10^44-4^99.

15. 16.

A/45+V20-\/80+\|r80.

\/72+a/98-V128-V32-\/50.
:

Simplify the following and find their numerical values, correct to two decimal places, using the square root table
17.

VT5.

18.

V63.

19.
2^

V60+\/l5.

20.

VI^-2VI2.

21.

VI28-VT62.

V56+V'72+V'90.
ix2-4 = 19.
^x2=Jx2_47.
Find
1

Solve, finding x to three decimal places

23. 26. 29.

x2=37***'^,
31x2=132.
27.

3x2+5=50.
i(3x2-ll)=53.
is

25. 28.

The area

of a circle

176 square inches

its

radius.

Square Roots of Numbers from


n

to 50

QUADRATIC HvltDS

233

It is thus seen that the product of two surds is found by multiplying the product of the rational factors by the product of the surd factors.

5V3x2V3=10.3=30,
also
It,

ay/cxhVc=abc.
therefore, follows that the product of two lik^ surds
i'

always a rational quantity.

Ex. 1.Multiply

VSO by Vl5.

Here the surds should be simplified before multipl3dng.


Since
.-.

V50 = 5V'2 and V75 = 5V'3, V50X V75 = 5\/2x5V3 = 25v'6.


2-i-2\/3

Ex.

2.

Multiply
2

+ 2V3

by 3-V2.

^~ ^^
6-j-6v3
~
6

Here the multiplication is performed ma manner similar to the multiplication oi a-\-b by x + v-

+ 6\/3-2\/2-2V6-

2v2 2v6 ~ ^

164.

Conjugate Surds.

If

we wish

to multiply

5V3+2V2
we may

by 5\/3-2\/2,

follow the same method as in the preceding example. These expressions, however, are seen to be of the same form as a-\-b and ab,
:.

(5\/3

+ 2V'2)(5V3-2\/2) = (5\/3)2-(2\/2)2=75-8=67.
(3+ V2)(3- V2) = 9-2 = 7,

Similarly,

and

(2-Vl0)(2+\/l0) = 4-10=-6.

Such surd quantities as these which differ only in the s)gn which connects their terms are called conjugate surds. Note that the product of two conjugate surds is always a
rational quaniitv.

234

ALOEBRA
E3XERCISB
112 (1-12. Oral)

Find the product of


1.

2V3,

3a/5.

2. 5.

5\/2, 6a/3.
{2\/3)2, (V2)2.
8.

3.
6.

aVb, bVa.
\/2+l, ^2.

4.
7.

W2,V3,V5.
A/3-1-V5,
-\/2.

\/a+V'6-l, Vc.

9.

V3 + V2, V3-V2.

10.
12.

Vro-

3,

VIO+3.
2v'2 VS.

11.

VxVy,Vx+Vy.
{\/3+a/2)2.
(3\/2-|-2V'3)2.

2\/2-|-\/3,

^?3a/6, 4V2.
15. 17. 19. 21. 23.

14.

3V^, 4\/7,J\/2.
(2\/6-V7)'-

16.
18. 20.

(Va+\^)2.
3\/2 + 2V3,

4-1-3V2, 5-3A/2.

5^2-3^3.

22. ZV5-W2, 2\/5+3"v/2. 3Va~2Vb, 2Va-3Vb. V5+V3+V2, ^5+^3-^2. ^Sk' V7+2V2-V3, V7-2V2+V3. 25. Va+b3, Va+b-]-2. Va+Va 1, Va Va 26.

1.

27.
29-.

(a/3+\/2-|-1)2.

28.
i^-^J!

(V'5+2\/2-\/3)^-

(Va+b+Va^)^.

(3V'^^-2Vx+y)^-

Simplify
31.

(6-2a/3)(6+2\/3)-(5-V2)(5+v/2).

.-/

^^,.32f"(\/3-\/2+l)2+{A/3-|-V2-l)2.

^8^r^50- VI8+ \/72+ V32) X iVS.


^3^/2(4\/3-|-3\/2)(3\/3-2\/2) + (5\/2"-3\/3)(4\/2+2\/3).
y5.
36.
(

V3+ \/2)(2\/3 - \/2)( \/3-2\/2)( V3-3V2).


squaring

By

VlO-f Vs and

Vs+Vl,

find whicii
lies

is

the greater

37. The product of 5\/3-f SV? and two consecutive integers ?

3^3 V?

between what

38.
taches.

Find the area of a rectangle whose sides are

5+ V'2 and 10 2^?

QUADRATIC SURDS
39.
inches.

235
7-(-4V2 and
7

The

sides of a right-angled triangle are

4v'2

Find the hypotenuse.

40.
is

The base

of a triangle is

2^3 + 3^2

inches and the altitude


places.

3V3+2\/2
165.

inches.

Find the area to two decimal

Division of Surds.

Since

VaxVb=V^,

:.

Vab^Va=J-'=Vb. y
a
,

Inh

Similarly,

Va ~ Vh = 7= = \/ Vh ^ h
3\/l5-f2V5=|V3.

and
Ex.
(1)

1.

Find
might

the numerical value of V5-ffind the square roots of 5

V2

or p^

V2

We

and

and perform the

required division.

(2)

,_,

V5

VSh- ^2 = 2-236-:- 1-414= 1-581 V'5-^V2=\/|=V'275= 1-581. \/lO 3-162_,_-\/5xV2


at once seen to be s.'^pler than

Here the third method


either of the others.

is

In

(3)
^

we changed
,

Vs
-^

into

VlO
,

that

is,

V2

we made
is

the

denominator a rational quantity.


rationalizing the denominator.

This operation

called

Ex. 2. Find the value of ^^.

if

^2=14142.
we
first

V2
by
a/2
-jr-

Here, instead denominator.

of dividing

1-4142,

rationalize

the

Then

1 lxV2 = ^ = ^ y^

1-4142

\/2

v/2xV2

2 = ^
2\/6
is

-7071.

_-_,

Ex. 3. Divide

6^8 by
2^2 sVa

10a/27.

6V8

6x2\/2

mV2i

JOxsVa

2xV2xV3 sxVsxVs

2x2-4496
15

= -32W

236

ALGEBRA
4.

hx.

Rationalize
by
its

the denominator of
will

-
be rational

We

have already seen that the denominator


it

if

we

multiply

conjugate

3 V

5.

2+\/5 _ C2+V'5)(3- \/5) 3+A/5 ~^4-a/5)(3-V5)

^ + Vh ^ + Vl ~ 9-5 ~ *
l l

Ex. 5. Divide 5 + 2^3 by 7-4\/3.


5
I

Write the quotient

in the fractional

form

2\/3
,_.

7-4\/3

rationalize the
.

denominator and simplify.

EXERCISE
Divide
:

113

(1

QUADRATIC SURDS
Solve, giving the value of x to

JSSTi

two decimal

places, using the table

28.
31.

a;\/2=3.

29.

xVz=V2.
32.

30.

a;V'3=\/2 + l.

x\/2-xV2=\.
x2(\/3-l)==2(\/3+l).

xV5-5=2x-y/5.

33. 34.

The area of a triangle is 2 square feet. V6-\-VZ feet. Find the base to three decimals.
35.

The

altitude

is

Simplify

2+VlO
4\/2+\/20-\/18-a/5
Equations.
is

166.

Surd

the

unknown quantity
of the terms.

surd equation is one found under the root sign,

in
in

which one or

more

Thus, V'a;+7 = 4,

Vx+Vx 5 = 5,

are surd equations.

Ex. 1. Solve
Square both
Verification

Vx^=2.
a; 3 = 4,
.'.

sides,

x=7.
2.

Vx^ = VT^ = Vi =

Ex. 2. Solve
Squaring,
Verification

V5^^-2a/^3=0.
\/5x\ = 2^/x-\-%.

Transpose 2\/x+3,

5x l = 4a;+12,
.-.

x=13.
the positive square root only,

-v/5a;

2 Vx4- 3= \/64 2-^/^6 = 8 8 = 0.


we have taken

Note that

in verifying

as defined in art. 63.

E3XBRCISB
Solve and verify
1.
4. 7.
:

114

(1-8,

Oral)

2Vx=Q.
\/x+2=4.

2.
5.

Vx 5=4. Wx=V20.
8.

3.

Q\/x=l.

6,

Vx 6=a.
'
.

m+V'i=n.

9.

Vx2+9=9-x.
V9x2-llx-5=3x-2.

10.

7 Vx 4=3. Vx2+llx+3=x+5=
2x-\/4x2-10x+4=4.

11.

CBL
44^,

2a+Vz+a^=b+a.

V(x-o)+2a6+6=*-a+6,

38

ALGEBRA
EXERCISE
116 (Review of Chapter XVIII)

Simplify

1*

QUADRATIC SURDS
^2f\

239

Find the value of

(2V2+V3){3V2- \/3)(3 V3- V2).


28.
If the sides of a right-angled triangle are
is

Vs+l

and

V3

1,

what
28C ^^
I

the length of the hypotenuse


c!r. Simplify -7=

oo^
30.

^5-1 Vo-2

V5-3 \/o + 3
jz:

and

j:z

V 3+V2 7^ V3-V2
of

V3-V2 ~
V3+A/2
7=

Find the value to two decimal places

x+y xy
4.,afr^ Multiply

x-y ^henx=2 + V'3, x+y

y = 2-V3.

2\/30-3V'5 + 5\/3 by

V3 + 2\/2-V5.

t,3gf^ Multiply

^7 + 2^6

by \/7-2V6.
is

The area of a rectangle is ISVlO 25 and one side 33. Find the other side to two decimal places.

3V5 V2-

CHAPTER XIX
QUADRATIC EQUATIONS
quadratic equation has already "been defined in In the same article we considered the method of solving some of the simpler forms of it. Quadratic equations frequenlly occur in the solution of problems as shown in the following examples.
167.
art.

104.

Ex.
462.

1.

Find

two consecutive numbers whose product


x-\-\.

is

Let the numbers be x and

a;(a;+l)
.-.

= 462,

x4-a;-462 = 0.

Ex. 2. The length of a rectangle is 10 feet more than the width and the area is 875 square feet. Find the dimensions.
Let
.".

x = the number of feet


a:+ 10 = the

in the width,

number

of feet in the length,

x(x+10) = 875,
.-.

a;+l Ox -875

= 0.

Ex.
Let

3.

Divide 20 into two parts so that the sum of their


be 36 more than twice their product.
a;

squares

may

= one

part,

20 x = the other part,


x x + (20-a;)*=2a;(20-a;) + 36, + 400-40x + x = 40x-2x* + 36,

.-.

4x- 80x4-364 = 0,
x-20a; + 91
J40

= 0.

QDADRATIC EQUATIONS
exercise:
116

Ul

Represent the number to be found by x and obtain, in its simplest torm, the quadratic equation which must be solved in each of the
following
1.*
2.
3.

The sum

of a

number and
is

its

square
less

is

132.
its

Find the number.


square.

Find the number which

156

than

The sum

of the squares of three consecutive

numbers

is

149.

Find the middle number.


4.
112.

The product

of a

number and the number increased by 6


five

is

Find the number.


times the width.

The length of a rectangle is 6 feet less than 5. The area is 440 square feet. Find the width.

The average number of words on each page of a book is 6 more 6. than the number of pages. The total number of words is 9400. Find
the

number
7.

of pages. of a rectangle
is

The area

88 square inches and the perimeter

is

38 inches.
168.

Find the length.

Standard

Form

of

the

Quadratic

Equation.

Every

quadratic equation

may

be reduced to the form

ax^-\-bx-\-c^O,

and c are any known numbers, except that a not be zero. The term not containing x is called the absolute term. It is frequently necessary to simplify equations to bring them to the standard form, and thus determine if they are quadratic equations.
in vv'hich a, b
'jan

Ex.

1.

(a;+l)(2a;+3)=4x2-22,
2x* + 5x + 3 = 4a;-22,
.-.

-2a;* + 5x + 25 = 0,

2a;-5x-25 = 0.
Here the equation is seen to be a quadratic. of a; is 5 and the absolute term is 25.
Or, o =
2,

The

coefficient of x*

is

2..

6= 5, c=-25.

242

ALOE BR A
2.
-j-

Ex.

4
:.

=
X

1,

7* + 4(a;-7) = 4x,

7x-28 = 0,

x-4 = 0.
Here a=l, 6 = 0,
c

= --4.
2x
a:

4-1

.-.

2x(x+2) + (x+l)(x-l) = 3(a;-l)(ar+2),


2x* + 4x+x*-l-=3x +

3x-6,

x+5 = 0.
Here a = 0, 6=1,
the coefficient of x*
c

= 5,

and the equation

is

not a quadratic, since

is

zero

EXERCISE
Reduce
in

117
of a, b

to the standard
is

form and state the values


:

and

c,

which a
1.*
3.

always positive

6x2=x+22.
19x=15-8x.

2.

6.

=
-^

5x.

QUADRATIC EQUATIONS
Ex. 1.Solve

243

2x^3x=0.
a;(2x-3) = 0,

x=0
x=
Verify both roots.

or or

2x-3 =0,
f.

Ex. 2.Solve
.'.

ax^+bz=0.
x{ax + b) = 0,

x= x=

or ax or

+ b = 0,
b

.'.

a
zero, the equation

(2)

\Mien the middle term


b}' factoring,

is

can always

be solved

or bj' extracting the square root.

Ex.Solve
.*.

3a;2 27=0.
3(z-3)(x + 3) = 0,
a;-3 =
or x-f3

= 0,

x=3.
Or thus,

3x-27 = 0,
.-.

x*

= 9,
when none of the quadratic expression, any of the methods previously
the
effected.

:.

x=3.

(3)

The equation
a,
b,

is

a complete quadratic
zero.
If

coefficients
ax^-\-bx-\-c,

is

given, the solution

can be factored b}' is then easily

Ex.

1.Solve
.-.

3x2-lla;=14.
3x-lla;-14 = 0,
(a;+l)(3a;-14) = 0,
a;

=1

or

Verify both of these roots.

Ex.

2.

Solve

x^mx-\-nxmn^=0.
t(x 7n)-(-n(x m) = 0,
.'.

(x

x=

m)(x + n) = 0, m or n,
E 2

244

ALGEBRA
EXERCISE
1-12.

118

Solve the equations in the preceding exercise and verify. Solve the problems in the
first

13-19.

exercise in this Chapter.

(Verify the results.)

Solve by factoring and verify

^^.
24. 26.

a:^ mx 6/7j2=rO.

^^. x^ ax 6x+a6==0.
25.
27.

x2-L2x(a+6)4-4a6=0.

2ax2+aa; 2x=l.

(x a)(x-6)=a6.
:

x^ a2=(x-a)(6+c).
whose

170.

sum

is

Find two numbers Consider the problem 100 and whose product is 2491.

Let

100 x=the
.-.

a:=one number, other number,

a;(100-x) = 2491,

x2- 100^+2491=0.

.solve this equation by the preceding method, we must two factors of 2491 whose sum is 100, but this is exactly what the problem requires us to find. The necessity is therefore seen for another method of

To

find

the quadratic equation when the factors of the quadratic expression cannot be obtained readily by inspection.
solving
171.

Solution

by

Completing

the

Square.

We know

that

(x-|-a)2=x2+2ax-i-a2, the middle term being twice the product


of X
If

and
the

a.

first

asc

two terms of a square are x^A^lax, we know that it must be the square of x+a, and, therefore, a- must be added to x^-\-2ax to make a
complete square. What is the area of the stiaded portion in

"

the

diagram

Similarly, x*-i-4x must be the first two terms in the square of x+2. To make x'-(-4x a complete square we must add 2* or 4. Also, x* 8x are the first two terms in the square of x 4, and, therefore, 4' or 16

must be added.

QUADRATIC EQUATIONS
To complete
added
is

245

the square,

it is

seen that the quantity to be


jc.

the square of half of the coelTicient of

Ex. 1. Factor

a;2^6a: 40.

Add
Then

9 to x'-\-6x to

make a complete

square.

x*

+ 6a;-40 = a;+6a;+9-9-40,
=a;*+6a;+9-49, = (*+3)*-7,

= (a;+3 + 7)(a; + 3-7), = (x+10)(x-4).


Ex. 2. Factor

x^+5xS0Q.

Add
Then

(I)- or

^^-

to x*-\-5x to complete the square. x*

+ 5x-806=x^+5x + ^-\^--S0Q, = (a;+|)_(V-)*, = (a:+ + V^)(x+-V). = (x+31)(x-26).

Ex. 3.Solve

a;2-100a;+2491=0.
x"- 100x4-2500-2500 + 2491 = 0, x2-100x + 2500- 9 = 0,
(x-50)2-32 = 0,
.-.

Add

50* or 2500 to complete the square.

(x-50 + 3)(x-50-3) = 0,
{x-47)(x-53) = 0,

x-47 = or x-53 = 0, x=47 or 53.

The

solution might be
:

contracted by writing

it

in the

following form

Transpose the absolute term,

Add 2500

to each side,
root,

/.

x-100x+2491 = 0. x^ 100x= 2491. x* - 100x + 2500= -2491 + 2500=9.


.'.

Take the square

/.

x 50=3,

x=503,

= 53

or 47.

246

ALGEBRA

Here the solution depends upon the same principle, but assumes a simpler form. It is thus seen that we effect the solution of a quadratic equation by finding and solving the two simple equations of
which
it

is

composed.

Thus by the first method of solving x2 100x+2491=0, we obtained the two simple equations x 47 = and x 53=0, and by the second x 50=3, and x 50= 3.
Ex. 4.Solve
Divide by 3 to

3.r2+x=10.
the
first

make
side,

term a square,

Add

(J) to

each

.-.

x+

ia;

+ 5V = V+irs = Vv-

Take the square

root,

.".

2;+i

= + Y. x=V- 8 =

or

2.

Verify both of these roots.

The
1.

steps in this

method

are

Reduce

the equation to the standard

form and remove

the

absolute term to the right.


2.
3.

Divide by the

coefficient of

x^

if not iinity.

Complete

the

square by adding

to

each side the square

of half the coefficient of x.


4.
5.

Take

the sqvxire root of each side.

Solve the resulting simple equations.

EXBRCISE
What must
square
1.
?

119

(1-8,

Oral)

be added to each of the following to

make

a comolete

x^+2x.

2.

a;2-4x.

3.
7.

x^+\Ox.
x2-|-4ax.

4. 8.

x^-Ur. x^
:

x^+3x.

6.

x2 5x.

.J.c

Factor, by

making the

difference of squares,

and verify
-fT.

^. x^+'Ltn. (l^ x-x-1640.

JO.

x2 54x-(-713.

x^ 2x 899

^^

x'-i^x+^.

(l4r'3x-|-16x-99.

QUADRATIC EQUATIONS
Use the method
verify the roots
:

247

of

completing the square to solve the following and

^
^.
21.

2;2^8x=9.

x2-9x+18=0.
2x2-3x=2.
If

&
22.

^. x2 6x=7.
x2+7x4-10=0.
2x2+x=1081.

Qi x^ 10x+9=0.

^
23.

x2-x=2.
6x2+5x=6.

24.

x2+x=l^,

find the values of x

+-

172.

Equations with Irrational Roots.

In

all

the quadratic

equations

we have

solved,

we found that when we had

completed the square on the left side, the quantity on the right was also a square. This would not always be the case.

Ex. 1.Solve

a;2-6a;-l=0.
a;-6x=l,
.-.

x-6x+9=10, x-3=v'IO,
x = 3v'ro.

The two

We
value

roots are 3 + VTO and 3-VTO. might go a step further and substitute roots

for

VlO

its

approximate

3- 16.

The two

would then be

3316 = 616
If

or

-16.

we

substitute- either of these values for x in x'

6x 1,

the result

will

not be exactly 0, as we might expect, because \^10 is not exactly 3-16, but the difference between and the value found for x' 6x will be very small.

Ex. 2.Solve

2x^+x=2.
x

+ ix=l,
x + i=\/H=iVl7.

x=-i^Vr7.
The two
substituting
roots are

-\-\-kV\l,

-\-kVV1,

or

'781,

-1-281, on

V 17 = 4- 123.

248
173.

ALOE BRA
Inadmissible Solutions of Problems.

is

a problem does not follow that the two roots of the equation wull furnish two admissible solutions of the problem.
solved by

When

means

of a quadratic equation,

it

Ex.
in 1^

man walked
less.

25 miles.

If his rate

had been one

mile per hour faster he would have completed the journey

hours

What was

his rate

Let his rate be x miles per hour.

The time taken

to walk 25 miles
his time-

25 =

At the supposed rate

X 25

hr.

r hr.

x-\-l

Simplifying,
Solving,

_ 25 " x+l~ ^' a;* + a; 20 = 0,


25

x=4
is

or

6.

Therefore his rate was 4 miles per hour, the other root giving a
solution which
inadmissible

BXE3RCISE
Solve,
finding

120
three

the roots approximately to

decimal places,

using the table


1.* k.

x^4x=\.
:c2+8a;=19.

^:
6.

a;2

10x+17 = 0.

-3.

H2x-6=0.
2a;2+3a;-4=0.

x(a;+3)=J.
:

^6.

Solve, expressing the roots in the surd form

7.

a;2 6x=2.

8.

x'^^9,x=\\.

9.

4a;2 4a;=7.

10.

4x2-8x=37.

11.

3x2-5a;-ll=0.

<12.

\x^^\x=\.
In solving
verify the

The

following problems reduce to quadratic equations.

the equations factor


results.

by inspection where
and

possible

and

13,

The sum

of

two numbers

is

11

their

product

is

30.

Find

the numbers.
14.

The sum

of

the squares of two consecutive numbers

is

85.

Find the numbers.

QUADRATIC EQUATIONS
15.

249
is

The

difference
is

and the area


I

between the sides of a rectangle 300 square inches. Find the sides.

13 inches

Find two consecutive numbers such that the square of their 16. 'sum exceeds the sum of their squares by 220.
17.

merchant bought

silk for $54.

price per yard exceeded the


of yards.

number

of yards

The number of cents in the by 30. Find the number

18.

18 rods
19.

The area of a rectangular field is 9 acres and the length more than the width. Find the length. The three
sides
of

is

right-angled

triangle

are

consecutive

integers.

Find the

sides.

20.

How

can you form 730

front of one will contain 4 21.

men into two solid squares men more than the front of the

so that the

other

The owner

of a rectangular lot 12 rods

double the size of the lot same amount. What should the increase be
22.
If x-\-2 in

by 5 rods wishes to by increasing the length and width by the


?

men

in

x+5
x.

days do

five

times as

much work
is

as a:+l

men
23.

x 1 A
of

days, find

rectangular mirror 18 inches by 12 inches


is

to be surrounded

by a frame

uniform width whose area Find the width of the frame.


24.

equal to that of the mirror.

What must
3.

a radius
25.

inches less

be the radius of a circle may be | as large ?


is

in

order that a circle with


>

One

side of a right triangle


is

10 less than the hypotenuse

afld the other

less.

Find the

sides.

spends $90 for coal, and finds that when the price is increased $1-50 per ton he will get 3 tons less for the same money. What was the price per ton ?
26.
27.

A man

A man

bought a number of

articles for $200.

He

kept 5 and
did he

sold the remainder for $180, gaining $2

on each.

How many
The sum

huy?
28.

The sum

of the
is

two

digits of a

number

is 9.

of the

squares of the digits


29.

f of the

number.

Find the number.


the rest

number

of cattle cost

a'vera^ed $10 T>8r head

more

$400, but 2 having died Find the number bou|;ht

250
30.

ALGEBRA
How much must
in

by 6 inches

be added to the length of a rectangle 8 inches order to increase the diagonal by 2 inches ?
31.

In
.

the

figure,

the

rectangle

AO 05= rect.

angle

CO OD.
(1) If
(2) If

.40=16, B0^3, C0= 8, ^0=10, 0=4, CD=13,


figure,

find

OD. OD.
circle,

find

32.

In the

when OA
4,
8,

is

a tangent to the

OA^=^OC .OD.
(1) If
(2) If (3) If

0C=
0.4=
0.4

CD=

5, find

OA.
00.

0Z)= 10,
0Z)=16,

find find

= 15,

OD.

33.

I sold

an

article for

$56 and gained a per cent,

equal to the cost in dollars.

What was

the cost

34. The denominator of a fraction exceeds the numerator by 3. If 4 is added to each term the resulting fraction is f of the original fraction. Find the fraction.

35. An open box containing 432 cubic inches is to be made from a square piece of tin by cutting out a 3 inch square from each corner and turning up the sides. How large a piece of tin must be used ?

36.

and
it

alone can do

in 12

can together do a piece of work in 14| days, and A days less than B. Find the time in which /I could

do

it

alone.

BXERCISE
1.

121

(Review of Chapter XIX)


?

What
it.

is

a quadratic equation

2.

Is (x+l)(a; 2)(a;+3)

{a;

4)(x l)(a; + 7) a quadratic equation?


its

Solve
3.

number
^4.
5.
6.

The sum of a positive number and to two decimal placas.

square

is

4.

Find the

SolvegA_+gA_ =
If x*y*

3;-L-^=l.
are the values of xy
1

6xy l = 0, what
7

Are x = 4 and x'=16 equivalent equations, that


Solve

is,

have they

the same roots

C9

x' xy+y* = 39, when y=7.

3t -r

*
251

QUADRATIC EQUATIONS
8.

may
is

Divide 14 into two parts so that the be greater than twice their product by 4.
If

sum

of their squares

9.

(a;-2)(x-3) = 7(x-3), does


?

it

follow that

x-2 = 7

What

the proper conclusion


10.

The distance
is

seconds

{s) in feet that a body falls from rest in given by the formula s=l6lt*. How long will it take

a body to

fall

6440 feet
1

ir.i Solve 3x - 4x -

= 0.
of the
?

number.
13.
r
.

Ten times a number is 24 greater then the square Does this condition determine the number definitely

x+2 Solve =3 + 4-a; ^^.

^R.
15.
16.

Find two consecutive odd numbers whose product


Solve(2a;

is

399.

3)-2(22; +

3) = 35.

The

units digit of a
of the digits
is 12.

number
7

is

the square of the tens digit

and the sum


._

Find the number.


;

17. 18.

Solve
If

a;+10

10

X 5

11
"E-

x+5
r-5

5 a; 3

15 =
5 3

2 hours less to travel 315 miles.

a train travelled 10 miles per hour faster Find the rate.

it

would require

<^.

Solve (3j;-7)(2j;-9)

(5a;-12)(a;-6) = (x-2)(2a;-3).
number which
is

20.* Find, to three decimal places, the positive 'ess than its square by unity.
^yJH*-.

If 4a;

3xy^-y'=14,

find x

if j/

= x + 3.
34 feet and the length of the

"'22.

The perimeter
is

of a rectangle
sides.

is

diagonal
23.

13 feet.

Find the

which

is

Solve x* + (x 4)* = 40. State the problem, the condition expressed by this equation.

in

24. A line 20 inches long is divided into two parts, such that the rectangle contained by the parts has an area of 48 square inches more than the square on the shorter part. Find the lengths of the
parts.

25.
26.

Solve x^ + !/^ =

9,

when y = Zx.
is

The diagonal
If 5 is

of a rectangle

39 feet and the shorter side

is

i'^

of the longer.

Find the area.


one root of x' 7x*-f-6x+20 = 0, find the other roota

27.

to three decimal places.

252
28.

ALGEBRA
Find the price of eggs per dozen when 10
less in a dollar's

worth

raises the price 4 cents per dozen.

The length of a field exceeds its breadth by 30 yards. If the were square but of the same perimeter, its area would be t^ greater. Find the sides.
29.
field

30.
31.

If

8x

= X

4, find

x to three decimal

pleices.

was S20. This was to be divided the men present. But four failed to contribute anytjjing, and thereby the cost to ea^h of the others was increased 25 cents. How many men were there ?
cost of an entertainment
equally

The

among

32.
in 3

If a man walked one mile per hour faster he would walk 36 miles hours less time. What is his rate of walking ?
If

A polygon with n sides has ^n(n 3) diagonals. 33. has 20 diagonals, how many sides has it ?
34.

a polygon

Solve aHa:-a) = 6*(x + a)*.

A can do a piece of work in 10 days less than B. If they work 35. together they can do it in 12 days. In what time could each do it alone ?
36.
If x'-L

X"

^ = 81.
*

find the value of x^

and
is

of x.

37.
6 acres

The length
is
?

of a rectangular field

and the area

5-4 acres.

How many

to the width as 3 to 2 rods longer must it be to contain

CHAPTER XX
RATIO AND PROPORTION
174.

Methods of Comparing Magnitudes.

When we
in

wish tc

compare two magnitudes, there are two ways comparison may be made.
(1)

which the

We may
We may

other.
(2)

determine by how much the one exceeds the This result is found by subtraction.
determine how
result
is

many

times the one contains


division.

;he other.

Here the

found by

Thus, if one line is 6 inches in length and another is 18 inches, we may say that the second is 12 inches longer than the first, or that the second is three times as long as the first.

Neither method of comparison can be used, unless the magnitudes compared are of the same denomination, or can be changed into equivalent magnitudes of the same denomination.
Thus, we can compare 3 lb. and 10 but we can not compare 5 lb. and 4 ft.
175.
Ratio.
lb.
;

2 yd.

1 ft.

and 2

ft.

9 in.

When two

are

compared by

division, the

magnitudes, of the same kind, quotient is called the ratio

magnitudes. Thus, the ratio of 3 to 4 is the same as the quotient of 3-1-4, which is usually written |. The ratio of 3 to 4 is written 3:4,
of the
.'.

3:4=3^4 = 1.
a
:

Similarly,

b=a^b =

-r.

254

ALGEBRA
may
be

It will thus be seen that all problems in ratio considered as problems in fractions.

Comparison of Ratios. To compare two ratios we 176. simply compare the fractions to which these ratios are
equivalent.

Ex.
i

1.

Which
is

is

the greater ratio, 3


changed into
:

4 or 7
is

The problem
or i

at once

"

Which

the greater fraction

"
?

To compare the
in the

fractions
f
,

we reduce them
it is

forms ^l and
also

and

m.ight

compare them

to the same denomination seen that the latter is'the greater. We by reducing the fractions to equivalent

decimals.

Ex.

2.

Which
a

is

greater, a

a-\-2 or

a+1 a+3.
:

_ a+2 ~ g+l _ a+3 ~


What
is

a(o+3)
(a

+ 2)(a + 3)
)

(a+l)(a + 2

(a+2)(a+3)
?

+ 2)(a + 3) _ a* + 3a+2 ~ (a+2)(a+3)"


(a

_ "~

a*+3a

the conclusion

177. Terms of a Ratio. In the ratio a b, a and b are called the terms of the ratio, a being called the antecedent and b
:

the consequent.
of

equivalent denominator.
o Thus, r
_,,

the

The antecedent corresponds to the numerator fraction, and the consequent to the
numerator -, denominator
-.

antecedent consequent

dividend
,.

divisor
is

178.

Equal Ratios.

Since a ratio

a fraction,

all

the laws

which we have used with fractions


ratios.

may
:

also be used with

Thus, since -

mb

j-

it

follows that a

b=^ma mb.
:

Hence bolh terms of a ratio may be multiplied or divided by the same quantity {zero excepted) without changing the value
of the ratio.
Thus, 6
:

=2

3, i

i .23 = 3
:

2, ^-:
'

= ^^ *-

RATIO AND PROPORTION

265

EXBRCISB
their lowest terms
1.

122 (1-15, Oral)

Simplify the following ratios by expressing them as fractions in

256
f28^

ALOE BRA
When
is

part

a sum of money is divided in the ratio 1 820 more than when it is divided in the ratio 2
:

2,
:

the smaller

7.

Find the

3um.
(_29.

What number must


i

be added to both terms of

to

make

it

equal to

(*Ji-^

(30^
13: 19
31.

What number
?

subtracted from each terra of 7

10 will produce

What must
is

What
32.

be added to each term of a the conclusion when c^=d^.

b to

produce

If a is a positive

number which

is

the greater-ratio,
, 7

SoTeet Soieet per second.


34.

Q
:

l+2a l+3a
The
rate of one train
is

14-3a
or

l+4a
is
?

30 miles per hour and of another

What

is

the ratio of their rates

Divide a line a inches long into two parts whose lengths are
:

in

the ratio b

c.

Ji^. ^'s income


expenditure =5
:

5's
If

income=3
spends

4,

6.
?

all his

5's and A'^ expenditure income, what per cent, of his


:

income does

save

^^^. Divide 8315 among A, B and C, so that as 3 4, and 's to (73 as 5 7.


:

A's,

share will be to 5's

37.

line is divided into

two parts

in the ratio of 5

two parts
division
38.

in the ratio

3:5.

If the distance

7 and into between the points of


:

is 1

inch, find the length of the line.

Two numbers

are in the ratio of 3:5, but


to

if

10 be taken from
reversed.

the greater and added the numbers.


39.

the smaller, the ratio

is

Find

Two

bodies

ra feet in

a seconds

The are moving at uniform rates. and the second n yards in b minutes.
?

first

goes
is

What
of

the ratio of their rates

179.

Proportion.

proportion

Is

the statement

the

equaUty of two
Thus, 3
:

ratios.
:

Therefore, 3

4=15 20, since | = i^. 4=15 20 is a proportion,


:
:

or

3, 4, 15,

20 are

said to be in proportion, or they are said to be proportionals.

RATIO AND PROPORTION


If a, b, c,

257

d are in proportion or a

.b=c

d,

then

a
r

= cJ a

ad=bc.
proportion a:b=c:d, a and d are called the and b and c the means. Since ad^bc, it is seen that <^e product of the extremes is

In

the

extremes

equal to the product of the means.


180.

Fourth Proportional.

When a:b=c:d,
10, 12,

is

called

the fourth proportional to


Thus,
then
if

a, b, c.

the fourth proportional to


10
.-.

15

is x,

12=15

:a;

10 or yg

15

10x= 12x15, x=18.

To find a Ratio, by Solving an Equation. From certain 181. types of equations in x and y, the value of the ratio oi x :y may be found.

Ex.
Since

1.

If

bx=Qy,

find the ratio oi x .y.

.*.

5x X X
:

= %, = ft/, _6
=
| or 6
:

the ratio of

5.

Ex. 2.If 3a;-|-4t/=3t/-7x, find -

3x+7x =

Sy iy,
10"

y~
If

the ratio ot x

each term in the equation is of the first degree in x or y, y can be found, but it can not be found if there is a term not containing x ox y.
:

Thus, from 2x 7i/=

10,

the value ol x

y can not be found.

258

ALGEBRA
{x-2y){2x-3y)
:.

Ex. 3. If 2x^lxy-\-6y^=0, find x:y.


Factoring,

= =

0,

x2y = 0or2x3y = 0,
2 or 5

Here there are two values


equation by
y*,

of -.

If

we

divide each term of the given

we

get

In this form

we

see that the equation

is

a quadratic in

and we

might naturally expect to find two values

for the required ratio.

Ex. 4. It

2x5y-\- 2=0,

3x+2/ 2z=0,
find the ratios of x, y,
If
z.

we eliminate

z in

the usual way,

we

get

7x-8y=0,
x

_8

RATIO AND PROPORTION


Find the value oi x:
13. 16.
y,

259

2x=ly
\x=^'iy.
a:2=42/2.

14,
17.

32/=

12a;.

15. 18.

2x-i/=0.
S^z+llx^O.

2x=-Zy.
4x2=91/2.

19.

20.

21.

(x-Sj/Kz 5i/) =

22.

If

= -, show
d

that c

= -,
a

and a

= -.
c
2, 3,
:

(^
k,

Find a fourth proportional to


\
;

18;

5,

7, 10;

\.

a, b, c

a, 26, 3c.

^4y

Find a fourth proportional to: -3a+2, a2 5a+6, a2-5a+4.

h,

{a-\-bY,

a^b^; and

What number must


(2&7
If

be added to each of the numbers


?

2, 4, 17,

25

8(^Jfiat the results will be proportionals

(Verify.)
x.

does the result


27. 28.

a+x, 6+x, c+x, d-j-^ mean when bc=ad


in order that
is
:

are proportionals, find


?

What

Find a
^'s age
their, ages.

a+3 a+15=3
:

4.
4.

to B's as 4

5.

Five years ago the ratio was 3

Find

29.* In an equilateral triangle the ratio of the altitude to either


of the equal sides
is

VS
is

2.

If

the altitude

is

10 inches, find the side

to

two decimal
30.

places.

When

a line

drawn

j^arallel to

the sides in the same ratio.

the base of a triangle it divides In the figure, AB=20, ADl'i and

AC=15.
31.
similar.

Find

AE and EC.

In the figure, the triangles

ADE

and

ABC

are
D/.

When

triangles are similar their correspond-

^ /\
\E
^

ing sides are in the

same
:

ratio, so that
:
:

AD AB=DE BC=AE
If

AC.

AB=8, BC^IO, AC=9, AD=6,


In the same
figure,

find the lengths of all the other

lines in the figure.

32.

the areas of the similar triangles are in

the
If

same

ratio as the areas of the squares

on their corresponding

sides.
is

AD^20

and AB=35,

find the area of

ADE

if

the area of

ABC

735.

s 2

260

ALGEBRA
The
side of the square

33
/^

A BCD

is

10 inches and

EF

is

parallel

to

DC.

If

the length of

AE

is

3 inches, find

the

length of
34.

FC

to three decimals.

If the bases of

two

triangles are in the ratio


:

4 and their heights in ratio 8

9, find

the ratio of

their areas.

J3tf^

From

these equations find z

y,
;

13x+5!/=9a;-f-13j/

ax+by=cx+dy
px-\-qy=0.

mxny^=nx-\-my

as^ Find
,>?r^
oi a
:

tv.'o

values ol x

y when
;

6x^l3xy+6y^=0
If

x^=4xy+5y^.
find the ratios

5a 36+2c=0
:

and a + b+c=0,

b,

c,

c.

38.
figure.
39.

Find the lengths

of

all

the other lines in this


3

If

a pole 10 feet high casts a shadow 17J feet


cast,

long,

time,

what will be the length of the shadow by a monument 84 feet high ?

at

the same

Write the equation 3x^-'\0xy-{-3y^=0 in a form showing


as the

unknown, and

find

y.
:

J^.

A number

of

two

digits bears the ratio 7


digits.
If

4 to the number

formed by reversing the find them.

the

sum

of the

numbers

is

66.

^2.

The length

of a
:

room
If

is

to the width as

6:5, and the length


is

i/ to the height as 3
find the dimensions.

2.

the area of the floor

187^ square

feet,

If 4 men and 3 women earn as much as 16 boys, and 6 men 43. and 5 boys earn as much as 10 women, find the ratio of the earnings of a man, woman and boy.

44. 46.

If

3ab+2b^:2a^-ab=9:5,
the angle
:

find a

b.

When

is

bisected,
:

AB AC^BD
(1) If
(2) If

DC.

AB=10, AC=S, BC=12, find BD and DC. AB=c, AC=h, BC=a, find BD and DC

RATIO AND PROPORTION


46.

261

The

ratio of the area of a rectangle to the area of the square


its

described on
47.

diagonal

is

13.

Find the ratio of the


7,

sides.

The

sides of
is

a triangle are

10

and
?

12.

The perimeter

of

similar triangle

72J.

What

are its sides

48.

If

=
5

= -~^-!
7

find x-.y.z.

When three numbers form a understood that the middle number is to be repeated. The three numbers are said to be in continued proportion, and the middle one is called the mean proportional between the other two.
182.

Mean

Proportional.

proportion,

it is

Thus,

4,

and 9 are

in

continued proportion, since


9.

4:6 = 6:9.

Here 6
If

is is

mean proportional between 4 and a mean proportional between 3 and 27,


the

X
.'.

~^,

27

..

= 81,

.-.

a;=+9. ~

the
3
i:

mean

proportionals between 3 and 27 are

+ 9.

Since
results.

tt;^

9 and 27

39 = - 9 27
;;

-,

it

is

seen that these are the correct

Similarly,

if

is

the
.

mean
a X

proportional between a and


.-.

h,

then

= -,

x=--Va&.

Therefore, the
is

mean

'proportional between

any two

quantities

the square root of their product.

183.

Third
is

Proportional.

If

a,

h,

are

in

continued
h.

proportion, c
Thus,
if

called the third proportional to a

and

is

the third proportional to 6 and 15,


-| 16

= lf,
X

6x=225,

.-.

x=37i.

262

ALGEBRA
BXSRCISB
124
;

o.

4063 and 'da%

Find a mean proportional between 4 and 16 (a-i)^ and (a + ft)^.


;
;

2a and 8a

J?r Find a third proportional to 2 and 4 x'^y^ and x ?/.

and 30

5a and

lOaft

A.
34

The mean proportional between two numbers whose sum


Find the numbers.

is

is 15.

(/4.

Three

numbers are

in

continued

proportion.

The middle
12

one

is

12 and the

sum

of the other

two

is

51.

Find the numbers.


3, 7,

5.

What number must

be added to each of the numbers


?

80 that the results will be in continued proportion

semicircle

figure, the angle BAC being in a a right angle. When AD is drawn perpendicular to the hypotenuse it is proven in

C*

In

the

is

geometry that

AD is a mean proportional between BD and DC, AB between BD and BC, and AC between CD and (1) If BD= 4, DC= 9, find AD. (2) If BD= 5, AB= 8, find DC. (3) If BC = n, AC=\2, find DC, AB, AD. (4) If AB= 3, AC= 4, find BC, AD, BD.
7.

BC.

How
(1)
(2)

would 30U use the preceding to


line

find
?
?

whose length

is

VG

inches
is

The

side of a square

whose area

12 square inches

A. tnem

Find two numbers such that the mean proportional between


is

4 and the third proportional to them


line 21

is

32.

9^ Divide a
loV^est
is

four times

the shortest and

inches long into three parts such that the the middle one is a mean

proportional between the other two.

184.

The

following examples will illustrate a

method which

has

many

appplications to problems with ratios or fractions.

RATIO AND PROPORTION


^^Since

263

^-^^
r

d'

P^^^^ '^^' ^^:i56^

= ^^35^
c=dk.

- let

each fraction
^.
.'.

k.

Then

a=bk,
c in

-,

= k,

.'.

Substitute these values of a and proven.

each side of the identity to be

3o+2b

a* 5b*
3c'

_ ~~
~

Sb^k*

+ 2b*
*

b*k*-5b*
d*A;-5d
3a

+ 2d _ 3d*P + 2d

cs_5d

b*(3k* + 2) _ 3k* + 2 ~ b^k^-5) ~ fc2-5 +2 _ rf(3fc* + 2) _ ~ d^(k^-5) ~ k'-5


__
3A:''

'

+ 2 6 _
prove

3c

+ 2d'

Ex. 2.-If -

=^=-

^^^qip^q:^

(a+6+c)3'

Let x = ak, y = bk, z = ck. of the fractions is equal to


,. If

Substitute as before and show that each


k^.

r 6

185.

c =T rf

^^
.

then

^ = cd ao
a-\-b
^

c-\-d !^

Prove

this

by

letting a

= bk and c = dk, as in the preceding examples.


was obtained by adding and suband
c

Here the fraction

tracting the terms of the fraction r,


in

was obtained

a similar

way from
is

-^

This principle

sometimes useful in simplifying equations.


4:X-\-3

Ex. 1.Solve
Adding and subtracting,
.-.

3a+46

4a: 3

3a 46
Si*

^=
646x

= _

36a,

9a

^~T6b'
Solve also, in the usual way, by cross multiplication.

264

ALOE BE A

^. 2.If a-\-b-\-c4-d = a+bcd --^ 4- ^ Ex. 'J a 0 c-f-a ao-\-cd


* AAAA u, Adding and subtiacting, ^^-^^

a prove r o

=d
-

2a+2d
a-\-d

2a-2d = ^^^,

ad c' + c~ 6 o + d _ b-{-c

ad
Adding and subtracting,
9^
a

bc

9~

'

b
*

d
a
'

b^c'
;2$
-%

EXBRCISB

^. ^

If -

=
:

h
If a

-, d^
:

prove that

2a+36

2c+3d
mc nd.

^.
3<

6=c

d, show that ma+nfe manh=^mc-\-nd


:

If

6=c
4

d,

prove a26d+6^c+6c=a62c+a6d+od.

5.

If

? y

= ?,

find the value of

??!^.
6x2/4-2/2

(Here x=|?/, substitute for x and simplify.)


T* If
a;

=y ^.
3

find the value of \^

,,

(2x+3w)(3x+2w) /, 32/)(3x 5t/) (5x /,,

^
^-^

If

? y
-

= 3 and ? = ?,
6

find the value of

"P^ 2az+3by
is

fii
.

^^
,

If

=-=
b

-,

prove that each fraction

equal to

o+fi-t-c

that

IB,

to

sum of numerators sum of denominators

\ n -^ =
\J
bc

ca

J^=

-^, ab

prove x(6+c)+2/(c+a)+2(o+6)=0.

RATIO AND PROPORTION

265
h^d-\-h(J^.

^?
d-c^

If

.1,

b, c,

d are proportionals, prove that ah-\-ac^,

and

h-d"^

are proportionals.
b
c

(3^"^
\^.
/i2J

nX
If
If T, If

+y
:

yz
:

z+x

prove that a^h-\-c.


b^.

b=b

c,

show that a c=a^


:

ab:a-\-b^=cd:c-\-d, prove a:b=c:d.

a+6+c+d = a 6+c
a-\-bcd
,

d
;
,

bc-\-a

show that

c -1^0 = -

6
16.

c ooJve
,

3x+46 3x-46
sum
of

= 5a+3b

5a- 36

If the

two numbers

is

to their difference as 7 to 4, find

the ratio of the numbers.

^a 16.
equals
17.

T* If

ab+l = bc + 3 = ca+5 bc+2 ca+4: a 6+6


;

show that each

,,

fraction

i-

f.

(Use Ex.

7.)

Solve
a

^^^=^^^^.
aa; 6+c

bx-\-ca
:

^.
19,

If

6=3
b

5, 6

c=7

9, c

d=^\f>-, 16, find the ratio

oin:A. si>.

If -

a
also equals

=^=c

show that each

fraction equals

5a 36+2c

^I

and

ma-\-nb

pc

y^^.
21.
find the

Find two numbers such that their sum, difference and product
9.
if

are proportional to 4, 2,
If a, b, c are

consecutive numbers and

c^b^

6^ a^=41

39,

numbers.
:

22. The length and breadth of a room are as 3 2, and if 2 feet be added to each, the new area of the floor is to the old as 35 27. Find the dimensions.
:

^^23.

If a

b=c

d,

prove a

a+6=a+c 0+6+c+d.
:

'4.

If

-^^^^
lOc+d 12c+d
a
:

show that ?
6
:

= -^
d
:

If

6=6

c,

then 0^+06

62=62+6c

c'

266

ALGEBRA
BXBRCISB
126

(Review of Chapter XZ)


forma
:

Write as fractions

in their simplest

w^7i:8|.
4.

^.
5,
1

x*-y*:{x-y)\
a*

^.
6^
.
,

+ 6
:

(a

+ 6)

l--i:l + -.
1

:1

a-1

^_

^
/f, x^
i

x*-5x+6' x'+z12
9.

,1 ^ >c^a+l + ^:a-l+^
3, 4,

Divide 144 into three parts proportional to

11.
it

*^
r

10.
to 6
:

What must
7
?

be added to each term of 4

7 to

make

equal

/* 4

11.

Write as a proportion
3. 6

in

two ways

= 2. 9; 2.5 = 3z; ab = cd (a + 6)(a-6) = 3 4 a*-5o + 6 = a* + 5o+4.


;

^i^
:)ther
131*.

If

the means are 7 and 12 and one extreme


?

is 3,

what

is

the

extreme

Find a fourth proportional to:


a, a',

7,

15,

a'

x + y, x y, x* w*

11
7.
:

35

r a-}-o

a*

b*.
)f

IfC
IS.

Find two numbers


504.

in the ratio 9

5,

the difference

whose

aquare^s

Two numbers

are in the ratio of 5

8,

and

if

1^3 and 2 be taken from the greater, the ratio numbers.


i^.

is

8 be added to the 14 15. Find the


:

Find two numbers

in the ratio 6

5 so that their

sum

is

to the

difference of their squares as 1:3.

17.* If the ratio a x


find X.

x
:

is

equal to the square of the ratio a

h,

^a.

If

2x+3?/

3x-5i/ = 9

11, find

y.

WT
20.

If (5x-72/)(2x-32/)
If If

= (4x-52/)(x-j/),
%x+2y
: :

find x

y.

4x-52/ = 2x+2?/, find

2x+32/.

^
21. 23.
24.

6x+152/*=19x2/, find X
:

2/.

Ifx* + x+l 62(x+l) = x*-x+l 63(x-l), find x. and 7x+6z/-92 = 0, find x:y,x:z and If 2x + 2/-23 =
:

x-.y.z

If

s.

find the value of

'^^^-

3x+llj/

RATIO AND PROPORTION

267

If az + by 26. X to y.

X
27;

Find a mean proportional to


:

a;*

and y^

bx-\-ay=9

11

and a

=3

2,

find

the ratio of

If

7'

show that each

of these fractions

is

equal to

mancpe mbndpf
j<?

Find two numbers whose sum, difference and product are 28. proportional to 5, 3, 16.

X
^
31. 32.

If

.b

=c

d,

show that
{ab

a+b
c+d'

2o*-36
2a*

+ 36*

2c--3d 2c*+3d'

+ cd)* = (a* + c*)(b* + d*).


7

u^

If

= ro-\-ca
any number

c-\-ao

= -T a-\-bc
-\-

prove that

x(b
If

c) -\-y{c a)

z(a

b) = 0.
show that each
ratio
is

of ratios are equal,

equal

to the ratio of the

sum

of all the antecedents to the

sum

of all the

consequents.
If 3x-22/ + 42 = 2a;-32/ + 2 = 0, find the ratios + 2/* + 2*= 150, find the values of x, y and z.

of x, y,

z.

If

also,

a;*

^[3'r

The hypotenuse
:

iide as 13

5.

If

of a right-angled triangle is to the shortest the perimeter is 120, find the sides.

and height of a room are proportional to each dimension be increased 2 feet, the area of the four walls will be increased in the ratio of 10 to 7. Find the dimensions of the room.
34.
3,

The

length, width

4,

2.

If

d
35.
If r

6 = -,

show that

CL

C^ l-C
rr

,-r

b^

+ d^+P

r-

CLC&

5-r;

bdf

36. If the sides of a triangle are 6 and 8 and the base is 4|, find the segments of the base when the bisector ot the vertical angle is

drawn.
37. 38.
If

yiZ^ = ^^P^,
z-x+y
x-\-y+z'

show that

z*

= x*+yK ^
are Bf

6:7.
39.

If

The incomes of A and B are as 2 3 and their expenses ^ saves 25% of his income, what % does B save ?
:

Find three values of the ratio x

ii

3{x*-ixh/ + 5xy*-2y*) = 2{x'-2xh/-2xy* + 3y).

CHAPTER XXI
THE GENERAL QUADRATIC EQUATION
186.

Type

of the General Quadratic,

The equation

is called the general quadratic equation, because every quadratic equation may be reduced to this form. If the factors of ax^-\-bz-\-c can be obtained, the roots of the equation can be found by solving the two equivalent

equations.
187.

Solution
the

of

Literal

Quadratics.

The
to

methoa

of

completing

square

may

be applied

the

solution of

quadratic equations with

literal coefficients.

Ex.

1.

Solve
x*
root,

z^-}'2mx=n.
side,

Complete the square by adding m' to each

+ 2mx-{-m* = n + m*.
x+Tn= Vn+m*,

Take the square

The two

roots are

x= m+ Vn-f-w*. m+Vn+vi*, mVn-^m*.


z^-\-pz-\-q^O.

Ex.

2.Solve
p*
~- to each side, x*

Transpose the absolute term,

x*+px= q.
p*
ir

Add

p* + px + ^ 9 + 4

'

p* 4o

'

Take the square

root,

2=
X

-p = -t:

Vp*-4q
;;

THE GENERAL QUADRATIC EQUATION


Ex. 3.Solve
Divide by a to

26&

aa;2+6a;+c=0.
the
r*
first

make

term a square.
0.

-r

+-= 'a

Transpose,

X*

-\

X
-rz

Add

-r.

to each,

4a

x* H

-\-

=
4a*

fe* 4ac

4a

4a^

Take the square

root,

+ 2^=
.'.

v'62_4ac
2a
h
K-

aJ

2a

+ V6* 4ac 2a
.

;j

'

_ -6\/&2 4ac ~
2a

The roots

of the general quadratic equation are

& V&^ 4ctc


2a
188. The roots bund by factoring

of the general

quadratic might also be


ax*-[-hx-\-c

as in art. 171.

0,

a]

(f

by^c

6M

is

Since the product is zero, one of the not zero, as the equation would not then

must be
quadratic.

zero.

But

+ 2^ +

270

ALGEBRA
eixbrcise: 127

Solve by either of the preceding methods


1.* a:-2aa:=3a2.
3. 6.
7.

2. 4.
6.
8.

a;2+46z-56*=0.
x^+4:pxp^=(i.

z* 6mx-H3m2=0.

z*-2az+6=0.
o+2ax=6.

x^^2bx-c=0.
ax2-i-26x+c=0.

9.

az* 6z c=0.
Solving

10.

px'^qx+r=().
roots
of

189.

by

Formula.

The

any particular

quadratic equation
of a, h

and

be found b}" substituting the values in the roots of the general quadratic.

may

Ex. 1.Solve

6a;2-7a;+2=0.

Herea = 6, 6=-7, c = 2.
Substitute these values in
,
.

x=

6+V6>-4ac

^^

2a

^=
=

+ 7V49-48
12

71
12

=r2'r2 = 3'2-

Verify by substitution.

Ex. 2.Solve
Here a=5,
6

5a:2+6x-l=0.
1.

'

= 6, c= _ ^~ -6\'36-(-20) _ -6+^56 ~ 10 10

'

~ -62Vl4 ~ -3VIi
10 5
if

In this case the roots are irrational, but,


substitute for

necessary,

we may

Vli
5

its

approximate value 3-742, when the roots become


-742

-33-742
Note.

-6-742
or
z.

5
pupil
ia

5
to

,_ = -HS

or

_. 1-348.
,

The

warned

be

careful

of

the

signs

whoD

Rubatituting, particularly

when

c is

negative.

THE GENERAL QUADRATIC EQUATION


Ex. 3.Solve
2a;2 5x+6=0.

271

0=2,

6= 6, c = 6.

+ 5+V'25-48 5+V^=^ = = -"^-4

^4

In the preceding result the numericannot be found even approximately, for there is no number whose square is negative. Such a quantity as 23 is called an imaginary quantity, and the roots in this case are said to be imaginary. This is merely another way of sajdng that there is no real number which will satisfy the equation 2x'^6x-\-Q=0.
190.

Imaginary Roots.

cal value of the roots

191.

Methods

of

Solving

Quadratic

Equations.

When

a
it

quadratic equation has been reduced to the standard form, may be solved
:

(1) (2)

By factoring, by inspection or by completing By substitution in the general formula.


is
is

the square.

The pupil this method


formula.

advised to try to factor by inspection,^ and if unsuccessful, then substitute in the general

As the general formula


absolutely essential that
it

will

be used very frequently, be committed to memory.


are

it

is

The

roots of

ax^+ox-\-c=0

2a
128

EXSRCISB
Solve, using the formula
1.*
:

3x2-5x4-2=0.

2.

24x2-46x+21=0.

^^.
5.

575x2-2x=l.
247x2+5x=12.

4.

2x2-6x-l=0.
2x2- 13x+ 10=0.
1200x2- 10x=l.

^O
1
^

X
l^
i>a.

JJff 391x2+4x=35.

^
ft.

x^+x{3b-2a)=6ab.

}y 6x^-x-l=0.

2x2-25x+77-^0.

1800x-5x-I=0

272
Solve by any method.
13. 16.
19.

ALGEBRA
Verify 13-18
14. 17.
:

27a;2_24a:=16.

15x2+7a:-2=0.

15.

12x2-T-6=0
5-26a;+5x2=0
2x2-2x=.
1 * 2 3"'"9'"x"
2

4a;2-17a;+4=0.

460x2-3x=l.
3a;2+2=9a;.

18.
21.

9x+4=5a;2.

20. 23.

22.

4x2-4a;=79.

f-2/=2/2

24.

25.

- -^ =0.
a;+2

26,

^4-1
2

^2 =
29.

w
ac+
X
a
2

27.

28.
30.

(x-4)2-3(x-9) = 15.

(x-2)(x+3)=x(5x-9)-2.

2ax2+x(a 2)=1.
2x(x-2)=a2-2.
12

31.

= - + ex.
X
X
3

32.

33.

2-^x
(x+2)2

+ x'
+ 3)2=(x+6)*.

x+1
^36.

^x+2

x+3'

-^5.

-(x

x'^xy^y^- -12.

n y=2,
,

find X.

^7.
38.
39.

x+1
,

x+1

find

X to three decimal places.

.,

2x

Find the sum of the roots of x^

3x=20.

40.

are expressed
41.

The area of a square in square feet and its perimeter in inches by the same number. Find the side of the square.
of a rectangular field exceeds the

The length
is

width by 16 rods

and the area

32 acres.

Find the length.


is

Find three consecutive even numbers whose sum 42. product of the first two.
43. A line 10 inches long square on the longer segment
is is

| of

the

divided into two segments so that the

equal to the rectangle contained by

the whole line and the shorter segment.

Find the segments to two

decimal places.
44.

Find two numbers whose difference


is

is

and the sum

of

whose

squares
45.

317.
of a square

The area

and 12 inches to the

other.

doubled by adding 5 inches to one side Find the side of the square.
is

TOJD QiUNERAL
46.

QUADRATIC EQUATION

27S

number by
47
A.D
48.
is

Three times the square of a number exceeds eight times the unity. Find the number to three decimals.

r^

Mr. Gladstone was born in the year a.d. 1809. he was x 3 years old. Find x.

In the year

The area

of a rectangular field
sides.
is 8.

is

half

an

acre.

The perimetei

201 yards.
49.

Find the

One

root of x^bx-\-d=(}

Find the value of d and the

other root.
50.
it

will

If a train travels 10 miles per hour faster than its usual rate, cover 480 miles in 4 hours less time. Find its usual rate.

51.

Divide 3 into two parts so that the

sum

of their squares

may

be 4.
52. I buy a number of articles for $4'80 and sell them but 2 at 6 cents a dozen more than they cost.
I

for $5'95 all of

How many
C

did

buy?
53.

straight line

AB,

12 inches in length,

is

divided at

so as

to satisfy

one of the following conditions.

Find, in each case, the

length of
(1)

4C

to

two decimals

AC^=2BC\ 3^C2=44 BG.


.

(2) (4)
in.

(3)
(5)

AC^=2AB BO. AC^+^BC^=2AB\


.

AC^-BC^^IQ

[.

(6)

AC{AB^BC)=2

sq. ft.

54. I buy a number of books for $6, the price being uniform. li they had been Subject to a discount of 5 cents each, I could have bought 6 more for the same money. What did each cost ? 55. Solve the equation ax^-\-bx-\-c=^0 by multiplying by 4a and completing the square of 2ax-\-h. =^ 56.

3+2a; c 1 Solve

2 3a;
2+a;

\^x-x^
a;2-4

2-x

1 =-

Verify the roots obtained.


192. Equations Solved like Quadratics. There are certain types of equations of a higher degree than the second, which may be solved by reducing them to the form of quadratics.

W4

ALGEBhA
x*- 10x2+9=0.
it
ii

Ex. 1.Solve
This
is ail

equation of the fourth degree, but we might write the form of a quadratic, thus
(x*)*-10(2:)
or
if

+ 9 = 0,
+ 9 = 0,
2/

we write y

for x*

it

takes the form


j/-102/

.-.

(3/-9){t/-l) = 0,

But

i/=aj*,

.".

x*

=9 =9

or
or

1.

1,

a;=3

or

1.-

We

see that this equation has four roots.


it is

This

is

what we might

expect, as

an equation

of the fourth degree in x.

Verify each of the four roots.

Ex. 2.Solve (x-2_5x)2+4(x2-5a;)-12=0.


Here we consider x* 5x as the unknown, whose value should
be found.
first

Let

THE GENERAL QUADRATIC EQUATION


w^ 4.-Solve c bX. A.
1

271

-^'4- 2a;

3 -. _^+^,^^ = 26
^

26

Complete Ex. 's 3 and 4 and verify the rational

roots.

Ex. 5.Solve
Factoring,
.-.

z3-1^0.
(x
l)(x*-f-x+
or x*

X 1 = x= 1

1) = 0, + x+l = 0,

or X

-1V^ ^

We
degree,

thus see that


or

if

one root of an equation of the third

a cubic equation, can he found by factoring, the equation can he completely solved. This equation might be written x^ = l, and each of the thi-ee roots when cubed must give unity, which shows that unity has three cube roots. This is what we might have expected, as we have already' .seen that unity has two square roots.

+1

and 1.

BXERCISE
Solve and verify the rational roots
:

128

,^.
3.

x*-5x2+4=0. V4-12=31y.

^J..

^.
i.

(.H5.^6)(.-9.+ U)=0.
(x2-4x+o)(x2-4x+2) = -2.
.,
^-9.

^
^t.

x- 13x2+36=0.
8i-65x3+8=0.

!i^+_^ =
x

2.

+ + =
a;

^^ a:2+x

.,***

(*2+x4-l)-4(x2++l)+3=0.

''!.
,11.

x3-4x2-4x+16=0.
6/'x

+ l)^-35/'x + lW50=0.

lA-

(i+x+x2)(x+x)=156.

t2

876

ALGEBRA
(Multiply
the
first

13.* (x+l)(4-2)(a:+3)(a;+4)=120.
factors

and

last

and the second and

third.)

14.

a;(x-l)(x-2){x-3) = 360.
Find the three cube roots of 8 by solving the equation x' 8=0.
Find the four fourth roots of 16 by solving the equation

16.
16.

:fc4_16=0.
17.
^ 18.

Solve

x' 19x4-30=0

being given that 3


^i^g

is

one of the roots,

Solve 12x3 29x2+23x 6=0 (ygg


It
is

factor theorem).

19.

evident that 4

is

a root of the equation

x(x-l)(x-2)=4.3.2.
Find the other two
20.
roots.

Find the

six roots of 8x

217x3+27=0.

21. 22.

Solve(x2 x)2 8(x2-x) + 12=0.


Solve x2
1 + jL + + X = 4
a;

(^Add to x?
\

X^

+ i-

the

x^

quantity

required to

make

it

the square of x H

) X )

BXERCISB
^\.
Illustrate

180

(Review of Chapter XZI)

Explain the different methods of solving quadratic equations them, by solving in full the equation 3x* 4x 15 = 0, by each method.

^.
(^
is

Solve 323x* + 2x=l.

The

104.

difference of two numbers Find the numbers.

is

and the sum

of their squares

V/C
5.

If

2( 1

+-

j,

find.

X to two decimal pie

What
?

in the price

is the price of meat per lb. if a reduction of 20^ would mean that 5 lb. more than before can be bought

fo^ $3
sji.

Solve

10x-I9x-9 = 0.

\^.

The

sides of a right-angled triangle are o,


?

o 10

and o+lO.

What

are the sides

THE GENERAL QUADRATIC EQUATION


Solve

27'3

x\

Xi

x 5 = xo

4.

The sum
9.

of two numbers Find the numbers.

is

45 and the sumi of their reciprocals

j/ioT
11.

Solve 6375x-10a;=l.
is

The length of a rectangular field The area is 3 J acres. Find the sides.
12.

5 rods

more than the width.


r

What
|

miist be the values of n in order that :


?

equal

when a = ^\

10n+21a

may ^

\13. The perimeter of a rectangle is 56 and the area is 192. Find the diameter of the circle which passes through its angular points.
14.

Solve

OOTox'+Tox^

150.

15.

By

solving

(x 2){x 3) = (a 2)(a 3),


3)

can be substituted for a in (a 2)(a

find a quantity whicL without changing its value.

,^.
17.

Solve x - 2a; - 89a; + 90 = 0.

Two
is

trains each run 330 miles.

One

of

them, whose average

speed

5 miles per hour greater than the other, takes ^ an hour less Find their average speeds. to travel the distance.

x+l a;+2 , Solve ^:j-j-+^-j:p2 = 2*--^


1
,

V.

9.

o Solve
,

^^^+-^ = ^.
,

a;

x*+\

109

I sell a 20. cost in dollars.

horse for $96 and gain as WTiat was the cost ?

much %

as

the

horse

^.
22.

Solve {a;*-3x-5)* + 8(x*-3a;-5) + 7 = 0.

Divide 25 into two parts so that the sum of the fractions formed by dividing each part by the other may be 4 25.

The sides of a rectangvdar 23. Find the area. diagonal is 50.


72 Solve x + a;+-r- =18.
x'-{-x

field

are

x+17 and

a;

17.

The

Solve (m n)x + 2x(m* + n*) + m* n = 0. Solve (x-5j(x-J )(x+V)tx+


27.
3)

= 60.
x'= 1 25.

Find

all

the roots of the equation

Since x* 8x+ 12 (x 2){x 6), for what values of x will the 28. expression x* 8x+12 be equal to zero, and for what values will it be negative ?

278
Solve

ALGEBRA
xa

29. 30.

xb

-.

a-\-b

Solve adx OCX* = hex hd

The area of a square is trebled by adding 10 inches to one 31. side and 12 inches to the other. Find the side of the square.
32.

Solve x(a-c*)-a;(o6 + 36c) -262 = 0.

33. 34.

Solve (x* + 6x+8)= + 3x(x2 + 6a;-(-8) = 0.

A man bought a number of acres for S300. If he had paid S5 more per acre, the number of acres would have been 2 less. Find the number bought.
35.

Solve
1

x-\-a-j-o

=
a

i +a+
b

^ b

36.

xa Solve z
b

x~b =

xa

a = xb

OX and OY are two roads at right angles. A starts at noon 37. along OX at 3 miles per hour. B starts at 2 o'clock along y at 4 miles per hour. Find to the nearest minute when they will be 20 miles apart.
38. 39.

Solve a*x-2oa; + a*-l Solve ax'

= 0.

+ 6 =cx
,

6x*.
is

40.
it is

gra%'el

path 2 yards wide


it

made round a square


field.

found that
Solve
8

of the field in

takes up j^^ of the area of the square yards.


ioT
t.

field and Find the area

41.

= vt+\(>t*

is

What positive integer is that, the sura of whose square and cube 42. nine times the next higher integer ?
43. 44.

Solve (x + x-2)-4(x* + x-2) + 3 = 0.

The

side of a square

is

34 inches.

sides the vertices of an inscribed square

Find at what points in the must be placed so that it

may have
45.

an area of 676 square inches.


in .

Write the equation ox* + 6x2/ + C2/*=0 aa a quadratic

What

are the values of -

and

of -

What positive integral 46. nearly equal to 1000 ?

value of x will

make x*+10x moat

CHAPTER XXII
SIMULTANEOUS QUADRATICS
193.

and the sum


Let

Consider the problem The sum of two numbers of their squares is 74. Find the numbers.
:

is

12

.'.

a;*

= one of the = the other, + (12-x) = 74.


a;

numbers,

12 a:

Solve this equation and find


If
.*.

x=7
7.

or

5.

x = 7 or

5,

then 12 x = 5 or
7.

the numbers are 5 and

Here we have used only one unknown.


solved by using two unknowns.
Let X and y be the numbers,
.-.

We

might have

a:

and

a;*

+ t/=12, + 2/* = 74.

How
194.

can we obtain from these two equations the original equation


?

in the preceding solution

Type

I.

280

SIM ULTANSO US Q U ADRAT


BXERCISE]
Solve and verify
1
1

W8

181

-6

x^y^l,
xy=12.

2/ x-y='k,
xy=60.

/3.

x-2y=Q,
x^-y^=27.

ji.

x-y=S,
a;2+y2=65.

^ x-y=6,
a;2-2/2=60.
8/;

0.

2x+2/=9,
x^-y'^=\b.

1* x+Zy=U,
^
\^tO
)

2x+Sy=l2,
a:2+2/2^13.

9^
'^

3a;-4t/=2,

x2+2/=27.

3x^+2y^=U02x -3i/=-]

x2+3a/ '-/2_^2x=3''

Kll\
I
J

3:K2_2a;2/+5x-?/=17

X j/=3.
12.
If

x3y=2

and x2 a;?/+2?/2=6,
of

tind the valu8 of x

and

i/

to three decimal places.

13.

The hypotenuse
is

a right-angled
sides.

triangle

is

25

and the
of the
their

perimeter

56.

Find the

14. .4 is 10 years older than B. Eight years ago the squares of the numbers representing their ages was 148.

sum

Find

15.
is

The diagonal

of a rectangle

is

50.

The

difference of the sides

10.

Find the area.

16.
of the

The area of a right-angled triangle is 96 and the difference two sides about the right angle is 4. Find the hypotenuse.
Solve

17.
18.

3x+5y=2, 3x^-l0y^-xy+28=0.
the product of these the digits are

K each digit of a number be increased by 2,


is

increased digits will be the original number.

When

interchanged the resulting number


the original number.
19.

thirteen times the tens digit of

Find the number.

The sum

of the areas of
is

two squares

is

40 square inches.

The

side of the smaller


larger.

10 inches less than three times the side of the

Find their sides to three decimals.


Solve

20.

+5^= 14, 2/-l=a;.


y^

195.

and

y,

When both equations are of the second degree in x they can not always be solved by elementary methods.

282

ALGEBRA
in
/

There are special cases


difficulty.

which they can be solved without

^^,y ^^'
II.

'

^ ^\
\s<^

^^

^\J(j\JL
r^-Cf"^

cUc^s*^
^

o:.

196.

Type

Solve

z^5xy-\-4:y'^=0,

(1)

(2)

Factoring

(1),

(x-4y)(x-y) = 0,
x=-ly or x = y.

We

are

now

required to solve
x

+ 2/ + 3x=29\
x,

x=iyf

^^^ x*+y* + Zx = 2%\

x=y /

Substituting the value of


16y

+ 3/*+12j/ = 29,
22/*

17i/+12j/-29 = 0,
(j/-l)(17r/+29) = 0,

+ 3t/-29 = 0,
3+V24I

y=\ x=4

or or

-H. Vt':

-3+V241
-3V241
4
J^ or

Here there are four solutions


X

= =

4 or

1/

or

3+^241

In this type the first equation contains only terms of the second degree. When that is the case the left-hand member may be factored and each of the resulting linear equations may be combined with the second equation, thus giving two
cases of

Type

I.

BXEROISB
Solve and verify 1-5
vl.

182

x2-?/2=0,

2.

x2+xi/+y2=36.
,

x2-4xy+3y2=0, x2+y2=10.
x2+2/2+2x=12,
3a!+2xy=y*.

3.

3x*-2x2/-j/*=0,

x+y+y*=32.

>

SI MULT Alii EOU 8


6.

QUADRATICS

283

4:c''+20a;y+V=0,
2ary+l=0.

(6>

^ + ^=14,
x\=y^.

^'^
9.

x^xyy==\.
Find four solutions

6a;^+4i/2=ll2;y.
of the equations

{x-y){x-2)^(i, (a:+y-6)(2/ + 3)=0.


197.

Type

III.

Homogeneous Equations.

Solve

x2 xi/=6,
y^+^xy^lO.

(1)

(2)

Multiply (1) by 5 and terms, and we get

(2)

by

and subtract, to eliminate the absolute


(3)
in

bx*-UTy-Zy* = Q.
This equation
Tj^pe II.
(3)
is

of

the
(1)

Grouping
(3),

(3)

with

same form as the first equation we proceed as before.

Factoring

{x-'iy){t.

284

ALGEBRA

The pupil should be on the look out for special methods of obtaining f'-om the given equations an equation of the first degree. Here we might have done so by simply adding the equations and taking the square root. Solve it by this
method.

BXHRCISB
Solve and verify 1-9 ^^ (jC. 3a;2-5j/2=28, Q^.
:

188

3xy-iy^^S.
3i/2-a;2+l=0.
7.

2x^-3y^=23, 2xy-3y^^3.

C^.
G.'

x^-xy+y^=2\, 2xy-y^=15.
x^xy=5^, xy-y^=18.
x--4y^^20, xy=\2.
32y^=^2xy+n, x^+iy^=lO.

A.^ 2z^3xy=U, '^p' x^+xy=66,


x^-y^=-U.
)i

x^+2xy^Z2, 2y^+xy^\6.
x^-3y^=i, x-+xy+y'^=28.

3x^-5zy+2y^=U,
2x^-5xy+3y^=6.

)^.

^.*
13,

J^

x^+xy+y^=7, 3x^l=xy.
14.

ht

^^

2x^dxi/+9y^=5, ix^-10xy+l\y^=3o.

x-+xy+y^=l,
2x^-{-3xy+i:y^^2A.

15. 10,

3x^-3xy+2y^=2x,

2x-

+ 3y^-xy=-ix.

Find, to two decimals, the real values of x and y which satisfy

x^xy=20 ana 3xyy"=50. 17. When a number is multiplied by


is

the digit on the left the product

105

when

the
is

sum

of the digits

is

multiplied by the digit on the right

the product
198.

40.

Find the number.

Special Methods.
it

Since {x-\-y)^={xy)^-{-'ixy,
of

follows that

if

the values

any two

of the quantities x-\-y,

xy

and xy are given, the

remaining one can be found.

Ex. 1,Solve
Squaring
Ftoxa. (2),

x+y=U,
xy=\%.

(1)

(2)

(1),

Subtracting,

+ 2xy-\-y^=\.2l. 4an/ = 72. x* 2xy-\-y* = A9, xy=l. :.


x*

SIMULTANEOUS QUADRATICS
If

285

+ 2/=ll,

38<

ALGEBRA
(a;+?/)2 5(x4-i/)

Ex. 5.Solve

6=0,

(1) (2)

Factoring

(1),

(x

+ 3/-6)(x-t2/+l) = 0, x-f^ = 6 or 1.
and
,

Now

solve

x + y=6, * xy = o.

x+" v=
'

xj/

It = 8.

'

EXE3RCISE
Solve,

184

by finding

x+y

and xy, and verify

J".

x+y=S, xy=

xy=4,
xy=12.
{x-y)^=l,

J^

x2+2/2=25,
X j/=l.

1;">.

4.

x2+w2=61

J7.

j^

xy=30.
8

x2 x2/+2/^=57. x-y^8.

x'^+xy+y^=19,

x^xy+y^=19,

x+y=5.
9.
i

x+y=U.
10,

5x2

+ xy+ 52/2=23,
x+y=l.

x2+?/2=:89,

xyiO.

^. x^-lxy+y^=-lOl,
xy=^30.

p.
l4.

2x2+3x2/+2y2=.8,
X2/= 6.

x^-y^=l9,
'J-

xy^l.
x^xy+y^^Z9,
x^+y^=35l.

^
1^
V-'

x3+2/'=1064,
x+t/=.14.
X*

+ x2?/2 + y4 = 21,

x2+xy+t/2=7.
X*-x2i/2 + y4^13^

17.

x+x2j/2 + 2/<=133.

18.

x^~xy+y^=lt
{x+y)^-S{x+y)-28=0, x-y^3.
(x-i/)2-(x-2/) + 12=0, X2/=12.
x22/2- 27x^4- 180=0,

xy=2.

x+y=8.
is

22j

The perimeter
Find the

of a rectangle

34 inches and the diagonal

is

[3 inches.

sides.

SIMULTANEOUS QUADRATICS
The diagonal
sides.

287
300.

of a rectangle

is

25 and the area

is

Find the

(24/ The sum


is

72-5.

of two numbers is 12 and the s^m of their squares Find the numbers. *'**t*'*****^ 7Cm .

The product

of

byB
26.
is

the product will be 180.

two numbers is 270. If each number is decreased Find the numbers. T/^^a^- /R.O

^^

A.
27.

The sum of two numbers is 10 and the sum of their reciprocals Find the numbers. 3-

Solve (a;-l)(i/+2)=9, 2xy=\5.

28.

A and B

.nore than B,

are two squares. and the perimeter

The area
of

of

is

63 square inches

is

12

inches

more than B.

Find the side of each.


29.

Find two numbers whose product


Is

is

and the sum

of

whose

reciprocals

2'i^.

30.

Solve x3-8!/3=56,

x-2y=2.

31. The sum of the two digits of a number is f of the number. The sum of the squares of the digits is 4 less than the number. Find

the number.
32.
is

The area of a rectangle is 1161 square yards, and 140 yards. Find the dimensions.
Solve 1 X

its

perimeter

33.

+1=
y
of a
is

-3,

-i

- 1=
y^

03.

x^

34.

The sum

reversing the digits

number of two digits and the number formed by 121. The product of the digits is 28. Find tlie
a right-angled triangle whose perimeter
is

number.
35.

Find the sides

of

is

24 inches and whose area


36.

24 square inches.
if

Prove, algebraically, that

two rectangles have equal


in all respects.

area.s

and equal perimeters, they are equal


37.
38.

Solve x^+xy-\-y^=l-15,

x^xy+y-=5-2o.

What must

7^ acres, if any other point

be the dimensions of a rectangular field containing the greatest distance from any point in its boundarv to
is

50 rods

288
39.

ALGEBRA
The sum
is
?

of the radii of

two

circles

is

8 inches
is

and the sum


9 inches.

of

their areas

of the area of a circle

whose radius

What

are their radii 40.

What must

oe the length of a rectangular

field

that contains

a square rods

and which can be enclosed by a fence

b rods long.

199.

Graphical

Methods.

What
?

is

the

distance

of

the

point P(4, 3) from the origin

Since
.-.

OP^=OM'^-\-MP\ OP2^42+32=25,

.-.

OP

=5.
the same

If

any point

{x^y) is

distance from the origin that

is,

then the point {x,y) must lie on a circle whose radius is 5 and

whose centre

is 0. But the square of the distance of the point {x,y) from the origin is x^-\-y'^, :.

x2+i/2=25.

It
circle

is

thus seen that the equation x^-\-y^=25 represents a whose radius is 5 and whose centre is the origin.

Similarly, x*-r2/*=16, x* + y* = 100, x* + y*=l8, represent circles with the origin as centres and whose radii respectively are 4, 10, Vl8.

It is seen that it is a simple matter to draw the graph of the equation of the circle in the form x^-'ry^r^. All we

require to do
jentre
is

is

to describe with the compasses a circle


is r.

whose

the origin and whose radius the radius


is

When
Here
a:

a surd as in x'^-\-y^=l8,

it

is

simpler

to find a pair of values of x

=3,

y=3

satisfies

and y which satisfy the equation. the equation, and the circle is then
(3, 3).

described through the point

SIMULTANEOUS QUADRATICS

289

290

ALOE BRA

BXSRCISB
/I,
"are

186
circles

On the same sheet draw the

graphs of the

whose equations

x^+y^=^'i,
2.

x^+y^=9, x^+y^=^lZ, x^+y^=3i.

Solve graphically x^-{-y^=l3,

xy=l.

Find graphically the positive integral roots of a;2+2/*=25 and 2x+3y=l8; x^+y^^lO and 2xy^5. Approximate to the
3.

other roots.
of two numbers is 8 and the sum Show, graphically, that this is impossible. the sum of the numbers is 7 instead of 8 ?

4.

The sum

of their squares
Is it impossible

is
if

25.

BXBROISB

186 (Review of Chapter XXII)

dy
2.
3.
is

Solve x + 2/ = 28,

X*- 2/* = 336.


10

Solve 5x-2i/=I2, 25x-42/* = 96.


of two numbers Find the numbers.

The sum

is

and the sum

of their squares

58.

4.
5.

Solve 2x-3j/ = 4, x + 2/* = 29. Solve 3x-42/ = 4, 2x* +


3xi/

= 56.
5

6.
18 g.

The sum

of

two numbers

is

and the sum


x + y = 3.

of their reciprocals

Find the numbers.


Solve x*-\-xy+2y*

7.

2x~7y + 5 = 0,

8.

Solve x* + xy62/* = 0. x*-|-3x)/ 2/* = 36.

A field whose length is to its breadth as 3 to 2 contains 664 9. square rods more than one whose length is to its breadth as 2 to 1. The difference of their perimeters is 60 rods. Find the dimensions of each field.
10. 11.

Solve x* + 2x2/ = 55,


Solve 2x*+3x?/ = 8,

x2/

+ 2?/* = 33.

y'-2xy = 20.

The area of a rectangle is 300 square feet. 12. If the length is decreased by 2 feet and the width by 3 feet, the area would be 216 square feet. Find the dimensions.
13.

Solve x(x4-t/)

= 150,

2/(x

2/)

= 75.
14.

14. 15.

Solve x(x 1/)= 15, 7/(x-(-y)=

Sodding a lawn at 9 cents a square yard coetB $108. If it had been 10 yards longer and 6 yards wider the cost would have been half a much again. Find the dimensions.

SIMULTANEOUS QUADRATICS
16.
17.

291

Solve x3-i/= 126, x4-a^ + j/ = 21. Solve x + 3an/-6y*4-2a; 2/=12, x + 2/ =


7.

and x*t/*-6x7/ + 8 = 0, find the (x4-t/)*-7(x4-J/)+12 = values of x + 2/ and xy, and thus solve these equations for x and y.
18.* If
19.

The product

of

two numbers

is

28 and their difference

is

Find the
20.

sum

of their squares, without finding the

numbers.

Solve 8x3 + y = 280, 2x-fy=10. Solve


2/

21.

= x+ a/2,

x*

+ 2/=l.

is

Find two positive integers whose sum multiplied by the greater 22. 192 and whose difference multiplied by the less is 32.
23.

Solve

+ y* = X*
a*
fe

ab
10,

xy

3.

24.

If

12x-41xi/ + 3oj/* = 0, find the values of -


of two numbers Find the numbers.
is

25.

The product
is 5.

and the difference

of their

squares
26.

Solve

^+y= y*

6,

x-w = 4. ^

\~.

Solve (x + 2/)(x+22/)

= 300,

^=
^

3.

A regiment consisting of 1625 men is formed into two solid 28. squares, one of which has 15 more men on a side than the other. What ia the number on a side of each ?
29.

Solve

1
-(

292
Solve

ALGEBRA

36. 36.

-+- =

-,

-4

=
xi/

Tr'

Solve (x + r/)-x-y = 20,

= 6.
two consecutive odd numbers

37.
is

The

difference of the cubes of

218.

Find the numbers.


Solve x x2/* +
162/*

38.
39.

= 28,

x*

+ 3x2/ + 42/= 14.

Solve x'+2/ = ?/* + x=3.

40.
is

The diagonal

of a rectangle

s.

What

are the lengths of the sides

obtained to find the sides of and one side is 7 inches longer than the other.

and the difference of the sides Apply the formula thus ? a rectangle whose diagonal is 13 inches,
is d,

Solve 9x* + i/* 21(3x + 2/)+ 128 = 0, xi/ = 4. (Make the first 41. equation a quEwlratic in 3x + i/, by adding to 9x* + ?/* what is necessary to make a complete square.)
42.

Solve x* + 4?/-18x-36i/-)- 112 = 0,

xi/

=8

43.

Solve x+3/3= 126,

x2/

+ x2/* = 30.

CHAPTER XXIII
INDICES

BXERCISB
1.

137 (Oral)
?

What

are the values of 3 2, 2^, 1*, P", 0^

2. 3.

Simplify 3

22

3x10'; SxO";

03-f-4.
:

When x=10, what


x3, 6x2,

are the values of

200^x, 500-^x2, Gx^-fx^

4.
5. 6.
7.

Givethevaluesof(-l)2, (-1)3, (-1)4, (-1)37^ (-1)'^

What

are the values of (2)2, (2)5,

( 2)
and

Find the difference between 2^ and

32, 2^

52.

What

does x*

mean

How many

factors are there in

x^xx*

8. 9.
is

Express in the simplest form a2xa3xa*.

How many factors will remain when x^


?

is

divided by x'

What

the quotient
10.

What

are the values of


X8

x*^x3, x^^x*
^;.3
Trr

O20
0^
?

a*b^^
a^b^
its

x'
11.

What does

(a2)3

mean

Read

value without the brackets.

12.

State the value in the simplest form of


(X2)2, (2/3)2, (^3)3^ (^3)4^ (^2)10.

13.

What

does

(06)*

mean

What

/a\3
does
(

mean

Read

their values without brackets.

14.

Express as powers of 10

100, 1000, 10,000, 10 X 100, 10^

IO2,

10-rlO.
298

29*
15.
to.

ALGEBRA
Simplify

(-l)x(-l)3x(-l)*; (-a)V (-a)*x(-a)'


the value of x
if
?

What

is

lO'^lOOO, 2^=16, 5'= 125, 3'=8I


17.

Express 32,794

in

descending powers of

10.

202.

Definitions of a"'.
a,

As a^

is

the product of three factors

each equal to
to a.

so a'"

is

the product of
to

factors each equal

a^"

a.a.a

factors.

Here
203.

it is

understood that

is

a positive integer.

The Index Laws.


(1)

We

have akeady seen that

a^xa*=a^^^=a\
a'=^a^'=a'^'^^a^.

(2) (3) (4)

(a2)3=a2^3=a.
{ah)*

= a*b*.
a'

/a\3

letters to

Let us now express these statements in general form, using denote the indices.
(1)
(2)
(.3)

n"'ya"=a"'*".
n"'

a"=a"''".

{fi"')"

= (i'"".

(4)

{ffhy"^a'"b*>^.

These are called the index


than
in n), to

laws.

The
(2)

letters

m
is

and n
greater

represent any positive integers, and in

?n>n {m

make

the division possible.

The

laws, as stated

the general form,

may

be proved as in particular cases.

INDICES
204.

295

Law

I.

Law

for Multiplication.

a'"xa"=a'"^".

By

definition,
a'"

=a = (o =a

a a
.

to

m factors.
n
factors,
.

0:^=0

a
.

to
.

o"

Xo''

to

m factors)(o

to

factors).

a ... to (m + n^

factors,

= 0"*+",
Also,
a'"

by

definition.

xa"xai =0"*+" x a^,

205.

Law n.
a"*

Law
a a
. . .

for Division,

rf"^-:- re '*=*'.

o"

a a
.

a
a

to to

m factors
n
factors
if

=a a = a~".
Here the n factors
in the

a ... to (m n) factors,

w>n,

in tiie denominator cancel with an equal number numerator, leaving factors in the numerator. If, however, n>m, the n factors in the numerator cancel with an equal number in the denominator, leaving ntn factors in the denomi-

mn

oator.

when m>n,
and when n>m,
206.

a"'-:-a"
a"*
-:-

= o'"",

a"

qh m

Law
(a"')''

in.
.

Law
o
o
. . .

of Powers.
. . .

('")" =a"*~.

= a'^ = (a
=a
.

a""
.

to n factors,
.

to 7n factors) (a

o ... to

factors)

the

brackets being repeated n times,

a =0".
Also, {(a"')"}p

a ... to

mn

factors,

= (a'"")^=o'"'v.
IV.
.

207.

Law
(06)"

Power
ab
.

of a Product.

(&)'="&'*.

= ab = (a

ab ... to n pairs of factors,


.

to

factors) (6

to

factors)

Also, (a6c)"=(o6)"

c"=o"6c".

296

ALGEBRA
Law
V.

208.

Power

of

a Quotient.

{l)"

= l-l-l---^oniactors.
to 6
.

n factors

to

factors

have given five index laws. They are not ail The second and thirH laws jnay easily be deduced from the first.
209.

We

independent.

(1)

When m>n,
.'.

o"'-^o"
o""

a" = a'^~" xa" by Law I. = o'"~", which is Law II.

(2)

xo'" = a"* "''" = o*'", by


xa'" = a"'+"'+'" = a'"',

Law

I.

Similarly,

a"'

xa
n

and

a"*

a"'

a"*

to

factors =0"'+'"+"'
(a'")"

to

n ter'js=aM

= o"'",
is

which

is

Law III.
called

For

this

reason the

first

law

frequently

the

fundamental index law.

EXERCISE
Simplify
1. 4. 7.
:

188 (1-18. Oral)

a^xa^xa^.
(a263)2.

2.
6. 8.

x'xx'-fa;'.
(32)2.

3.

(x^)*^x^.
(33)2^(32)3.

6. 9.

(abf^a^b.
6'

5*^5*.

((-2)2)3.
(oft)"

(-1)7

13.
16.

oC'xxf'Xaf.
x
+

14. 17.

a'.av.a'-v.
(a263c*)2.

15. z"*-"xa;2' + .
18.
a:"

*-^x-^

x* + '

a5

'

".

-'Ko^y^*-

^-

2<'

+ *va;2*'*''x JC*"

*"*

INDICES

297

t^

\b)
dTn + n y^

\c)

\a)

^'

7i>-'^ TT^-^ 7^-^

a"*P

2^
26.

Express 4" as a power of 2 and 9' as a power

of 3.
3.

Divide 27^ by 9' by expressing each as a power of

^
a'"

2nx2"-i X
.

22

Simplify

and

9"x3

+6

g^^i

28.

Solve

52x+i_5i+3. 41=2**'; 93**s__27^"S


210.
Fractional, Zero

2*. 4^

8^=16^=^"'.

and Negative

Indices.

We

have defined
This

to

mean

the product of

w
tell

factors each equal to a.

definition requires

to be a positive whole number.

Thus, the definition will


tell

us

what

a^ means, but will not

us what a^ means, nor what a~* meaas, nor

what

means.
If
it is

we wish
it

to use in algebra such quantities as a^, a"*, a",

necessary that

we

define their meanings.

would be very inconvenient if we gave to these new forms of indices such meanings that the index laws, alreadj' established for positive integral indices, would not apply to them. We will, therefore, give to fractional, zero and negative indices such meanings as will make the index laws
valid for
211.

Now

them

as well as for positive integral indices.


of a Fractional Index.
if

Meaning

Since x^Y.x^=x^^'^, then

we suppose
it
a;.

that the same

law applies to fractional indices,


xi

follows that

X x^ = x^ "^ ^ = xi=

Thus, x' when multiplied by x^ gives the product x, or the square of x^ is x. But we have already represented the quantity whose square is X by Vx,

298

ALOE BRA
this
is

That
follows

a reasonable value to attach to x^

might appear

aa

We know
case.
If

under the root sign

x*= Vx^, x*= Vx*, x^= Vx*, the index of the quantity in each case being hiMpi the index in the preceding now we take half of the index on each side again, it wouIq
that

seem but natural that x- should be equal to Vx.


Similarly, x^

X x^ X x^ = x*

*"

"*"

= x.

x^='^x
Also, x*

(the cube root of x).

= Vx (the

fourth root of

x).

and

x^= y/x

(the nth root of x), where n

is

a positive integer

Thus,

4^=v/4 = 2, 125^=^125 = 5, 32^=1^32=2.


III,
.-.

By Law

(x^)

= x*,

x=-^x.

Similarly,

(x?) =x-P,
E
.".

= V xP, where p and q are positive integers

9i

We

thus see that

if

as to positive integral

the same laws apply to fractional indices indices we are led to the conclusion
q are
the

that x^^^vx^,

when p and
is

positive

integers, that

is,

when

the.

index

indicates the root to

denominator of the fraction be taken arid the numerator the power.

a fraction,

By Law

III,

x
p

= UV

=(xP),

?'

So that

x^

means that the


g""

p"'

power

of the

q'^

root

is tc

be taken, or the
Thus,
Of

root of the

j"'

power.

8=i?/8=-^6i = 4,
8^

= (^8) = 2 = 4.

INDICES
It will

299
first

be seen that

it

is

simpler to take the root

and then the

power.

Thus,
212.

32'^

= (\/32) = 2 = 8.

Meaning
I,

of a

Zero Index.
o''xa'"

By Law

= o+'" = a".

Therefore,
positive

if

integral

the same law applies to zero indices as to indices, we are led to the conclusion that
index zero
is

any quantity
Thus,
213.

{zero excepted) to the

equal to unity.

30=
Meaning
I,

1,

(ox)''=l,

(-2)=1, (-106)0=1.

of a

Negative Index.
a

By Law

Similarly,

We
to

thus see that, any quantity

to

a negative index

is

equal

unity divided by the same quantity to the corresponding positive

index.

Thus,

300
Thus.

ALGEBRA

-^^

-^ = 8; -^ =
a^/8^x V^IG^
;

4x-a3 = -^

Ex.-SimpHfy
i^S'x

{^^J^
16 9a6

VW^ = 8^ X

16*

= (23) X (2)^ = 2x 25=32.


g-

/ 9a* \'g

9"'

64
27ae6

EXERCISE
What
1.
is

189 (1-82. Oral)

the meaning of

ai

INDICES
Simplify

301

^*

16" i

59.

Solve a;^=4, a;^=32, x*=27, x'^=Z, a;"2=8.

215.

Operations with Fractional and

Negative

Indices.

The

following examples will illustrate

applied to

the

multiplication,

the index laws may be division, etc., of quantities

how

involving fractional and negative indices.

The work will usually be simplified if all expressions are arranged in descending or ascending powers of some common
letter.

Thus, 5+x^+x-^2_^ powers of x, thus


:

-i_|_a;2

would be written

in

descending

Ex. 1.Multiply 2x^ + 3 x-i by Sx^ 2 5x-i. Ex.


2.

Divide

^x"^

ab by

ahh^.
(2)

(1^

3a;i_2
&x

a a^b^

+9x^- 3 -4x^- 6+

ix'^

-10-15x~^ + 5a;-i
8a;

+5x^-I9-13"* + 5a;-i

302

ALGEBRA
of 9.r 12^*4-

Ex. 3. Find the square root

10 4.r-i4-x-i.

9a;-12x* + 10-4a;"'^+a;-i 9x

3x^-2 + x'^

^-2
I

-12a;^+10

-12x^+
6x^-4 + x'^

6-4x'^ + x-^
6

4x
^

^-j-x'^

Verify by squaring 3x^ 2-\-x check by putting a;=l.

by the method"

of

art. 93.

Also

EXBRCISB
Multiply
1.* a;2+3, a;^-2.
3.
5.

140

2.
4.
(J.

x+x^ + l,z^-X,
3x-2a;^+5, x~2x^

x-^-x+x^-1, x^+l.
a^_l-f-2a~i o2 + l-2a"i

{a-a^ + l)^.
(x^

7. 9,

x+5xi+6xKx^-l-x~^.

8.

x+x^^+y, xx^y^+y.

10.

+ 2)*. (a- 1)^.

Divide and verify


11. 12.

a+5aM+66
a;3 a;2+a; 2

by a^+2bk

2a;-

2 2a;-8 by

a;+24-2a;

*.

13. 14.

x^-\-x*y^-^y^ by z^+^^^J/ +2/

1 5a;" X by 1 x^+3a;^,

"

as far as four terms.

Find the square root and verify


15.

+ 6a^+9

and 25x2 10+a;

16. 17.

a2+4a2-f6a+4o^+l.
4a;^'-20a;3+37a;-30a;^+9i

INDICES
18.

SOJ

49-30x^-24a;~^+25x34-l6z"i

19.

Show

that

'-'^^+^^
by x- a;

= (a^+fe^)-'.
2.

^^
21.
12.

'Divide x^-z~-

Divide

10a3'"_32a"' 27a2'" + 14 by 2a'" 7.

SimpUfy (a;+xi+l)2+(a;-a;^ + l)2.


Simplify

^3.
24.

(Vo+l)(\/o-l)-(\/3a+\/2)(V3a-V'2).

Find the square root of


Contracted

x^AxVx+\0x\2\/x+9.
will

216.

illustrate

how

The following examples Methods. contracted methods may be employed.

Ex.
If

1.

Multiply

x+z^4 by

a;4-x^+4.

x + x- be considered as a single term, the product

= (x + x2)-4,
= a;* + 2x^+x-16.
Ex. 2. Divide
Since a
2;3

[{a

+ b){a-b) = a*~b*.'\

a+b by a^+bK
b of b^, this is similar to

is

the cube of o^ and

dividing

j/3

by x + y.
{x^

Since so

+ y^)~(x-ry) = x* xy + y*, (a + b)^{a^ + b^) = a^-nh^-irbK


the cube root of

Ex.

3.

What
is

is

This
last
.'.

is

evidently the cube of a binomial whose

first

term

is

2x' and

term

3t/^.
is

the cube root

2x* Sy*,

if

the given expression

is

a perfect

cube.

Check when x = y=l.

Usir^ the method of art. 155, the cube root of complicated expressions may be found.

more

804
217.

ALQEBRA
Factors with Fractional or Negative Indices.
If

we

are

permitted to use fractional or negative indices, many expressions may be factored which were previously considered algebraically prime.
Ex.
1.

a b 3iay be written as the difference of two squares, thun

(a^)-(6i).
.-.

a-6 = (a^ + 6^)(a^-6-).


by
cross

Ex.

2.

3a5

The factors are


Ex.
It
.'.

(Sa;-

2x~^ niay be factored + 2a;'^)(x* -x'-).


is

multiplication

3.

x*-\-xy-\-y*

an incomplete square.

may

be written

{x-\-y)*

{x-y^)*.
-\-y){x

the factors are {x + x^y^

x^y^ -\-y).
141

EXSRCISE
Use contracted methods
in the

foUowing
a^l)^-

yV'i.* Multiply
2.

x^-2 by

x^ + 2

by J+fet

Multiply

a^l+a"^ by
of

a^ + l+o~i

3.

Find the square

x x- 1

and
l

of

2a 2 o~^.

4.

Write down the cube of a-

and

of

1 x-.

5.

Multiply

z+x-y^+y, xx^y^+y, x^xy-\-y\

11

11

6.

Divide x-{-y by x* x^y^ -\-y^K Divide

7.

a+2aM+6 c
common
r

by a^-\-lhc^.

8.

Find three factors of x^y^.


Find a
factor of

9.

a+a^b^ 2b, ab.


a'^+ab+b^
j^.

10.
^-

Simpufy

x5xi+Q
x+x^-6

a-b
.

-j

::z-

a3-63

a+y/ab+b

INDICES
3
jL

306

11.

What

is

the cube root of a;2_6x-i-12a;^

8,
?

and of
12.

x3-3J + 6a:2-7a;^ + 6x-3a;5 + l


What
is

the

square root

of

4x-*-fl2-?-3_|_ 93.-2^

and

of

x2+4a:+2-4a;-i+a;-2>

EXERCISE
1.
2.

142 (Review of Chapter XXIII)

State the index laws.

Explain

how meanings

are

assigned

to such

quantities

as

/S.*

When x= 16, = 9,
t/

find the values of

jk.

Find the numerical values


8^, 9~',

of

i/\2b*, 32-*, 16-i. -25"^ (-64)"^.

Jb.

Show that

i'%*

X ^^8^=

108.

^6.
^/7.

Simplify 32"^-i-(^i.)J

^nd 812^^(iVrV-

Find, to two decimals, the value of 10*

when

x= i,

1, i^ |.

J&.
J9.

Simplify 2*+\0'>-'^^-{l)'^ + {)*^{y/J^)-^.


Find, to three decimals, the value of (3*)~^x v'27.

710.
JlI.
yl2.
13.

Simplify

16^+ 16'- 16"*- 16"^ and 32=-32*+32' + 32~*.

Simplify 5^ X 5^

5^

16^''

X 16" x

16"".

Solve

a;^

= 8,

2^ .4^

= 64
-^'3

SimpUfy 4^x 6'*x


eSimphfy -5 X

and
,

{83

+ 42)x 16"^.

^A 14.
15.

1*

2"+!

6-"
jgr-

3.2''-4.2"-i

and
:

^-g^^.
a^+ab *-*'

Reduce to lowest terms

o+SVo-f 15 3x^+5x^+2 a+7\/a4 12' j.*+l

'

506
16.

ALOEBKA
Multiply

x^y^-2xy + ^x^y^ by x^ + 2y^


-

17.

Miiltiply x' y

-\-\-\-x~^y^

by x^y~^ \-\-x~^y^.
4

18.

Divide

x*

y-*

by x^-\-x^y

3-8 ^

-
-\-y

'

and
19.
20.

o^+1-f.a"^ by
Divide a*"'-6*'" by a"'-6'".

a^+]+a"i

The dividend
is

is
1.

J/-+22/*

St/ 2, + x~^)*

the quotient

is

J/- i/'-l,

the remainder
21.

3i/^

Find the
{x

divisor.

Find the square root of

-4(x x~ij.
j'-fc^j'

22.

What
If

is

the cube root of ia*-ia^6i' + a^6*-'-

23.

x = a+l and

2/

= a->+l,

show that

^ + ^~y = xyx+y

o.

24.

Simplify -008^
If

1-728^,

2-25s,

-0625'^.

25.

x + y = a^ and

x~y = a
26

^,

find the values of xy

and x*+v*

in terras of a.

26. 27.

If 2a

= 2^-t-2-^

and

= 2^-2"^

find a*-b*.

Find the square root of (e* e~')*-f-4 and of

x*-'ix^y^+l0x*y-l4:x^y^+l3xy*-6x^y^+y'.

28.

Simplify

-^
c

-rb

-i
''

29.

Factor

x*

y,

x 5x-

6, x 1, 4a 6* and
^nd
IQi-Bosii.

x-4x+10-12x-i + 9x-.
30. 31.
If 10-30103 If 7*<i

= 2,

find the value of IQ-soJoe

= 50

and

7<'6

= 55,

find the value of T*"'"'.

2n+l
32.

/2r.-l)+l
^.;,

Simplify

y^^^^ x

INDICES
33
34. 36.
36,

307

Solve 3'+

+ 2" = 35,

3'^

+ 2''+ = 41.

Divide

x-2{x^-x'h + 2(x^^ -x'^-^'^ by x^-x'K


is

Show

that x^'^^={xVx)''

satiafied

by x=2l.

Find the square root of

U2Vx)^-2x^+x+4x*+ Vi-i-

^.

'ii

CHAPTER XXIV
bURDS AND SURD EQUATIONS
218. In Chapter XVIII. we have ah-eady dealt with elementary quadratic surds It was there shown by squaring

that

VaxVb = Vab.
might now deduce
IV,
.-.

We

it

from the index laws.

From Law
Letn = i,

(afe)"

= a"6".

{ab)^

= ah^\ Vab=VaxVb.

Similarly,
.-.

L L L (a*)"=o"6",

\^ab=VaxVb.

219.

Orders of Surds.

We

have already defined a quadratic


is
is

surd as one in which the square root surd is one in which the cube root

to be taken.
to be taken.
. . .

A
n"",

cubic

When
they

higher roots are to be taken as the fourth, fifth n}^ orders. are called surds of the fourth, fifth
.
. .

may

Changing the Order of a Surd. A surd of any order be expressed as an equivalent surd of any order which is a multiple of the given order.
220.
n

Thus,

\/x=x^=x^=x^ = x^'\
\/x=-\/x*='\/x^= Vx".
n

Similarly,

x^ = x'' = x^ = x^'',

08

SURDS AND SURD EQUATIONS


221.

309

may

To Compare Surds of Different Orders. Any two surds be reduced to surds of the same order and their values

compared.
Thus, to compare the values of
v/2 = 2^
if/3

V2

and

"^3,

= 2 = v/2 = v^8.
than V2.

= 3* = 3^ = V''3*=\5'9.

It is thus seen that -^3 is greater

222.

Changes

in the

Form

of Surds.

Any mixed

surd can

be expressed as an entire surd.


Thus,

2^5= ^2 X ^5= -^40.

TO

n).

^TO + n =
1

/m n

/ 'V^
.

(to

+ n)*
,

..

mn=
TO

+n

Vto

^ n.

Conversely,

^250 = -^125 x ^2 = 5^2.

^I'=^fil =

^^x ^Too=i^Ioo.
148

iJ^^^8T=^^^7x^3^ -3-^3,
BXBRCISEl
Express as mixed surds
1.
:

\
V32^,
V363a62.

^27", A/lOOa \/562, VSo^ft,

2.

-^16,

-^SaS^Six*; -^125^, ^-"8la3,-^^=^|a*:


-t^J^, ^^64, V8a;2+16xy+82/2.
:

3.

v'32, -^243;

Express as entire surds


4.

2V3, 10^2, 3Va, o\/5, a6V6, (o 6)\/a 6. 2^3, 3^7,

5.

-2^16^

a^S^fe,

-^^,

2v^5.

6.

(a+6)y^. (TO+)yzLi.

^y^.

310
1
.*

"^

Z^^ ALGEBRA
to surds of the
;

Reduce ^2, V^S

same

order.
V^S.

Also reduce

^2
8.

and V^S

\/2,

f3

and

Which
;

is

the greater:
;

3\/2 or 2\/3

Sv^G or 7\/3

V"^ or v'^lO
:

1-26 or \'/2

^^3 or y/51

Reduce
9.

to like surds

and simplify

\/8

+ \/l8+V98.

10. 12.

^500+^80-^20.
'^l6--^r28+v^^250.

11.
13.
15.

3V32+5\/50-iV'r28.
1^96 -2^ ^^12 + ^'324.

14.

-^32+ -1^162^+ \/l250.

\/75-3\/l2+5V'300+2v/48-7Vl47+3\/i.
x\/x^-y-ir\/x^+xhf-\/{x-\-yf-V{x^-y^){x-y).
Express as equivalent surds of a lower order
\/9, v/l25,
:

16.
17.

\/^,

\/l6^6,

A?^32.

18.

If

'^2=1-26 approximately,

find the values of

^l6. ^54, ^2000,


^9.

i^J, ^^002,

f^6^.

Show

that

2xV2xv^2xv^4=4.

223.

Rationalizing a Surd Denominator.

When

the numerical

value of a fraction with a surd

the value

is

denominator is required, more easily obtained when the denominator is

rational (art. 165).

When the denominator contains only two terms, it may be rationaUzed by multiplying by its conjugate (art. 164).
BXBRCISB
Multiply
1.
:

144
^'

t-

r^j
2.
5.

Oa .^

_^^
"ff

2^3,

3\/5.
i/5.

^201,

-N/3ai.

3. 6.

Vi, Vxy.
-^

4,

^4,

6Vl4, JV21.
'^-'""^^^

-2,

^-4.

7.

^^=^. ^^+6,
's/i+2,

^iH^.

8.

V-3, \/z-2, ViH-3.

SDMDS AND SURD EQUATIONS


9.

311

V2+V3-V5, V2+V3+V5.

^'
'

'

10. 11.
12.

^-1, ^a-2,

i^a+3.

Ve-VIT, Ve+VTT.
(Vl8+\/l2+\/8)2.
is

What
each case
13.

the simplest quantity by which the following


?

must be

multiplied, to produce rational products


?

What

is

the product in

3V2.

312
39,

ALGEBRA
The three dimensions
to the nearest inch.
of

room
is

are equal.

If

the longest

diagonal from the ceiling to the floor

18 feet, find the length of the

room

quadratic surd, and the equation

equation contains a single is written with the surd alone on one side, the surd may be removed by squaring both sides of the equation (art. 166). If the equation contains three terms, two or three of which
224.

Surd Equations.

When an

are surds, the operation of squaring

must be performed
'

twice.

Ex. 1.Solve
Squaring,

l-\-Vx=Vx-h25.
l-f-2\/x+a;=x+25,
,-.

2Vx=24:,

Vx=\2,
Verification:

x=\U. l+V'x=l + \/r44=13.


v/a;+25=Vl69=ia

Solve by squaring in the form


l

VxVx+2^l
solutions.

and

in

the form

= \/x+26 Vic,
Ex. 2.Solve
Squaring,

and compare the three


1
\.

\/x=a/x+25.

2^/x-\-x = x-\-2o, .2Vi = 24,


-Vi=12,
x=144.
,p .P

\.y^

Compare, line by line, this solution with Ex. 1. The answer is the same to both, although the equations are different. We have verified Ex. 1, and we know that a;= 144 is the correct result.
Let us

now

verify Ex.

2.

Verification:

1 -v/x=l- ^144= - 11.


\/x+25=V'l69=13.

seen that our attempt at verification shows that the correct root of the equation in Ex. 2.
It is

x=

144

is

not

If in verifying we could say tliat \/l44 is 12, the equation would But this is not allowable, as the symbol \^~ always be satisfied. repreeente the positive square root (art. 63).

SUBDS AND SURD EQUATIONS


This
(1)

318

may

be explained as follows
\

The equation
is

Vx=Vx-\-26

is

impossible of solution,
l

as Vz-{-25

a positive quantity, and therefore

Vx

must

be positive, that is, x must be less than 1. But it is evident that no value of x which is less than 1 can satisfy the equation.
(2) If

we square both
is

sides of

an equation, the resulting

equation

not necessarily equivalent to the given equation.


is

simple example will show that this

the case.

Let
Squaring,
.'.

K= 6.
x*

= 36.
6, and
is,

the equation x* = 36 has two roots -pB and therefore, not equivalent to the given equation.

Now

This

is

similar to the case in

which both

sides of

are multiplied Let


Multiply by
a;

by a

factor containing the

an equation unknown.

x = 2.

-3,

.-.

x{x-3) = 2(x~3),

.-.

cc*-5x+6 = 0.
3, is

The equation a;* 5x+6 = 0, which has the roots 2 and equivalent to the given equation.
225.

not

Extraneous Roots.
b}'

Roots which are introduced into

an equation
roots.

sTJimring or multiplying are called extraneous

Thus, 6 in the extraneous roots.


of

first

equation and 3 in the second equation are

Refer to Ex. 4, art. 145, where reference is made to the effect dividing both sides of an equation by a factor containing the

unknown.

have already seen the necessity of verifying the results In the case of surd equations there is an added reason for verifying, for although there may be no error in the work, the root which is found may not be a
in the solution of equations.

We

toot of the given eauatica.

^1-.~7r '-:3c '=^-3c314

ALGEBRA
BXERCISB
146
:

Solve and verify 1-15.


1.

Reject extraneous roota


2.

\/2x^-3=0.
3a;^=a;^+4.

\/3x-2=2\/x^.

3.

^4. ^5x-7=2.
fi.

5.

2x^

= Z.

2^3a;-25+3=7.

7.

2(x-7)^ = (x-14)i

^.
JO.
12.

Vi+Vx+5=^.
l

9.

Vx+i5-\-Vz=9.

+ Vx+2==Vx.

11.

VxTi+V'^+T5=ll.
Va;

\/4a:2+3a;-16=2a;+2.

13.

-7-== + 5 = Va;

14.

^
V4x

= 4.

15.

Vi-3^V5+1,
Vx+Z
a/5-2
(12+x)^-fx^=6.

16.*

Vx+4-V^4=4.
(x+8)^-(x4-3)^=2a;i

17.

18.

19.

Vx+16_Vx+29 Vx+U Vx+4


rz:

"

*U.

6\/x-ll_2\/x+l
-77^

3Vx
22.

Vx+6
36

z=:

21.

Vx2 a2+62=x 0+6.


\/53T5-fV'x=-,-=^-

Vx-{-Vx9--

Vx

23.

vx 15

24.

\/x+3+V'xHF8=2V'x.

25.

\/x+4a--\/x=2\/6+x.
,

26.

^x3-6x2+llx-5=x-2.

27.

5(70x+29)^=9(14x-15)*.

28.

\/^+6+Vx=6_ -^-^_-_^^_3.

29.

V5x+1

^!r^

^1 + ^^-^
2

30.

vg^+v^ ^c.
-v/o+x Va

<1

SURDS AND SURD EQUATIONS


Surd Equations Reducing
to Quadratics.

315

226.

Ex. 1.Solve
Transposing,
Squaring,

x-\-\/x+6=l.

Verification

v/x+5 = 7 x. /. a;+5 =49 14a;+x^ a;-15a;+44 = 0, (x-4)(x-ll) = 0, x = 4 or 11. When x = 4, x-f Vx+5=4+ \/9 = 7, When x=ll, x4- \/x+5= 11 + VT6=15,
.*.

.'.

liK>

.-.

.".

the correct root


11
is

is 4.

evidently a root of x

y/x-\-o =7.

Ex. 2.Solve
Transposing,
Squaring,

V8x+I-Vx+I=V3i.
.-.

\''8x+

= -\/3x+ Vx-\- 1.
'.

/.

8x+l = 3x+2V'3x* + 3x+x+l,


4x = 2\/3x + 3x,

2x= V'3x + 3x",


4x = 3x* + 3x,

x-3x=0,
x = 0or3.

^ y^^^ ^_^i^ ^

-zl

f^

^.^(^^i^ot'^

Here we

find on verifying that both roots satisfy the given equation.

Ex. 3.Solve

2V2x+l=S-3Vx^.
is

Solve as in the preceding and the roots are 4 or 364, neither of which satisfies the equation.

Of what equation

4 a root.'

Of what equation

is

364 a

root.^

Ex. 4.Solve
If the

x^-3x-6Vx^-3x3=-2.
to one side,

surd

is

removed

we get

x-3x-L2=-6v'x-3x 3.
If

we now square both

sides to

remove the surd, we

will

obtain an

equation of the fourth degree which

we cannot

easily solve.

i ^0

316

ALGEBRA
from x
to

\ X*
,

We may obtain the solution by changing the unknown 3x 3. similar to the method employed in art. 192. Let Vx* 3x3 = y, X' 3x-3 = y*, r*-3x = y* + 3.
y*
:.

Substituting in the original equation,

+ 3~6y=-2,
y = 5 or
1.

y*-Gy + 5 = 0,
or

.-.

.-.

.".

x*-3x-3 = 25, x-3x-28 = 0, x=7 or 4.


have four roots:
7,

x*-3x-3=l, x*-3x-4 = 0,

x=4
4,
;

or

1.

We,

therefore,

4,

1.

each of these and show that they all satisfy the given equation. Here both values of y were positive if either of them had been negative it could at once be discarded as impossible.
V^erify

Ex. 5.Solve

x-{-yVx-\-y=20,

(1) (2)

xy2Vxy=l20.
From From
(1), (2),

Vx+y = 5
V'xy=

or 4.

12 or

- 10.

Here the negative values of the surds are discarded,


.-.

Vx+2/ = 5, Vxy=^l2.
x + y = 2b,
x!/=144.

Solving these,

x = 9 or

16,

!/=16 or

9.

EXERCISE
Solve and verify 1-17.
1.

146
:

Reject extraneous roots


2.
4.

x+Vx=20.

x-\/x=20.

3.
5.

V'3x^+V^^=3.
3x4-a/5x2+11+5=0.
2 -7=
/= 5 2Vx.

\/3x-5-V'x-2=3.
3x + 5=-v/5x2+ll.

6.

7.

8.

3x-2A/7a;+4=15,

SURDS AND SURD EQUATIONS


Vz+16

317

V4X
10.

V4-X
11
13. 14.

Vx-fl6
12.

a;3_u 2x2

10a;+5=i-l.

Vx+a+Vx+b=Vab.

V2^+5-\/x^=2.

4(x+a;+3)^=3(2a;2+5a;-2)^.

3(a;+\/2^^)=4{z-V2^^).

15.

+ 4 - \/2a;2 + V3x2 + 4 4- V2x2 + ^ 7'


VSx^
i 1 1

16.

2
18.

fx-1

= 7.

17.

2Vx+3Vy=12,
3V'x+2V^=13.

xy V a-?/=30,
x+y=13.

19.

x+Vxy+y=28, zVxy+y=12.

20.

x+2/+V^+^=30,
a; 2/+Va; /=12.

21.

x^+xy+y^=^\,

22.

x+Vxy-^y=\^.
23.
24.

x^-Zx^&-^/x^-Zx+%=2.
vx^

x_6=36.
X+2/-10.

25.

Vi"^Vi~^'
227.

Square Root

of a

Binomial Surd.

(V3 + V2)2=3+2+2V6 =5+2\/6.

(V5-\/3)2=5+3-2\/l5=8-2\/l5. (3-^2)2=9+2-6^^2 =ll-6\/2.

(Va+V6)2=a+6+2Va6.
(\/a-\/6)2=a+6-2Va5.
The square of Va+V6 is made up of a rational quantity a+6, which is the sum of the quantities under the root signs, and a surd quantity 2\/ab, the ab being the product of the
quantities under the root signs

318

ALOE BRA

The form of the square of Va\-\''h will show us how we can sometimes find, by inspection, the square root of a binomial surd.
>/ a-\-h-\-2Vab=y/a+Vh,

\^a-\-b2Vab = Va Vb.
Ex.
4 and
1.

Find

the square root of 7


of 12,

+ 2\/l2.
is 7.

Here we want two factors


3.

whose sum
4

They

are evidently

7
.-.

+ 2V'l2

_=

+3+

__ 2V'4.

3.

'\/7

+ 2V'l2=\/4 + \/3=--2+\/3.
- a/3.

Similarly,

V'l-2Vl2 = 2

Verify by squaring

2+ VS

and

2 VS.

Ex.

2,

Find

the square root of


form a + b 2VcA,

14 6V5.
first

To put

this into the

change

6^5 into

2\/46,

14-6V5=14-2\/45 = 9 + 5-2V45,
.-.

Vl4-eV5=V9-V'5 = 3-\/5.
BXBRCISB
Oral)

147
?-

(1-7.

Find the square root and verify


1.

_^

r-^

SURUS AND SURD EQUATIONS


Ex.
1.

319

Find the square root of 56 24a/5.


56-24V5 = 56-2\/720.

Here we require two factors of 720 whose sum is 56. When the numbers are large, as here, it may be difficult to obtain the factors by inspection. When this is the case we may represent the factors by a and b and find the values of a and 6 from the equations
o6 = 720,

a + 6 = 56.
Solve these equations by the method of obtain
art.

194 or of

art.

198 and

a=36
6

or 20, or 36.

= 20

The required
.-.

factors of 720 then are 36

and

20.

56-24-^/5=56-2^/720 = 36 + 20-2^36.

20.

.-.

\/56-24\/5=\/36-V'20=6-2\/5.

Verify by squaring.

Ex.

2.

Find

the square root of


9
/g

+ VS.

4"^
the square root

^ 9 + 4\/5 ^ 9 + 2\/20 ~ 4 4

is

or -^

(-

1.

Ex.

3.

Find

the square root of 2V'lO+6V'2. and we get

First take out the surd factor y/2,

2VT0 + 6\/2=\/2(6 + 2V5),


the square root

=\/2(l +

\/5).

EXERCISE
Find the square root and verify
1.
4. 7.
:

148

94-42\/5.
107-24\/r5.
67

2.

38+12\/l0.
94 + 6V245.

3. 6.
9.

47-12\/T5.
101-28\/T3.

5. 8.

7v^72.

28-5\/l2.

xy^2yVxf^.

320
10.* Find the valve of

ALGEBRA

l^Vl6 6a/7

to 3 decimal places.

11.

Find the value, to three decimals, of the square root

of

^I

-.

7+4\/3
12.

By

first

removing a simple surd

factor, find the square roots of

7V2+4\/6, 10+6V5, 7V3-12, 59\/2+60.


13.

Showthat V'l7 + 12\/2 + \/l7-12\/2=6,


(2)

(1)

by taking the

square roots,

by squaring.

14.

Simplify

3+'v/i2+\/49+8a/3.

15

^ By changmg
.

V/- into 4-2V3 3


-z

find the square root of 2

V 3,

also of S

+ V2
less

and

of

V+^Vl
1,

IG.

From

the result of Ex.

show that 94 42\/5

is

a positive

quantity
17.
If

than unity.

a;2(14-6\/5)
sides

= 21-8\/5,

find

x to three decimals.
are

18.

The

of a right-angled

triangle

Vs

and 3-f2V2.

Find the hypotenuse.

228. Imaginary Surds. When we solve the equation we obtain a;= 3, and we know that this is the correct

a;2=9,
result,

for the square of either


If

+3

or

is 9.

x^=5, we say that the value of a; is Vs, and we can approximate to the values of the roots as closely as we wish bj'^ finding the square root of 5 by the formal method. If we are asked to solve a;2= 9, we might say that the solution is impossible, as there is no number M'hose square is 9. This statement is correct, but we find it
solve

we

convenient to say
if

x2^-9,
~

then

= V^^.

SUBDS AND SUED EQUATIONS


9 is called an imaginary Such a quantity as and must be distinguished from such quantities as

321
quantity,
5,

^,

jV7,
is

etc.,

We may

which are called real quantities. define an imaginary quantity as one whose square

negative, or as the square root of a negative quantity.

have already seen how imaginary quantities sometimes appear in the solution of quadratic equations (art. 190). We will assume that the fundamental laws of algebra, which we have applied in using real numbers, apply also to imaginary numbers.
Thus,

We

V^^=V9xV^l = 3V^^.
\/

a*=Va*x

V^=aV^.

These examples show that an imaginary quantity cau always be expressed as the product of a real quantity and the
imaginary quantity

V
the

The quantity

1 is

sometimes

called the imaginary unit.

229.

Powers
is real,

of

Imaginary Unit.
is

Any even power

of

v/ 1

and any odd power

imaginary.
definition,

Thu3,
.-.

(\/^)= - 1, by

(V^)3=-v'^n[,
(V^^)=(-i)*= + i.

.-.

.-.

(\/^6_(V^n:)x

v^=+a/-i,

etc.

230.

Multiplication and Division of Imaginaries.

Ex.1.

V^^x V'^=V'2

V^X V3 V^,
.

= V2 V3x(V^^)*=-V6.
.

Note that the product

iiere is

VG,

not %/6.

322

ALOE BRA

Ex.2.

sV^xsV^^eV^xisV^i,

Ex.3.
Ex.
4.

^
V-2
(x {a

= 90(v'^)*=-90.
=.

^,?X yZ^ = y^l = V9^3. \/2 \/2xV-l

+ 2/v'^) = x* + 3/(V^* + 2a;y\/^, = x i/* + 2a:n/\/^. + bV^l)(a-bV^^) = a*-b*iV^\)* = a* + bK


3
.

Ex. Ex.

5.

6,

Rationalize the denominator of


1 -f-

Multiply both terms by


3{l

V 2 and we get
3(1

+ V^^)_ ^
V-2)

(l-\/-2)(l +

l-(-2)

+ V32) _

^32

BXBROISB
1.

149

(1-0,

Oral)

Express as a multiple of

V^

V4,

V \6, V 81, V a*
?

V^^Wd, V^^9x*,
2.

V^^-b)\

What

is

the value of (V^-l)^


:

(V^S (V^-l)*, V^l)'


V^^, V^OOO, \/^^49. a + Vb\ abVl.
9.

Find the sum of


3. 6.

V^, V^. 4+\/^, 2 V 16.


V^l,
is

4.
6.

What
7.

the product of
8.

V^, V^.

^-25, V^IOO.

V'^^^

V^P

Simplify
10.*
11.

3V^3+2V^^5-4:V^-[2+5V'^^48.
(3-|-6\/^)(3-5\/^)+(5-3\/^)(5 + 3\/^).
(4-3\/-i)2-|-(2+6\/^)2.

12.
13. 15. 16.

2-^(l-V^).

14.

(-l +

v/^)^(-l-V^).

{a+bV^)^+(a-b\-iy\
Show
that
i(
l

+ \/^)2=i(-l-V^).

SURDis
17.
is

AND SURD EQUATIONS


+ VS),
l

323
this quantity

By

finding the cube of ^(

show that

a cube root of unity,


18.

(art. 192,

Ex.

5.)
?

Are
If

2 V^^,

the roots of

x^-ix+7=0

19.

a=2+3V'^
Impossible

and

6=2 3V^,

show that a+b, ab and

o'+ft* are real quantities.

231.

Problems.

We
?

obtained
question,

the
"

imaginary
is

number 9 in answer number whose square is

to the

What
said,

the
is

"

As we have

this

arithmetically an impossible problem.

When we obtain an imaginary result in solving may conclude that the problem is impossible.
Ex. 1. The sum Find the number.
Let x=the number,

a problem, we

of a

number and
-

its

reciprocal

is

1|.

.*.

its reciprocal,

2a;-3a;+2 = 0,

3+\/9^=T6

3V^^

Here the roots are imaginary, and we conclude that there is no number which answers the condition of the problem. In fact, it may be shown that the sum of a positive number and its
reciprocal
is

never

less

than

2.

Change IJ into 2^ and solve the problem.

per yard.

Ex. 2. For $30 Find z.

can buy z yards of cloth at S(10 x)

The

total cost in dollars


.-.

= x(

x) = 30.
10+ V'^=20 x=--2

a;*-iOa;+30=0,

What

conclusion do you draw


7

Would

it

be impossible

if

for $30

we substitute $25

$20

y2

324

ALGEBRA
BXBROISB
150
:

Solve and determine


1.

if

these problems are possible


is

line

which

is

10 inches long

divided into two parts so that


is

the area of the rectangle contained by the parts

40 square inches.

Find the lengths of the parts.


2.

The length

of a rectangle

is

twice

its
1

width.

If

the length be
is

increased 10 feet and the width decreased

foot, the area


is

doubled
1

Find the dimensions.


3.
it

Solve also

when the width


If

increased

foot.

A man
take him

has 20 miles to walk.

he walks at x miles per hour


?

will
4.

%x

hours.

At what

rate does he walk


is

If it is possible

that

x(12 x) = 36+a, and a


?

not negative,

what must the value

of a be

BXERCISB
!.
2.

161

(Review of Chapter XXIV)


y/Q-y/2.

Multiply Multiply

+ \/3-V'2by

\/3+ \/2 by

Find the square roots of: 3. ll-\/72, 12-6\/3.


4.

14+ V 180, 25-4\/21, 22+\/420,

Find to two decimal places the values of


1 + V3 VB-Vs' ^5+2v6. 2V3+V8'
/

3 \/8+V20

V5+V2

Solve and verify


5.

x+\/x^=\\.

6.

V4x+7+\/4x+3 = 6.

7.

\/z+\/z^=

Vx
35

,-

8.

V6x+7-Vx+2=V2Vx+i.

9.

Vx+5+ Vx-IQ=V'l^+x

Vx+5
^^

10.

Vl6-a:

Vl9+

'

SURDS AND SURD EQUATIONS


11.

325

12.

V'2+V3+V5 by v'5+V3-V'2 and x+y + 2Vx+y by x + y 2Vx+y. When x = 2+V3, y=2V3, find the value or
Multiply

2x+y

x--2y

xy
13.
result

x+y
Check the

Expand and simplify {V2a+Vb + V2aVb)*. by substituting a= 13,. b= 100.


Solve

14.
15. 16.

p + xV2px+x* = q.

Fina the product of 2\/3 + 3v'2+


Find the continued product of

VSO and

^2+^3- VB.

x-l + \/2, x-\-V2, x+l + VS and z+l-VS.


,^ 17.
o-

vf Simplify

-7=77= - 77=r + V6+\/3 V3+1


,

3\/2

2V6

Ve

V3+V2
,

18.

When

x = a-\-Va^\, find the values of


1
,
,

X
19.

x'
:

x'

Express in the simplest form

A'27-\/8 + A/l7-f 12V2-V'28-6V3


and
^
20.

Vll + 6V'2+\/l9-4\/r2 + V5-\/2i.


Simplify
o-

.r

V 12+6\/3 and ,T
,

V3-(-l
21. 22. 23.
If

^ / TO n
.

/m+n

\ TO+n

/m 7'

x=-l-f-2V'^,

find the value of

x-12x.

Find the square roots of


Solve 2x*-f

T+ViS

and 2a \'4a' 4.

6x 6-\/x* + 3x 3 = 45.

24.

SimpUfy (Vs^-

^3+ V'2) + (\/54-\/3- a/2)


+
(

V'5- a/3+ V2)* + C\/5- \/3giving

^/2)*.

26.

Solve

3x*

9x+

11

=4Vx* 3x + 5,
.

the

roots

to

two

decimal
26.

pleices.
^^t Simplify

Va + b-{-Vab

Vo+6 Vo 6

Va + bVa~b
Va+b-^-Vab
, ,

326

ALGEBRA
Show Show
that
,

27.

V16 + 2V'63
28.

V16-2V63

^=

3.

unless a* b

is

cannot be expressed in the form that a perfect square.

va+Vb
\

Vx+Vy

29. K

=' ''" Simplify

^ (-^)+(^:^==)
'

<

30.

Simplify (3-2\/2)'^

+ (3 + 2\/2)'.

31.

Find the value

of

x^+x*4-x-i-i when

x= v'3-trV.

CHAPTER XXV
THEORY OF QUADRATICS
232.

Sum and

Product of

the Roots.

Solve these equations


(1)
(2) (3)

x2-lla;4-10=0.
2.r2- 3.r-

The

roots are 10,



11,
f,
2 6

1.

5=0.

f,

-L
=
10.

15a;H26a;+ 8=0.
the

,,

-|, -^.

In In
Tn AiX

(1) (2)

sum

,,

of the roots

,,

,,

,,

= =

product

,,

=-f.
j^.
8

CW \fJ
/

jg,

Examine the sum and the product in each case and state how they compare with the coefficients in the given equations.
Every quadratic equation may be reduced to the form
ax^-{-bx-\-c0.

Ihe roots are

b+Vb^-'iac
^

2a

J and

-6 V62-4ac
^
.

2a

For brevity represent these roots by


m-\-n

m and

n,

= -b+Vb^-4:ac-b-Vb^-4MC
2a

-26
2a

and

mn =

{-b-^Vb^-4ac){-b-Vb^-4:ac)
4a2

(-6)2 (\/62 4ac)2


4a2
atf

62-62+4ac
4a*

4ac

~4a=

328

ALGEBRA
results with the coefficients a,
:

Comparing these
equation,

b, c

in the

quadratic equation, in the standard form, is equal to the coefficient of x with its sign changed, divided by the coefficient of x^, and the product of the roots is equal to the absolute term, divided by the coefficient of x^.

we see that The sum of the roots of any

Sum

coefficient of

x
a;*

of the roots
coefficient of

Product of the roots


See
if

absolute term
coefficient of
a;*

efficients of the three

these two statements apply to the roots and coequations preceding.

The formulae for the sum and product of the roots furnish a convenient means of verifying the roots.
the roots of 3x*-\-x2 = to be 5, 1, but the sum of and the product is |, which agree with the sum and product given by the formulae. Therefore, these are the correct

Thus,

I find
is

and

roots.

Are the roots of 14a;- 19x-60 = 0, V,


233.

Reciprocal Roots.

If the

roots of ax^-\-bx-\-c=0 are


is

reciprocals (like
c

and

),

their product

unity,

and therefore

=1
is

or

c=a.
the
coefficient

So that any quadratic equation, in which


x^
Thus, the roots of 6x* 13x + 6 =
is f

of

equal to the absolute term, will have reciprocal roots.


are reciprocals, since their product

or

1.

Verify this by finding the roots.

234.

Roots equal
(like

in

Magnitude but opposite


their

in Sign.

If the

roots of ax'^-\-bx-{-c^=0 are equal in magnitude but opposite


in sign

and 3),

sum

will

be zero, therefore

_- =
a

or 6

= 0.

So that any quadratic equation in which the second term is missing will have roots equal in magnitude but opposite in sign.
and ox* c = 2a;* 9 = Verify by finding the roots.
Thus,

have such

roots.

THEORY OF QDADRATIC8
EXERCISE
State the
1.

32^

152 (Oral)
:

sum and product

of the roots of

x2 7a:+12=0.
a;2+6a;+l=0.
3a;2-12a;-7=0.

2.

x2_5a._ii^0.
2a;2_i0a;+6=0.

3.
5. 7.

4.
6. 8.

4a;2-17x+4=0.
ax^{b-\-c)x-\-a=0.

ax^bxc=0.
px^q=0.
3x^-4:z=6.

9.

10. 12.

ax^+a=0.
(ai-b)x^-x+a^-b^=0.
have reciprocal roots
?
V

11. 13.

Which

of

the

preceding

equations

Which have
14.
15.

roots equal in magnitude but opposite in sign


?

Are 4 and 5 the roots of x2-9a;+20=0 Are

S+V2, 3V2

the roots of

x^6x+7=0

In which
16. 18.

of the following are the correct roots given


5, 2.

x2-7a;J-10=0;
a;2-13a;4-36=0;

17.

x-2+3a;-28=0

7,
;

-4.

4, 9.

19.

x2-12x+27=0
2a;2-5a;+2=0
is
;

4, 8. 2, J.

20.

x2-4a;-5=0

5, 1.

21.

In solving a;^ 2a; 1599=0, one root must the other be ?


22.
23.

found to be

41.

What

How

roots of

would you show that x2-3-293x+2-439=0 ?

1-125

and

2-

168 are the correct

24.
of a
?

If the roots of

Qx^lOx-^a=0

are reciprocals,

what

is

the value

25.

If

the roots of

jnx^{m^9)x-'rm^=0
is

are equal in magnitude


?

but opposite in sign, what the product of the roots ?

the value of

What would

then be

235. To form a Quadratic with given Roots. First Method. In the equation x^-^-px-^-q^O, the sum of the roots is p, and the product is q. Since every quadratic equation may be reduced to the form x^-}-px-]-q=0, by dividing by the coefficient of x^, any quadratic equation may be written thus
;

X'^X (sum

of roots)

+ (product of roots)=0.

330
If

ALOEBRA
the roots are given, the equation can at once be writi^a

down.
Thus, the equation whose roots are 3 and 5
is

x*

a;(3 + 5) + 3
is

= 0,

orx-8x+15 =
The equation whose
roots are
2-}-

\/3 and

2 VS
is

x-x(2+V3-|-2-V3) + (2+\/3)(2-\/3) = 0,
The equation whose
roots are a

or

x*-4x+l = 0.

+b

and a b

x*

2ax-f-a* 6* = 0.
p and
q

is

Second Method. {xp){xq)=^Q.


The equation whose

The equation whose


roots

roots are

are

and

is

t-^

3)(x 5) = 0,

or

x*-8x+15 = 0.
The equation whose roots are and J is (3x-2)(4x+3) = 0. The equation whose roots are 2 \/3, 2 \/3 is
(x

)(x+|) = 0,

or

(x-2- V'3)(x-2+\/3) = 0,

or

(x-2)*-3 =

or

x*-4x 4-1 = 0.

Either method is simple enough to apply, but the first is probably easier when the given roots are not simple numbers. The second method may be applied to form an equation with any number of given roots.
Thus, the equation whose roots are
2, 3,

is

(x-2)(x-3)(x + 5) = 0, or x3_

19a;

+30=0.

EXERCISE
-7. -1.

153 (1-16, Oral)

State, without simplifying, the equations


1.
5.
3, 7.

whose roots are


7.

2.
6.

3, i,

3.
7.

-3,

4.

-3,
I, J. 0,

-7

ii
a, a.

-h-\.
3, 0.

8.

9.

10, 14.

a, b.

11.

12.
16.

m.

13.

3,4,5.

2,3,-1.

15.

a,b,c.

a, 6, 0.
:

Reduce
17.*
20.
23.

to the simplest form the equations


18.

whose roots are


19.

m+n, mn.
1|,

2a b, 2o+6.
-2, -4,
6.

3+V3, 3-A/a
i, \, \.

-2i.

21.

22.

Show

that 1-25 and 4-64 are the correct roots of

x00a;''-589*+580==0.

THEORY OF QUADRATICS
24.

331
is

Construct an equation in which the


is

sum

of the roots

and the

difference of their squares

14.

25.

Form

the equation

whose roots are a and

where a^-\-b^=2bi

a-\-b=l.
26.

Form

the equations whose roots are


a-\-b

ab
27.
(1) (3)

-;; -^, a-^b

ab

i(4\/7).
-

,_

Find the sum and the product of the roots of

(x-2)2=5x-3.
x{x-p)=p{x-q).
Solve
If

(2) (4)

{x-a){x-b)=ab.
(2-+a)2+(a;+6)2=(a;4-c)2.
is 5.

28. 29.

x* 21x2 20x=0,

being given that one root


is

one root of a;^


of a.

12x+a=0

double the other, find the roots

and the value


30.
If

one root of x^-\-px-'ri&Q

is

three times the other,

what are

the values oi

236.

Functions of the Roots.


b
,

When m and n

are the roots

of ax^-\-bx-\-c=0,

m-\-n=
Here
it

mn = -

a
the product of the
while the separate

will

be seen, that the

sum and

roots do not contain surd roots do.

expressions,

If we wish to find the sum of the squares of the roots, we can do so in the following way
:

m^+n^

= (m-\-n)^2mn.

62

2c

62_2ac

It

and adding the results. the same result.

can also be found by taking the square of each root Find it that way and see if you get

332

ALGEBRA
1.

Ex.

When
1

ni

and n are the roots


-

of a,r2-|-6.T-f c=0.

find the values of


1

m
_

+ -.
n
b
_

n
c
'

-|

m^-i-n^,

m~n.

m-\-n

b
c

m
f>^
,

n
''^

mn

6_
n

2c

m
,

_ w* + n* _
mn

(m + n)* 2mn

a*
c

b^

2ac
ac

mn
,

3 3 m^ + n^

/ N3 / (m4-tiy o 3m,n(m + n)
,

= fe
a^

-= 3a6c-6
36c
^

a*

a'

or
/

m,^-^n^

= =

(7n4-n)(m* mn + n*) = {??!+ 'i)l(w

+ n) *3mn} =
>

etc.

(m n)'

.\.

(m + n)* 4wn
/

V.

= .
a*

6*

4c

6* 4ac
a*
s

mn = H ~

Vfc*

4ac
.

The same two values of the last expression might have been found by simple subtraction, the sign depending on the order in which the roots were taken.

Ex.

2.

If

equation whose roots are


Here m-\-n=

and n are the roots m^ and n^.


and mn=q.

of x^-\-px-\-q=0, find the

The sum

of the roots of the required equation is

m* + n* = (m + n)* 2mn=p* 2g.


The product
..

of the roots

the required equation


3.

is

= m*n* = g*. x* x{p*2q)-\-q* = 0.


13x 8=0.
3,

Ex.

Find the equation whose roots are each greater by


J.
'

2 than the roots of 6a;2

Solve the given equation and the roots are


.'.

.'.

the roots of the required equation are V, the required equation is


x*

{i^+i)x+'-,*. = 0,
1

or

6x-37a;+42 = 0.

We

might have solved the problem without finding the actual roots

of th) given equation.

THEORY OF QUADRATICS
Let

333

p and

g be the roots of 6a;* iSa; 8

Then
.'.

P + <1=V- and pq=

= 0. tis

the

sum

of the roots of the required equation

p+2 + g+2=p + g+4 =


and the product

i5-

+ * = V-.

= (p + 2){q+2)=pq+2(p + q) + 4,= -i + -^^+i = 7.


.'.

the required equation

is

x*-^x+7 = 0,
When would the second method
Ex.
4.

or 6a;*-37x + 42

= 0.
tirst ?

be simpler than the

Find the equation whose roots


P

are the reciprocals

of the roots of mx^-^7iz-\-k=^0.

Let p and q be the roots of the given equation,


then

+q=

and pq

k = m
q

The

roots of the required equation are -

and

p
Find the sum and product of - and p complete the solution. Compare the new equation with the not write down, mentally, the equation
of the roots of q

- in terms of m, n and k and

given one and see if you could whose roots are the reciprocals

any given equation.

The following method will be found useful in solving 237. such problems as the three preceding.
Ex. 1. Find the equation whose roots are each greater by 5 than the roots of 4a;2 5x+7 0.
Let y be the unknown
in the required equation.

Then
Substitute

y = x + o or x y x y5
in

5.

the

given

equation,

and

the

required

equation
or

is

Of

5)* 5(i/ 5)-f 7 = 0, 42/-40y+100-5y + 25 + 7 = 0, 4j/-45y+ 132 = 04(2/

S34

A
2.

WEBRA

Ex.

Find

the equation whose roots are the squares of

the roots of ax^-\-bx-{-c=0.


Let y be the unknown
in the required equation.

Then
.'.

y x* or
is

x=+Vy,

the required equation

a{Vy)* + b(Vy) + c = 0,

or or

ay-\-c= '+bVy,
a*y* + 2acy + c*

= b*y,
this

or

a*y*^y(2ac b*) + c* = 0.
Solve Ex.'s
2, 3,

4 proceding,

by

method.

BXEROISE
1.* If

154

m
,

-|

n
1

m
,

m
I

and n are the n , ,


,

roots of a;^ 5x+3=0, find the values of

m^-j-mn-j-ri'.

2.

Find the sum of the squares of the roots of

x2-7x + l=0 and


3.
li

of Sa;^ 4x+5=0.

p and

q are the roots of

3x^+2a: 6=0,

find the values of

-, V^'P'-P?+9'+ p' p g2

4.

Find the sum of the cubes of the roots of

2x2 3a;+4=0 ^nd


6.

of

z^x+a=0.
are

Find

the

equation

whose

roots

double

the

roots

of

a;* 9x4-20=0, (1) by solving, (2) without solving.


6.

Find the equations whose roots are each

less

by 3 than the

roots of (1)
7.

x2-llx4-28=0,

(2)

x2-x-l=0.

Find the equations whose roots are the reciprocals of the roots

of (1)

2x2+x-6=0,
If

(2)

x^-pz+q=0.
find the equationfl

8.

and n are the roots of 3z'' 2x+5=0,


:

whose roots are

(1)

i and

(2)

- and n

(3)

m' ad

n.

THEORY OF QUADRATICS
9.

SP

Find the sura of the squares and the sum

of the cubes of the

roots of x'^-\-ax-{-h=0.
10.

Find the equation whose roots are the squares of the roots

of x^-\-pxq=:0.

11.

Find the equation whose roots are each greater by h than the

roots of ax^-\-hx-\-c=0.

12

Find the equation whose roots are the reciprocals of the roots

of x^-\-x=\.

13.

If

and n are the roots

of

x^px-\-q=Q, show that m-\-n

and

mn

are the roots of x^x{p-\-q)-\-pq=0.

14.

Form

the equation whose roots are

and

n,

where

m2+n2=20, w+7? = 6.
If m and n are the roots of x'^-\-px-\-q=0, show that m-{-2n 16. and 2m-\-n are the roots of x^+3px-\-2p^+q=0.

16.

If

p and

q are the roots of ax^-i-bx-'rc=0, find the value of


a, b

p*-\-p^q^-\-q* in

terms of

and

c.

238.

Character of the Roots of a Quadratic Equation.


:

Solve the equations


(1) (2)

x2 6.r+ 9=0,
6a:2+ a:- 15=0,

the roots are


3,

3,

|,

-.
10

(3)

5x^-{-lx- 2=0,

-7V89

(4)

2a;2-3x+ 2=0,
the roots are equal.

3V^
We
might say that there is two roots,

In

(1),

only one root, but

we

prefer to say that there are


to be equal.

which
In

in this case

happen

(3), the roots are irrational, but we can approximate to their values by taking the square root of 89.

In (4), the roots are also irrational, but we can not even approximate to their values. Here the roots are imaginary, while in each of the others the roots are real.

336

ALGEBRA

These statements might be written thus


In
(1), the roots (2), the roots

are equal, real

and
and

rational. rational.

In

are unequal, real

and

In
In
If

(3), the roots are irrational


(4), the roots

real.

are irrational

and imaginary.

we examine

the roots of the general quadratic equation

we
is

will see

the reason why, under particular conditions, there

this difference in the character of the roots.

The

roots of ax^-\-hx-\-c=0 are

6+v'62 4ac
2a

and

6 a/62 4ac
2a
:

From
(1) If

these roots

we may conclude
a, b, c
is

the particular values of

are such that

b^^ac=0,

then the roots are equal, for each


In equation
(1),

evidently equal to

a=\,

fe=

6, c = 9.
6-4ac = 36-36 = 0.
its

.-.

(2)

If

b^4:ac

is

a perfect square, then

square root can

be found exactly and the roots are rational.


In equation
(2),

.-.

= 6, 6= 1, c= 15. 6*-4ac=l + 360 = 361=l9*.


not a perfect square, but
is

(3)

If

b^^ac

is

positive, the

roots are real but irrational.

Find the value of


(4) If

6*

4ac

in equation (3).

b^4:ac

is

negative, the roots are imaginary.

Find the value of

6*

4ac

in equation (4).

Hence, the roots of ax^-j-bx+c^O are real and equal if b^ 4ac=0, real and unequal if b^ 4ac is positive, imaginary if b^ 4ac is negative, real and rational if b* 4ac is a perfect
square.

THEORY QF QUADRATICS

^Sl

The Discriminant. We see then, that we can deter239. mine the character of the roots of a quadratic equation without actually finding the roots. All we require to do is to find the value of b^4ac. This important quantity is called the discriminant of the
equation ax^-{-bx-\-c=0.

Ex.

1.

Determine the character


3x^-\-5xU^0. a;2- a;+ 3=0.
of the discriminant (6*
is

>i

the roots of

(1)

(2)
(4)

i2x^-2ox-\- 12=^0.

(3)

2a;2-16x+32=0.

The value

4ac)

in (1)

157,
49,

.".

the roots are real and irrational, the roots are real and rational,
the roots are imaginary, the roots are real and equal.

in (2) in (3)

is is

.'.

11,
0,

.'.

in (4) is

.'.

Ex.
The
that
is,

2.

For
?

what values
if

of

k will 4:X^kx-]-4=0 have

efqual roots

roots will be equal


if

6* 4ac = 0,
or
if

^-*-64 =

i=8.
if

Substitute these values for k and see

the roots are equal.

Ex.
Here

3.

Show that the roots are rational of


3mx^x(2m-\-3n)-\-2n=^0:
6*-4ac = (2m+3n)* 24/.n,

= 4m* 127/in+9n2 = (2/n 3n)*.


Since 6* 4ac is a square, the roots are rational. Verify by finding the roots.

EXERCISE
1.

155

(1-5,

Oral)
?

What What

is

the discriminant of x^-{-4iX-\-4:=0


?

What
? ?

is

the

character of the roots


2. 3.
4.
is is

the nature of the roots of x^-ir3x-{-20


peculiar about the roots
if

What
What
If

6^ 4ac=0

kind of roots have

x^5x+7=0, x^ 6x-l-9=:0,
a;2 4a; 6=0
is
?

x'x6^Q,
5.

the discriminant

is -25,

what

the character of the roota^


7.

J38

ALGEBRA
:

Determine the character of the roots of


6.*
8.

23-H 5x4-3=0.

7.

3r2 72- 5=0.

4x2+7a;+15=0.
abx'^+x{a'^+h^)-\-ah=0.

9.

9x2-12r+4=0.
x'^-mx\=^.
all

10. 12.

11.

Show
If If

that x^-\-ax-\-h=0 has real roots for

negative values

Jh.
13.
14.

9a;24-12x+it=0 has equal

roots, find k.
a.
all

ax-\Qx+a=0
k.

has equal roots, find

16.

Show
If

that the roots of x^x{\-\-k)-\-k=0 are rational for

values of
16.
17.
it

x*+2x( 1+0)4-0^=0 has equal


solving the equation x^
less

roots, find a.

By

4x4-5=t,
X

show that

if a;

is

real,

cannot be
18.

than
the

1.

Show
is

that

roots

of

| 1

are

real

if

x4-a

x4-o

a^ab + b^
19.

positive.

Eliminate y from the equations y=mx-\-c end?/*=4ax, and find the value of c if the resulting equation in x has equal roots.
20.
of
If

2mx24-(57n4-2)x4-(4m4-l)=0 has equal


verify.

roots, find the values

and

240.

Factors of a Quadratic Expression.

When

and n are the roots

of ax^-^bx-\-c=0,

m 4- n
,'.

b
,

mn =
a;

c --

a
4- j

ax^-]-hx-\-c=a(x^ 4- -

=a{x^ {m-\-n)x-{-inn} =a{xm){xn).


So that,
if

and n are the roots

of ax^-\-bx-\-c^O, the

factors of the quadratic expression


ax'^-\-bx-\-c

are a{xm){xn).

THEORY OF QUADRATICS

339
like

We

can,

therefore,

find

the factors of a trinomial

ax^-\-hx-]-c

by solving the corresponding equation.

Solving by formula,

Ex. 1. Factor 6a;2+a; 40. we find the roots of 6x* + a;-40=0 are ^,
.-.

|.

6a;+x-40=6(a;-5)(a;+f)

= (2a;-5)(3a;+8).
Ex. 2. Factor 12x2-47a:+40.
The
roots of the corresponding equation are
.-.

\,

|,

12x-47x + 40=12(x-|)(x-|)

= (4x-5)(3x-8).
241.

Character

of

the

Factors

of

Trinomial.
it

Since

ax^-{-bx-\-c=0 has equal roots

when 6^ 4ac=0,
is

follows that

ax^+bx+c
..

has equal factors, or

a square, when

b^ 4ac=0.
factor 3

Thus, in 3x*-30a: + 75,


3iC*

30a;+7o

ia

a perfect square

6*-4ac = 900-900 = 0. when the numerical

is

removed.

// 6'^ 4ac is a perfect square, the expression ax^-\-hx-\-c has two rational factors, for under this condition the corresponding

equation has rational roots.


Thus, in 20x x-12,

6-4ac = 961 = 31.


Find the
factors.

2Qx* x 12 has rational factors.


242.

Surd

Factors

of

a Trinomial.

trinomial can be factored,

we

usually

\Mien we say that a mean that it can be

expressed as the product of rational factors.

As we have
is

seen, this can always be done when h'^AiOC a perfect square. When there are no rational roots we may use the preceding

method
Ex.

to find surd factors.


of

Find two surd factors


a;

x^ 6a;-[-4.

nx-6a;+4 = 0,
.-.

^^^^ = 3 Vs.
e2

a;-6x4-4 = (x-3-V5)(a;-3+'\/6).

Verify by multiplication.

340

ALGEBRA
EXERCISE
156

Factor, by trial

if
:

you can, otherwise by solving the corresponding

equations and verify


1,

3x2-17x+10.
a;2_2.f-1783.

2. 4.
6.

20a;2+3x-108.

3.
5. 7.

lS00a--5a-l.
221a:2-458ax+221a2.

299^2+ lOx-1.

Show

that 12x^ 15x+4 has no rational factors,

8.*
of
it.

x-+4x 3
If If

has no rational factors.

Find two surd factors

9.

X- 8x4-Z;

is

a perfect square, find


is

k.
is

10.
find k.

ax^kx-'rQa

a perfect square, when the factor a

removed,

11. 12.

Express
Factor

x^ 6x 11

as the product of

two surd
this

factors.

144a;* 337a;2?/^-|- 1442/*.

When

expression
y.

is

equal to zero, find four values of the ratio of x to


13.
If

x2

is

a factor of 120x3 167x-ax+56, find the value of

a and find the other two factors.


14.

By

finding the square root of


a, b

ax^+bx+c,
is

find the relation

which

must connect
243.

and

when

this expression

a perfect square.

Equation cannot have more than two have seen that the equation ax^-\-bx-{-c0 has two roots, and since this equation represents every quadratic, it follows that every quadratic equation has two roots. It cannot have more than two roots.

Quadratic

Roots.

We

Let

m and n be the roots of ax^-{-bx-\-c=^0.


ax^-{-bx-\-c=a{xm){xn)
(art. 240).

Then

a{xm){x7i)0.
Since this product is zero, one factor must be zero. But a is not zero, for the equation woukl not then be a quadratic.
Therefore, either

xm=0

or

X'

nO.

THEORY OF QUADRATICS
But no values
of
.r

341

other than

and n

will

make

either of

these quantities equal to zero.

m
roots,

and n are the only

roots.

Since the quadratic equation ax^-{-bx-\-c=^0 has only two then the quadratic expression ax^+bx+c can be resolved

into linear factors in only one

way.

EXERCISE
1. 2.

157 (Review of Chapter

XXV)
?

What

is

the

sum and

the product of the roots of ax^ + bx-\-c =


reciprocals

When
3.
(3)

are they equal in

Under what condition are the roots of ax*-i-fea; + c = magnitude but opposite in sign ?

When
If

are the roots


(4) rational
?

of

ax*-\-bx

+ c = 0,
:

(1)

equal,

(2)

real,

imaginary,
4.*

p + 9 = 4 and pq=5,

find the values of

6.

Find the sum and the product

of the roots of

(3a;-2)(a;-3) = (x-l)(a;-5).

of

Find the equation whose roots are each one-half 6. 4a;-20x+21 = 0.


7. 8.

of the roots

Find the sum of the squares of the roots of 3a;* lla;+

= 0.

Find the equation whose roots are twice as great as the roots

of

24x*-38a;+15 = 0.
9.

For what value of k

will

x* I0x=k have equal

roots

10.

Find the equation whose roots are

and n when m^ + n^^l^

and mn=35.
11.
12.

Factor 5256a;*+a;-l and 221a;*- 8a; 165.

Form

the equation whose roots are

and n where m' + ^'=28

and

m+w = 4.
Find the sum of the roots of {xa)* + {xb)* = {xc)*.

13.

Construct the equation whose roots are the reciprocals of the 14. roots of 17x* 53x 97 0.

16,

Express x* + 6a;-f-7 as the product of two linear factors.

342

ALGEBRA

16. Construct the equation whose roots are each greater by 7 than the roots of 2x*+ llx-21 = 0. 17.
of ax*

Find the equation whose roots are each three times the roots

+ 6x + c = 0.
If

18.

and n are the roots

of ax*-^bx

+ c = 0,

find the equation

19.
roots.

Show that {a + b + c)x* 2x{a + b) + a + b c = What are they ?

has

rational

If one root of 20. 2p* = 9q.

x*px+q =

is

double of the other, show that

If and n are the roots of x*+/)x g = 0, 21. are the roots of x* + x(m + /i mn) mn(m + n) = 0.

show that p and

22.
all

Show

that the equation (mx-\

=4ox

has equal roots for

values of m.
23.

Find the values of k


x*

for

which the equation

x{2 + k) + k + 37

has equal roots.


Since x* 8x-20 = (x- 10)(x+2), for what values of x is the 24. expression x* 8x 20 equal to zero ? For what values is it negative ? For what values is it positive ?

Show that it is impossible to divide a line 6 inches in length 25. mto two parts such that the area of the rectangle contained by them

may
26.

be 10 square inches.

Verify

For what values of k your result.


Find the sum

is

4x* x(i+8) + A; + 5 a perfect square?

27. 28.

of the cubes of the roots of x*

+ mx + n = 0.

Find the sum of the squares of the roots of

x-x(l4-o) + J(l+a + a*) = 0.


29.

Find the

sum and
a

the product of the roots of


^

X
30.
find a
If

_
b

'^

c
equals their product,

the

sum

of the roots of a*'

6x+ 12a =
(x

and

verify.

31.

It is evident that a is

one root of

c)

(x 6) = (a c) (o 6).

Find the other root.

THEORY OF QUADRATICS

343

If a;* 5x 3a and x* lla;+3a have a common factor, it must 32. be a factor of their difference. Make use of this; to find the value of a for which x'' 5.t 3a = and x* lla;+3a = will have a common Verify by finding the roots. root.

The absolute term in an equation of the form x*-\-px + q = is 33. misprinted 18 instead of 8. A student in consequence finds the roots to be 3 and 6. What were the roots meant to be ?
34.
If
1

and n are the roots of

ax^-\-bx-\-c

0,

show that

m+n

and

are the roots of aca;* + 6x(a4-c) + 6* = 0.

35. Two boys attempt to solve a quadratic equation. After reducing it to the form x^-\-px-\-q = Q, one of them has a mistake only in the absolute term and finds the roots to be 1 and 7. The other has a mistake only in the coefficient of x, and finds the roots to be 1 and 12. What were the corrnct roots ?

36.

Express x*-\-2hx-\-c ae the product of two linear factors

in x.

CHAPTER XXVI
supplementary theorems and exercises
Additional Examples in Factoring
244.

Product of two Trinomials.

If

we multiply

the product

a-2b-3c by 2a &4-c may be written 2a^-5ab+2b^-5ac+bc-ZcK


three terms of the product, which do not contain

The

first
c,

are evidently the product of a 2b and 2a b. The last term, Sc^, is the product of 3c and c. If we wish to factor the product of two trinomials, we may do so by the method of cross multiplication, which we used to

the letter

factor a trinomial.

Ex. 1.Factor 2a^ - 5ab-{-2b^ Sac -\-bc-3c^.


First factor 2a*5ab-{-2b*, and then choose such factors of 3c* as will give the remaining terms in the product when the complete
miiltiplication is

performed

2a*-5o64-26-5ac-{-6c- 3c 26 3c a

b 2a + r,. the terms of the factors are written under the terms from which they are obtained, it is not difficult to obtain by trial the factora of an expression of this type.
If

Ex. 2.Factor 4a2+362_l2c2-8a6-8ac.


Arrange the expression thus
2a 2c
:

4o-8a6 + 36*-8ac-12c*

-I-

2c

-36

6c

Show by

multiplication that these factors are correct-

SUPPLEMENTARY THEOREMS
BXBRCISE! 158
Write, mentally, the products of
1.
r

346

a-26-i-c

2.

Zxy-\-\

8.

a~ b-c
4.
3x-|-

3x+y2
5.

3^46+ c 2a- b-2c


2x 5y+
z

o 6+4
2a

6.

2x3y+3
7.

3x~-2ySz

2a~3b-5c 2af36

8.

3m 2n+l 2m 3+4

9.

a^2a+3
a^-\-3a2

Factor and verify


10.

a^~^ab-r4:b^-a+2b-12,
2x'^-i-xy6y^+2xz+llyz~4z*.

11.
12.

2a2-+-662~3c2+76c 5ca 7ao.


2a;2-7a;t/-222/2-5a;+35i/-3.

13.
14.
15.

6a^+at>-l2b^-2a+3lb-20.
a;2 a;z 6z2 2xy+&y3.

16.* Divide the product of 6a^5ab-i-b^i-na4:b+3 and


!i-i-6-2

by 3a2-f 2a6 62_5a-f 36-2.

17.

Reduce to lowest terms


4p'^+2lq^- 18r^+33qr+Qrp~3lpq

^p^7pq-r3q^~2pr+3qr-Qr^
18.
If

3x-\-2y5z

is

a factor

of

3x2-faa;?/ 6i/2-t-6a;z4-cy3 lOz*,


tt^hat

are the values of a, b

and

19.

z'^-{-x{yl){2y^10y-\-12).

Write the expression x^4-xy2y^x-{-l0y~l2 in the form Solve the corresponding equation for

X and thus find the factors of the given expression.


20.
fche

Solve

x25ax+6a*+7a;-17a+ 12=0. (W bv

factoring. {2\

by

general formula.

346
21.

ALOmBRA
Express, in the factor form, the L.C.M. of

6a2 5a64-6--(-ac c^
and
245.

62-ffl6_262 oc+46c 2c2.

Sum and
aS + 63

Difference of Cubes.

We

have seen that

= (a + 6)(a-a6 + 6*),
(a

a^-b'' = (a-h){a'^-\-ab-\-b*).

Similarly,

+ 6) + c = (a + 6 + c){(a + 6)*-c(a + 6) + c*;

and

(a-6)3-cS = (o-fe-c)|(a-fe) + c(a-6) + c*J.

Ex. 1. Factor a^+h^^c^Zabc. Add to a' + 6' sufficient to make the sum
add 3a6 + 3a6^

the cube of a + 6, that

is,

Then

a^ + b^ + c^ Zahc, = a3 + 63 + 3a6_^3oj4.cS_3aJft_3a5i_3a6c, = (a + fc) + c-3a6(a + 6 + c), = (o + 6 + c)!(a + 6)*-c(a + 6) + c*-3afe}, = (o + 6 + c)(a + 6* + c*-a6-6c-ca).

The factors of this expression are important, and the pupi' should endeavour to retain them in memor3^
The expression
is

the

sum

of the cubes of three

quantities

diminished by three times their product.

One
the

factor is the

sum

of the three quantities,

and

the other is

sum

of their squares diminished by the

sum

of their products

taken two at a time.

We

form as

should recognize expressions which are of the same this type expression.

Thus, a' + 6' c' + 3a6c


a''

may

be written in the form


of a, b

+ 6' + (-c)S-3a6(-c),
and

and it is now seen to be the sum of the cubes by three times their product.

c, diminished

The factors of a^ + ^^ c' + 3afec may at once be written down from the factors of the type form by merely substituting c for c.
.-.

a* + b^-c^ + 3abc = {a + b-c){a* + b' + c*-cih + bc + ca). o_fe3-c8-3a6c = o + (-6)' + (-c)-3a(-6)(-c), = (a-b-c){a* + b* + c* + ah-bc + ca).

8x'

+ 27!/-2 + 18a^2 = (2a;) + (3j/) + (-2)3-3(2a;)(32/)(-z), = 'Zx+3yz)('ix* + 9y* + z* 6xy + dyz4-2zx.

SUPPLEMENTARY THEOREMS
Ex. 2.~Factor
a^-\-h^^l '6ah.

34T

= (a + 6+l)(a*+6+i--a&-a 6V
Ex.
3.

Find one factor of


same form
as a*

(a:+t/)H(y+2)3+(z+a:)3-3(a;+2/)(y+2)(3+a:).
This
is

of the

+ 6' + c^ 3o6c, where a = x-\-y,


is

b=y-\-z,

cz+x. One factor is x-\-y-\-y-^z-\-z-\-x The other factor is lengthy, but

or 2{x-\-y-\-z). easily written down.

Ex. 4. If
This
is

a+6+c=0, show

that

a^+h^+c^=Zahc.
if

equivalent to showing that a' + 6^+c' 3a5c = 0.

Now
to zero,

this quantity will be equal to zero,


is

one of

its factors is zero.


it is

But a-\-b+c

already seen to be a factor, and since


a3 + 6 + c-3a6c=0, or

given equal

.,

a3+6+c' = 3a6c.

We

have thus shown that

zero, the

sum

if the sum of three quantities is of their cubes is equal to three times their 'product.

Prove

this also

by substituting h c

for a.

Ex. 5.Show that

(a-6)3-^(5-c)3+(c-a)3=3(a-6)(6-c)(c-a).
Here the sum
result follows at

of a h, h c, c a is zero, and therefore the once from the preceding theorem.

Similarly, (a

since the suro

+ 26- 3c)3 + (6 + 2c- 3a)' + (c + 2a- 36) = 3(a 26 - 3c)(6 + 2c - 3a)(c + 2a- 36 of a + 26 3c. 6 + 2c-- 3a, c + 2a 36 is zero.
-f-

),

EXERCISE
Factor
1.

159

(a+26)9-c3. (a+6)'+8c'.

2. 4. 6.
8.

a'^-{h-c)\
(aJ-6)3-t-(c+i)'.

3. 6.
7,

{x-y)^~{a-h)\
(3a-6)'-(a-36).
a*b^-\-c*+Zabc

[2x-yY-\-{x-2y)K
8(3a-6)3-27(2a-3i)s.
%i^-iry^-\^z^~%xyz.

9,

tO.

348
11.

ALGEBRA
a3+6-l+3a6.
8x3-1/3-1 253=' -SOxyz.
is

12.

l+c-rf3+3c<f.

13.*

14.

(a4-6)HcH'-3c(a4-6l

What
15.
16.

the product of

abc 1-a-b

and a^-\-b^+c^-{-ab+acbc.

2x-y+3z and 4xH2/^+922+2Ti/-6xz+32/a.


and l+a^+fts+a+fi-aft.

17. 18.

2a-36-4 and 4a2+6a6+962-126+8a+16.


is

What
19.

the quotient of

1a^+b^+Zab by la+b.

20.
21.

27m3 n^- 1 9mn by 3w n 1.


a3+ 12563- l + l5o6 by a24.2562+l-5a6+a+56.
is

What
22. 23.

one factor of

(4a+36)3-(a+26)3.

(a:2-3x+7)3+8.

24. 25. 26.

(o2-3a+2)3-(a2-5a+7)3.
{a+b)^+{c+d)^-l+3{a^b){c+d).
Prove that the
is

difiference of

the
of

cubes

of

4a2+a+l and

2a2_2a-f3
27.

divisible

by the product
is

2a

and a+2.

Show

that

a^+b^+c^ 3abc

equal to

^{a+b+c)>{a-b)^Mb-c)^-\-{c-a)^\.
28.

Write down a quantity of the type a^ -\-b^-{-c^3abc, of which


is

^x2y+z
29.
If If If

a factor.

What

is

the other factor

aH-6 c=0, show that a^+b^i-3abc=c\

30. 31.

x=y+z, show

that x^=y^'^z^+3xyz

a+b+c=0, show that


that

(o+26)3+(6 + 2c)3 + (c+2a)3=3(a + 26)(6+2c)(c+2a).


32.

Show
Show

(x-y)3-l-(y-2)3+(z-x)3-3(x-J/)(i/-2)(z-x)=0.
33.

that

(a+36-4c)3+(6 + 3c -4a)3+{c+3o-46)3 =3(a+36-4c)(6 + 3c-4a)(c+3a-46).

STTPPLJfl,f^fJ7TARY
34.
If

THEOREMS
that x'^-\-y^^Zxyz=^z^.

349

z^ah, y=a+b, z=2a, show

35.

Find the value of

a^-h^->rC^-\-'ioiic.

when a=-32,

6=--46, c=-14.

36.

Reduce

to lowest terms

2a2-5a6+3624-a-3c2
37.

(a:-2?/)2+(22/-z)2+(3-a;)3*

Find two factors of the

first

degree of

{ax-\-hy-\-azY-\-{hx+ay+bzY.
38.

When x=h-\-c, y=c+a,

z=a-\-b, prove that

x^-\-y^+z'^Zxyz=2{a^-\-h^+c^Zahc).
39.

increased, or each decreased

Prove that a^+h'^+c^ahachc is unaltered by the same quantity.

if

a, 6, c

be each

40.

Solve (a;-a)3

+ (6-.T)3+(a &)^=0.

246.

Grouping Terms.

We

have

already seen

(art.

91)

that

we can

frequently obtain a factor of an expression

by

suitable arrangement of the terms.

The
this

following examples will give further illustrations oi

method.

Ex. 1. Factor a\h-c)-\-h'^{c~a)+c\a-h).


Arrange in descending powers of
a,

and the expression

'

= a2(6-c)-a(62-c2)~L6c(6-c), = (b c){a^ ab axi-{-bc), =^{bc)a{ah) c{ab)\, = (6 c)(a 6)(o c).


When
the factors are written in cyclic order
(art. 140),

o*(6-c) + fe^(c-a) + c*(a-6)=-(a-6)(6-c)(c-a).


This expression may also be factored by writing it in the equivalent form {a--h^){b-c)-{a~b){b-^~c^). In this form a b and b c are seen to be factors. Complete the factoring by this method.

The expression
.-.

cr(6*

c2) + 6(c^ a^) + c(o* 6^)

differs

on.y in sign

from a2(6-c)+fc(C"a) + c2(a-6), a(62 c2) + 6(c2-a2) + c(a-62) = (a-6)(6-c)(c-o).


o6(o 6) + 6c(6 c) + ca(c o)=

Also,

(a 6)(6 c)(c a).

350

ALGEBRA
=a^(6 c) 0(6' c') + 6c(6* c*),

Ex. 2. Factor a\h c)+h^{c-a)^c^{ah).


The expression

= (6 c)(a3 afe*-aAc ac* + 6*c + 6c*).


Now
arrange the second factor in powers of
b,

and proceed as before

and obtain

(a 6)(6 c)(c a)(a-r^ + c).

Factor also by using the second method of Ex. 1, writing the expression in the form (a' 6^)(6 c) (a fe)(6' c*). What are the factors of 0(6^ c') + 6(c* a')+c(a* 6^), and of a6(a'-6) + 6c(6*-c*) + ca(c-a) ?

Ex. 3. Factor a2(6+c)4-62(c+a)+c2(a+6)+2a5c.


Arrange
in

descending powers of

a,

and the expression

= a*(6+c) + a(6* + 26c + c*) + 6c{6 + c), = {6+c)(o+o6+oc+6c), = (6+c)(a + 6)(a+c) = (o + 6)(6 + c)(c+a).
Ex. 4. Factor
are easily factored

{a'^h^)x^-{-{a'^-{-h^)x+ah.

Expressions of this kind,

when

written in descending powers ot

Z.

by

cross multiplication in the usual

way.

(a-6)x* + (a*-6*)a; + a6
(o
(a

+6 )x b)x
and
(a

+a +6
6)x+6.
160

The

factors are (a

+ 6)x4-a

BXERCISB
Factor and verify 1-8
1.
:

acx'^+x{ad-{-bc)-\-hd.

2.
3. 4. 5.

mpx*-\-xy(qmpn)nqy*.
a;2(o-62)+4a6a;-(a2_62)_

(p*-g')y*+22/(p2+g2)+l)2-g2.

z*(a+6)+a;(a+26+c)+6+c.
a;*(a2_a)^a.(2a2_3a^2)+o2-2a.
o{6+c)+a(6+36c+c)+6c{6+c).
o6(a+6)+6c(fe+c)+co(c+o)+3a4*'

6. 7.

8.

SUPPLEMENTARY THEOREMS
9.*
10. 11.

351

xHy-z)+y^z-x)-\-z^x-y).
xy{xy)+yziyz)+zx{zx).
x{y^-z^)+y{z^-x^-)+z{z^-y^).
a(b^-c^) + b{c^-a^)+c{a^-b^).

12.
13.

a^b*-c*)+b\c*-a*)+c^{a*-b*).
Divide

14.

a^b-c)^b^{c-a)+c^a-b) by a2(6-c)+62(c-a)+c2(n-6).
Solve and verify
15.

abx^x{ad-\-bc)-\-cd=0.

16.
17.
18.

{a^-b^)x^-Aabx=a'^-b\
x2(c-6)+a2(6-a:J+62(x-a)=0.

a6x2-x(a2+62)+a2-62=0.
{a^-ab)x^+{a^+b^)x=^ab+b\
Find a

19. 20.

common

factor of

abx^-\-x{a^2abb^)a--r-b^ and

a^x^a^xabb\

The Factor Theorem. Ws have already seen that 247. any expression is divisible by x~a, if the expression vanishes when we substitute a for x (art. 101). Any expression whose value depends on the value of a; is
called a function of x (art. 114).

Any function of x may be conveniently represented by the symbol /(a:), which is read " function x." The factor theorem might be stated thus
f{x)
Thus,
if

is

divisible
f{x) /(2)

by

oca

if

f{a)=0.

= x^-lx^+nx-2, = 8-28+22-2 =
2,

.'.

' 7x*+ 11a; 2

If

then
,*,

T* 4x*a

-|-

by x 4xa+5xa*+10a', /(.c) = x3 /(-o)=-a3-4as-5a3+10a3 = 0, 5xa* + 10a* is divisible by x-\-a.


is

divisible

362
248.

ALGEBRA
Factors ol x'^

a**.

We

nave already seen that

x* a'^ = {xa)[x-{-a^,

a;*-o*={x*-a)(x* + a2)=(x a)(x+o)(x+a>-

Here we see that x a is a factor of each. Is x a& factor of z^ a^ ?


Vhen we substitute a for
x,

a;o
(1) Is

is

a factor of a;^o^
?

xa

When we

a factor of x" o" substitute a for x,


a;"

a" a" a"


?

ST!

U-

" a"
(2) Is x-\-a

is

divisible

by a; a,
a;"

a factor of

o"

When we

substitute

~a

for x,

a;" a"

= ( a)" o".
( o1" = cf", and
tins

Now (a)" a"


J9

will

true only

when
is

be equal to zero only when n is even,

X"a^
Thus.
*"

divisible

by x+ct when n

is

even-

a;*--a*, re* a*, a;* a*, etc., are divisible


etc.,

by x+a, but a:* o*.

o*,

are not divisible by x-\-a.

(3) Is x-\-a

a factor of

x"-{-a" ?
it is

Examine
* is

this, as in

the preceding, and show

factor only wiiOD

odd. a; a a factor of a;"+o"


?

(4) Is

We thus conclude that, when w is a positive integSK, (1) x a is always divisible by xa (2) x" a" is divisible 6y x+a when n is even. (3) x+a" is divisible by x+a when n is octd. (4) x+a" is never divisible by s a.
249.
1)

Quotient on dividing

a?""
a:* a*

by act.
,
,

^2 o

xa

= x-f-a.
-

xa
^^^^

, , =sx'-\-x'aA-!ea*+a*.
. ,

~
=a*4-ir'C--i-a*

=af* J-a?"o+a;*o*+a'-i-<**

SUPPLEMENTARY THEOREMS

353

Verify these results by division or miiltiplication. Notice that the signs are all positive, and that the powers of x are descending and those of a are ascending.
Similarly,

= x^ + x^a-\-x*a*-\-x^a^-{-x*a*-\-xa^+a*,
X a

and
(2)

a;"~^+a;"~*a+a;"~*o*-f
x-\-a

+xo""*+a"~*.

x-j-a

=xa.

=x^x^4-xa*a\

Verify and note the peciiliarity in the signs.

Write down the value of

and
-

of

x-\-a

x-\-a

(3)

^ = x* x+o

xa-\-a*.

=* x'a+a;*o* xa'+a*.
x-\-a

Write down the value of

x'

+ a'

x-\-a

and

x*"+^
of

+ o'"+';

x-\-a

EXERCISE
1.
If

161

/(a;)=a;3-8a;2+19x-12, find the values of


/(I), /(2), /(3), /(4), /(5).

What
2.

are the factors of

x^ Sx^+lGx 12

If

/(x)=x* 2x3 x2+2x,

find the values of

/(2), /(I). /(O).

/(-I), /(-2).
?

What
3.

are the factors of /(x) in this case


is

Prove that x^* ?/^*

divisible

by

xy

and x+y.

4.
5.

Prove that x^ 21 Prove that

is

divisible

by x1, x+l> a^^+l ^j'+l*


by
x-\-y

x^'-j-y^' is divisible

and that x^+32

is

fivisible

by x+2.
:

Write down the quotients in the following divisions


a;3-i-3

6.

^^

7,

a* 6* ^.

x-j-y

ab
a:*-81
'

8.

a*b*
a-\-b

9.

x'\ =.

10

*-i+32 a;+2

^'-"'
^^
'

j^
'

13
'

(+fe)^-l
(o+ft)

x-\-Z

x^a'

+r

A A

364
14.

ALGEBRA
State one factor of
;

x^-b^, a-'+b\ x3-64,

m''+,, {x+y)^-l.

What
15. 16. 17. 18. 19.

is

the product of

o^4-a*+o+l and

a 1. in^-^m^m-\-l and m*

m-\-l.

a^+a*b^+a^b*+b'^ and a^b\


Prove that x^+3x*+ix^+224L
is

divisible

by x'+l.

Show
If

that

x+y,

x^+y^, x^+y^, x*

+ y*,

x^+y^ and a;i+y"

are factors of x^*y^*. 20.

X a

is

a factor of a;^+pa;+? find the relation between

o,

p
is

and

g
If

21.

f{x)=Tnx^+nx-]-r,

find

/(a)

and show that f(x)f{a)

divisible

by xa.
a; 1
is

22.

If

a factor of x^k^x^

+ lOkx10,

find the values of

k and verify.
23.

Write down the quotient when


(1) (2)

(3)
(4)

xa x+a xa x+a

is is is

divided by x^a^. divided by x*+a*. 1 1 divided by x^a^. divided by x^-\-a".

is

250.

Symmetrical Expressions.

An

expression
if

is

said to be

symmetrical with respect to any two letters when those two letters are interchanged.

it is

unaltered

Thus, x + y and x* + 2/* are symmetrical with respect to x and y, but x^ + xy is not symmetrical. Similarly, a + b + c and ab + bc + ca are symmetrical with respect to a and b, b and c, c and a, for if any two be interchanged the expressions remain unaltered.

An expression is said to be Symmetry. Cyclic 251. symmetrical with respect to the letters a, b and c, if it U unaltered when a is changed to 6, 6 to c and c to a, that is, when the letters are taken in cyclic order-

SUPPLEMENTARY THEOREMS
Thus,
a,

366

a* \-b^-\-c*
c,

ab~bc ca
6*

and

for wh"'n the letters

symmetrical with respect to are changed in cyclic order the result is


is

+ c*+o* 6c ca a6,

which
a,

equal to the given expression. The expression a^ + 6* + c' 3o6cd is symmetrical with respect to b and c, but not with respect to a, b, c and d.
is

with respect to
k{a-\-b-\-c).

The only expression of the first degree which is symmetrical a, b and c is a~\-b-\-c or some multiple of it as
of the second degree, a^-\-b^-\-c^ of

There are two expressions and oi + 6c +ca,. and the sum

any multiples

of these, such as

k{a^-\-b^-i-c^)+l{ab-{-bc+ca),

which are symmetrical with respect to


252.

a, 6, c.

Symmetry
be

applied

to

Factoring. of

The

factor theorem

may

applied to

the

factoring

many symmetrical

expressions.

Ex. 1.Factor a{b^-c^)+b{c^-a^)-^cia^-b^).


If

we put a = b, the

expression equals zero,

a6

is

a factor.

Since the expression is symmetrical and a b is shown to be a factor, it follows that b c and c a must be factors. We have thus found three factors each of the first degree. But the given expression is of the third degree, and, therefore, there cannot be another literal factor. There may be a numerical factor.

Suppose
.-.

A;

is

a nimaerical factor,

a(6*-c*) + 6(c*-a) + c(a2-fe*) = t(a-6)(6-c)(c-o).


is

Since this relation


let

true for

all

values of
6

a, b, c,

then

:.

= 2, c = 0, l(4-0) + 2(0-l) + = Jfc(l-2)(2-0)(0-l), 2 = 2k, or k=\, a(6*-c*) + 6(c-a) + c(a*-65i) = (o-6)(6-c)(c-a).


a=l,
.-.

In finding the vaUie of k, any values of a, 6, c may be used provided they do not make both sides of the identity vanish on substitution.

A A 2

366

ALGEBRA
(a+6+c)3+(a-6-c)3+(ft-c-o)34-(c-a-6)3.

Ex. 2. Factor

If
.".

we put a = 0,
o must be a

the expression vanishes,


factor, and, therefore, b

and

c.

Complete the solution aa before, and show that the expression equals
24a6c.

Ex. 3. Factor a^{h-c)+h\c-a)-^c^{a-h).


in Ex. 1, show that a b, b c, c a are factors. Since the expression is of the fourth degree it must have another factor of the first degree. The remaining factor must be of the form A-(a + 6 + c).

As

.-.

a^b-c) + b^(c-a) + c^a-b)=^k(a-b)(b-c){c-a){a + b + c).


a, b

Substitute numerical values for

and

and show that the

factors are

-(a-6)(fe-c)(c -a){a + b + c).

Ex.

4.

Simplify
26)2+(a+6+c)2.

(a-6-2c)2+(6-c-2a)2+(c-a

This expression is symmetrical with respect to a, b and c and is of the second degree. In the simplified result there can be only two kinds of terms, squares like a* and products like ab.

1+4+1 + + fe*+c*). The coefficient of ab is -2-4+4 + 2 = 0, the complete result is 7(o* + 6*+c*). Check by letting o = 6 = c=l.
The
/.

coefficient of a* in the result is


is

or

7,

one part of the result

7(o*

.*.

Ex.

5.

Simplify
1

(a+6)(a+6-2c)+(6+c)(&+c-2a) + (c+a)(c+a-26).
The
/.

coefficient of a* in the result is

one part of the result

The
.*,

coefficient of ab is 2

the

;.

the

+ 2-2 or 2, other part of the result is 2(a6 + 6c+ca), complete result is 2(o*-i- 6* + c* o6 6c ca).
is

2(a*

fe*

+ 1 or + c*).

2,

SUPPLEMENTARY THEOREMS
FIXBRCISB
With
1.

S5l

162 (1-12, Oral)

respect to

what

letters are these


2.

symmetrical
3.

a+b.

a+cb.
5.

x'^+y'-'rxy.

4.
6.

ab+bc+ca.

a^+b^+c^^3abc.

x^+y^+x-y.

7.

3{p^+q^+r^)-2{pq+qr+rp).
first

8.

What

is

the simplest expression of the


1

degree which
a, b, c

is

symmetrical with respect to x and y


9.

a, b

and

and d

What

expression similar to a^-\-b^-\-Zab


?

is

symmetrical with

respect to o, 6 and c
10.

Simplify

{a^by-+{b+cy+(c+a)^
11.
to a, 6
li a-{-b is
a,

and

{a-b)^+ib-c)^+{c-a)\

factor of

and

c,

what other

factors

any expression, symmetrical with respect must it have ?


is

12.
of a^

When
is

{a-\-b)^-\-{b-{-c)^+{c+a)^
is

simplified,

the

coefficient

2,
?

of a^b

3 and of abc

is 0.

What must

the simplified

form be

Simplify
13,*
14. 15.

{a-b+c)^+{b-c+a)^+{c-a+b)\
{a+b){a+bc) + {b+c){b+ca) + {c+a){c+ab).
{xy){px-\-pyz)^{yz){py+pzx) + {zx){pz+pxy).

16.

(a-6)H(6-c)^+(c-a)3.

Factor
17.

x^{y-z)+y^{z-x)+z^{x-y)
xy{xy)+yz{yz)+zx{zx).

18.
19.

a'^{b+c)+b%c+a)+c\a+b)+2abc.

20.
21.

(a+6+c)3-(a+6-c)3-(6+c-o)3-(c+a-6)3
(x-y)3+(2/-2)3+(3-a;)3.
a(6-|-c)2+6(c+a)2-|-c(a+6)2-4a6c.

22. 23.

ab{a^-b^)+bc{b^-c^)+ca(c^-a^).

24.

a2(6*-c)-|-6V-a*)+c^(a*-6*).

358
Simplify
25.
:

ALGEBRA
^(y+g) {x-y){z-x)
y(8+a^)
, ,

2(a:4-y).

(yz){x-y)

{z-x){y-z)'

26.

^
(a:-y)(x-z)
?^

+
+
'

y"

{y-z)(y-z)

+ +

!!

iz-z){zy)
"^

27.

^
+

(c-a)(c-6)
28.

{a-b){a-c)^ {b-c){b-a)

bc{ab){ca)
29.

--4-

--J
c^ba
"^

ca{bc)(ab)

-+

ab(ca){bc)

b^ac
(a-b){b-c)
"^

(b-c)(c-a)
cajc+a) {bc){ba)

a^cb {c-a){ab)
ab{a-\-b)

30

bc(b+c)

(ab)(ac)

(ca){cb)

31.

, .:. ^. ... 1^ (x j/)(z a;) + (/ 2)(a; .. +; (z a;)(?/ z) t/)


.

32.

(a-6)''+(&-c)^+(c-a)' a(62_c2)+6(c2-a2)+c(o2-62)

33.

SimpUfy

(a+b+c)'^-{b+c)^-(c+af-(a+bf+a^+b^+c',
is

being given that a

a factor of

it.

34.

Sliow that

ab

is

a factor of

o"(6 c)+6"(c a)+c"(a 6).

What may
35
.

be inferred regarding other factors


expression
is

An

symmetrical

in x,

y and

two dimensions. x=l, y=2, z=3,

When x=t/=z= 1,
it

the expression equals 15, and

and each term is ol when

equals 64.
it is

Find the expression.


obviously impossible for the following

Point out wherein 36. statements to be true


:

(1)
(2)

(a*+6*+c2)(a+6+c)=o3+6'+o2(64-c)+62(c+a)-

a^+b^+c'-Sabc=(a+b+c){a^+b'^+c^-3ab).
(a-6)(6-c)(c-o)=o6+6c+ca*-ac2_6c*-6a*.

(3)

SUPPLEMENTARY THEOREMS
253.
Identities.

359
of
is,

We
all

algebraic

expressions

have already had many examples which are identically equal, that
values of the letters involved.

which are equal for


Thus,
(.T-f ?/)(x

2/) = a;* .V*,

(o+6)3=a3+3a26 + 3a6* + ft',


a3

+ c3-3a6c = (a + 6+c)(o*4-62 + c2-a6-6c-ca).

Any of these may be shown to be identities by performing the operations necessary to remove the brackets on one side, when the result is the same as the other side.

Ex.Show

that (a+fe+c)^

=a3+&3^c3-3a6c+3(a+6+c)(a64-&c+ca).
Here the cube of a-\-h-\-c may be found by multiphcation or by any other method. The brackets are then removed from the right and the terms
collected.

The two
is

sides are
also
:

now

the same, which shows that the given statement


first

an identity. We might

have changed the second side into the

by

factoring, thus

(a3 + 63-[-c3 3a6c) + 3(a + 6 + c)(a6 + 6c + ca), = (o + 6 + c)(a + 62-j-c*-a6-6c-ca) + 3(a + 6 + c)(a6 + 6c + co), ==(a+6 + c)(o* + 6+c* + 2a6 + 2ac+26c), :=-^(a + + c)^, which proves the proposition.
fe

254.

result

When two expressions are to be shown may frequently be obtained by showing


is

equal, the

that their

difference

zero.

The
(1) (2)

diflference

may

be zero,

because because

all
it

of the terms cancel, or

has a factor which

is

equal to zero, identically,

or which is given equal to zero.

.iGO

ALGEBRA
1,

Ex.

Prove
(a-6)3 + (6-c)3 + (c-a)'-3(a-6)(6-c)(c-o) = 0,

(a-ft)3+(6-c)3+(c-a)3=3(a-6)(6-c)(c-a).
Here we may prove that
by removing the brackets when
all

(1) (2)

the terms cancel,


factor
245).

by observing that (a 6) + (6 c) + (c a) is a expression and this factor is identically equal to zero (art.

of

the

Ex.

2.

If

a^h=^c, show that a'^-\-hc=h'^+ca.

Here, as in the preceding, we may show that a* + 6c t* ca = 0. by showing that a-\-h c is a factor of it and this factor is given equal to zero, or by substituting c = a-\-b in each side or in the difference. Solve this problem both ways.

Ex. 3. If

a+6+c=0, show
c,
for

that

{a-\-b)(b+c){c-\-a)-\-abc=0.

For

a-{-b

substitute

b-\-c

substitute

a, and

for

c+a

substitute

b and (a + b)(b-\-c){c + a) + abc = {-c){-a){~b'+abc = 0.

Ex.

4.

If 2s=a-\-b-j-c,

prove that

s2+(s-a)2+(s-6)2-i-(5-c)2=a2+6Hc2.

When

the

first

side

is

simplified

it

= 4 2(a + 6 + c) + o+6* + c*, = 4s-2(2s) + a* + + c*, = o*+6* + c*, which was required.
i'*

Of course,
s

this could
is

have been proven by substituting the value


eeisier,

ol

at once.

It

usually

however, to substitute in the last step.

BXBRCISB
Prove the following
1. 2.
identities
:

les

a(6+c)2+6(c+a)24-c(a+6)2 4a6c=(a+6)(6+c)(c+a).
{x+y)*+x*+y*=2{x'+xy-hy^)''.

3.

(o+6)3+(a-6)3+6o(o+6)(o-6)=8a.

StTPi'LEMENTABY THEOREMS
4.
5.
If

361

2(a3+6'+c3-3a6c)=(a+6+c);(a-6)2+(6-c)2+(c-a)2|.
a(b-c)^4-b{c-a)^+cia-b)^={a-b){b-c){c-a){a+b+c).
that

a+6+c=0, show

6.
7.

(3a-26+4c)2-(2a-36+3c)2=0.

a2^62_c2^2a6=0 and c^ a6=62 ac.


(a+6)(6+c)+(6+c)(c+a)+(c+o)(a+6)=a6+6c+co.
a*+6*+c*=2a262+262c2+2c2o2.

8.

9.

10. 11. 12.


13.

(3a-6)3+(36-c)3+(3c-a)3=3(3a-6)(36-c)(3c-a).

a(62+6c+c2)+6(c2+ca+a2)+c(a2+a6+62)=o.
If

a+6=l,

prove that {a^-b^-)'^=^a^-]-b^-ab.

If

a;+=23, prove that

a; z
14.
If

-^ + y = -^

z
-,

2.

= ^-^, b =
X
y

show that a-{-b+c-\-abc=0.

15.

If

-H a a
a;

c ab
2/

r,

prove that a
-\

+ - = -. c b
x^
-\-

16.

If

+-

show that x^

= y^2
^

= y^ 3y

x*^-=y*-4:y^-\-2.
X*
If 2s=:a-\-b-\-c,

show that

17. 18.
19.

s{s~a)+{sb){sc)=bc.

a(s-a)+6(s-6)+c(s-c)+252=2(a6+fic+ca).

(s-a)2+(s-6)2+(s-c)2+2(s-a)(s-6)+2(s-6)(s-c)

+ 2(s c)(5 a)=5*.


20. 21.

(2a5+6c)(26s+ca)(2c5+a6)==(a+6)2(6+c)2(c+a)2

J_ + -L + -L_l=
sa
sb
sc
s

'^
s{sa){sb){sc)

22.

I6s(s-a){s~b){s-c) = 2bh^2c^a^+2c^a^-a*-b*-c\

23.

If6

+ -=l5C + - = l, prove + r =

and a6c

1.

362

ALUEH HA n
If

24.*

a 4- -

= 3,

find the value of a^

+1

26.

a=a;(6+c), h=y[c-\-a), c=z{a-\-h), show that


xy-\-yz-\-zx-^1xyz=^\.

26.
in

If

4-y=o and xy=b^,


6.

find the values of

x^+y^ and x'+y^

terms of a and
27.

Eliminate

and

?/

from

the

equations

x-{-y=a,

xy=b^,

28.

Eliminate x and y from z+2/=a, x^+y-=6^, x^+!/^=c'.


If

29.

x=a+6 c, y=6+c a,

z=c-{-ab, show that

xH2/3^z3-3x2/2=4(o3+fc^+c^-3a6c).

EXERCISE
1.

164 (Review of Chapter


is

XXVI)
x-\-y-\-z,

Show

that x' + y' + z^ 3xyz

divisible

by

and hence

show that
2.

(fe-c)'

+ (c-a)3 + (o-6) = 3(a-fc)(6-c)(c-a).

Prove that

3.

If

o + 6 + c + d = 0, prove that

4.

Prove
is

+ b){a + c){a + d) = {b + c){b + d)(b + a). that (a 6)" + (6 c)" + (c o)" is divisible
{a (o

by

6)(6 c)(c a),


I

when n

an odd

integer.
is

If 6. 23+i-|.i

n is a positive integer prove that 12" by 24, 7"-l by 48.


of the

divisible

by

11,

6.*

Write down a quantity

same typa

is x^

of which
7.

^x+}y ^z
that
a,

+ y' + z' 3xyz

is

a factor.

Show

a x and a 2x are factors ot

(a-6)(o-6-x)(o + 26-2x) + 6(6-x)(3c-26-2r>;}.


8.

Show
is

that {x + y)'' x" y"

is

always divisible by xy{x + y),


find x in terms of a

when n
9.

an odd integer.

If (^

o)(l a) = (^ 6)(1 6)^x,

and

only.

SUPPLEMENTARY THEOREMS
10.
If
a;

363

-f 2/

(1) x* (2)

+ 2 = 0, prove that + xi/ + ?/^ = .y^ f 2/2 + z* = 2* + 2a; + x'.


(3/

(a;+2/-2)' +

+ 2-a;)3 + {z + x-2/)3 + 24x2/2 = 0.


b
c

11. 12.
13.
14.

Simplify

Solve

_ c) + ca(6 - c)(6 - o) "^ ab{c - a)(c - 6)' (x-a)3 + (x-6)3 + (x-c)3 = 3(x-a)(a;-6)(x-c).
tc(

fc)(a

Show
If 2s
(1)

(2)
(3)

that (o + 6)5-a5-65 = 5a6(a-r6)(a* + a6 + 6*). = o + 6 + c, show that s(5 6) + (5 a)(s c) = ac. s+(5 o)( 6) + (5 6)(s-c) + (s-c)(5-o) = o6 + 6c+ca. (s-o)3 + (s-c)3 + 36(s-o){s-c) = 63.
is

15.

Prove that a''(62_c2) + 6(c2 o*) + c"(a2-62)


X

divisible

by

(ab){b c){c a) and find the quotient when w = 3.


ifi
'

17.

18.

19.

x c x b - c) "^ 6(6 - c)(6 - a) "^ c(c - a){c - b) If x = a^ bc, y h^ ca, z = c* ab, prove that ax + by-\-cz = {a + b-\-c)(x-\-y-{-z). a(6 - c) + 63(c a) + c3(o 6) Simplify -^r .,. 'J, ^ ^ (6 c)3 + (c a)* + (o o)' If a6 + 6c + co = 0, show that (1) (a + fe+c) = o* + 6 + c*. (2) (a + 6+c) = o4-6' + c3-3o6c. (3) (o + 6 + c)* = a* + &* + c*-4a6c(o + 6+c).
If *i"^P*"y
?

a{a

6)(a

'

20.

Show

that a;"+^ a;" a;+

is

divisible

by (x

1)*,

when n

is

positive integer.

21.

Write down the quotient on dividing


X* a* by X a, x* +
l

by x*+l, o* 32 by a 2.

22. 23.

Factor
,.,

x*-l-3(x-l) + 4{x-l).
o(fe*

Simplifv

-f

b){a (a Tv; r c)

6c

+ c*) + two similar fractions. ..,,..


,

24.
V
is

Show
If

that x(y*z^) + y{z*x*) + z{x* y*)


x-\-a,

is

not altered when

changed to

y to

y-\-a, z to

z+a.

25.

r*=+l, show

that x = 5x+3.

364
26.

ALGEBRA
Find two
linear factors of

(ax+h)^-\-{hx+cy + {cx\-a)3-~3(ax-\-b){hx-rC)(,cx+a).
27. 28.
If

x^+y^ = z^, show

that {x^-{-y^-z*)^ + 21x^y^z*

= 0.

If a

+ b + c = 0, prove that a^ + b^ + c^ + 3(a + b)(b + c){c + a) = 0.


expression of two dimensions
is

29.
in X, y,

A homogeneous
z.

symmetrical

Its value is 42
it.

when x = y = z = 2 and
a;

is

16

when x=l, 2/= 2,

2=0.
30.
31.
32.

Find

Eliminate x and y from


If If

+ = o,
2/

xy = b, x^-\-y^ = c.

x+3/ = 3 and

x^-\-y-

= b, find the values of x^-\-y^ and x*+y*.


a+6+c*

o + 6+c=- 10 and a6 + 6c + ca = 31,find the values of

Bnd a' + ft-'+c* 3a6c.

ANSWERS
TO

HIGH SCHOOL ALGEBRA

ANSWERS
No answers are given to elementary examples, oral examples or examples which may be verified or checked without difficulty. In each exercise the number of the first example to which the answer is given is marked with a star.

Page 8
15. 108,
38, 10. 32, 60.

16.

3, 14, 39, 0.

17.

9, 29, 18.

19.

2.

21

25.

22.

44, 7.

23. 154, 616.

Page 10
9. 47.

10.

70.

12. lOz+10.

13. xft.E.

14.

15.

16. 2a8+2a2+3a.

17.

11,5a;.

Page 12

24.

37.

25.

17.

26.

34.

27.

1.

28.

1.

29.

^.

Page 14
30. (1
4-

A^ hours.

31 . (5a;+20y-72)
34. 1234, 4019.
20, 20.

cents.

32.
3,

^^

ceata.

33. 5x+10y+5(K.

35.
24.

02. 2, 16, 05, 03. 2*.

36. X

+ -. y

37.

38.
S67

39.

968

ALGEBRA
Page 38

7.

la+eb-15c.

8.

lOx^-Ux+9.

9. lOa-76.

10. 4o+46+4c.

11. 'ia-8b+3c-5d. 14. 6a2+862_6c2.

12. 8a; 62/+5. 15. 3a4-36+3c+3jl

13. 8o-4fe.
16. 2x+2yz.

Page 40
18.
0.

19. 15x+5y.

20.

46c.

21. 10a2+o6.

22.

4.V*.

23.

2o 6 ^.

Page 41
13. -3xy. 18.
0.

14. 19.
z.

0.

15. 4p2.

16. 10TO-3n.

17. 6.V-4?.

Page 44
15. 3a-2b.

16. 2a+5c.

17. -Sx^.

19. 4a2_4o-15.
23. z^+Gar-S 26. 2a;-3.

20.

26.

21. 13r-p.

22. 36-oc-2a. 25. a+b+c.

24. 2a2+a-12. 27. 10x+2a;2+8x4-2.

Page 46
11. 3x+2y.
15. 3a+6-3c.

12. -2a-36. 16. 3x2-3.

13. a+b. 17.


7.

14. 6-a. 18.


y, 0.
11.

22.

6, 4, 4, 6, 10.

23. 4a+46-15c, 4o-46+4c-4rf,

ANSWBBS
Page 47
1. 6.
o.

am

2. 4x46. 7.

3. 5n.

4. 8.

5c.

5. 46,8a. 9. 14,
6.

2a+26+6c.

a;+y-5z.
12.

3a-2b2c.

10. -7. 14. 18.

11. y-x. 15. 4a;-9.

31.

13. a-2c, 2c-o.

9m-2n.

16.

x-U.
2x.

17. 26-4c.

56 5a, 0+36 4c, 7a 6 6c.


22. 52-3a:.
29.

19.

20. 3a; 6.
l

21. o-^6+^c.

25.

20.

26.

+ 2x.

27.

7.

38. 7n+4a;-2TO.

3-o-6-c.

30. 5c-36.

Page 52
19. 21.
1, 4, 5,

-3, -1, -8, -9,


1,

7.

20.
22. 28.
23.

3, 16, 35.

a2,

-o3, -8, -1,

81, 32.
30.

29, 81.

23.

24.

25.

90.

26.

6.

27.

29. -20.

30. -50.

31. -100.

Page 53
13. 8o+76+9c.
17. 4o+i6.

14. x-4y. 18. 6x2+8x.

15. dm.

16. 9o-6.

19. a^

20. x^-9x^+10x.
28. 7a*-5a.

21. -4a6. 29.


2as/,

27. 6x2- 15a;.

3x^+xy+Zy^, x^+5xy+y^.

Page 56
19. 2af2+4x-4.

20. 5a2-8a-22.

23. 214.

24. 'ki^-db\

25. 2x^+2y^, ^y. 28. x2-6x-7.

26. a^+ab+4ib^. 29. 2x-10.


31. 12x2+12.

27. 14a;+30.
32. 3x2+10.

33.

15a.

34. 3x2+12x+14.

B B

i70

ALOEBHA
Page 60

18.

1.

19. x^-Zx+2.

20.

2y.

21.

5.

22. o-b.

23. a\

24. x+13.

Page 60
2.
9, 16,

-12,

25,

-7,

27,

-64,

91.

3.

1,

-1,

1, 16,

-27.

4. 4a2.
9. 12x2+12.

5. 8a2-9a.

6. Sa^-Sft^.

8. 30a+406.

10. 4x2+12xi/-9i/2. 18, 5a2_3ai_452.

13. 13m2+137i2_24TOn.
'20. 3a2-12a-rl4.

16. 4x2.

21. 6x2-2x!/-6!/2.

22. 4-a.

23.

8, 19.

24. x*-16.

25. 8o2-9a-l, 6-lOa, 3a-4. 21. 20b^-obc. 28.


0.

26. a^-b\2,a^-b\

Page 81
22.
27.
7,

-2.

23.

-8.

24.

5,

-2.

25.

5, 6.

26.

^, 2.

6. 1.

Page 83 21
.

4, 5.

22. -3, -3.

23.

4, 9.

24.

5, 3.

25.

12, 12.

26.

19, 3

27.

15,

-56.

Page 92
28.
2a.

29.

1.

30. x+1.

31. 3a;-8.

32. x+5, a+b. 35. a:(x-7)(a;-l).

33. 2(x-2)(x-3). 36. 5, 1.

34. 3(a+4)(a-3).

Page 93
17. 7,-11. 18. 10,-4.

19.

5.

20. 10

in.

21.

7 in.

22. 14

in.

23. 3^

in.

ANSWERS
Page 95
17. 2a;+2.

371

18. 2a^+2b^.

19. 5ar+5.

20.

4tah.

21. 5OT*-10TOn.
24. a;2~4.

22. 5z^+24:xy-5y^. 26.

23. 3x*+12a;+14.

25. 18a- 15.

3x2-4a;2/4-6i/2.

27. 16z-34.

28.

36a;.

29. 9a^-8ab+%^.

31.

8.

Page 98
25. 5o-5. 26. 3a2-862.
30.
15.

27.

0.

28. 19q'-4:pq. 32. 5(x+2)(x-2). 35.


5(l

29. x<-a.

31. S{x+y){x-y). 34. m(x-a)(a;+a).

33. a(o4-l)(o-l).

+ 3p)(l-3p).
41.
2a;, 7.

36. {x+y){x-y){x^+y^).
38. {x+l){x-l){a+b).

37. 40. a2-2a6-362.

;7(ii;+r)(i?-r).

42.

4,

8.

Page 102
22. a;(x+l)(a;-l), 3(x-2)(x4-2), a(a-l)(a-2).
24.
2.

23. 8,-2

2,

-1.

26. 2(x-2)(a;+2)(a;2+4), (a+2)(a-2)(a+3)(a-3), 27. 262+2c2^4.


31. 43,23,17,13.

2m(m+3)(m-3), {x+y){x-y){a+b){ab).
28. 3x2-5i/2. 32. Sx+6y.
30. 12a6-3862.

Page 104
13. a + 2b.
18. a-2.
23. 2(3a+26).

14.

a 6.

15. tw n.

16. x+y.

17.

m + 2.

19. x-3.

20. j/-1. 25.

21. a+b.
2, 3.

22. x-5.

24. a(a-l).

Page 106

22.^+1.
X

23. -J^.

y^

24.^:^. X
29. ^^1-^
2

25.^^.
m
30. x^^l

26.-^.
a-f46

27.

x+2y

^^.

28. o+l. ^

BB

372

ALOE BRA
Page 107

10.?.

11.4.

12.-.
ay

13.^ 13.^
x-\-2

14.^^.
a

IB.

Page 108
10. 14.
o(a

+ l).

11- 3x(x+2).

12. ab{b+c).

13. 2(a;-l). 16. aA(a-6).

ar(x+j/)2.

15. (x4-l)(a;-l)(2;-2).

17. (a+6)(a-6)2.

18. x(x-l)(x+l).

19. 4a:(x- l)(x+l).

20. (t/_l)(j/+l)(y_2).

Page 109
22. ?^^.
6

23.

^^-^y
.

24.

^^.
12

25.

?-t^.
x^-y^

26.

0.

21.^.
X

28.
2

-"^

29.^-^.
ah(ah\

6(x+2)

30

^+^^
(o+l)(a4-2)(o+3)

31.

(a-l)(a+2)

Page 111
2.

X 2,60(x 2).
a6

3. x+y,xy{x+y).

4.

x-2, (x 2)(x+4)(x 5).


X

x3/i6
a

e^
"

.yj

46'c

'

^-

a
'

x4-2 2(x 3)

a+&

9
13.

2a

6-4c
46c

18.

x-2 x-3

ANSWERS
Page 116
18. 7x+6.

373

19. 0+6.

20. 3~xy. 23. 10x2-5a;+15.

21. 3z+3^+3z.
24. a;2+8x 12.

22. 22x2 ^24x-ll. 29.


a.

30.

2.

31. 4|f.

Page 119
17. 3a;2-12x+ll.

18.

0.

19. 2ad+2bc.

20.

0.

21. 12x2 + 12.

22. l-x8.

23. x*-10x3+35x2-50x+24.
26.
13.

24. x*-10x2+9.
28.
0.

25. a6-l.
l

27.

0.

-^

29.

+ 3x+6x2+10x3.

30. 2x3+9x2+3x-l.
33.

31. 28x^+x*y-S3x^ij^+3lx^y^+20xy*-12y\

2-x+4x2-2x3

34.

195.

35.

5i.

36. abx*+x'^ib^-ac)+adx^+x{bd-c^)+dc.

37. p^x^+x{pr-q^)+qr, x3(a2_a)+x2(a2+a_i)_i.


38. 2a^y^-2b^y+2bc. 39. px^+x{p^+Zp+3).

Page 123
30. x2+2x+l. 33. -2xy.

31. -x3+9x2-l.
34.
2a.

32. a^+a+1, a^-a+1.


36. a-2. 37.

35.
2aa;.

6.

a^+Sa 2.

38. x^+xy+y^.

39.

40. x+c.

41. a:+p 1.

42. axbc.

43. ay+a+l.

Page 125
1.
4.

2.

-5.

3. 22.

4. 2/+5.

5.

2j/'.

6.

-2y\

7.\
1^.

-l-. + x+l

8.1+. ab
12.

9.2--^.
a-\-b

10.

5x-3+~^.
x+2

l+x+x2+x3.

1x+x^x^
16.
6.

13.

+ 2x+2x2+2x3
7,

14. l+2a+3o+o'.

15. o-3.

17. z-i,

374

ALGEBRA
Page 126

15. a3+63+c3 3aAc.

16. a- 256.

17. x2-4x+8.

19.

21.
7.

20. llx2-7x-8. 26.


5!/.

22. x^{c-a) + x{d-b) + (f-c).


27. a;+6i/-22.
30.
9.

23.

28. 2(a6+6c+ca).
35.

29. bx^-bcx^^x{ac-a+b)bc. 32. 3x-8. 37. -52. 33. 38.


3*.
4.

31.
G.

x* 4a;2+12x.
36.

34.

35.

p^+p-2.
41.
9.

40. x*+2x^+3x^+2x+l.

45. x*+x^(b+p)+x^q+bp+c)+x(bq+pc)+cq.

46. 3a+2bc.

47. x*+x^+x^a-a^)+x(l-2a)-l.

48. a^+2a^bc+iabh2+Sb^c^. 50.

49. a;3 x2(a-)-6+c)+x(a6+6c+ca) aic. 51. 3a:5_10a;* + 3x3-14a;2-7x.

6 c.

52. x^+y^+xy-2x-'iy+i.

Page 129
21. 3(x-2)(x2+l), a(x-l){y-l). 22. a+b, x-1.

23. (2x-yX5x-32), aJ(a+c)(a-36). 25. (6x-7y)(8o4-56).

24. x-3.

26. {x+y){x+y-\-A), {a-b){2a-2h-l).

Page 131
34. 14x2+191/2.
35. 3a2+362+3c2.
36. 2x+6x2+2.

37. 3a2-f362+3c2-2a6-2ac-26c.
45. 3(x+l)2, a(a+26)2.

38. 8{x^-z^-xy+yz).

46. (a+6+2c)2, (a+b-c-d)^.

48.

3.

49.

14.
0.

50. (x2+2/2)(a2+62+c2).

51. {ax+by)^+{ay-bx)\

52.

Page 134
11. a2-62-c2+26c. 13. p^-iq*-9r^+l2qr.
15. a*-62+c2-d2_2ac-26d.

12. 4x2+ 12xy+9j/ 225.


14. l+x2+x*

16. a^+Ab^-c^-4:d^-4dab+'icd.
(a-x)(a+x)(a2+x2).

44. 2(x+2)(x-2),

o(o+l)(a-l),

46. 6{o-6+2c)(o-6-2c), (x-3b){x-b}{x-5b).

AN S WEBS
Page 134 {continued)

376

46. {b+c){bc){a+d){ad), (a+b+c)(a+bc){ab+c){a--bc).


%7. {ax-\-c+by){ax-'t-cby),

{7nn+Smn){mnSmn)f
50. {x+y){xy)(x+y+a){x+ya).

48. (a;+l)(x-l)(3a;-2), x{x-l)(x-3)(x+3).

49. 2a2 2a6+26c 2c2.


51. 2a2_6a-fl,
12xz24yz,

24o+9a2 Ga',

20a;2|/2 40a;3?/.

52. a+6*+c*-2a262-262c2_2c2a2. 54. (a-6)(c-a).

53. (x-y){y-z)(z-x).

Page 137
13. 2(x2+2x+2)(x2-2a;+2), x{x^+x+l)(x^^x+l).
14. {a-b)(a+b)(3a-b)(3a+b).

15. (a;2-a;+l)(x2+2;+l)(a;*-a;2+l).

16. (a+6+c)(a+6-c)(a-6+c)(a-6-c).

17. (a2+3)(3a2+l).

Page 139
31. 3(a;+8)(x-9).

32. 2(a+l)(a+3).

33. a:(3a;-l)(2a;-l).

^4. (x+l)(a:-l)(x+2)(a;-2).
36.
(a

35. a(a-l)(a+l)(a-3)(a + 3).

+ l)(a-l)(3a+l)(3a-l).

37. (a;+l)(x+3){a;-l)(a;+5).
40. a;2-5a;+6.

38. (x-2)(x-7)(a;4-l)(x-10).

41. 4a;2-16a;+15. 43. 33a^-38ab-Sb\ 46. x^+l.

42. 1, 11, 19, 41.

44. (a;+2/)(a:+42/+l).

45. (3a+26)(a-6+2).

Page 141
22. 2(a-2)(o2-|-2a+4).
24. a{a+l){a^-a+l).

23. 3(y+3)(y^-3y+9).
25. 6(a4-6)(a2-a6+62).

26. {a^+b^){a*-a^b^+b*).

27. (a;+2/+a)(x2+2x2/+2/2-aa:-ai/+o2).
30. 2o(a2+362).

28. x(x2-6x+12).
31. x+y.

29. {2a-b){a^-ab+b^).

33. o2(a*-6a26c+1262c2),

2/22(3a._2^2)(9a.2^3a;y2+t/222).

34. (a-6)(a+i)(a2+62)(o2_o5^t2)(a2_,_(^^52)(4_o2ft2^ft4). 35. (x+l)(x-2)


36.
2.

376

ALGEBRA
Page 143

7. (x-l)(2a;2-9x-4). 9. (x-l)(x-2)(x+3).

8. {x-\){x-^\){x-2).

10. (a;-2)(x-3)(x-f5).

11. (a-l)(o-2)(a+4).

12. ''?^/')2fa-26;. 20.


8.

16. -12.

17. (a-6)(a+26Ka.+ 36).

-4

21

2, 3.

Page 147
4. 2a;2+2a2-262.
7.
14a2

5. 4x3.

B. 4a2+462+4c^
8. 19997.

+ 1462+14c2+14a6-10'?/:-22ftc.
10. -5.
!1-

9. 14,860,000.

2a+19'7.

30. (x-2)(4x-9). 32. 8(a+c)(c a 6).

31. 3a(2a-6)(4a2+2a6+6-;

33. (3x-4)(4x+5). 35. (x+2/)(x-j/+l).


37. {xy){x^xy-\ry^).
39. (a 64-c)(a 6 c).

34. 4(3a-5)(9a2+15a+25).

36. (x-3)(x+3)(x2+2).
38. {x+Uy){x-\2y).

40.

(x+2/)3.

41. (x-36)(ax-2).

42. (a+26)(a-26-3).
44. (a+&)(a+6+c). 46. (x-j/)(x2+xy+j/2+x+2/4-iy
48. (2a+56)(2a-56 + l).

43. {2x-y){2x+y+a).
45. (o-6)(a-6-l).

47. a6(a+6)(o-6)2. 49. 96(4a2+2a6 + 62)


51. (a2-62+a-3)(a2-62-a + 3). 53. (a-l)(3a2_2a-10).

50. (x2+4x2/-2/2)(x2-4x?/-!/2).

52. (x-l)(x2-10x-3).

54. (x4-l)(x-l)(c+l)(c2-c^l;.
55.
(a

+ l)(a-l)(a+2)(a2+l)(a'*-2tt^

1,

60. (a-6)(6-c)(c-a).

61. (x-2)(2x+3)(3x-2).

62. (x-l)(x-2)(x-3Kx-4). (x4-lHx+3)(x-2Vx-6).


63.
1,

5,

6;
5.

0,

1,

6,

-7.

65. x^c\

66. o^-ftM c^+2ac.

67. 4,

68. (x-a)(2x+a+6).

69. (oi+cd)2-(ac+M)2, {ab-cdf-{ac-U)'^.

ANSWERS
Page 153
16. 21.
30.
16.

377

17.

13.

18. 1,3,10.

19.

29.

20. 1,3.5.

Page 155
6. 45. 7.
4.

8.

1.

Page 169
6.

About 2

h.

35 m. after

7. (a) At 10.55, 2 m. from

A started C towards
;

31 m. from Toronto.

D.

(6)

22 m., 17 m.

(c) 11.10.

Page 172
5. 13. 9. 54. 6.

square, 16.
5, 6i, 8.

7. Right-angled,

4.

8.

M.

10. 16;

11.

6.

Page 177
3. 5, 10, 13.

4. 6|.
(5, 3).

5.

13.

6. 7|,

4.

7. 30, 30.
24.

8.

(1,

-7), (-3, -17),


'

9. 112i.

10.

17.

24.

18.(4,4).

25.

(3, 2), 90.

26. S1200,

12th.

Page 181
I. 4xyh, 24ax^y*z':

2. xy,xy{x^y^).

3. a-\-b,b{ab){a+b)^.

4.

x-Z,

(a;-3)(a;-4)(a;+3)(a;+5).

5. a+5, (a+5)(o+3)(a-7)(a-2).
7.

6. 3(a:-2), 3(a:+l)(a;+2)(a;-2)2. 8.

x-y, y{x-y){x+z),

m-2, 4m^2(^_^2)(m-2)2(m2+2m+4).
10. a+6-c, a(a+6-c)(a+6+c).

9. 2(a+o6+62), ^{a^-b^).

II. o+ft+c, (a+6-f-c)(a 6-c)(6-c a)(c a 6).

J78

ALGEBRA
Page 181 [continued)

12. r^-3-.v + y, (r-f yKx^+aV-rJ'';-

13. 3z-2, (3a;-2){a;+3)(a:-3)(2a;-3). 14. 5x-l, (5x-l)2(5z+l)(2o+3c).

15. a;-3, a:(a;-3)(x-2)(a;+5).

16. u v, {uv){u+v)(u^-\-v^){u^-{-uv-\-v^).
17. x2-8,
(a;2_8)(a;4-2)(a;4-3).

19. -a.

20.

2;2_3a;y_^2?/2,

x^+xy-B?/^.

Page 182
1. 3.

x-l,(x-l)(x-2)(x2_5x4-3).

2. a-l,(a-l)(a-5)(o''-18o-l).

x-2, (x-2)(x24-4)(2x2-3x-6).

4. a-1. (a-l)(a2+l)(3a2+a+6). 5. x-l,x(x-l)(x-f4)(x2+x-6).


7.
,

6.

(x-2)(x2+5x+l)(x2-2x-l).

a^+2ab-\5b^

7^,

. ,

-1.
2x4-4

8.

x3-6x2+llx-6, x3-9x2+26x-24.

Page 186
1.

x-5.

2. (o-3)(a-4).
6.

3. 2(3x2+2x+2).

4. 2x-9.
8.

5.

262-6-5.

2x-ly.

7. a-2.

x-3.

9. 3a2(o-l).

10. x-1.

11. (x-3)(x+l)(x+2){x2-x+l). 13. {2x+3)(3x-4)(x2-f3x-l). 15. (5x2-l)2(4x2+l)(5a;2+x+l). 19.


11.

12. (x+l)(x+2)(x+3)(x+4).

14. (x-l)(x-2)(x-3)(x-4).
16.
3.

17.

35.

18. x2^5x-14.

Page 187
1. 2.

x-U,

(x-9)(x-10)(x-ll)(x-13).

x-3, (x-3)(x-12)(x2-2)(x24-3x+9).

3. a-b, (a-6)2(o+6)(o-+a6-f 62), 4. x-f 3, x(x+3)(x+2)(x-4)(x-5).

^. (2a+l)(o-3), (2a+l)(a-3)(a+3)(2o-l),

ANSWERS
Page 187 {continued)
6.

3T9

x-b, {x-a){x-b){x-c).

7. a;-l, (a:-l)(a;-2)(a;-3)(x+2)(a;+3).

8. (x-l)(a;+3), (a;-l)(a;+3)(a;2+a;+4)(a;2-6a;-4). 9. (a-r3)(2a+l), (a-2)(2a+l)(a+2)2(a+3)2.

10. {x-y)\ {x-yf{x-2yf{x-\-2y)-'. 11.


a;2_a;y4-2/2, (a;2_a;2/+2/2)2(x'+a:2/+2/2).

13.

3.

14. a;*-a:2a2+o*.

17.

1, 3.

18. a;2-3a;+2, a;2-6x+5.

19.

3a;+2 2a;+3

Page 191
""^^

o2+o+l
11
'

8.-^.
a;-2

~^
9.

4o2+3a-6
'
'

10.

="+2
23?{x+\)

^^
x+1

12
'

=^'-^

13

-3

14
"

^-3
2a;- 1"

l2x^-lx-4:

a-2o3+2a-5

Page 192
^

a'+b\ o 6*

g_

2y
a;2-^2

3.-1^.
a;2-y2

4.

o-7a+10
.

5.

J^.
a-6*

6.-^^.
1-a*
3

7.^^. a;*-?/^
^^

8.

^
(a;+T/)(2z y)

9.0.
5^

10.

3x^-5ry-2y\
x2-i/2

(x+^){x+5){x+l)
14.

^^
17,

a;2-5a;+6

13. -iL..

xy

a^b^
19.

^.

^5.^^
xy

16.0.

x^Y
21.
2.

18.

? 2o-36
0.

? (a;-l)(a;-2)(x-3)

20.

0.

22.

23.

1.

24.

2(a+l)
26.

25 29 29.

^^

^^
30.

x'^-y^' x*-2/*

-M,.
a4i_y8

27.

81-x*

^*^.

3' 28.

c-62"

^?L 44.

-^
l_a;8

2.

8W

ALGEBRA

Page 196

1.

^~
oa;(a+a:)

2.

3. J?_.

2a-36
g
'

x-2

4.

~^
x^-V
g
'

26+3a
ab{ah)

x+ax
a{xa)

^
'

a;'-a:+2

_1_

x^1
12.

x2
20

9.

-^..
x^-y^
'+^' ac{ab)

10.0.

11.^. b-3a
15.:
(c o)(c 6)

3a+2x,

^.
16.

13.

x*-l

14.

(x o)(a; 6)
21.
0.

17.

_y_.
x{x-y)

18.

_1
1-9x2

19.

0.

20.

0.

22.

1.

""^ 23. "

^" -.^y-y^-^

24. -1.

25.

0.

(x y)(y-2)(z z)
d.

26.

27.

.^\

28

^^"^

{a+b){a^+b-^)

(x-9)(x-25)

29.

^(o-fl)(o+2)(a+3)(a+4)

30.

^^
(a;-l)(x-9)

Page 198

1.

2.

o 6*

-J.
8.

3.
'

^I^.

x+4

4 ?!i'.
'

5
'

o 6*

-^.
1.

y*

x*+y
11.

7. x*+l-f-l. X*

o*+l.
a*

9.

1.

10.

'^.
X

12. y(~^)

13.

a+&

14. ?

+ ??+l.

15.

^l-l

+ g.

ANSWERS
Page 198 (continued)
16. --^-+J!-*.

381

17.

x^+y^
20.

<^.
{a
2)^

18.?!^Z^.
a^
1.

19.i2. x2
24.

--.
o* x2

21.

^.
o2

22.

23.

J-.

^(+f a 66

Page 200

1.

_?_.
106c

2.

Z^.
56 8.

3.

0^62
9.

-i-.
1.

4.

^.
a:i/

5. 1.

6.

-?.
6

7.

-1^.

1.

10.

^y+\
16.

11. i'.

12.^-^;.

13.?.

14. a+6.

15. a+6.

^,.

17. ^,

Page 201

___^a* 6*

1.

-i^.
8.

2.

?^^
6c

3.-?^.
--

4.0.

a;2 9i/2

5. ^

-?-.
la*

6.2.
x

7.1.
2

1^'.
2a;

9.

6+c

^^

10.

^+^
x^+iar.+S

11.-^+^
o_oi+68
15.0.
,..

12.

?!4\
0+6
,

13.

5+-)
a*+ax+x2

14.

i^.
^

16.1.

17.

^"^"^^

(2x+3)(3x-2)
25.

18.

-i-. x-1
32.

20.x.

21.1.

22.1.
*'

x+x^

26.

?.

27. -1.

"^ a+6

33.
(a+b){a^+b')

34.

0.

35.

4^^:10x
(-l)(a:-2)(x-3)(x-4)

gg Q

^__
x+j/

38S

ALOBBRA
Page 210

13.

^^.
a+6
a2

-.

a-\-hcd

14.

-.
a-\-h

15. ah.

16.
a-\-hc

17
'

18

"''+^+^^

a+b

19
'

^
'

20
' "

_^^
2
'

21
'

6*

22. ^'.

23. a-b.

24.
a6

?^.
2
'

25.

6+c

-^,

^.
64-c

26 ""+^ om 6
TO+n' m-{-n
o/\

__

-g 3a6 3a^

_g
,
*

wn(a+fe)

ah

o+3
-^ 31.

mnmn
,

30.

oft cd

-.

a-f-bcd
32.
8-,r-\-rl,

a+l

2s

2s In 28 an
,

^-"+
r

li:^.

33.

?^=^, H^=^.
2t
fi

Page 213
13.
a,

-h.

14. ^i:^, ^illjl.


a,

Oj
.

Cj

0,

15.

2a,

-b.

16.

o, 6

^^^

oA-oA^ a^V-oA
"2^1

O1C2

O1C2

18. b+a, b-a.

19.

^a.

ife.

fflaCj

22.

c, 0, a.

23. 4a -36.

Page 214
25.

^'f 36
.

26. 30.

afcc.

27. 5,5.
31. -3.

28. 21Jl, 27A. 32. $543, $457. 37. 1540, 880,


616.

29. 2,-^. 33.


|.

J, 2.

36. $16400, $13600.

38.

??i::!^^,
2n

^Z^.
n^n
42. 182040.

39. 2a='+26^

40.

35,

41. $2100, $560.

ANSWERS
Page 219
18. a^-fSa+l.

383

19.

6.

20. -4.

21. {z+l)(x+2)(x+3).

22. l-a:-2T2, 2-3x-|a;2.

Page 221
19. ix+y)^-2{x+y)-\-l.
22. a2+62.

20. x^-3ax+a^.
.

21. 26.
13.

024.6?.

23.

x^

+2+-

24. -6.

Page 223
21. 2a^+{iab\ 6a^b + 2b^

23. 2a^+2b^+6a^b+Qab^+6ac^+6bc\

27.

27.

28. 242.

Page 225

M.

x'+x+l.

18. l-2a;+3a;2. 21.


l-a;^.

19. 22.
4c.

?_1J.?.
3

20. 3a2-4a4-l-

23. x-l.

24. a-3.

25. x2.

Page 226
1. 3z2_4xy+22/2.

2.

x3+2x2-3x+l.

3. x+3a;*-2a;2+2. 6. 2x2 9.

4.

hx^-lx+l.

5. 5a;2-2ax-3a2. 8. (a;+l)(x-5)(2x-3).

+ 3a+7.
a;

7. (a:+2)(x+3)(a;+4).

2x2-5a:+2-?.
12.
a.

10. 3-5a;. 13. l-a;-^a;2,


18.
8a3.

11. 2x^-x-^l.

\-^a-la^ 2+^x-^x^
0,

15. 7a;2-2a;-3.

21.

-81/3.

24. 2x^-3x^+x-2.

25.

16.

27. 6x-4.

28. 7a;*-2ar+l.

984

ALOEBRA
Page 230

32. Vl3.
37. 4\/2, 12\/2.

34. V2\.

35. 2\/2, 4\/2.

38. 5\/2.

Page 232
9. 10\/3.

10. 7\/2.

11. 5\/5.

12.

-3V7.
17. 866.

13. 7^2.
18.
7*94.

14. 8VTT. 19.


11-62.

15. 7a/5. 20.


5-20.

16. -4\/2.

21. -141.

22.

25-46.

23. 6-083. 27. 9-592.

24. 3-873. 28. 13711.

25. 6-782.

26. 6-481

29. 7483.

Page 234
13. 24\/3.
17, 30 + 12\/6. 14. 12\/7.

15. 5+2V6.

16. 27-4\/36.
19. 2+3\/2.

18.

a+6+2\/^

20. 12+V6. 23. 6+2\/l5.

21. 6 + \/l0.

22. 6a+66-13\/^.

24. 4V6-4.

25.

a+6-6-\/^.

26.

1.

27. 6+2\/3+2\/2+2V6.

28. 16+4Vl0-2Vi5-4\/6.
30.

29. 2a-\-2Va^^'^.
32.

nx-5y-\2^^^y\
34.
70.

31.

1.

12-W2.

33. 6V6.

35. 30-5\/6'.
46.

36. V8+\/7. 40. 30-92.

37.

42, 43.

38.

39. 9\/2.

Page 236
13. 14+8\/3. 17. Vl5.
18.

14.

6V2+W3.
.

15. 5+2^6.
-577.

16.

~_^
21.
-817.

^^~^^

19.

20.

3-636.

ANSWERS
Page 236 {continued)
22. 27. 32.
-318.
-072.

386

23.

1-491.

24. 29.

084.
-82.

25.
30.

1-225.
1-39.

26.
31.

-894.

28. 212.

3-15.

11-71.

33. 2-73.

34.

1-008.

35. V2.

Page 238
1. 12a/2. 6.

2. 12\/6.
7.

3. 10V3.

4. 62.

5. 191. 9.
^.

-4^2.

22-12^2.
11.
1.

8.

12-W6-2VZ+W2.
Vl2+Vl0.
17.

10. 9-4-V/5.
14.
I.

12.

2VT3+2V2.
16.

13. 74+11V6.
1,

15.

1-732.

2.

18. 2V2, ^V6, iV30,

^VU,

i(4v'2-2\/3).
1-291, -518.

25. 8-661, 7-937, 9-899, 26.


-817, -447, -414, -757, -337.

27. 25V3.
31.
30.

28. 2\/2.
6.

29. lilZ^, 4\/6.

30. 202.

32.

33.

4-83.

Page 241
1. a;2+a;-132=0. 4. a;2+6a;- 112=0. 7. 2. a;2-a;-156=0. 5. 5a;2- 6a; -440=0.
6. 3. a;2-49=0.

x2+6x-9400=0.

a;*-19x+88=0.

Page 242
1. 5.
6,
1,

-1, -22.
-10, -19.
9.

2. 6.

6,
2,

-25,
-5,

21.

3.

8, 19,
1, 4,

-15.

4. It, -11,2. 8.
6,

2.

7.

-32.
0. 2, 7.

-27,
0, 1,

28.

9. 9.

44.

10.

2,

-6, -3.

11.

12.

-1.

386

ALGEBRA
Page

2^
3. 1-646, -3-646. 6. -851, -2-351.

I. 4-236

--236.

2. 7-828, 2172.
5. -232, -3-232.

4. 1-916, ^9-916. 7.

3Vll.
5_V157^
o

8.

-43\/3.
,2. -JiV2-2.

9.

iV2.

10. l^vTl.

II.

Page 251
20.
1-618.
14,

21.
4.

2^,

-1.
25.
a;

22.

5, 12.
1.

23.

6,

-2.

24.

6 or 16,

or

2,

t/=2 or

26. 540.
1-449, --949.

27. 3-236, -1-236. 31. 35.


20.

28.

20c.

29.
33.

60, 90.

30. 34.

32. 3 m. perhr. 36.

8.

^, ab

a+b

20, 30.

x=2

or

^.

37.

4.

Page 255
18.
2-54, 1-0936
:

19. 8

27.

20.
:

-192, 1-302,

21. 3937

6336.

22. 4:5, 11

27,

a+3 o+5.
:

23. 11:15,13:18,2:3.3:6. 33. 4:6.


34.

31
35.
10.

-^^
cd
39. 20fem
:

32.
an.

1^ l+4a

b+c b+c

-^

Page 259
29.
11-55.

30.

10|,

^.
:

31.
3.

AE=6l DE=1\.
35.
2,

32. 240.
,

33. 9-899.

34. 2

c m

-^ n
,

ANSWERS
Page 259 (continued)
36. ior, 5or -1. ^
^

387

37.- = -=
5

3-8
^.

38.

AC=20, AE=5, DE=i.


44. 47. 17^,
^

39. 147

ft.

40. 3 or
61, 5J
;

43.5:4:2.
46. 2
:

or

-i.
48. 110
:

45.
15
:

-^,.
b-{-c

b+c

-^

3.

25, 30.

17.

Page 262
6. 6
;

7^

U^,

5,

4,%

5, 2f, 1*.

8. 2,

8.

9.

3, 6, 12.

Page 264
4.

H-

5. -17^.

6.

Ih

14.

'^.

IB. V-

17.

-^.

18. 7:

16.

Page 266

17.-^.
23. 1

18.^.

19.

2, |.

20.

f.

21.

I, |.

22.6.
26.

= 1 = 5.

24.

ft.

25.^,-1.
39. 1,3,4.

i.

32. 10, 5, +6.

38. 41|.

Page 270
1. 3a, -a. 5.

2.

b,

-5b.
6.

3.

ZmmV6.
7.

4. - 2p^\/6.

oV5236.

-6a/P+c.
feA/6''+4ac
.^^ '

-l^--f
2p

1.

g^

--bV^ac^
a

'2a

?vg^^^
CO
2

M8

ALGEBRA
Page 271

1.

1.

2.

I,

I.

3.

^,
7.

-55.

4.

i\Vn.

5.

A, -T%.

6. i,3^V89.

A. -^3.
i,

8. .^. -j\j.

9. 2a, -36.

10.

7,

J^i.

11.

-\.
^,

12.

:j>5,

-jig.

19. ,%Ti5\/l6l. 32.

20. 3Jv/57.
33.

21.

-^.

22. ^2\/5.

12V2
3.

V6.
6-18,

35. l2\/6.
382.

38. 2'414, --414.


46. 2-786 or -120.

39.

43.

63.

7-03, 8-78, 8, 2-29, 6-42, impossible,

Page 276
13.
15.
1,

-6,

-|^V^39.
16. 2,

14.

6,

-3, %\y/^^l\.
17.
3,

2,

-l\/33.
I,
\,
|.

2\/^.

2,

-5.

18.

1,

19. -|^V^=23. 21.


2, 3,

20. 22.

3,

-ii\/^, -ft^v'^a.
-l\y/l.

-1, -2.

1.

1,

Page 277

25. '*^^, !ii^29.

26. -4,3,

-i^V^^.
-,

27.

5,

-f|A/=^.

a+hV'^^db+b\

30.

--
a

31.
12.

15.

32.

o c o+c

33. -1, -2, -4,


36. a+6.

-8

34.

35. -o, -6.


39.

'+^*
0,

37. 5.10 p.m.

38. ai.
a

a-f 6

0+6 o+6

-^.

^.

ANSWERS
Page 277 (oorUintied)
40. 6076
nearly.

389

41.

->^^^+64.
32
in.

^g.
46.

3.

43.

-^^V5I, -|iVl3.

44. 10

from a comer.

27.

Page 281
7.
(6. 2).

(-V.

8.

(3, 2), (J^, ^f).

9.

(6, 4).

(-f^, -ff ).
(-5,

10.

(4, j.),

i-l -^).
(-2-275, -1-425).
k)-

11.

(2, 1),

-^).

12.

(2-525, -175),

19.

4-196, 4-732.

20. (-2, -1), (-1,

Page 283

8.

4921\/5
(2, 1),

-7 + 3\/5\

2
7.
(4, 3),

2/

(-1,

-3),(3^(-I,

_).

8.
9.

(1. 2).

(-5, -10).

{-'-/'',

=^^y

(2, 4), (3, 3), (2,

-3), (-3, -3).

Page 284

23

II

12.

(3.:p^),(^,^^j. 13.(4.l).(13V,V5^/^).

14. (1. 2).

(^, +^).
17.
35.

15.

(0. 0). (1. 1).

(^. A).

16. ^6-32, 316).

ALGEBRA
Page 286

16. (2, 1), (1, 2).


18. (2, 1), (1, 2), ('\/^l, 19. (5,2),(-i, -i).

17. (3, 2), (2, 3).

+2\/^),

(2V^, tV=1).
40. ^b+^Vb^-l6a.

20. (6,2),(-2. -6),(|^\/57)(-i|\/57).


39.
7-32, -68.

21

(5, 3), (3, 5), (6, 2), (2, 6).

Page 291

18.

(2,

2),

(2,

1),

(1, 2),

(2\/2, 2 + V2), (3i\/=7, i+iA/=7).

19.81.

21.

(-^-.^).

23

27. (10, 5), (5V2, 5V'2).


36.
(3, 2), (2, 3),

(-2V^. -2q:V^).
-1),

39. (-1,

2), (2,

{-it^vTS, -i|\/T3).
41,
(4^ 1)^ (2, 2), (J. 12), (f, 6).

40.

^+^^'-^'', -s+V2d^-s\
43.

42.

(4, 2), (2, 4), (8, 1).

(5, 1), (1, 5).

Page 296
19.
be
.

20.

x*"***.

21.

1.

22.

1.

23. a\
2,

24.

1.

25.

22", 312.

26. 3

27.

2, 9.

28.

7,

3, 2.

Page 301
45.
51.
J.
Tig.

46.

8.

47. 625.
32.

48.
4.

11 J.
.

49.
55.
^.

125.
^.

50. 56.

J.

52.

53.

54.
16^ g^ gi,

J|.

57. ^.

58. -^"53.

59.

1^

ANSWERS
Page 302
1. x-\-xi-6. 5.

391

2. x2_i.

3. a;2_i.

4. 6.

3a;2_8x2+9x-10xi

01+40-^40-1.

a2 2a-^ + 3a 2a^+l-

7. a;i+4x-lla;^-6a:^.
9. x^+xy+y'^.

8. a:2+8a;^+24x+32a;2

+ 16.

10. a'^ 3a+3a^ 1.

20. x^+x-^-l+x'^+x'^

21. 5a^"+4a"'-2.

22. 2x2+6a:+2.

23. l-2a.

24. a;-2Vz+3.

Page 304
I.
3.

x-4, a3-63.

2.

a+l+a-i.

x^-2x^-x+2xi+l, 4a2-8a+4o-i+o-2.
a2+3a+3aJ+l, 1 3x5+3x-a;i
5.

4.
6.

x*+xy +

!/*.

x^+yK

7. a^-\-b^+ck
i

8. (x+2/)(x2 y2)(x^+2/5),

9.

J-bk
x-x^ +
1.

10.

?^,
x2

a^+oM+fti a-Vod+fc.
X+2-X-1.

II. x^-2,

12. 2x-2+3x-i,

Page 305
3.
5, fo, 49.

4. 4, J^, 25, 4,
1-333, 5-62.

^,

8,

,^5.

6. S^g,

2.

7. 3-162, 1-778,

8.

4.

9. 1-732,
2.

10. 9f, -lif.

11. 100.
15.

12.

4,

13.

i,

1^.

14.

I,

f.

Vo+4

^^,

3xi+2,

^p^o26 62

16. x^y^Sx^y^

an

ALGEBRA
Page 305 {continited)

17. x^y-^ + 1+x sy 19. o'"'+a2"'6"+a"'62"._|_b3m

18.

z^y~\ a^ l+a"i
21.

20. t/+2?/^ + l. 25.

* 2 a;-V
i/'o

22. ia'-ift".

24.

-OOie, 1-44, 3-375, 8.

Vs -lY

+l
a6c
2,

26. 30.

1.

27. e'+e-',
32.

x^ 2a:2/^ + 3^y yl
32.
^.

M.
33.

4,

31. 2750.

34. x^ +2x^ + l + 2z~^ +x'^.

36. 2z+x,^-a;"i.

Page 310
7. >^4,

^27;

V^16, ^27";

^64, ^8l, ^125.


9. 12\/2.

8. 3a/2, 5\/6, Vo, 1-26, ^5.

10. 12V5.

11. 33\/2.

12. 3^2.

13. 7-^12.

14. 10v^2.

15. 9V3.

16.

0.

17. VS, V5,


126.
-63,

xV^,

r^ix, V2.

18.

2-52, 3-78,

126, 1-26.

Page 311
25. 3\/iO.

26. i\/5.

27. ^VE. 31.

28. V2.

29.

U2V2-V3).

30. ^02+6*4-6.
33. 2-517,
1-364.

Va+b-Vc.
34.
194.

32. -(z-Vx-y2). 36. 37(V3-V2).


39. 10
ft.

36. M(7-V6), 2V5.

37. i\(18-34V5).

in.

ANSWERS
Page 314
16. 21.

393

No
a.

root.

17. 22.
25.

4.

18. 23.
64.

f|.

19.
root.

100.

20. 25.

9.

24. No

5^1^
2a

26.

3.

27.

10.

28.

10.

29.

|.

30.

C2-1-1

J^

Page 317
18.
(4, 9), (9, 4). (9, 1), (1, 9). (2, 8), (8, 2).

19. 22.

(4, 16), (16, 4).

20.(17,8).
2, 1.

21. 25.

(2, I), (|, 2).

23.

24.

7,

-6.

Page 320
10. 2-823.

11.

-196.

12. 2^(2+^3), 5i(\/5+l), 3^(2- V3),2i(5\/2+3).


15.

14. 1+VS.

^, ^. ^. VS V2 V2
,

17. 2-309.

18. 2+3V2.

Page 322
0. 25\/^.
14. 11.
68.

12. -25.

13.

1+V^.

-l-y^.
2

i5_ 2a2-262.

Page 324
I.
5.

2+2v^-2\/3.
9.

2. 2+|\/6.
7.
7i.

4. 1-98, 3-15, 1-39, 3-55.


7,

6. IJ.

8.

-1.

9. 20.

10.

13.

II. 6-f2vl5, x^+2xy+y^-Ax-'iy.

12. |V3. 15.


12.

13. 4a4-2\/4a2-6.

14. ^(i'*-2p?+g2).

3M

ALGEBRA
Page 324 {continued)

16. x*-lx^+2x+2.

17.

0.

18. 2a, 4.a^-2, 8a^-i


2m,

19.

ANSWERS
Page 338
6. Rational. 9. Real and equal. 7. Real and irrational.

396

8. Imaginary

10. Rational.

11. Real.

13.

4.

14. 5.

16. -i.

19. -
tn

20.

2,

-?.

Page 340
8.

(x-{-2+Vl){x+2-Vl).

9. 16.

10. 6a.

11. (x-3 + 2V5)(a;-3-2\/5).


12. (3x-4t/)(3a;+42/){4x-32/)(4z+32/); |, J.

13. 174; (8a;+7)(15x-4)

14. 62^4a.

Page 341
4.
6,

^,

|, 4,

-14.

5.

f,

A.

6.

16x2-40x+21=0.

7. 12^,

8. 6x2-19a;+15=0.

9. -25.

10. x212x+35=0.
12. x^-'kc+3=0.

11. (72x+l)(73x-l), (13x+ll)(17x-15).

13. 2a + 26-2c.
15. (x+3+V'2)(x+3-a/2).
17. ax^ + 3bx+9c=0. 19.
1,

14. 97x2-53x-17=0. 16. 2x*-17x=0.


18. acx^x{b^2ac)+ac=0.

+^~^
a+ft+c

23. 12.

26. 4.

27.

3mn-m^
32.

28.

a.

29. 33.

t^ 0+6 c o+o

^
1.

30.

31. c+6-a.

o=8

or

8,

35.

6. 2.

36. (x+b-{-Vb^^){x+b-Vb*^).

SM

ALGEBRA
Page 345

16. 2a-6+3.

M.Pllll^. vq+r

18.-7,1,19.
20. 2o-3, 3o-4.

19. (x-y+3)(a;+2!/-4). 21. (2a-6+c)(3o-6-c)(3a+26-2c).

Page 348
13. (2a;-7/-53)(4a;2+2/24-2522+2x?/+10xz-5i/2).

14. (a+6+c+l)(a2+62+c2+2a6-ac-6c-a-6-c+l).
15. a3-63-c3-3a6c.
17. l-a3-t3-3a6. 19. 16. 8x3-?/3+2723+18x?/2.

18. 8a3-27fe3-64-72a6.

l+a2+&2+a-6+a*.
22. 3a+fe.

20.

9w2+n2+l+3TOn+3m-.
24. 2a-5. 35.
0.

21. a+56-1.

23. x2-3a;+9.

25.

a+6+c+d-l.
2a-36+3c
'

28. 27x3-8!/3+334.i8xj/z.

^
40. a,b.

"l

^^- (-+2/+^)(a+6).

Page 351
9. (-2/)(t/-3)(a;-2).

10. {x-y){y-z)(x-z).
12. (a-6){6-c){c-o)(o+6+c).
14. a+6+c.
a, 6.

11. (x-?/)(y-z)(z-x).

13. {a-6)(6-c)(c-a)(a+6)(6+c)(c4-a).

15.

^,

^.
6

16. ?-^

^-:::^-

a 6

17.

18.

b-\-a

^^ a

^=^.
b

19.

*,

*^.

ba

20. ax-a-b.

ANSWERS
Page 353
10. x*-2x^i-4x^-Sx+l6.

307

11. r-3a;2+9a;-27.

12. x^+x'>a+3^a^+x^a'^+a*. 14. a; 6, a+6,


17. a8-6.
1

13. (a+6f-(a+6)2+o+5-l.
15.
o*

X 4, m+-, a;+y 1.

1.

16. m*+l.

20. a2+ap+3=0.

22.

1, 9.

23.

II

'11

a;3

4-a;3a3_|_a3^

x^ x3a3_^a3^

ii

5.1

a2

la

a;5_|_a;5a5_j_a;5a5_j_x5a5-j_o5^

ai -k I la 4 4 a;6 x%5+x5o6 ar^a^+os.

Page 357
13. 3(a*4-6^+c'^)-2(a6+6c+ca).
16. -3(a26-afe2_^62(,_jc2+c2a-ca2).

14. 2(a2+62+c).

15.

0.

17. {x-y){y-z)(x-z).
20. 24a6c.

18. (j;-r/)(^-2)(x-2). 21. 3(x-?/)(j/-2)(z-x).

19. (a+6)(6+c)(c+a).

22. {a+b){b+c)(c-^a).

23.

(a-fe)(6-c)(c a)(a+6+c).
25.
1.

24. {a-b){b-c)(c-a){a+b){b+c){c-\-a). 27. 32.


1.

26.

1.

28.

-4-abc

29.

0.

30. a+b+c.

31. -{x+y+z).

3.

33.

6abc.

35. 3(x2+?/2+z2) + 2(xy+2/z+zx).

Page 362

24.

18.

26. 02-262, a^-Sab^.

27. a=c*+26.

28. o+2c=3a6.

808

ALGEBRA
Page 362

e. k^+ri\y^-^iZ-{-kxvz.

9. x=(a-l)(l-6).

H-

aoo

12. l(a+b+c).

15. -{ab+bc+ca). 23.

16.

~ooc

18. -i(a+6+c).

22. {x-\){3^-2x-^+2xi2).

-a-6-c.

26. (x+l)(a+6+c).
31. 9,17.

29. 3(a:*+i/2^z2) + |(a;j/+j/2+2x).


32.
38, 70.

30. o2=26+c.

HR

38-05

4^'J4

.(

CURRKUUM
--

LABORATORY
f8ei7 sot^eo t72t76 s.,

--

Mi

hM

Auvusn o a dO xxisuaAiNO

u./

LIBRARY USE ONLY


lO

,1111

\9^

university of
british

Columbia

ET-6

Вам также может понравиться